Sat Practise Test 1

  • May 2020
  • PDF

This document was uploaded by user and they confirmed that they have the permission to share it. If you are author or own the copyright of this book, please report to us by using this DMCA report form. Report DMCA


Overview

Download & View Sat Practise Test 1 as PDF for free.

More details

  • Words: 43,883
  • Pages: 82
Answer Sheets

1

A B D E C O O O O O

6

A B D E C O O O O O

11

A B D E C O O O O O

16

A B D E C O O O O O

2

A B D E C O O O O O

7

A B D E C O O O O O

12

A B D E C O O O O O

17

A B D E C O O O O O

3

A B D E C O O O O O

8

A B D E C O O O O O

13

A B D E C O O O O O

18

A B D E C O O O O O

4

A O

B D E C O O O O

9

A O

B D E C O O O O

14

A O

B D E C O O O O

19

A B D E C O O O O O

5

A B D E C O O O O O

10

A B D E C O O O O O

15

A B D E C O O O O O

20

A B D E C O O O O O

1

A B D E C O O O O O

7

A B D E C O O O O O

12

A B D E C O O O O O

17

A B D E C O O O O O

SECTION

2

A B D E C O O O O O

8

A B D E C O O O O O

13

A B D E C O O O O O

18

A B D E C O O O O O

2

3

A B D E C O O O O O

9

A B D E C O O O O O

14

A B D E C O O O O O

19

A B D E C O O O O O

4

A B D E C O O O O O

10

A B D E C O O O O O

15

A B D E C O O O O O

20

A B D E C O O O O O

5

A O

11

A O

E O

16

A O

E O

21

A B D E C O O O O O

6

A B D E C O O O O O

1

A B D E C O O O O O

9

A B D E C O O O O O

17

A B D E C O O O O O

24

A B D E C O O O O O

2

A B D E C O O O O O

10

A B D E C O O O O O

18

A B D E C O O O O O

25

A B D E C O O O O O

SECTION

3

A B D E C O O O O O

11

A B D E C O O O O O

19

A B D E C O O O O O

26

A B D E C O O O O O

3

4

A B D E C O O O O O

12

A B D E C O O O O O

20

A B D E C O O O O O

27

A B D E C O O O O O

5

A O

E O

13

A O

E O

21

A O

E O

28

A B D E C O O O O O

6

A B D E C O O O O O

14

A B D E C O O O O O

22

A B D E C O O O O O

29

A B D E C O O O O O

7

A B D E C O O O O O

15

A B D E C O O O O O

23

A B D E C O O O O O

30

A B D E C O O O O O

8

A B D E C O O O O O

16

A B D E C O O O O O

1

A B D E C O O O O O

8

A B D E C O O O O O

15

A B D E C O O O O O

22

A B D E C O O O O O

2

A B D E C O O O O O

9

A B D E C O O O O O

16

A B D E C O O O O O

23

A B D E C O O O O O

E O

17

A O

E O

24

A B D E C O O O O O

SECTION

1

SECTION

B O

B O

C O

C O

D O

D O

E O

B O

B O

C O

C O

D O

D O

B O

B O

C O

C O

D O

D O

3

A O

E O

10

A O

4

A B D E C O O O O O

11

A B D E C O O O O O

18

A B D E C O O O O O

25

A B D E C O O O O O

5

A B D E C O O O O O

12

A B D E C O O O O O

19

A B D E C O O O O O

26

A B D E C O O O O O

6

A O

B D E C O O O O

13

A O

B D E C O O O O

20

A O

27

A B D E C O O O O O

7

A B D E C O O O O O

14

A B D E C O O O O O

21

A B D E C O O O O O

1

A B D E C O O O O O

7

A B D E C O O O O O

12

A B D E C O O O O O

17

A B D E C O O O O O

SECTION

2

A B D E C O O O O O

8

A B D E C O O O O O

13

A B D E C O O O O O

18

A B D E C O O O O O

5

3

A B D E C O O O O O

9

A B D E C O O O O O

14

A B D E C O O O O O

19

A B D E C O O O O O

4

A B D E C O O O O O

10

A B D E C O O O O O

15

A B D E C O O O O O

20

A B D E C O O O O O

5

A O

11

A O

16

A O

21

A B D E C O O O O O

6

A B D E C O O O O O

4

B O

B O

C O

C O

D O

D O

E O

B O

B O

C O

C O

D O

D O

E O

B O

C O

D O

B D E C O O O O

B O

C O

D O

E O

Copyright © 2005 Thomson Peterson’s, a part of The Thomson Corporaton SAT is a registered trademark of the College Entrance Examination Board, which was not involved in the production of and does not endorse this product.

1

Answer Sheets

SECTION

1

A B D E C O O O O O

5

A B D E C O O O O O

9

A B D E C O O O O O

13

A B D E C O O O O O

6

2

A B D E C O O O O O

6

A B D E C O O O O O

10

A B D E C O O O O O

14

A B D E C O O O O O

3

A B D E C O O O O O

7

A B D E C O O O O O

11

A B D E C O O O O O

15

A B D E C O O O O O

4

A O

8

A O

12

A O

16

A B D E C O O O O O

B D E C O O O O

B D E C O O O O

B D E C O O O O

SECTION

7

2

For Questions 1–13: Only answers entered in the ovals in each grid area will be scored. You will not receive credit for anything written in the boxes above the ovals.

1

2

3

4

5

6

7

8

9

10

11

12

13

Copyright © 2005 Thomson Peterson’s, a part of The Thomson Corporaton SAT is a registered trademark of the College Entrance Examination Board, which was not involved in the production of and does not endorse this product.

Practice Test

1 Section 1 20 Questions j Time—25 Minutes 12345678901234567890123456789012123456789012345678901234567890121234567 7 12234567890123456789012345678901212345678901234567890123456789012123456 7 1234567890123456789012345678901212345678901234567890123456789012123456 7 1234567890123456789012345678901212345678901234567890123456789012123456 7 1234567890123456789012345678901212345678901234567890123456789012123456 Directions: Read each of the passages carefully, then answer the questions that come after them. 7 1234567890123456789012345678901212345678901234567890123456789012123456 34567890123456789012345678901212345678901234567890123456789012123456 7 1234567890123456789012345678901212345678901234567890123456789012123456 1 The answer to each question may be stated overtly or only implied. You will not have to use 7 7 12234567890123456789012345678901212345678901234567890123456789012123456 outside knowledge to answer the questions—all the material you will need will be in the passage 7 1234567890123456789012345678901212345678901234567890123456789012123456 7 1234567890123456789012345678901212345678901234567890123456789012123456 itself. In some cases, you will be asked to read two related passages and answer questions about 34567890123456789012345678901212345678901234567890123456789012123456 1234567890123456789012345678901212345678901234567890123456789012123456 77 1234567890123456789012345678901212345678901234567890123456789012123456 their relationship to one another. Mark the letter of your choice on your answer sheet. 7 1234567890123456789012345678901212345678901234567890123456789012123456 1234567890123456789012345678901212345678901234567890123456789012123456 77 1234567890123456789012345678901212345678901234567890123456789012123456 7 1234567890123456789012345678901212345678901234567890123456789012123456 7 1234567890123456789012345678901212345678901234567890123456789012123456 7 1234567890123456789012345678901212345678901234567890123456789012123456 Musical notes, like all sounds, are a result of 1. In this passage, musical notes are used 7 1234567890123456789012345678901212345678901234567890123456789012123456 7 1 the sound waves created by movement, like the primarily to 2 34567890123456789012345678901212345678901234567890123456789012123456 1234567890123456789012345678901212345678901234567890123456789012123456 77 1234567890123456789012345678901212345678901234567890123456789012123456 (A) illustrate the difference between 7 1 rush of air through a trumpet. Musical notes 7 1234567890123456789012345678901212345678901234567890123456789012123456 are very regular sound waves. The qualities of human-produced and nonhuman7 12234567890123456789012345678901212345678901234567890123456789012123456 34567890123456789012345678901212345678901234567890123456789012123456 7 1234567890123456789012345678901212345678901234567890123456789012123456 these waves—how much they displace molproduced sound. 7 1234567890123456789012345678901212345678901234567890123456789012123456 7 1 ecules, and how often they do so—give the note (B) demonstrate the difference between 7 12234567890123456789012345678901212345678901234567890123456789012123456 its particular sound. How much a sound wave 7 1234567890123456789012345678901212345678901234567890123456789012123456 musical sound and all other sound. 34567890123456789012345678901212345678901234567890123456789012123456 7 1234567890123456789012345678901212345678901234567890123456789012123456 displaces molecules affects the volume of the 7 1234567890123456789012345678901212345678901234567890123456789012123456 (C) provide an example of sound 7 1 note. How frequently a sound wave reaches 7 1234567890123456789012345678901212345678901234567890123456789012123456 properties common to all sound. 7 1234567890123456789012345678901212345678901234567890123456789012123456 your ear determines whether the note is high2 34567890123456789012345678901212345678901234567890123456789012123456 7 1234567890123456789012345678901212345678901234567890123456789012123456 (D) convey the difference between 7 1234567890123456789012345678901212345678901234567890123456789012123456 or low-pitched. When scientists describe how 7 1234567890123456789012345678901212345678901234567890123456789012123456 musical pitch and frequency pitch. 7 1234567890123456789012345678901212345678901234567890123456789012123456 high or low a sound is, they use a numerical 7 1 (E) explain the connection between 2 34567890123456789012345678901212345678901234567890123456789012123456 7 1234567890123456789012345678901212345678901234567890123456789012123456 measurement of its frequency, such as “440 7 1234567890123456789012345678901212345678901234567890123456789012123456 number and letter names for sounds. 7 1234567890123456789012345678901212345678901234567890123456789012123456 vibrations per second,” rather than the letters 1234567890123456789012345678901212345678901234567890123456789012123456 77 1 musicians use. 7 12234567890123456789012345678901212345678901234567890123456789012123456 7 1234567890123456789012345678901212345678901234567890123456789012123456 7 1234567890123456789012345678901212345678901234567890123456789012123456 7 1234567890123456789012345678901212345678901234567890123456789012123456 1 34567890123456789012345678901212345678901234567890123456789012123456 7 2 34567890123456789012345678901212345678901234567890123456789012123456 1234567890123456789012345678901212345678901234567890123456789012123456 77 1 7 12234567890123456789012345678901212345678901234567890123456789012123456 1 34567890123456789012345678901212345678901234567890123456789012123456 7 7 12234567890123456789012345678901212345678901234567890123456789012123456 1 34567890123456789012345678901212345678901234567890123456789012123456 7 1234567890123456789012345678901212345678901234567890123456789012123456 7 1234567890123456789012345678901212345678901234567890123456789012123456 7 1234567890123456789012345678901212345678901234567890123456789012123456 7 12345678901234567890123456789012123456789012345678901234567890121234567 Copyright © 2005 Thomson Peterson’s, a part of The Thomson Corporaton SAT is a registered trademark of the College Entrance Examination Board, which was not involved in the production of and does not endorse this product.

3

12345678901234567890123456789012123456789012345678901234567890121234567 1234567890123456789012345678901212345678901234567890123456789012123456 7 34567890123456789012345678901212345678901234567890123456789012123456 7 12 2. All of the following are true statements 7 3. The passage cites Walker’s interaction 1234567890123456789012345678901212345678901234567890123456789012123456 34567890123456789012345678901212345678901234567890123456789012123456 7 12 about pitch, according to the passage, with Langston Hughes as 34567890123456789012345678901212345678901234567890123456789012123456 7 12 7 1234567890123456789012345678901212345678901234567890123456789012123456 EXCEPT: 34567890123456789012345678901212345678901234567890123456789012123456 7 12 (A) instrumental in her early work being 2 34567890123456789012345678901212345678901234567890123456789012123456 7 1 34567890123456789012345678901212345678901234567890123456789012123456 7 12 (A) Nonmusical sounds cannot be published. 34567890123456789012345678901212345678901234567890123456789012123456 7 12 34567890123456789012345678901212345678901234567890123456789012123456 12 referred to in terms of pitch. (B) influential in her decision to study at 7 34567890123456789012345678901212345678901234567890123456789012123456 7 12 2 34567890123456789012345678901212345678901234567890123456789012123456 7 1 (B) Pitch is solely determined by the Northwestern University. 34567890123456789012345678901212345678901234567890123456789012123456 7 12 34567890123456789012345678901212345678901234567890123456789012123456 7 12 frequency of the sound wave. (C) not as important at the time it 34567890123456789012345678901212345678901234567890123456789012123456 7 12 34567890123456789012345678901212345678901234567890123456789012123456 7 12 (C) Pitch is closely related to the happened as it is now, due to 7 1234567890123456789012345678901212345678901234567890123456789012123456 34567890123456789012345678901212345678901234567890123456789012123456 7 12 vibration of molecules. Hughes’ fame. 34567890123456789012345678901212345678901234567890123456789012123456 7 12 34567890123456789012345678901212345678901234567890123456789012123456 7 12 (D) Pitch cannot be accurately described (D) a great encouragement for Walker’s 34567890123456789012345678901212345678901234567890123456789012123456 7 12 2 34567890123456789012345678901212345678901234567890123456789012123456 7 1 with letter names. confidence as a poet. 34567890123456789012345678901212345678901234567890123456789012123456 7 12 34567890123456789012345678901212345678901234567890123456789012123456 7 12 (E) Humans’ perception of pitch is not (E) important to her choice to study at 34567890123456789012345678901212345678901234567890123456789012123456 7 12 34567890123456789012345678901212345678901234567890123456789012123456 7 12 affected by the intensity of the New Orleans University. 34567890123456789012345678901212345678901234567890123456789012123456 7 12 34567890123456789012345678901212345678901234567890123456789012123456 7 12 sound wave. 34567890123456789012345678901212345678901234567890123456789012123456 7 12 4. The passage suggests that Walker’s 34567890123456789012345678901212345678901234567890123456789012123456 7 12 Line Margaret Walker, who would become 34567890123456789012345678901212345678901234567890123456789012123456 7 12 decision to become a poet 2 34567890123456789012345678901212345678901234567890123456789012123456 7 1234567890123456789012345678901212345678901234567890123456789012123456 one of the most important twentieth7 1 34567890123456789012345678901212345678901234567890123456789012123456 7 12 (A) occurred before she entered college. century African-American poets, was 34567890123456789012345678901212345678901234567890123456789012123456 7 12 34567890123456789012345678901212345678901234567890123456789012123456 12 (B) was primarily a result of her interac- 7 born in Birmingham, Alabama, in 1915. 34567890123456789012345678901212345678901234567890123456789012123456 7 12 34567890123456789012345678901212345678901234567890123456789012123456 7 12 tion with Hughes. (5) Her parents, a minister and a music 34567890123456789012345678901212345678901234567890123456789012123456 7 12 34567890123456789012345678901212345678901234567890123456789012123456 7 12 (C) was not surprising, given her teacher, encouraged her to read poetry 34567890123456789012345678901212345678901234567890123456789012123456 7 12 2 34567890123456789012345678901212345678901234567890123456789012123456 7 1234567890123456789012345678901212345678901234567890123456789012123456 upbringing. and philosophy even as a child. Walker 7 1234567890123456789012345678901212345678901234567890123456789012123456 7 1234567890123456789012345678901212345678901234567890123456789012123456 (D) occurred after her transfer to completed her high school education at 1234567890123456789012345678901212345678901234567890123456789012123456 77 1 Northwestern University. Gilbert Academy in New Orleans and 34567890123456789012345678901212345678901234567890123456789012123456 7 12 34567890123456789012345678901212345678901234567890123456789012123456 7 12 (E) was sudden and immediately (10) went on to attend New Orleans Univer7 1234567890123456789012345678901212345678901234567890123456789012123456 successful. 7 1234567890123456789012345678901212345678901234567890123456789012123456 sity for two years. It was then that the 34567890123456789012345678901212345678901234567890123456789012123456 7 12 2 34567890123456789012345678901212345678901234567890123456789012123456 7 1234567890123456789012345678901212345678901234567890123456789012123456 important Harlem Renaissance poet 7 1234567890123456789012345678901212345678901234567890123456789012123456 7 1 Langston Hughes recognized her talent Questions 5–10 are based on the following 34567890123456789012345678901212345678901234567890123456789012123456 7 12 34567890123456789012345678901212345678901234567890123456789012123456 7 12 and persuaded her to continue her passage. 34567890123456789012345678901212345678901234567890123456789012123456 7 12 (15) education in the North. She transferred 34567890123456789012345678901212345678901234567890123456789012123456 7 12 7 1234567890123456789012345678901212345678901234567890123456789012123456 Line F. Scott Fitzgerald was a prominent to Northwestern University in Illinois, 7 1234567890123456789012345678901212345678901234567890123456789012123456 1234567890123456789012345678901212345678901234567890123456789012123456 American writer of the twentieth century. 7 where she received a degree in English in 2 34567890123456789012345678901212345678901234567890123456789012123456 7 1234567890123456789012345678901212345678901234567890123456789012123456 This passage comes from one of his short 7 1234567890123456789012345678901212345678901234567890123456789012123456 1935. Her poem, “For My People,” 7 1234567890123456789012345678901212345678901234567890123456789012123456 stories and tells the story of a young John 7 1234567890123456789012345678901212345678901234567890123456789012123456 which would remain one of her most 7 1 (5) Unger leaving home for boarding school. 7 34567890123456789012345678901212345678901234567890123456789012123456 12 (20) important works, was also her first 34567890123456789012345678901212345678901234567890123456789012123456 7 12 John T. Unger came from a family 34567890123456789012345678901212345678901234567890123456789012123456 7 12 publication, appearing in Poetry maga34567890123456789012345678901212345678901234567890123456789012123456 7 12 that had been well known in Hades—a 7 1234567890123456789012345678901212345678901234567890123456789012123456 zine in 1937. 2 34567890123456789012345678901212345678901234567890123456789012123456 1234567890123456789012345678901212345678901234567890123456789012123456 small town on the Mississippi River—for 7 7 1234567890123456789012345678901212345678901234567890123456789012123456 7 1234567890123456789012345678901212345678901234567890123456789012123456 several generations. John’s father had 1234567890123456789012345678901212345678901234567890123456789012123456 77 1 (10) held the amateur golf championship 34567890123456789012345678901212345678901234567890123456789012123456 7 12 7 1234567890123456789012345678901212345678901234567890123456789012123456 through many a heated contest; Mrs. 34567890123456789012345678901212345678901234567890123456789012123456 7 12 7 1234567890123456789012345678901212345678901234567890123456789012123456 Unger was known “from hot-box to 34567890123456789012345678901212345678901234567890123456789012123456 7 12 2 34567890123456789012345678901212345678901234567890123456789012123456 7 1 hot-bed,” as the local phrase went, for 7 1234567890123456789012345678901212345678901234567890123456789012123456 1234567890123456789012345678901212345678901234567890123456789012123456 77 1234567890123456789012345678901212345678901234567890123456789012123456 12345678901234567890123456789012123456789012345678901234567890121234567 4

Copyright © 2005 Thomson Peterson’s, a part of The Thomson Corporaton SAT is a registered trademark of the College Entrance Examination Board, which was not involved in the production of and does not endorse this product.

12345678901234567890123456789012123456789012345678901234567890121234567 1234567890123456789012345678901212345678901234567890123456789012123456 7 7 12234567890123456789012345678901212345678901234567890123456789012123456 her political addresses; and young John So the old man and the young shook 7 1 34567890123456789012345678901212345678901234567890123456789012123456 7 12234567890123456789012345678901212345678901234567890123456789012123456 (15) T. Unger, who had just turned sixteen, hands, and John walked away with tears 7 1234567890123456789012345678901212345678901234567890123456789012123456 7 1 34567890123456789012345678901212345678901234567890123456789012123456 had danced all the latest dances from streaming from his eyes. Ten minutes 7 12234567890123456789012345678901212345678901234567890123456789012123456 34567890123456789012345678901212345678901234567890123456789012123456 7 1 New York before he put on long troulater he had passed outside the city limits 7 12234567890123456789012345678901212345678901234567890123456789012123456 7 1234567890123456789012345678901212345678901234567890123456789012123456 sers. And now, for a certain time, he was (60) and he stopped to glance back for the last 7 1234567890123456789012345678901212345678901234567890123456789012123456 7 1234567890123456789012345678901212345678901234567890123456789012123456 to be away from home. time. Over the gates the old-fashioned 7 1 34567890123456789012345678901212345678901234567890123456789012123456 7 12234567890123456789012345678901212345678901234567890123456789012123456 (20) That respect for a New England Victorian motto seemed strangely 7 1234567890123456789012345678901212345678901234567890123456789012123456 7 1234567890123456789012345678901212345678901234567890123456789012123456 education which is the bane of all attractive to him. His father had tried 7 1234567890123456789012345678901212345678901234567890123456789012123456 34567890123456789012345678901212345678901234567890123456789012123456 1 provincial places, which drains them time and time again to have it changed to 7 7 12234567890123456789012345678901212345678901234567890123456789012123456 7 1234567890123456789012345678901212345678901234567890123456789012123456 (65) yearly of their most promising young something with a little more push and 7 1234567890123456789012345678901212345678901234567890123456789012123456 7 1234567890123456789012345678901212345678901234567890123456789012123456 men, had seized upon his parents. verve about it, such as “Hades—Your 34567890123456789012345678901212345678901234567890123456789012123456 7 1 7 12234567890123456789012345678901212345678901234567890123456789012123456 (25) Nothing would suit them but that he Opportunity,” or else a plain “Welcome” 7 1234567890123456789012345678901212345678901234567890123456789012123456 7 1234567890123456789012345678901212345678901234567890123456789012123456 should go to St. Midas’s School near sign set over a hearty handshake pricked 7 1234567890123456789012345678901212345678901234567890123456789012123456 34567890123456789012345678901212345678901234567890123456789012123456 1234567890123456789012345678901212345678901234567890123456789012123456 Boston—Hades was too small to hold out in electric lights. The old motto was a 7 7 1234567890123456789012345678901212345678901234567890123456789012123456 7 1234567890123456789012345678901212345678901234567890123456789012123456 (70) their darling and gifted son. Now in little depressing, Mr. Unger had 1234567890123456789012345678901212345678901234567890123456789012123456 77 1234567890123456789012345678901212345678901234567890123456789012123456 Hades—as you know if you ever have thought—but now ... 7 1234567890123456789012345678901212345678901234567890123456789012123456 7 1 (30) been there—the names of the more So John took his look and then set 7 12234567890123456789012345678901212345678901234567890123456789012123456 fashionable preparatory schools and his face resolutely toward his destination. 7 1234567890123456789012345678901212345678901234567890123456789012123456 7 1234567890123456789012345678901212345678901234567890123456789012123456 colleges mean very little. The inhabitants And, as he turned away, the lights of 7 1234567890123456789012345678901212345678901234567890123456789012123456 7 1234567890123456789012345678901212345678901234567890123456789012123456 (75) have been so long out of the world that, Hades against the sky seemed full of a 7 1234567890123456789012345678901212345678901234567890123456789012123456 7 1234567890123456789012345678901212345678901234567890123456789012123456 though they make a show of keeping warm and passionate beauty. 7 1234567890123456789012345678901212345678901234567890123456789012123456 34567890123456789012345678901212345678901234567890123456789012123456 7 1234567890123456789012345678901212345678901234567890123456789012123456 (35) up-to-date in dress and manners and 7 1234567890123456789012345678901212345678901234567890123456789012123456 7 1234567890123456789012345678901212345678901234567890123456789012123456 literature, they depend to a great extent 7 1234567890123456789012345678901212345678901234567890123456789012123456 5. The tone of line 28 can best be described as 7 1 on hearsay, and a function that in Hades 2 34567890123456789012345678901212345678901234567890123456789012123456 1234567890123456789012345678901212345678901234567890123456789012123456 77 1234567890123456789012345678901212345678901234567890123456789012123456 (A) compassionate. would be considered elaborate would 7 1 (B) sincere. 7 1234567890123456789012345678901212345678901234567890123456789012123456 doubtless be hailed by a Chicago 7 12234567890123456789012345678901212345678901234567890123456789012123456 (C) sardonic. 34567890123456789012345678901212345678901234567890123456789012123456 7 1234567890123456789012345678901212345678901234567890123456789012123456 (40) beef-princess as “perhaps a little tacky.” 7 1234567890123456789012345678901212345678901234567890123456789012123456 (D) dismayed. 7 1 John T. Unger was on the eve of 7 12234567890123456789012345678901212345678901234567890123456789012123456 (E) understated. 7 1234567890123456789012345678901212345678901234567890123456789012123456 departure. Mrs. Unger, with maternal 34567890123456789012345678901212345678901234567890123456789012123456 1234567890123456789012345678901212345678901234567890123456789012123456 77 1234567890123456789012345678901212345678901234567890123456789012123456 fatuity, packed his trunks full of linen 1 6. The “Chicago beef-princess” (lines 39–40) 7 7 1234567890123456789012345678901212345678901234567890123456789012123456 suits and electric fans, and Mr. Unger 7 1234567890123456789012345678901212345678901234567890123456789012123456 can best be described as representing the 2 34567890123456789012345678901212345678901234567890123456789012123456 7 1234567890123456789012345678901212345678901234567890123456789012123456 (45) presented his son with an asbestos 7 1234567890123456789012345678901212345678901234567890123456789012123456 Chicago upper class by way of which 7 1234567890123456789012345678901212345678901234567890123456789012123456 pocket-book stuffed with money. 7 1234567890123456789012345678901212345678901234567890123456789012123456 literary device? 7 1 “Remember, you are always welcome 2 34567890123456789012345678901212345678901234567890123456789012123456 7 1234567890123456789012345678901212345678901234567890123456789012123456 here,” he said. “You can be sure, boy, (A) Anachronism 7 1234567890123456789012345678901212345678901234567890123456789012123456 7 1234567890123456789012345678901212345678901234567890123456789012123456 that we’ll keep the home fires burning.” (B) Simile 7 1234567890123456789012345678901212345678901234567890123456789012123456 7 1 (50) “I know,” answered John huskily. (C) Apostrophe 2 34567890123456789012345678901212345678901234567890123456789012123456 1234567890123456789012345678901212345678901234567890123456789012123456 77 1234567890123456789012345678901212345678901234567890123456789012123456 “Don’t forget who you are and where (D) Metaphor 7 1234567890123456789012345678901212345678901234567890123456789012123456 7 1234567890123456789012345678901212345678901234567890123456789012123456 you come from,” continued his father (E) Neologism 7 1 2 34567890123456789012345678901212345678901234567890123456789012123456 7 1234567890123456789012345678901212345678901234567890123456789012123456 proudly, “and you can do nothing to 7 1 7 12234567890123456789012345678901212345678901234567890123456789012123456 harm you. You are an Unger—from 1 34567890123456789012345678901212345678901234567890123456789012123456 7 7 12234567890123456789012345678901212345678901234567890123456789012123456 (55) Hades.” 7 1 34567890123456789012345678901212345678901234567890123456789012123456 1234567890123456789012345678901212345678901234567890123456789012123456 7 1234567890123456789012345678901212345678901234567890123456789012123456 7 1234567890123456789012345678901212345678901234567890123456789012123456 7 12345678901234567890123456789012123456789012345678901234567890121234567 Copyright © 2005 Thomson Peterson’s, a part of The Thomson Corporaton SAT is a registered trademark of the College Entrance Examination Board, which was not involved in the production of and does not endorse this product.

5

12345678901234567890123456789012123456789012345678901234567890121234567 1234567890123456789012345678901212345678901234567890123456789012123456 7 34567890123456789012345678901212345678901234567890123456789012123456 7 12 7. The phrase “maternal fatuity” (line 7 10. The names Hades, St. Midas, and Unger 1234567890123456789012345678901212345678901234567890123456789012123456 34567890123456789012345678901212345678901234567890123456789012123456 7 12 42–43), suggests that suggest that the passage can be considered 7 34567890123456789012345678901212345678901234567890123456789012123456 12 7 1234567890123456789012345678901212345678901234567890123456789012123456 a(n) 34567890123456789012345678901212345678901234567890123456789012123456 7 12 (A) John will not need linen suits and 2 34567890123456789012345678901212345678901234567890123456789012123456 7 1 34567890123456789012345678901212345678901234567890123456789012123456 7 12 electric fans at St. Midas’s. (A) epic poem. 34567890123456789012345678901212345678901234567890123456789012123456 7 12 34567890123456789012345678901212345678901234567890123456789012123456 7 12 (B) John’s mother packed frantically and (B) euphemism. 34567890123456789012345678901212345678901234567890123456789012123456 7 12 2 34567890123456789012345678901212345678901234567890123456789012123456 7 1 ineffectively. (C) aphorism. 34567890123456789012345678901212345678901234567890123456789012123456 7 12 34567890123456789012345678901212345678901234567890123456789012123456 7 12 (C) John’s mother was excessively (D) satire. 34567890123456789012345678901212345678901234567890123456789012123456 7 12 34567890123456789012345678901212345678901234567890123456789012123456 7 12 doting. (E) allegory. 7 1234567890123456789012345678901212345678901234567890123456789012123456 34567890123456789012345678901212345678901234567890123456789012123456 7 12 (D) John resented his mother packing for 34567890123456789012345678901212345678901234567890123456789012123456 7 12 34567890123456789012345678901212345678901234567890123456789012123456 7 12 him. Questions 11–20 are based on the following 34567890123456789012345678901212345678901234567890123456789012123456 7 12 2 34567890123456789012345678901212345678901234567890123456789012123456 7 1 (E) John never enjoyed linen suits or passage. 34567890123456789012345678901212345678901234567890123456789012123456 7 12 34567890123456789012345678901212345678901234567890123456789012123456 7 12 electric fans. 34567890123456789012345678901212345678901234567890123456789012123456 7 12 This passage discusses the work of Abe Kobo, a 34567890123456789012345678901212345678901234567890123456789012123456 7 12 2 34567890123456789012345678901212345678901234567890123456789012123456 7 1234567890123456789012345678901212345678901234567890123456789012123456 Japanese novelist of the twentieth century. 8. From the conversation between John and 7 1234567890123456789012345678901212345678901234567890123456789012123456 7 1234567890123456789012345678901212345678901234567890123456789012123456 his father in paragraphs 3–6, it can be 7 1234567890123456789012345678901212345678901234567890123456789012123456 Line Abe Kobo is one of the great writers of 7 1234567890123456789012345678901212345678901234567890123456789012123456 inferred that John feels 7 1234567890123456789012345678901212345678901234567890123456789012123456 postwar Japan. His literature is richer, 7 1 34567890123456789012345678901212345678901234567890123456789012123456 7 12 less predictable, and wider-ranging than (A) rejected and angry. 34567890123456789012345678901212345678901234567890123456789012123456 7 12 34567890123456789012345678901212345678901234567890123456789012123456 7 12 that of his famed contemporaries, (B) melancholic but composed. 34567890123456789012345678901212345678901234567890123456789012123456 7 12 34567890123456789012345678901212345678901234567890123456789012123456 7 12 (5) Mishima Yukio and Nobel laureate Oe (C) impassive and indifferent. 34567890123456789012345678901212345678901234567890123456789012123456 7 12 Kenzaburo. It is infused with the passion 7 34567890123456789012345678901212345678901234567890123456789012123456 12 (D) resigned but filled with dread. 34567890123456789012345678901212345678901234567890123456789012123456 7 12 and strangeness of his experiences in 34567890123456789012345678901212345678901234567890123456789012123456 7 12 (E) relieved but apprehensive. 34567890123456789012345678901212345678901234567890123456789012123456 7 12 Manchuria, which was a Japanese colony 7 34567890123456789012345678901212345678901234567890123456789012123456 12 34567890123456789012345678901212345678901234567890123456789012123456 7 12 on mainland China before World War II. 7 9. John’s meditation on the town’s sign in 1234567890123456789012345678901212345678901234567890123456789012123456 2 34567890123456789012345678901212345678901234567890123456789012123456 1234567890123456789012345678901212345678901234567890123456789012123456 (10) Abe spent his childhood and much of his 7 paragraph 6 serves in the passage prima7 1234567890123456789012345678901212345678901234567890123456789012123456 7 1 youth in Manchuria, and, as a result, rily to suggest a contrast between 1234567890123456789012345678901212345678901234567890123456789012123456 7 34567890123456789012345678901212345678901234567890123456789012123456 7 12 the orbit of his work would be far less 34567890123456789012345678901212345678901234567890123456789012123456 7 12 (A) John’s love of Victorian things and 34567890123456789012345678901212345678901234567890123456789012123456 7 12 controlled by the oppressive gravitational 7 1234567890123456789012345678901212345678901234567890123456789012123456 his father’s love of modern things. 34567890123456789012345678901212345678901234567890123456789012123456 7 12 pull of the themes of furusato (home34567890123456789012345678901212345678901234567890123456789012123456 7 12 (B) his father’s commercialism and 2 34567890123456789012345678901212345678901234567890123456789012123456 7 1234567890123456789012345678901212345678901234567890123456789012123456 (15) town) and the emperor than his contem7 1234567890123456789012345678901212345678901234567890123456789012123456 John’s sentimentality. 7 1 poraries’. 7 1234567890123456789012345678901212345678901234567890123456789012123456 (C) John’s previous role as a part of the 7 1234567890123456789012345678901212345678901234567890123456789012123456 Abe, like most of the sons of Japa2 34567890123456789012345678901212345678901234567890123456789012123456 7 1234567890123456789012345678901212345678901234567890123456789012123456 town and his new role as nostalgic 7 1234567890123456789012345678901212345678901234567890123456789012123456 nese families living in Manchuria, did 7 1234567890123456789012345678901212345678901234567890123456789012123456 outsider. 1234567890123456789012345678901212345678901234567890123456789012123456 return to Japan for schooling. He entered 7 7 1 (D) his father’s naivety and John’s 34567890123456789012345678901212345678901234567890123456789012123456 12 (20) medical school in Tokyo in 1944—just in 7 pragmatism. 34567890123456789012345678901212345678901234567890123456789012123456 7 12 34567890123456789012345678901212345678901234567890123456789012123456 12 time to forge himself a medical certificate 7 (E) the old-fashioned atmosphere in the 34567890123456789012345678901212345678901234567890123456789012123456 7 12 7 1234567890123456789012345678901212345678901234567890123456789012123456 claiming ill health; this allowed him to town before John’s father influenced 2 34567890123456789012345678901212345678901234567890123456789012123456 7 1234567890123456789012345678901212345678901234567890123456789012123456 avoid fighting in the war that Japan was 7 1234567890123456789012345678901212345678901234567890123456789012123456 it and its current modernity. 7 1234567890123456789012345678901212345678901234567890123456789012123456 already losing and return to Manchuria. 7 1234567890123456789012345678901212345678901234567890123456789012123456 7 1 (25) When Japan lost the war, however, it also 2 34567890123456789012345678901212345678901234567890123456789012123456 1234567890123456789012345678901212345678901234567890123456789012123456 77 1 lost its Manchurian colony. The Japanese 7 34567890123456789012345678901212345678901234567890123456789012123456 12 7 1234567890123456789012345678901212345678901234567890123456789012123456 living there were attacked by the Soviet 34567890123456789012345678901212345678901234567890123456789012123456 7 12 2 34567890123456789012345678901212345678901234567890123456789012123456 7 1 Army and various guerrilla bands. They 7 1234567890123456789012345678901212345678901234567890123456789012123456 1234567890123456789012345678901212345678901234567890123456789012123456 77 1234567890123456789012345678901212345678901234567890123456789012123456 12345678901234567890123456789012123456789012345678901234567890121234567 6

Copyright © 2005 Thomson Peterson’s, a part of The Thomson Corporaton SAT is a registered trademark of the College Entrance Examination Board, which was not involved in the production of and does not endorse this product.

12345678901234567890123456789012123456789012345678901234567890121234567 1234567890123456789012345678901212345678901234567890123456789012123456 7 7 12234567890123456789012345678901212345678901234567890123456789012123456 suddenly found themselves refugees, 7 accurate to say that the novel simply 1 34567890123456789012345678901212345678901234567890123456789012123456 7 12234567890123456789012345678901212345678901234567890123456789012123456 (30) desperate for food. Many unfit men were marked a turning point in his career, 7 1234567890123456789012345678901212345678901234567890123456789012123456 7 1 34567890123456789012345678901212345678901234567890123456789012123456 abandoned in the Manchurian desert. At when Abe turned away from the experi7 12234567890123456789012345678901212345678901234567890123456789012123456 34567890123456789012345678901212345678901234567890123456789012123456 7 1 this apocalyptic time, Abe lost his father (75) mental and heavily political work of his 7 12234567890123456789012345678901212345678901234567890123456789012123456 7 1234567890123456789012345678901212345678901234567890123456789012123456 to cholera. earlier career. Fortunately, he did not 7 1234567890123456789012345678901212345678901234567890123456789012123456 7 1234567890123456789012345678901212345678901234567890123456789012123456 He returned to mainland Japan once then turn to furusato and the emperor 34567890123456789012345678901212345678901234567890123456789012123456 7 1 7 12234567890123456789012345678901212345678901234567890123456789012123456 (35) more, where the young were turning to after all, but rather began a somewhat 7 1234567890123456789012345678901212345678901234567890123456789012123456 7 1234567890123456789012345678901212345678901234567890123456789012123456 Marxism as a rejection of the militarism more realistic exploration of his continu7 1234567890123456789012345678901212345678901234567890123456789012123456 34567890123456789012345678901212345678901234567890123456789012123456 7 1 (80) of the war. After a brief, unsuccessful ing obsession with homelessness and 7 12234567890123456789012345678901212345678901234567890123456789012123456 7 1234567890123456789012345678901212345678901234567890123456789012123456 stint at medical school, he became part of alienation. Not completely a stranger to 7 1234567890123456789012345678901212345678901234567890123456789012123456 7 1234567890123456789012345678901212345678901234567890123456789012123456 a Marxist group of avant-garde artists. his earlier commitment to Marxism, Abe 34567890123456789012345678901212345678901234567890123456789012123456 7 1 7 12234567890123456789012345678901212345678901234567890123456789012123456 (40) His work at this time was passionate and turned his attention, beginning in the 7 1234567890123456789012345678901212345678901234567890123456789012123456 7 1234567890123456789012345678901212345678901234567890123456789012123456 outspoken on political matters, adopting sixties, to the effects on the individual of 7 1234567890123456789012345678901212345678901234567890123456789012123456 34567890123456789012345678901212345678901234567890123456789012123456 7 1234567890123456789012345678901212345678901234567890123456789012123456 (85) black humor as its mode of critique. Japan’s rapidly urbanizing, growth7 1234567890123456789012345678901212345678901234567890123456789012123456 7 1234567890123456789012345678901212345678901234567890123456789012123456 During this time, Abe worked in the driven, increasingly corporate society. 1234567890123456789012345678901212345678901234567890123456789012123456 77 1234567890123456789012345678901212345678901234567890123456789012123456 genres of theater, music, and photogra7 1234567890123456789012345678901212345678901234567890123456789012123456 7 1 (45) phy. Eventually, he mimeographed fifty 11. The word “infused” in line 6 most closely 7 12234567890123456789012345678901212345678901234567890123456789012123456 copies of his first “published” literary 7 1234567890123456789012345678901212345678901234567890123456789012123456 means 7 1234567890123456789012345678901212345678901234567890123456789012123456 work, entitled Anonymous Poems, in 34567890123456789012345678901212345678901234567890123456789012123456 1234567890123456789012345678901212345678901234567890123456789012123456 77 1234567890123456789012345678901212345678901234567890123456789012123456 (A) illuminated. 1947. It was a politically charged set of 7 1234567890123456789012345678901212345678901234567890123456789012123456 7 1234567890123456789012345678901212345678901234567890123456789012123456 (B) saturated. poems dedicated to the memory of his 1234567890123456789012345678901212345678901234567890123456789012123456 77 1234567890123456789012345678901212345678901234567890123456789012123456 (C) influenced. (50) father and friends who had died in 7 1234567890123456789012345678901212345678901234567890123456789012123456 7 1234567890123456789012345678901212345678901234567890123456789012123456 (D) bewildered. Manchuria. Shortly thereafter, he 1234567890123456789012345678901212345678901234567890123456789012123456 77 1 (E) nuanced. published his first novel, For a Signpost 7 12234567890123456789012345678901212345678901234567890123456789012123456 7 1234567890123456789012345678901212345678901234567890123456789012123456 at the End of a Road, which imagined 7 1 34567890123456789012345678901212345678901234567890123456789012123456 12. The author refers to “the orbit” of Abe’s 7 1234567890123456789012345678901212345678901234567890123456789012123456 another life for his best friend who had 7 12234567890123456789012345678901212345678901234567890123456789012123456 work (lines 12–13) to emphasize that 34567890123456789012345678901212345678901234567890123456789012123456 7 1234567890123456789012345678901212345678901234567890123456789012123456 (55) died in the Manchurian desert. Abe was 7 1234567890123456789012345678901212345678901234567890123456789012123456 7 1 also active in the Communist Party, (A) his work covers a wide range of 7 12234567890123456789012345678901212345678901234567890123456789012123456 7 1234567890123456789012345678901212345678901234567890123456789012123456 organizing literary groups for workingthemes. 34567890123456789012345678901212345678901234567890123456789012123456 1234567890123456789012345678901212345678901234567890123456789012123456 77 1234567890123456789012345678901212345678901234567890123456789012123456 men. (B) the emperor is often compared to a 7 1 7 1234567890123456789012345678901212345678901234567890123456789012123456 Unfortunately, most of this radical sun. 1234567890123456789012345678901212345678901234567890123456789012123456 7 2 34567890123456789012345678901212345678901234567890123456789012123456 7 1234567890123456789012345678901212345678901234567890123456789012123456 (60) early work is unknown outside Japan C. Abe’s travels were the primary 7 1234567890123456789012345678901212345678901234567890123456789012123456 7 1234567890123456789012345678901212345678901234567890123456789012123456 and underappreciated even in Japan. In themes in his work. 1234567890123456789012345678901212345678901234567890123456789012123456 77 1 early 1962, Abe was dismissed from the D. Abe’s work is so different from his 7 12234567890123456789012345678901212345678901234567890123456789012123456 Japanese Liberalist Party. Four months contemporaries’ that it is like 7 1234567890123456789012345678901212345678901234567890123456789012123456 7 1234567890123456789012345678901212345678901234567890123456789012123456 later, he published the work that would another solar system. 7 1234567890123456789012345678901212345678901234567890123456789012123456 7 1 34567890123456789012345678901212345678901234567890123456789012123456 (65) blind us to his earlier oeuvre, Woman in (E) conventional themes can limit an 2 34567890123456789012345678901212345678901234567890123456789012123456 1234567890123456789012345678901212345678901234567890123456789012123456 77 1234567890123456789012345678901212345678901234567890123456789012123456 the Dunes. It was director Teshigahara author’s individuality. 7 1234567890123456789012345678901212345678901234567890123456789012123456 7 1234567890123456789012345678901212345678901234567890123456789012123456 Hiroshi’s film adaptation of Woman in 7 1 2 34567890123456789012345678901212345678901234567890123456789012123456 7 1234567890123456789012345678901212345678901234567890123456789012123456 the Dunes that brought Abe’s work to 7 1 7 12234567890123456789012345678901212345678901234567890123456789012123456 the international stage. The movie’s fame 1 34567890123456789012345678901212345678901234567890123456789012123456 7 7 12234567890123456789012345678901212345678901234567890123456789012123456 (70) has wrongly led readers to view the novel 7 1 34567890123456789012345678901212345678901234567890123456789012123456 7 1234567890123456789012345678901212345678901234567890123456789012123456 as Abe’s masterpiece. It would be more 1234567890123456789012345678901212345678901234567890123456789012123456 77 1234567890123456789012345678901212345678901234567890123456789012123456 12345678901234567890123456789012123456789012345678901234567890121234567 Copyright © 2005 Thomson Peterson’s, a part of The Thomson Corporaton SAT is a registered trademark of the College Entrance Examination Board, which was not involved in the production of and does not endorse this product.

7

12345678901234567890123456789012123456789012345678901234567890121234567 1234567890123456789012345678901212345678901234567890123456789012123456 7 34567890123456789012345678901212345678901234567890123456789012123456 7 12 13. From the sentence beginning “He entered 7 16. Which of the following does the passage 1234567890123456789012345678901212345678901234567890123456789012123456 34567890123456789012345678901212345678901234567890123456789012123456 7 12 medical school. . . “ in lines 19–24, it can present as a fact? 34567890123456789012345678901212345678901234567890123456789012123456 7 12 7 1234567890123456789012345678901212345678901234567890123456789012123456 be inferred that 34567890123456789012345678901212345678901234567890123456789012123456 7 12 (A) Abe was a better playwright than 2 34567890123456789012345678901212345678901234567890123456789012123456 7 1 34567890123456789012345678901212345678901234567890123456789012123456 7 12 (A) Abe entered medical school because novelist. 34567890123456789012345678901212345678901234567890123456789012123456 7 12 34567890123456789012345678901212345678901234567890123456789012123456 7 12 he was sick. (B) Abe’s early work was of greater 34567890123456789012345678901212345678901234567890123456789012123456 7 12 2 34567890123456789012345678901212345678901234567890123456789012123456 7 1 (B) sick people were sent to Manchuria quality than his later work. 34567890123456789012345678901212345678901234567890123456789012123456 7 12 34567890123456789012345678901212345678901234567890123456789012123456 7 12 during World War II. (C) The group of avant-garde artists of 34567890123456789012345678901212345678901234567890123456789012123456 7 12 34567890123456789012345678901212345678901234567890123456789012123456 7 12 (C) Abe wanted to help the ill and which Abe was a part were influ7 1234567890123456789012345678901212345678901234567890123456789012123456 34567890123456789012345678901212345678901234567890123456789012123456 7 12 injured in World War II, rather than enced by Marxism. 34567890123456789012345678901212345678901234567890123456789012123456 7 12 34567890123456789012345678901212345678901234567890123456789012123456 7 12 fight. (D) The themes of furusato and the 34567890123456789012345678901212345678901234567890123456789012123456 7 12 2 34567890123456789012345678901212345678901234567890123456789012123456 7 1 (D) illness would excuse one from emperor have precluded Japanese 34567890123456789012345678901212345678901234567890123456789012123456 7 12 34567890123456789012345678901212345678901234567890123456789012123456 7 12 military duty in World War II Japan. literature from playing a major role 34567890123456789012345678901212345678901234567890123456789012123456 7 12 34567890123456789012345678901212345678901234567890123456789012123456 7 12 (E) Abe never intended to practice in world literature. 34567890123456789012345678901212345678901234567890123456789012123456 7 12 34567890123456789012345678901212345678901234567890123456789012123456 7 12 medicine. (E) Abe’s work is richer than his 34567890123456789012345678901212345678901234567890123456789012123456 7 12 34567890123456789012345678901212345678901234567890123456789012123456 contemporaries’ because he included 7 12 34567890123456789012345678901212345678901234567890123456789012123456 7 12 14. The author uses the word “apocalyptic” autobiographical elements. 2 34567890123456789012345678901212345678901234567890123456789012123456 7 1234567890123456789012345678901212345678901234567890123456789012123456 7 1 to emphasize that 34567890123456789012345678901212345678901234567890123456789012123456 7 12 34567890123456789012345678901212345678901234567890123456789012123456 7 12 17. The phrase “blind us” in lines 65–66 34567890123456789012345678901212345678901234567890123456789012123456 7 12 (A) Manchuria suffered intensely as a 2 34567890123456789012345678901212345678901234567890123456789012123456 7 1234567890123456789012345678901212345678901234567890123456789012123456 refers to the 7 1234567890123456789012345678901212345678901234567890123456789012123456 result of the use of nuclear weapons 1234567890123456789012345678901212345678901234567890123456789012123456 77 1234567890123456789012345678901212345678901234567890123456789012123456 in World War II. (A) absence of film adaptations for Abe’s 7 1234567890123456789012345678901212345678901234567890123456789012123456 7 1234567890123456789012345678901212345678901234567890123456789012123456 (B) Abe was deeply affected by the loss other novels. 7 1234567890123456789012345678901212345678901234567890123456789012123456 7 1234567890123456789012345678901212345678901234567890123456789012123456 of his father. (B) excessive critical attention to Abe’s 1234567890123456789012345678901212345678901234567890123456789012123456 77 1 (C) there was massive famine in Mannovel, Woman in the Dunes. 34567890123456789012345678901212345678901234567890123456789012123456 7 12 34567890123456789012345678901212345678901234567890123456789012123456 7 churia at the end of World War II. 12 (C) difficulty in reconciling Woman in 7 1234567890123456789012345678901212345678901234567890123456789012123456 (D) postwar Manchuria experienced the Dunes and other later works 7 1234567890123456789012345678901212345678901234567890123456789012123456 34567890123456789012345678901212345678901234567890123456789012123456 7 12 exhilarating change. with the form and content of his 2 34567890123456789012345678901212345678901234567890123456789012123456 1234567890123456789012345678901212345678901234567890123456789012123456 77 1234567890123456789012345678901212345678901234567890123456789012123456 (E) conditions in Manchuria after World earlier works. 7 1 34567890123456789012345678901212345678901234567890123456789012123456 7 12 War II were generally horrific. (D) challenge of interpreting Abe’s more 34567890123456789012345678901212345678901234567890123456789012123456 7 12 2 34567890123456789012345678901212345678901234567890123456789012123456 7 1234567890123456789012345678901212345678901234567890123456789012123456 experimental works. 7 1234567890123456789012345678901212345678901234567890123456789012123456 7 1 15. The word “avant-garde” (line 39) could (E) overwhelming power of Abe’s novel, 1234567890123456789012345678901212345678901234567890123456789012123456 7 7 1234567890123456789012345678901212345678901234567890123456789012123456 best be replaced by Woman in the Dunes. 34567890123456789012345678901212345678901234567890123456789012123456 7 12 34567890123456789012345678901212345678901234567890123456789012123456 7 12 34567890123456789012345678901212345678901234567890123456789012123456 7 12 (A) experimental. 34567890123456789012345678901212345678901234567890123456789012123456 7 12 7 1234567890123456789012345678901212345678901234567890123456789012123456 (B) dramatic. 2 34567890123456789012345678901212345678901234567890123456789012123456 7 1234567890123456789012345678901212345678901234567890123456789012123456 (C) novel. 7 1234567890123456789012345678901212345678901234567890123456789012123456 7 1234567890123456789012345678901212345678901234567890123456789012123456 (D) profound. 7 1234567890123456789012345678901212345678901234567890123456789012123456 7 1 (E) realistic. 2 34567890123456789012345678901212345678901234567890123456789012123456 1234567890123456789012345678901212345678901234567890123456789012123456 77 1234567890123456789012345678901212345678901234567890123456789012123456 7 1234567890123456789012345678901212345678901234567890123456789012123456 7 1234567890123456789012345678901212345678901234567890123456789012123456 7 1 34567890123456789012345678901212345678901234567890123456789012123456 7 12 1234567890123456789012345678901212345678901234567890123456789012123456 7 34567890123456789012345678901212345678901234567890123456789012123456 7 12 1234567890123456789012345678901212345678901234567890123456789012123456 7 34567890123456789012345678901212345678901234567890123456789012123456 7 12 1234567890123456789012345678901212345678901234567890123456789012123456 7 1234567890123456789012345678901212345678901234567890123456789012123456 7 1234567890123456789012345678901212345678901234567890123456789012123456 7 1234567890123456789012345678901212345678901234567890123456789012123456 7 12345678901234567890123456789012123456789012345678901234567890121234567 8

Copyright © 2005 Thomson Peterson’s, a part of The Thomson Corporaton SAT is a registered trademark of the College Entrance Examination Board, which was not involved in the production of and does not endorse this product.

12345678901234567890123456789012123456789012345678901234567890121234567 1234567890123456789012345678901212345678901234567890123456789012123456 7 7 12234567890123456789012345678901212345678901234567890123456789012123456 18. The author’s main purpose in the passage 20. The author’s attitude toward Marxism 7 1 34567890123456789012345678901212345678901234567890123456789012123456 7 12234567890123456789012345678901212345678901234567890123456789012123456 is to can best be described as 7 1234567890123456789012345678901212345678901234567890123456789012123456 1 34567890123456789012345678901212345678901234567890123456789012123456 7 7 12234567890123456789012345678901212345678901234567890123456789012123456 (A) defend Abe’s later works against the (A) contemptuous derision. 7 1 34567890123456789012345678901212345678901234567890123456789012123456 7 12234567890123456789012345678901212345678901234567890123456789012123456 prevalent criticism of it. (B) reverent espousal. 7 1234567890123456789012345678901212345678901234567890123456789012123456 7 1234567890123456789012345678901212345678901234567890123456789012123456 (B) advocate for Abe’s work over that of (C) skeptical tolerance. 7 1234567890123456789012345678901212345678901234567890123456789012123456 34567890123456789012345678901212345678901234567890123456789012123456 7 1 his contemporaries. (D) respectful interest. 7 12234567890123456789012345678901212345678901234567890123456789012123456 7 1234567890123456789012345678901212345678901234567890123456789012123456 (C) explain the differences between Abe’s (E) restrained impatience. 7 1234567890123456789012345678901212345678901234567890123456789012123456 7 1234567890123456789012345678901212345678901234567890123456789012123456 earlier and later works. 34567890123456789012345678901212345678901234567890123456789012123456 7 1 7 12234567890123456789012345678901212345678901234567890123456789012123456 (D) argue that Abe is an even greater 7 1234567890123456789012345678901212345678901234567890123456789012123456 7 1234567890123456789012345678901212345678901234567890123456789012123456 writer and artist than generally 7 1234567890123456789012345678901212345678901234567890123456789012123456 34567890123456789012345678901212345678901234567890123456789012123456 7 1 perceived. 7 12234567890123456789012345678901212345678901234567890123456789012123456 7 1234567890123456789012345678901212345678901234567890123456789012123456 (E) demonstrate that Abe’s work became 7 1234567890123456789012345678901212345678901234567890123456789012123456 7 1234567890123456789012345678901212345678901234567890123456789012123456 less interesting after he left Manchu34567890123456789012345678901212345678901234567890123456789012123456 1234567890123456789012345678901212345678901234567890123456789012123456 77 1234567890123456789012345678901212345678901234567890123456789012123456 ria. 1234567890123456789012345678901212345678901234567890123456789012123456 77 1234567890123456789012345678901212345678901234567890123456789012123456 7 1234567890123456789012345678901212345678901234567890123456789012123456 19. The author of the passage is most likely a 7 1234567890123456789012345678901212345678901234567890123456789012123456 7 1 7 12234567890123456789012345678901212345678901234567890123456789012123456 (A) film critic. 7 1234567890123456789012345678901212345678901234567890123456789012123456 7 1234567890123456789012345678901212345678901234567890123456789012123456 (B) literary critic. 34567890123456789012345678901212345678901234567890123456789012123456 1234567890123456789012345678901212345678901234567890123456789012123456 77 1234567890123456789012345678901212345678901234567890123456789012123456 (C) avant-garde artist. 7 1234567890123456789012345678901212345678901234567890123456789012123456 7 1234567890123456789012345678901212345678901234567890123456789012123456 (D) translator. 1234567890123456789012345678901212345678901234567890123456789012123456 77 1234567890123456789012345678901212345678901234567890123456789012123456 (E) novelist. 7 1234567890123456789012345678901212345678901234567890123456789012123456 1234567890123456789012345678901212345678901234567890123456789012123456 77 1234567890123456789012345678901212345678901234567890123456789012123456 7 1 7 12234567890123456789012345678901212345678901234567890123456789012123456 7 1234567890123456789012345678901212345678901234567890123456789012123456 1 34567890123456789012345678901212345678901234567890123456789012123456 7 1234567890123456789012345678901212345678901234567890123456789012123456 7 7 12234567890123456789012345678901212345678901234567890123456789012123456 7 1234567890123456789012345678901212345678901234567890123456789012123456 7 1234567890123456789012345678901212345678901234567890123456789012123456 1 34567890123456789012345678901212345678901234567890123456789012123456 7 7 12234567890123456789012345678901212345678901234567890123456789012123456 7 1234567890123456789012345678901212345678901234567890123456789012123456 34567890123456789012345678901212345678901234567890123456789012123456 1234567890123456789012345678901212345678901234567890123456789012123456 77 1234567890123456789012345678901212345678901234567890123456789012123456 7 1 1234567890123456789012345678901212345678901234567890123456789012123456 7 1234567890123456789012345678901212345678901234567890123456789012123456 7 7 12234567890123456789012345678901212345678901234567890123456789012123456 7 1234567890123456789012345678901212345678901234567890123456789012123456 7 1234567890123456789012345678901212345678901234567890123456789012123456 7 1234567890123456789012345678901212345678901234567890123456789012123456 1 34567890123456789012345678901212345678901234567890123456789012123456 7 2 34567890123456789012345678901212345678901234567890123456789012123456 1234567890123456789012345678901212345678901234567890123456789012123456 77 1234567890123456789012345678901212345678901234567890123456789012123456 7 1234567890123456789012345678901212345678901234567890123456789012123456 7 1234567890123456789012345678901212345678901234567890123456789012123456 7 1 7 12234567890123456789012345678901212345678901234567890123456789012123456 7 1234567890123456789012345678901212345678901234567890123456789012123456 7 1234567890123456789012345678901212345678901234567890123456789012123456 7 1234567890123456789012345678901212345678901234567890123456789012123456 1 34567890123456789012345678901212345678901234567890123456789012123456 7 2 34567890123456789012345678901212345678901234567890123456789012123456 1234567890123456789012345678901212345678901234567890123456789012123456 77 1 7 12234567890123456789012345678901212345678901234567890123456789012123456 1 34567890123456789012345678901212345678901234567890123456789012123456 7 7 12234567890123456789012345678901212345678901234567890123456789012123456 Do not proceed to the next section until time is up. 7 1 34567890123456789012345678901212345678901234567890123456789012123456 1234567890123456789012345678901212345678901234567890123456789012123456 7 1234567890123456789012345678901212345678901234567890123456789012123456 7 1234567890123456789012345678901212345678901234567890123456789012123456 7 12345678901234567890123456789012123456789012345678901234567890121234567

STOP

Copyright © 2005 Thomson Peterson’s, a part of The Thomson Corporaton SAT is a registered trademark of the College Entrance Examination Board, which was not involved in the production of and does not endorse this product.

9

Section 2 21 Questions j Time—25 Minutes

Reference Information

12345678901234567890123456789012123456789012345678901234567890121234567 34567890123456789012345678901212345678901234567890123456789012123456 7 12 34567890123456789012345678901212345678901234567890123456789012123456 7 12 2 1 34567890123456789012345678901212345678901234567890123456789012123456 7 34567890123456789012345678901212345678901234567890123456789012123456 7 12 Directions: Solve the following problems using any available space on the page for scratchwork. 34567890123456789012345678901212345678901234567890123456789012123456 7 12 34567890123456789012345678901212345678901234567890123456789012123456 7 12 Mark the letter of your choice on the answer sheet that best corresponds to the correct answer. 34567890123456789012345678901212345678901234567890123456789012123456 7 12 2 34567890123456789012345678901212345678901234567890123456789012123456 1234567890123456789012345678901212345678901234567890123456789012123456 77 Notes: 1234567890123456789012345678901212345678901234567890123456789012123456 7 1234567890123456789012345678901212345678901234567890123456789012123456 1. You may use a calculator. All of the numbers used are real numbers. 7 1234567890123456789012345678901212345678901234567890123456789012123456 1234567890123456789012345678901212345678901234567890123456789012123456 77 1234567890123456789012345678901212345678901234567890123456789012123456 2. You may use the figures that accompany the problems to help you find the solution. Unless 7 1 34567890123456789012345678901212345678901234567890123456789012123456 7 12 the instructions say that a figure is not drawn to scale, assume that it has been drawn 34567890123456789012345678901212345678901234567890123456789012123456 7 12 34567890123456789012345678901212345678901234567890123456789012123456 7 12 accurately. Each figure lies in a plane unless the instructions say otherwise. 34567890123456789012345678901212345678901234567890123456789012123456 7 12 34567890123456789012345678901212345678901234567890123456789012123456 7 12 34567890123456789012345678901212345678901234567890123456789012123456 7 12 34567890123456789012345678901212345678901234567890123456789012123456 7 12 34567890123456789012345678901212345678901234567890123456789012123456 7 12 2 7 1234567890123456789012345678901212345678901234567890123456789012123456 2s r 2x 7 1234567890123456789012345678901212345678901234567890123456789012123456 w r s x 7 1234567890123456789012345678901212345678901234567890123456789012123456 c h h h b 7 1234567890123456789012345678901212345678901234567890123456789012123456 7 1 34567890123456789012345678901212345678901234567890123456789012123456 w 2 34567890123456789012345678901212345678901234567890123456789012123456 7 1234567890123456789012345678901212345678901234567890123456789012123456 s b 3x a A 5 pr2 7 1234567890123456789012345678901212345678901234567890123456789012123456 1 2 2 2 2 7 1 h c 5 a 1 b Special Right Triangles V 5 ,wh V 5 pr C 5 2pr A 5 ,w bh A 5 7 1234567890123456789012345678901212345678901234567890123456789012123456 2 34567890123456789012345678901212345678901234567890123456789012123456 7 12 2 34567890123456789012345678901212345678901234567890123456789012123456 7 1234567890123456789012345678901212345678901234567890123456789012123456 The number of degrees of arc in a circle is 360. 7 1234567890123456789012345678901212345678901234567890123456789012123456 The measure in degrees of a straight angle is 180. 7 1 34567890123456789012345678901212345678901234567890123456789012123456 7 12 The sum of the measures in degrees of the angles of a triangle is 180. 34567890123456789012345678901212345678901234567890123456789012123456 7 12 2 34567890123456789012345678901212345678901234567890123456789012123456 1234567890123456789012345678901212345678901234567890123456789012123456 77 1234567890123456789012345678901212345678901234567890123456789012123456 7 1 7 1234567890123456789012345678901212345678901234567890123456789012123456 1. What percentage of 75 is 12? 2. If a circle is inscribed in a square of area 7 1234567890123456789012345678901212345678901234567890123456789012123456 2 34567890123456789012345678901212345678901234567890123456789012123456 7 1234567890123456789012345678901212345678901234567890123456789012123456 36, what is the area of the circle? 7 1234567890123456789012345678901212345678901234567890123456789012123456 (A) 8% 1234567890123456789012345678901212345678901234567890123456789012123456 77 1234567890123456789012345678901212345678901234567890123456789012123456 (B) 12% (A) 36p 7 1 2 34567890123456789012345678901212345678901234567890123456789012123456 7 1234567890123456789012345678901212345678901234567890123456789012123456 (C) 16% (B) 24p 7 1234567890123456789012345678901212345678901234567890123456789012123456 7 1234567890123456789012345678901212345678901234567890123456789012123456 (D) 18% (C) 12p 1234567890123456789012345678901212345678901234567890123456789012123456 77 1 (E) 20% (D) 9p 34567890123456789012345678901212345678901234567890123456789012123456 7 12 34567890123456789012345678901212345678901234567890123456789012123456 7 12 (E) 6p 34567890123456789012345678901212345678901234567890123456789012123456 7 12 34567890123456789012345678901212345678901234567890123456789012123456 7 12 1234567890123456789012345678901212345678901234567890123456789012123456 7 2 7 1234567890123456789012345678901212345678901234567890123456789012123456 1 34567890123456789012345678901212345678901234567890123456789012123456 7 34567890123456789012345678901212345678901234567890123456789012123456 7 12 1234567890123456789012345678901212345678901234567890123456789012123456 7 34567890123456789012345678901212345678901234567890123456789012123456 7 12 1234567890123456789012345678901212345678901234567890123456789012123456 7 1234567890123456789012345678901212345678901234567890123456789012123456 7 1234567890123456789012345678901212345678901234567890123456789012123456 7 1234567890123456789012345678901212345678901234567890123456789012123456 7 12345678901234567890123456789012123456789012345678901234567890121234567 10

45

60

30

45

Copyright © 2005 Thomson Peterson’s, a part of The Thomson Corporaton SAT is a registered trademark of the College Entrance Examination Board, which was not involved in the production of and does not endorse this product.

12345678901234567890123456789012123456789012345678901234567890121234567 1234567890123456789012345678901212345678901234567890123456789012123456 7 7 12234567890123456789012345678901212345678901234567890123456789012123456 3. Which of the following could have a slope 4. The first four terms of a series are 1, 4, 9, 7 1 34567890123456789012345678901212345678901234567890123456789012123456 7 12234567890123456789012345678901212345678901234567890123456789012123456 of one? and 16. What is the eight term of this 7 1234567890123456789012345678901212345678901234567890123456789012123456 7 1 34567890123456789012345678901212345678901234567890123456789012123456 series? 7 12234567890123456789012345678901212345678901234567890123456789012123456 (A) 34567890123456789012345678901212345678901234567890123456789012123456 7 1 7 12234567890123456789012345678901212345678901234567890123456789012123456 (A) 49 7 1234567890123456789012345678901212345678901234567890123456789012123456 7 1234567890123456789012345678901212345678901234567890123456789012123456 (B) 56 7 1234567890123456789012345678901212345678901234567890123456789012123456 34567890123456789012345678901212345678901234567890123456789012123456 7 1 (C) 64 7 12234567890123456789012345678901212345678901234567890123456789012123456 7 1234567890123456789012345678901212345678901234567890123456789012123456 (D) 72 7 1234567890123456789012345678901212345678901234567890123456789012123456 7 1234567890123456789012345678901212345678901234567890123456789012123456 (E) 81 34567890123456789012345678901212345678901234567890123456789012123456 7 1 7 12234567890123456789012345678901212345678901234567890123456789012123456 7 1234567890123456789012345678901212345678901234567890123456789012123456 7 5. 1234567890123456789012345678901212345678901234567890123456789012123456 7 1234567890123456789012345678901212345678901234567890123456789012123456 34567890123456789012345678901212345678901234567890123456789012123456 7 1 (B) 7 12234567890123456789012345678901212345678901234567890123456789012123456 7 1234567890123456789012345678901212345678901234567890123456789012123456 7 1234567890123456789012345678901212345678901234567890123456789012123456 7 1234567890123456789012345678901212345678901234567890123456789012123456 34567890123456789012345678901212345678901234567890123456789012123456 1234567890123456789012345678901212345678901234567890123456789012123456 77 1234567890123456789012345678901212345678901234567890123456789012123456 7 1234567890123456789012345678901212345678901234567890123456789012123456 7 1234567890123456789012345678901212345678901234567890123456789012123456 What is the area of the above parallelo7 1234567890123456789012345678901212345678901234567890123456789012123456 7 1234567890123456789012345678901212345678901234567890123456789012123456 gram? 7 1 7 12234567890123456789012345678901212345678901234567890123456789012123456 7 1234567890123456789012345678901212345678901234567890123456789012123456 (A) 16 7 1234567890123456789012345678901212345678901234567890123456789012123456 34567890123456789012345678901212345678901234567890123456789012123456 7 1234567890123456789012345678901212345678901234567890123456789012123456 (B) 18 7 1234567890123456789012345678901212345678901234567890123456789012123456 (C) 7 1234567890123456789012345678901212345678901234567890123456789012123456 (C) 22 1234567890123456789012345678901212345678901234567890123456789012123456 77 1234567890123456789012345678901212345678901234567890123456789012123456 (D) 24 7 1234567890123456789012345678901212345678901234567890123456789012123456 7 1234567890123456789012345678901212345678901234567890123456789012123456 (E) 32 1234567890123456789012345678901212345678901234567890123456789012123456 77 1234567890123456789012345678901212345678901234567890123456789012123456 7 1 6. Which of the following fractions has the 7 12234567890123456789012345678901212345678901234567890123456789012123456 7 1234567890123456789012345678901212345678901234567890123456789012123456 greatest reciprocal? 7 1 34567890123456789012345678901212345678901234567890123456789012123456 1234567890123456789012345678901212345678901234567890123456789012123456 7 7 12234567890123456789012345678901212345678901234567890123456789012123456 2 7 1234567890123456789012345678901212345678901234567890123456789012123456 (A) 7 1234567890123456789012345678901212345678901234567890123456789012123456 9 (D) 7 1 34567890123456789012345678901212345678901234567890123456789012123456 7 12234567890123456789012345678901212345678901234567890123456789012123456 4 7 1234567890123456789012345678901212345678901234567890123456789012123456 (B) 34567890123456789012345678901212345678901234567890123456789012123456 7 1234567890123456789012345678901212345678901234567890123456789012123456 5 7 1234567890123456789012345678901212345678901234567890123456789012123456 7 1 7 1234567890123456789012345678901212345678901234567890123456789012123456 7 7 1234567890123456789012345678901212345678901234567890123456789012123456 (C) 2 34567890123456789012345678901212345678901234567890123456789012123456 7 1234567890123456789012345678901212345678901234567890123456789012123456 3 7 1234567890123456789012345678901212345678901234567890123456789012123456 7 1234567890123456789012345678901212345678901234567890123456789012123456 2 7 1234567890123456789012345678901212345678901234567890123456789012123456 (D) 7 1 3 2 34567890123456789012345678901212345678901234567890123456789012123456 1234567890123456789012345678901212345678901234567890123456789012123456 77 1234567890123456789012345678901212345678901234567890123456789012123456 7 1234567890123456789012345678901212345678901234567890123456789012123456 3 (E) 7 1234567890123456789012345678901212345678901234567890123456789012123456 (E) 7 1 13 2 34567890123456789012345678901212345678901234567890123456789012123456 1234567890123456789012345678901212345678901234567890123456789012123456 77 1234567890123456789012345678901212345678901234567890123456789012123456 7 1234567890123456789012345678901212345678901234567890123456789012123456 7 1234567890123456789012345678901212345678901234567890123456789012123456 7 1 7 12234567890123456789012345678901212345678901234567890123456789012123456 1 34567890123456789012345678901212345678901234567890123456789012123456 7 7 12234567890123456789012345678901212345678901234567890123456789012123456 1 34567890123456789012345678901212345678901234567890123456789012123456 7 7 12234567890123456789012345678901212345678901234567890123456789012123456 1 34567890123456789012345678901212345678901234567890123456789012123456 7 1234567890123456789012345678901212345678901234567890123456789012123456 7 1234567890123456789012345678901212345678901234567890123456789012123456 7 1234567890123456789012345678901212345678901234567890123456789012123456 7 12345678901234567890123456789012123456789012345678901234567890121234567 Copyright © 2005 Thomson Peterson’s, a part of The Thomson Corporaton SAT is a registered trademark of the College Entrance Examination Board, which was not involved in the production of and does not endorse this product.

11

12345678901234567890123456789012123456789012345678901234567890121234567 1234567890123456789012345678901212345678901234567890123456789012123456 7 34567890123456789012345678901212345678901234567890123456789012123456 7 12 7. 9. 7 1234567890123456789012345678901212345678901234567890123456789012123456 34567890123456789012345678901212345678901234567890123456789012123456 7 12 34567890123456789012345678901212345678901234567890123456789012123456 7 12 1234567890123456789012345678901212345678901234567890123456789012123456 7 34567890123456789012345678901212345678901234567890123456789012123456 7 12 1234567890123456789012345678901212345678901234567890123456789012123456 7 34567890123456789012345678901212345678901234567890123456789012123456 7 12 34567890123456789012345678901212345678901234567890123456789012123456 7 12 34567890123456789012345678901212345678901234567890123456789012123456 7 12 34567890123456789012345678901212345678901234567890123456789012123456 7 12 2 1 34567890123456789012345678901212345678901234567890123456789012123456 7 34567890123456789012345678901212345678901234567890123456789012123456 12 According to the chart above, Company B 7 34567890123456789012345678901212345678901234567890123456789012123456 7 12 34567890123456789012345678901212345678901234567890123456789012123456 7 12 produced approximately how many more 34567890123456789012345678901212345678901234567890123456789012123456 7 12 2 34567890123456789012345678901212345678901234567890123456789012123456 7 1 widgets than Company A? 34567890123456789012345678901212345678901234567890123456789012123456 7 12 34567890123456789012345678901212345678901234567890123456789012123456 7 12 34567890123456789012345678901212345678901234567890123456789012123456 7 (A) 75,000 12 34567890123456789012345678901212345678901234567890123456789012123456 7 12 (B) 150,000 2 34567890123456789012345678901212345678901234567890123456789012123456 7 1 34567890123456789012345678901212345678901234567890123456789012123456 7 12 (C) 225,000 34567890123456789012345678901212345678901234567890123456789012123456 7 12 34567890123456789012345678901212345678901234567890123456789012123456 7 12 If a 5 60 and c 5 50 then x = (D) 300,000 34567890123456789012345678901212345678901234567890123456789012123456 7 12 2 34567890123456789012345678901212345678901234567890123456789012123456 7 1234567890123456789012345678901212345678901234567890123456789012123456 (E) 375,000 (A) 40 7 1234567890123456789012345678901212345678901234567890123456789012123456 7 1234567890123456789012345678901212345678901234567890123456789012123456 (B) 65 7 1234567890123456789012345678901212345678901234567890123456789012123456 x 7 1234567890123456789012345678901212345678901234567890123456789012123456 is odd, and x is an integer, then the 10. If x (C) 75 1234567890123456789012345678901212345678901234567890123456789012123456 77 1 value of x must be (D) 85 34567890123456789012345678901212345678901234567890123456789012123456 7 12 34567890123456789012345678901212345678901234567890123456789012123456 7 12 (E) 110 34567890123456789012345678901212345678901234567890123456789012123456 7 12 (A) odd. 2 34567890123456789012345678901212345678901234567890123456789012123456 1234567890123456789012345678901212345678901234567890123456789012123456 77 1234567890123456789012345678901212345678901234567890123456789012123456 (B) even. 7 1234567890123456789012345678901212345678901234567890123456789012123456 8. @~22 1 22!21#22 5 (C) less than one. 7 1234567890123456789012345678901212345678901234567890123456789012123456 7 1234567890123456789012345678901212345678901234567890123456789012123456 1 (D) an irrational number. 7 1234567890123456789012345678901212345678901234567890123456789012123456 (A) 7 1234567890123456789012345678901212345678901234567890123456789012123456 64 (E) None of the above. 7 1234567890123456789012345678901212345678901234567890123456789012123456 1234567890123456789012345678901212345678901234567890123456789012123456 77 1 1 34567890123456789012345678901212345678901234567890123456789012123456 7 12 (B) 34567890123456789012345678901212345678901234567890123456789012123456 7 12 16 1234567890123456789012345678901212345678901234567890123456789012123456 7 7 1234567890123456789012345678901212345678901234567890123456789012123456 1 34567890123456789012345678901212345678901234567890123456789012123456 7 12 (C) 2 34567890123456789012345678901212345678901234567890123456789012123456 7 1234567890123456789012345678901212345678901234567890123456789012123456 8 7 1234567890123456789012345678901212345678901234567890123456789012123456 7 1 (D) 16 34567890123456789012345678901212345678901234567890123456789012123456 7 12 34567890123456789012345678901212345678901234567890123456789012123456 7 12 (E) 64 34567890123456789012345678901212345678901234567890123456789012123456 7 12 34567890123456789012345678901212345678901234567890123456789012123456 7 12 1234567890123456789012345678901212345678901234567890123456789012123456 7 1234567890123456789012345678901212345678901234567890123456789012123456 7 1234567890123456789012345678901212345678901234567890123456789012123456 7 34567890123456789012345678901212345678901234567890123456789012123456 7 12 34567890123456789012345678901212345678901234567890123456789012123456 7 12 34567890123456789012345678901212345678901234567890123456789012123456 7 12 34567890123456789012345678901212345678901234567890123456789012123456 7 12 1234567890123456789012345678901212345678901234567890123456789012123456 7 2 7 1234567890123456789012345678901212345678901234567890123456789012123456 7 1234567890123456789012345678901212345678901234567890123456789012123456 7 1234567890123456789012345678901212345678901234567890123456789012123456 7 1234567890123456789012345678901212345678901234567890123456789012123456 1 34567890123456789012345678901212345678901234567890123456789012123456 7 2 34567890123456789012345678901212345678901234567890123456789012123456 1234567890123456789012345678901212345678901234567890123456789012123456 77 1234567890123456789012345678901212345678901234567890123456789012123456 7 1234567890123456789012345678901212345678901234567890123456789012123456 7 1234567890123456789012345678901212345678901234567890123456789012123456 7 1 34567890123456789012345678901212345678901234567890123456789012123456 7 12 1234567890123456789012345678901212345678901234567890123456789012123456 7 34567890123456789012345678901212345678901234567890123456789012123456 7 12 1234567890123456789012345678901212345678901234567890123456789012123456 7 34567890123456789012345678901212345678901234567890123456789012123456 7 12 1234567890123456789012345678901212345678901234567890123456789012123456 7 1234567890123456789012345678901212345678901234567890123456789012123456 7 1234567890123456789012345678901212345678901234567890123456789012123456 7 1234567890123456789012345678901212345678901234567890123456789012123456 7 12345678901234567890123456789012123456789012345678901234567890121234567 12

Copyright © 2005 Thomson Peterson’s, a part of The Thomson Corporaton SAT is a registered trademark of the College Entrance Examination Board, which was not involved in the production of and does not endorse this product.

12345678901234567890123456789012123456789012345678901234567890121234567 1234567890123456789012345678901212345678901234567890123456789012123456 7 7 12234567890123456789012345678901212345678901234567890123456789012123456 The following description applies to questions 14. If the determinant of this matrix is 26, 7 1 34567890123456789012345678901212345678901234567890123456789012123456 7 12234567890123456789012345678901212345678901234567890123456789012123456 11–13. what is the value of n? 7 1234567890123456789012345678901212345678901234567890123456789012123456 1 34567890123456789012345678901212345678901234567890123456789012123456 7 7 12234567890123456789012345678901212345678901234567890123456789012123456 The factorial of a number is the product of all n 4 7 1 34567890123456789012345678901212345678901234567890123456789012123456 7 12234567890123456789012345678901212345678901234567890123456789012123456 the integers from one to the number. For 5 27 7 1234567890123456789012345678901212345678901234567890123456789012123456 7 1234567890123456789012345678901212345678901234567890123456789012123456 example, 5 factorial is 5 3 4 3 3 3 2 3 1. The 7 1234567890123456789012345678901212345678901234567890123456789012123456 (A) 22 notation for a factorial is the number followed 34567890123456789012345678901212345678901234567890123456789012123456 7 1 7 12234567890123456789012345678901212345678901234567890123456789012123456 by an exclamation point. 7 1234567890123456789012345678901212345678901234567890123456789012123456 3 7 1234567890123456789012345678901212345678901234567890123456789012123456 (B) 25 7 1234567890123456789012345678901212345678901234567890123456789012123456 5 Thus 5! 5 5 3 4 3 3 3 2 3 1. 34567890123456789012345678901212345678901234567890123456789012123456 7 1 7 12234567890123456789012345678901212345678901234567890123456789012123456 (C) 27 7 1234567890123456789012345678901212345678901234567890123456789012123456 7 1234567890123456789012345678901212345678901234567890123456789012123456 6! 7 1234567890123456789012345678901212345678901234567890123456789012123456 (D) 28 5 11. 34567890123456789012345678901212345678901234567890123456789012123456 7 1 3! 7 12234567890123456789012345678901212345678901234567890123456789012123456 3 7 1234567890123456789012345678901212345678901234567890123456789012123456 (A) 2 7 1234567890123456789012345678901212345678901234567890123456789012123456 (E) 28 7 1234567890123456789012345678901212345678901234567890123456789012123456 4 (B) 16 34567890123456789012345678901212345678901234567890123456789012123456 1234567890123456789012345678901212345678901234567890123456789012123456 77 1234567890123456789012345678901212345678901234567890123456789012123456 (C) 30 1234567890123456789012345678901212345678901234567890123456789012123456 15. If p 1 q 5 2q 1 6, which of the following 7 7 1234567890123456789012345678901212345678901234567890123456789012123456 (D) 88 7 1234567890123456789012345678901212345678901234567890123456789012123456 statements must be true? 7 1234567890123456789012345678901212345678901234567890123456789012123456 (E) 120 7 1 I. p is even 7 12234567890123456789012345678901212345678901234567890123456789012123456 7 1234567890123456789012345678901212345678901234567890123456789012123456 2 II. q is even 7 1234567890123456789012345678901212345678901234567890123456789012123456 12. If f(x) 5 (x!) then f(3) 5 34567890123456789012345678901212345678901234567890123456789012123456 7 1234567890123456789012345678901212345678901234567890123456789012123456 III. pq is even 7 1234567890123456789012345678901212345678901234567890123456789012123456 (A) 16 7 1234567890123456789012345678901212345678901234567890123456789012123456 7 1234567890123456789012345678901212345678901234567890123456789012123456 (A) I only (B) 36 1234567890123456789012345678901212345678901234567890123456789012123456 77 1234567890123456789012345678901212345678901234567890123456789012123456 (B) II only (C) 172 7 1234567890123456789012345678901212345678901234567890123456789012123456 7 1234567890123456789012345678901212345678901234567890123456789012123456 (C) II and III only (D) 1080 1234567890123456789012345678901212345678901234567890123456789012123456 77 1 (D) I, II, and III (E) 6282 7 12234567890123456789012345678901212345678901234567890123456789012123456 7 1234567890123456789012345678901212345678901234567890123456789012123456 (E) None 1 34567890123456789012345678901212345678901234567890123456789012123456 7 7 1234567890123456789012345678901212345678901234567890123456789012123456 13. If y 1 2 5 x , and y and x are integers, 7 12234567890123456789012345678901212345678901234567890123456789012123456 16. 34567890123456789012345678901212345678901234567890123456789012123456 7 1234567890123456789012345678901212345678901234567890123456789012123456 y! 7 1234567890123456789012345678901212345678901234567890123456789012123456 then 5 7 1 x! 7 12234567890123456789012345678901212345678901234567890123456789012123456 7 1234567890123456789012345678901212345678901234567890123456789012123456 1 34567890123456789012345678901212345678901234567890123456789012123456 7 1234567890123456789012345678901212345678901234567890123456789012123456 (A) 7 1234567890123456789012345678901212345678901234567890123456789012123456 y 1 2 y 1 1 ~ !~ ! 7 1 7 1234567890123456789012345678901212345678901234567890123456789012123456 (B) y 7 1234567890123456789012345678901212345678901234567890123456789012123456 2 34567890123456789012345678901212345678901234567890123456789012123456 1234567890123456789012345678901212345678901234567890123456789012123456 77 1234567890123456789012345678901212345678901234567890123456789012123456 1 1234567890123456789012345678901212345678901234567890123456789012123456 For the above graph, for which values of x 7 (C) 2 7 1234567890123456789012345678901212345678901234567890123456789012123456 y is y . 0? 7 1 2 34567890123456789012345678901212345678901234567890123456789012123456 1234567890123456789012345678901212345678901234567890123456789012123456 77 1234567890123456789012345678901212345678901234567890123456789012123456 y (A) 23 , 21 and 1 , x , 3 7 1234567890123456789012345678901212345678901234567890123456789012123456 (D) 7 1234567890123456789012345678901212345678901234567890123456789012123456 x (B) x , 23 and 21 , x , 1 and x . 3 7 1 2 34567890123456789012345678901212345678901234567890123456789012123456 7 1234567890123456789012345678901212345678901234567890123456789012123456 (E) y(y 2 1) (C) x , 23 and 1 , x , 3 and x . 3 7 1234567890123456789012345678901212345678901234567890123456789012123456 7 1234567890123456789012345678901212345678901234567890123456789012123456 (D) x . 3 and x . 21 1234567890123456789012345678901212345678901234567890123456789012123456 77 1 (E) 23 . x . 3 7 12234567890123456789012345678901212345678901234567890123456789012123456 1 34567890123456789012345678901212345678901234567890123456789012123456 7 7 12234567890123456789012345678901212345678901234567890123456789012123456 1 34567890123456789012345678901212345678901234567890123456789012123456 7 7 12234567890123456789012345678901212345678901234567890123456789012123456 1 34567890123456789012345678901212345678901234567890123456789012123456 7 1234567890123456789012345678901212345678901234567890123456789012123456 7 1234567890123456789012345678901212345678901234567890123456789012123456 7 1234567890123456789012345678901212345678901234567890123456789012123456 7 12345678901234567890123456789012123456789012345678901234567890121234567

S D

Copyright © 2005 Thomson Peterson’s, a part of The Thomson Corporaton SAT is a registered trademark of the College Entrance Examination Board, which was not involved in the production of and does not endorse this product.

13

12345678901234567890123456789012123456789012345678901234567890121234567 1234567890123456789012345678901212345678901234567890123456789012123456 7 34567890123456789012345678901212345678901234567890123456789012123456 7 12 17. 19. Raising n2p 7 1234567890123456789012345678901212345678901234567890123456789012123456 n by which of the following will 34567890123456789012345678901212345678901234567890123456789012123456 7 12 34567890123456789012345678901212345678901234567890123456789012123456 7 12 give the result n? 1234567890123456789012345678901212345678901234567890123456789012123456 7 34567890123456789012345678901212345678901234567890123456789012123456 7 12 7 1234567890123456789012345678901212345678901234567890123456789012123456 2p 34567890123456789012345678901212345678901234567890123456789012123456 7 12 The numbers from the number set pn 34567890123456789012345678901212345678901234567890123456789012123456 7 12 (A) 34567890123456789012345678901212345678901234567890123456789012123456 7 12 {9, 11, 12, 15, 16} must be put in the 34567890123456789012345678901212345678901234567890123456789012123456 7 12 n 2 34567890123456789012345678901212345678901234567890123456789012123456 7 1 above boxes according to these condi(B) np 34567890123456789012345678901212345678901234567890123456789012123456 7 12 34567890123456789012345678901212345678901234567890123456789012123456 7 12 tions: 34567890123456789012345678901212345678901234567890123456789012123456 7 12 p 34567890123456789012345678901212345678901234567890123456789012123456 7 12 (C) Boxes A, C, and D contain numbers 2 34567890123456789012345678901212345678901234567890123456789012123456 7 1 n 34567890123456789012345678901212345678901234567890123456789012123456 7 12 divisible by three. 34567890123456789012345678901212345678901234567890123456789012123456 7 12 2n 34567890123456789012345678901212345678901234567890123456789012123456 7 12 (D) 34567890123456789012345678901212345678901234567890123456789012123456 7 12 Box B contains a prime number. p 2 34567890123456789012345678901212345678901234567890123456789012123456 7 1 34567890123456789012345678901212345678901234567890123456789012123456 7 12 34567890123456789012345678901212345678901234567890123456789012123456 7 n1p 12 Which number must be in Box E? 34567890123456789012345678901212345678901234567890123456789012123456 7 12 (E) 34567890123456789012345678901212345678901234567890123456789012123456 7 12 p 2 34567890123456789012345678901212345678901234567890123456789012123456 7 1234567890123456789012345678901212345678901234567890123456789012123456 (A) 9 7 1234567890123456789012345678901212345678901234567890123456789012123456 7 1234567890123456789012345678901212345678901234567890123456789012123456 (B) 11 20. If n is a positive integer and n, n 2 2, and 1234567890123456789012345678901212345678901234567890123456789012123456 77 1234567890123456789012345678901212345678901234567890123456789012123456 (C) 12 n 1 2 are each prime numbers, then the 7 1234567890123456789012345678901212345678901234567890123456789012123456 7 1 (D) 15 set of those three numbers is called a 34567890123456789012345678901212345678901234567890123456789012123456 7 12 (E) 16 34567890123456789012345678901212345678901234567890123456789012123456 7 12 prime triplet. How many different prime 34567890123456789012345678901212345678901234567890123456789012123456 7 12 2 34567890123456789012345678901212345678901234567890123456789012123456 7 1234567890123456789012345678901212345678901234567890123456789012123456 triplets are there where none of the set is 7 1234567890123456789012345678901212345678901234567890123456789012123456 18. greater than fifty? 7 1234567890123456789012345678901212345678901234567890123456789012123456 1234567890123456789012345678901212345678901234567890123456789012123456 77 1234567890123456789012345678901212345678901234567890123456789012123456 (A) 1 7 1234567890123456789012345678901212345678901234567890123456789012123456 7 1234567890123456789012345678901212345678901234567890123456789012123456 (B) 2 7 1234567890123456789012345678901212345678901234567890123456789012123456 7 1234567890123456789012345678901212345678901234567890123456789012123456 (C) 3 7 1 2 34567890123456789012345678901212345678901234567890123456789012123456 7 1234567890123456789012345678901212345678901234567890123456789012123456 (D) 4 7 1234567890123456789012345678901212345678901234567890123456789012123456 7 1 (E) None. If,AC 5 8, AB 5 4, and D and H are 1234567890123456789012345678901212345678901234567890123456789012123456 7 34567890123456789012345678901212345678901234567890123456789012123456 7 12 midpoints on CE and AG respectively, 34567890123456789012345678901212345678901234567890123456789012123456 7 12 34567890123456789012345678901212345678901234567890123456789012123456 12 21. If a circle has four tangents, each of which 7 what percentage of the rectangle is 1234567890123456789012345678901212345678901234567890123456789012123456 7 34567890123456789012345678901212345678901234567890123456789012123456 7 12 is perpendicular to two of the other shaded? 34567890123456789012345678901212345678901234567890123456789012123456 7 12 2 34567890123456789012345678901212345678901234567890123456789012123456 tangents, then 1234567890123456789012345678901212345678901234567890123456789012123456 77 1234567890123456789012345678901212345678901234567890123456789012123456 (A) 12.5% 7 1 (A) at most one pair of tangent lines is 7 1234567890123456789012345678901212345678901234567890123456789012123456 (B) 16.33% 7 1234567890123456789012345678901212345678901234567890123456789012123456 parallel. 2 34567890123456789012345678901212345678901234567890123456789012123456 7 1234567890123456789012345678901212345678901234567890123456789012123456 (C) 20% 7 1234567890123456789012345678901212345678901234567890123456789012123456 (B) at most two pairs of tangent lines are 7 1234567890123456789012345678901212345678901234567890123456789012123456 (D) 25% 7 1234567890123456789012345678901212345678901234567890123456789012123456 parallel. 7 1 (E) 30% 2 34567890123456789012345678901212345678901234567890123456789012123456 7 1234567890123456789012345678901212345678901234567890123456789012123456 (C) all four tangent lines are parallel. 7 1234567890123456789012345678901212345678901234567890123456789012123456 7 1234567890123456789012345678901212345678901234567890123456789012123456 (D) a square is inscribed in the circle. 1234567890123456789012345678901212345678901234567890123456789012123456 77 1 (E) one particular diameter could be 34567890123456789012345678901212345678901234567890123456789012123456 7 12 34567890123456789012345678901212345678901234567890123456789012123456 7 12 perpendicular to all four tangent 34567890123456789012345678901212345678901234567890123456789012123456 7 12 34567890123456789012345678901212345678901234567890123456789012123456 7 12 lines. 7 1234567890123456789012345678901212345678901234567890123456789012123456 2 34567890123456789012345678901212345678901234567890123456789012123456 1234567890123456789012345678901212345678901234567890123456789012123456 77 1 34567890123456789012345678901212345678901234567890123456789012123456 7 12 1234567890123456789012345678901212345678901234567890123456789012123456 7 34567890123456789012345678901212345678901234567890123456789012123456 7 12 Do not proceed to the next section until time is up. 7 1234567890123456789012345678901212345678901234567890123456789012123456 1234567890123456789012345678901212345678901234567890123456789012123456 7 1234567890123456789012345678901212345678901234567890123456789012123456 7 1234567890123456789012345678901212345678901234567890123456789012123456 7 12345678901234567890123456789012123456789012345678901234567890121234567

STOP

14

Copyright © 2005 Thomson Peterson’s, a part of The Thomson Corporaton SAT is a registered trademark of the College Entrance Examination Board, which was not involved in the production of and does not endorse this product.

Section 3 30 Questions j Time—25 Minutes 12345678901234567890123456789012123456789012345678901234567890121234567 7 12234567890123456789012345678901212345678901234567890123456789012123456 7 1234567890123456789012345678901212345678901234567890123456789012123456 34567890123456789012345678901212345678901234567890123456789012123456 7 1 Identifying Sentence Errors 7 12234567890123456789012345678901212345678901234567890123456789012123456 7 1234567890123456789012345678901212345678901234567890123456789012123456 7 1234567890123456789012345678901212345678901234567890123456789012123456 7 1234567890123456789012345678901212345678901234567890123456789012123456 Directions: Mark the letter of your choice on the answer sheet that best corresponds to the 34567890123456789012345678901212345678901234567890123456789012123456 1234567890123456789012345678901212345678901234567890123456789012123456 77 correct answer. 1234567890123456789012345678901212345678901234567890123456789012123456 1234567890123456789012345678901212345678901234567890123456789012123456 77 1234567890123456789012345678901212345678901234567890123456789012123456 Notes: 7 1234567890123456789012345678901212345678901234567890123456789012123456 1234567890123456789012345678901212345678901234567890123456789012123456 77 1 1. The following questions test your knowledge of the rules of English grammar, as well as 7 12234567890123456789012345678901212345678901234567890123456789012123456 7 1234567890123456789012345678901212345678901234567890123456789012123456 word usage, word choice, and idioms. 7 1234567890123456789012345678901212345678901234567890123456789012123456 34567890123456789012345678901212345678901234567890123456789012123456 1234567890123456789012345678901212345678901234567890123456789012123456 77 1234567890123456789012345678901212345678901234567890123456789012123456 2. Some sentences are correct, but others contain a single error. No sentence contains more 7 1234567890123456789012345678901212345678901234567890123456789012123456 7 1234567890123456789012345678901212345678901234567890123456789012123456 than one error. 1234567890123456789012345678901212345678901234567890123456789012123456 77 1234567890123456789012345678901212345678901234567890123456789012123456 3. Any errors that occur will be found in the underlined portion of the sentence. Choose the 7 1234567890123456789012345678901212345678901234567890123456789012123456 7 1234567890123456789012345678901212345678901234567890123456789012123456 letter underneath the error to indicate the part of the sentence that must be changed. 7 1234567890123456789012345678901212345678901234567890123456789012123456 7 1 2 34567890123456789012345678901212345678901234567890123456789012123456 7 1234567890123456789012345678901212345678901234567890123456789012123456 4. If there is no error, pick answer choice (E). 7 1234567890123456789012345678901212345678901234567890123456789012123456 7 1 7 1234567890123456789012345678901212345678901234567890123456789012123456 5. There will be no change in any parts of the sentence that are not underlined. 7 12234567890123456789012345678901212345678901234567890123456789012123456 34567890123456789012345678901212345678901234567890123456789012123456 1234567890123456789012345678901212345678901234567890123456789012123456 77 1234567890123456789012345678901212345678901234567890123456789012123456 7 1 7 12234567890123456789012345678901212345678901234567890123456789012123456 7 1234567890123456789012345678901212345678901234567890123456789012123456 3. The FDA did not conclude that the 1. Despite the enormous voter drive , there 34567890123456789012345678901212345678901234567890123456789012123456 1234567890123456789012345678901212345678901234567890123456789012123456 77 1234567890123456789012345678901212345678901234567890123456789012123456 A A B 7 1 1234567890123456789012345678901212345678901234567890123456789012123456 are still many city-dwellers who are not negative side affects of the drug offset the 7 1234567890123456789012345678901212345678901234567890123456789012123456 7 7 12234567890123456789012345678901212345678901234567890123456789012123456 C B C 7 1234567890123456789012345678901212345678901234567890123456789012123456 registered to vote. No error drug’s positive benefits . No error 7 1234567890123456789012345678901212345678901234567890123456789012123456 7 1234567890123456789012345678901212345678901234567890123456789012123456 D E D E 1 34567890123456789012345678901212345678901234567890123456789012123456 7 2 34567890123456789012345678901212345678901234567890123456789012123456 1234567890123456789012345678901212345678901234567890123456789012123456 77 1234567890123456789012345678901212345678901234567890123456789012123456 2. Debating the energy bill was the first 4. Over the last decade, the information 7 1234567890123456789012345678901212345678901234567890123456789012123456 7 1234567890123456789012345678901212345678901234567890123456789012123456 A A 7 1 2 34567890123456789012345678901212345678901234567890123456789012123456 7 1234567890123456789012345678901212345678901234567890123456789012123456 order of business for the Senate; to set the industry had grown into a multi-billion7 1234567890123456789012345678901212345678901234567890123456789012123456 7 1234567890123456789012345678901212345678901234567890123456789012123456 B C B 1234567890123456789012345678901212345678901234567890123456789012123456 77 1 calendar for the upcoming session dollar industry that employs tens of 7 12234567890123456789012345678901212345678901234567890123456789012123456 7 1 34567890123456789012345678901212345678901234567890123456789012123456 C D 7 12234567890123456789012345678901212345678901234567890123456789012123456 thousands of workers. No error was to follow . No error (E) 1 34567890123456789012345678901212345678901234567890123456789012123456 7 7 12234567890123456789012345678901212345678901234567890123456789012123456 E D E 7 1 34567890123456789012345678901212345678901234567890123456789012123456 1234567890123456789012345678901212345678901234567890123456789012123456 7 1234567890123456789012345678901212345678901234567890123456789012123456 7 1234567890123456789012345678901212345678901234567890123456789012123456 7 12345678901234567890123456789012123456789012345678901234567890121234567

12345678901234567890123456789012123456789012345678901234567890121234567 1234567890123456789012345678901212345678901234567890123456789012123456 7 34567890123456789012345678901212345678901234567890123456789012123456 7 12 5. Reading widely in her field, making herself 8. Of Armigo’s two films, most critics agree 7 1234567890123456789012345678901212345678901234567890123456789012123456 34567890123456789012345678901212345678901234567890123456789012123456 7 12 34567890123456789012345678901212345678901234567890123456789012123456 7 A B A B 12 7 1234567890123456789012345678901212345678901234567890123456789012123456 available to students, and that the second is best . No error 34567890123456789012345678901212345678901234567890123456789012123456 7 12 2 34567890123456789012345678901212345678901234567890123456789012123456 7 1 C D E 34567890123456789012345678901212345678901234567890123456789012123456 7 12 34567890123456789012345678901212345678901234567890123456789012123456 7 12 her sophisticated research paid off for 34567890123456789012345678901212345678901234567890123456789012123456 7 12 9. Most people in the neighborhood 34567890123456789012345678901212345678901234567890123456789012123456 7 12 C D 2 34567890123456789012345678901212345678901234567890123456789012123456 7 1 Professor Jackson: she was awarded 34567890123456789012345678901212345678901234567890123456789012123456 7 12 34567890123456789012345678901212345678901234567890123456789012123456 7 12 agree that it is reasonable for the represen34567890123456789012345678901212345678901234567890123456789012123456 7 12 34567890123456789012345678901212345678901234567890123456789012123456 7 12 tenure last year. No error A B 7 1234567890123456789012345678901212345678901234567890123456789012123456 34567890123456789012345678901212345678901234567890123456789012123456 7 12 tative to not acquiesce to the demands of E 34567890123456789012345678901212345678901234567890123456789012123456 7 12 34567890123456789012345678901212345678901234567890123456789012123456 7 12 C D 34567890123456789012345678901212345678901234567890123456789012123456 7 12 6. One cannot perform multiple tasks the transit authority. No error 2 34567890123456789012345678901212345678901234567890123456789012123456 7 1 34567890123456789012345678901212345678901234567890123456789012123456 7 12 E 34567890123456789012345678901212345678901234567890123456789012123456 7 12 34567890123456789012345678901212345678901234567890123456789012123456 7 12 simultaneously if one is easily distracted 34567890123456789012345678901212345678901234567890123456789012123456 7 12 10. Although in many ways Canada is a 2 34567890123456789012345678901212345678901234567890123456789012123456 7 1234567890123456789012345678901212345678901234567890123456789012123456 A B C 7 1234567890123456789012345678901212345678901234567890123456789012123456 by one’s surroundings. No error A 7 1234567890123456789012345678901212345678901234567890123456789012123456 7 1234567890123456789012345678901212345678901234567890123456789012123456 staunch ally of the United States, D E 1234567890123456789012345678901212345678901234567890123456789012123456 77 1234567890123456789012345678901212345678901234567890123456789012123456 B 7 1 34567890123456789012345678901212345678901234567890123456789012123456 7 12 7. In many ways emblematic of the sweeping they have made their differences 34567890123456789012345678901212345678901234567890123456789012123456 7 12 34567890123456789012345678901212345678901234567890123456789012123456 7 12 A C 34567890123456789012345678901212345678901234567890123456789012123456 7 12 changes the state’s agricultural industry known on a number of important issues. 34567890123456789012345678901212345678901234567890123456789012123456 7 12 34567890123456789012345678901212345678901234567890123456789012123456 7 12 B D 34567890123456789012345678901212345678901234567890123456789012123456 7 12 34567890123456789012345678901212345678901234567890123456789012123456 7 12 has undergone, strawberry farming No error 34567890123456789012345678901212345678901234567890123456789012123456 7 12 34567890123456789012345678901212345678901234567890123456789012123456 7 12 C E 34567890123456789012345678901212345678901234567890123456789012123456 7 12 had exploded in Central California. 34567890123456789012345678901212345678901234567890123456789012123456 7 12 7 1234567890123456789012345678901212345678901234567890123456789012123456 D 2 34567890123456789012345678901212345678901234567890123456789012123456 1234567890123456789012345678901212345678901234567890123456789012123456 77 1234567890123456789012345678901212345678901234567890123456789012123456 No error 7 1 7 1234567890123456789012345678901212345678901234567890123456789012123456 E 34567890123456789012345678901212345678901234567890123456789012123456 7 12 2 34567890123456789012345678901212345678901234567890123456789012123456 1234567890123456789012345678901212345678901234567890123456789012123456 77 1234567890123456789012345678901212345678901234567890123456789012123456 7 1 34567890123456789012345678901212345678901234567890123456789012123456 7 12 34567890123456789012345678901212345678901234567890123456789012123456 7 12 2 7 1234567890123456789012345678901212345678901234567890123456789012123456 7 1234567890123456789012345678901212345678901234567890123456789012123456 1 34567890123456789012345678901212345678901234567890123456789012123456 7 1234567890123456789012345678901212345678901234567890123456789012123456 7 1234567890123456789012345678901212345678901234567890123456789012123456 7 34567890123456789012345678901212345678901234567890123456789012123456 7 12 34567890123456789012345678901212345678901234567890123456789012123456 7 12 34567890123456789012345678901212345678901234567890123456789012123456 7 12 34567890123456789012345678901212345678901234567890123456789012123456 7 12 1234567890123456789012345678901212345678901234567890123456789012123456 7 2 7 1234567890123456789012345678901212345678901234567890123456789012123456 7 1234567890123456789012345678901212345678901234567890123456789012123456 7 1234567890123456789012345678901212345678901234567890123456789012123456 7 1234567890123456789012345678901212345678901234567890123456789012123456 1 34567890123456789012345678901212345678901234567890123456789012123456 7 2 34567890123456789012345678901212345678901234567890123456789012123456 1234567890123456789012345678901212345678901234567890123456789012123456 77 1234567890123456789012345678901212345678901234567890123456789012123456 7 1234567890123456789012345678901212345678901234567890123456789012123456 7 1234567890123456789012345678901212345678901234567890123456789012123456 7 1 34567890123456789012345678901212345678901234567890123456789012123456 7 12 1234567890123456789012345678901212345678901234567890123456789012123456 7 34567890123456789012345678901212345678901234567890123456789012123456 7 12 1234567890123456789012345678901212345678901234567890123456789012123456 7 34567890123456789012345678901212345678901234567890123456789012123456 7 12 1234567890123456789012345678901212345678901234567890123456789012123456 7 1234567890123456789012345678901212345678901234567890123456789012123456 7 1234567890123456789012345678901212345678901234567890123456789012123456 7 1234567890123456789012345678901212345678901234567890123456789012123456 7 12345678901234567890123456789012123456789012345678901234567890121234567 16

Copyright © 2005 Thomson Peterson’s, a part of The Thomson Corporaton SAT is a registered trademark of the College Entrance Examination Board, which was not involved in the production of and does not endorse this product.

12345678901234567890123456789012123456789012345678901234567890121234567 1234567890123456789012345678901212345678901234567890123456789012123456 7 7 12234567890123456789012345678901212345678901234567890123456789012123456 7 1 34567890123456789012345678901212345678901234567890123456789012123456 Improving Sentences 7 12234567890123456789012345678901212345678901234567890123456789012123456 7 1234567890123456789012345678901212345678901234567890123456789012123456 1 34567890123456789012345678901212345678901234567890123456789012123456 7 7 12234567890123456789012345678901212345678901234567890123456789012123456 Directions: 7 1 34567890123456789012345678901212345678901234567890123456789012123456 7 12234567890123456789012345678901212345678901234567890123456789012123456 1. The following questions test your knowledge of English grammar, word usage, word 7 1234567890123456789012345678901212345678901234567890123456789012123456 7 1234567890123456789012345678901212345678901234567890123456789012123456 choice, sentence construction, and punctuation. 7 1234567890123456789012345678901212345678901234567890123456789012123456 34567890123456789012345678901212345678901234567890123456789012123456 7 1 7 12234567890123456789012345678901212345678901234567890123456789012123456 2. Every sentence contains a portion that is underlined. 7 1234567890123456789012345678901212345678901234567890123456789012123456 7 1234567890123456789012345678901212345678901234567890123456789012123456 3. Any errors that occur will be found in the underlined portion of the sentence. If you 7 1234567890123456789012345678901212345678901234567890123456789012123456 34567890123456789012345678901212345678901234567890123456789012123456 7 1 believe there is an error, choose the answer choice that corrects the original mistake. 7 12234567890123456789012345678901212345678901234567890123456789012123456 7 1234567890123456789012345678901212345678901234567890123456789012123456 Answer choices (B), (C), (D), and (E) contain alternative phrasings of the underlined 7 1234567890123456789012345678901212345678901234567890123456789012123456 7 1234567890123456789012345678901212345678901234567890123456789012123456 portion. If the sentence contains an error, one of these alternate phrasings will correct it. 34567890123456789012345678901212345678901234567890123456789012123456 7 1 7 12234567890123456789012345678901212345678901234567890123456789012123456 4. Choice (A) repeats the original underlined portion. If you believe the underlined portion 7 1234567890123456789012345678901212345678901234567890123456789012123456 7 1234567890123456789012345678901212345678901234567890123456789012123456 does not contain any errors, select answer choice (A). 7 1234567890123456789012345678901212345678901234567890123456789012123456 34567890123456789012345678901212345678901234567890123456789012123456 1234567890123456789012345678901212345678901234567890123456789012123456 77 1234567890123456789012345678901212345678901234567890123456789012123456 5. There will be no change in any parts of the sentence that are not underlined. 7 1234567890123456789012345678901212345678901234567890123456789012123456 1234567890123456789012345678901212345678901234567890123456789012123456 77 1234567890123456789012345678901212345678901234567890123456789012123456 7 1234567890123456789012345678901212345678901234567890123456789012123456 7 1 7 12234567890123456789012345678901212345678901234567890123456789012123456 11. Her first novel having been published, the 13. After working on his serve for several 7 1234567890123456789012345678901212345678901234567890123456789012123456 1234567890123456789012345678901212345678901234567890123456789012123456 author began to take notes for her second. days, rumors circulated that the challenger 7 34567890123456789012345678901212345678901234567890123456789012123456 1234567890123456789012345678901212345678901234567890123456789012123456 77 1234567890123456789012345678901212345678901234567890123456789012123456 would win the rematch. (A) Her first novel having been published 7 1234567890123456789012345678901212345678901234567890123456789012123456 7 1234567890123456789012345678901212345678901234567890123456789012123456 (A) After working on his serve for (B) Having been her most recent novel 7 1234567890123456789012345678901212345678901234567890123456789012123456 7 1234567890123456789012345678901212345678901234567890123456789012123456 several days, rumors circulated that published 7 1234567890123456789012345678901212345678901234567890123456789012123456 7 1234567890123456789012345678901212345678901234567890123456789012123456 the challenger would win the (C) Her first novel, having been pub7 1234567890123456789012345678901212345678901234567890123456789012123456 7 1 rematch. lished 2 34567890123456789012345678901212345678901234567890123456789012123456 1234567890123456789012345678901212345678901234567890123456789012123456 77 1234567890123456789012345678901212345678901234567890123456789012123456 (B) After working on his serve for (D) When having had her first novel 7 1 7 1234567890123456789012345678901212345678901234567890123456789012123456 several days, the challenger circupublished 7 12234567890123456789012345678901212345678901234567890123456789012123456 34567890123456789012345678901212345678901234567890123456789012123456 7 1234567890123456789012345678901212345678901234567890123456789012123456 lated rumors that he would win the (E) Having published her first novel 7 1234567890123456789012345678901212345678901234567890123456789012123456 7 1 rematch. 7 12234567890123456789012345678901212345678901234567890123456789012123456 7 1234567890123456789012345678901212345678901234567890123456789012123456 12. Van Gogh’s early work has often been (C) Rumors circulated that the chal34567890123456789012345678901212345678901234567890123456789012123456 1234567890123456789012345678901212345678901234567890123456789012123456 77 1234567890123456789012345678901212345678901234567890123456789012123456 described as being in sharp contrast with lenger, after working on his serve for 7 1 1234567890123456789012345678901212345678901234567890123456789012123456 his later work, despite there is a fundaseveral days, would win the rematch. 7 7 1234567890123456789012345678901212345678901234567890123456789012123456 mental continuity between the two. 2 34567890123456789012345678901212345678901234567890123456789012123456 1234567890123456789012345678901212345678901234567890123456789012123456 (D) After having worked on his serve for 7 7 1234567890123456789012345678901212345678901234567890123456789012123456 7 1234567890123456789012345678901212345678901234567890123456789012123456 several days, the rematch was (A) with his later work, despite 1234567890123456789012345678901212345678901234567890123456789012123456 77 1 rumored to be won by the challenger. (B) with his later work; despite the fact 7 12234567890123456789012345678901212345678901234567890123456789012123456 7 1234567890123456789012345678901212345678901234567890123456789012123456 (E) After working on his serve for that 7 1234567890123456789012345678901212345678901234567890123456789012123456 7 1234567890123456789012345678901212345678901234567890123456789012123456 several days, rumors circulated, the (C) with his later work, rather, 7 1 34567890123456789012345678901212345678901234567890123456789012123456 challenger would win the rematch. 2 34567890123456789012345678901212345678901234567890123456789012123456 7 1234567890123456789012345678901212345678901234567890123456789012123456 (D) with his later work, but 7 1234567890123456789012345678901212345678901234567890123456789012123456 7 1234567890123456789012345678901212345678901234567890123456789012123456 (E) with his later work, notwithstanding 1234567890123456789012345678901212345678901234567890123456789012123456 77 1 7 12234567890123456789012345678901212345678901234567890123456789012123456 1 34567890123456789012345678901212345678901234567890123456789012123456 7 7 12234567890123456789012345678901212345678901234567890123456789012123456 1 34567890123456789012345678901212345678901234567890123456789012123456 7 7 12234567890123456789012345678901212345678901234567890123456789012123456 1 34567890123456789012345678901212345678901234567890123456789012123456 7 1234567890123456789012345678901212345678901234567890123456789012123456 7 1234567890123456789012345678901212345678901234567890123456789012123456 7 1234567890123456789012345678901212345678901234567890123456789012123456 7 12345678901234567890123456789012123456789012345678901234567890121234567 Copyright © 2005 Thomson Peterson’s, a part of The Thomson Corporaton SAT is a registered trademark of the College Entrance Examination Board, which was not involved in the production of and does not endorse this product.

17

12345678901234567890123456789012123456789012345678901234567890121234567 1234567890123456789012345678901212345678901234567890123456789012123456 7 34567890123456789012345678901212345678901234567890123456789012123456 7 12 14. The artist thought that it was important 16. Coffee shops, which were formerly found 7 1234567890123456789012345678901212345678901234567890123456789012123456 34567890123456789012345678901212345678901234567890123456789012123456 7 12 only in urban settings and near college both to portray the subject truthfully, no 34567890123456789012345678901212345678901234567890123456789012123456 7 12 7 1234567890123456789012345678901212345678901234567890123456789012123456 campuses, have been expanding in the last matter the difficulty, and revealing 34567890123456789012345678901212345678901234567890123456789012123456 7 12 2 34567890123456789012345678901212345678901234567890123456789012123456 7 1 something new about the subject. few years outside these circumspect 34567890123456789012345678901212345678901234567890123456789012123456 7 12 34567890123456789012345678901212345678901234567890123456789012123456 7 12 domains. 34567890123456789012345678901212345678901234567890123456789012123456 7 12 (A) and revealing something new about 34567890123456789012345678901212345678901234567890123456789012123456 7 12 2 34567890123456789012345678901212345678901234567890123456789012123456 7 1 (A) which were formerly found only in the subject. 34567890123456789012345678901212345678901234567890123456789012123456 7 12 34567890123456789012345678901212345678901234567890123456789012123456 7 12 urban settings and near college (B) and so he revealed something new 34567890123456789012345678901212345678901234567890123456789012123456 7 12 34567890123456789012345678901212345678901234567890123456789012123456 7 12 campuses about the subject. 7 1234567890123456789012345678901212345678901234567890123456789012123456 34567890123456789012345678901212345678901234567890123456789012123456 7 12 (B) being formerly found only in urban (C) and to reveal something new about 34567890123456789012345678901212345678901234567890123456789012123456 7 12 34567890123456789012345678901212345678901234567890123456789012123456 7 12 settings and near college campuses the subject. 34567890123456789012345678901212345678901234567890123456789012123456 7 12 2 34567890123456789012345678901212345678901234567890123456789012123456 1 (C) which have been found formerly only 7 (D) having thereby revealed something 34567890123456789012345678901212345678901234567890123456789012123456 7 12 34567890123456789012345678901212345678901234567890123456789012123456 7 12 in urban settings and near college new about the subject. 34567890123456789012345678901212345678901234567890123456789012123456 7 12 34567890123456789012345678901212345678901234567890123456789012123456 7 12 campuses (E) and revealing something about the 34567890123456789012345678901212345678901234567890123456789012123456 7 12 34567890123456789012345678901212345678901234567890123456789012123456 7 12 (D) which were formerly found only in subject that is new. 34567890123456789012345678901212345678901234567890123456789012123456 7 12 urban settings or near college 34567890123456789012345678901212345678901234567890123456789012123456 7 12 34567890123456789012345678901212345678901234567890123456789012123456 7 12 15. Max Planck was not only one of the campuses 2 34567890123456789012345678901212345678901234567890123456789012123456 1234567890123456789012345678901212345678901234567890123456789012123456 77 1 founders of quantum mechanics, but an (E) that were formerly found only in 34567890123456789012345678901212345678901234567890123456789012123456 7 12 34567890123456789012345678901212345678901234567890123456789012123456 7 12 accomplished pianist. urban settings and near college 34567890123456789012345678901212345678901234567890123456789012123456 7 12 2 34567890123456789012345678901212345678901234567890123456789012123456 7 1234567890123456789012345678901212345678901234567890123456789012123456 campuses 7 1234567890123456789012345678901212345678901234567890123456789012123456 (A) mechanics, but an accomplished 1234567890123456789012345678901212345678901234567890123456789012123456 77 1234567890123456789012345678901212345678901234567890123456789012123456 pianist. 7 1234567890123456789012345678901212345678901234567890123456789012123456 17. Until the Chin dynasty changed this 7 1234567890123456789012345678901212345678901234567890123456789012123456 (B) mechanics; but he was also an 7 1234567890123456789012345678901212345678901234567890123456789012123456 practice, most Chinese intellectuals did 7 1234567890123456789012345678901212345678901234567890123456789012123456 accomplished pianist. 7 1234567890123456789012345678901212345678901234567890123456789012123456 not travel to the imperial court but 7 1 (C) mechanics; and he was also an 2 34567890123456789012345678901212345678901234567890123456789012123456 1234567890123456789012345678901212345678901234567890123456789012123456 remained in their native provincial centers. 7 7 accomplished pianist. 1234567890123456789012345678901212345678901234567890123456789012123456 7 1 (A) but remained in their native provin(D) mechanics, and an accomplished 7 1234567890123456789012345678901212345678901234567890123456789012123456 34567890123456789012345678901212345678901234567890123456789012123456 7 12 cial centers. pianist. 2 34567890123456789012345678901212345678901234567890123456789012123456 1234567890123456789012345678901212345678901234567890123456789012123456 77 1234567890123456789012345678901212345678901234567890123456789012123456 (B) and remained in their native provin(E) mechanics, but also an accomplished 7 1 34567890123456789012345678901212345678901234567890123456789012123456 7 12 cial centers. piano. 34567890123456789012345678901212345678901234567890123456789012123456 7 12 2 34567890123456789012345678901212345678901234567890123456789012123456 7 1234567890123456789012345678901212345678901234567890123456789012123456 (C) but rather they remained in the 7 1234567890123456789012345678901212345678901234567890123456789012123456 7 1 native provinces. 1234567890123456789012345678901212345678901234567890123456789012123456 7 7 1234567890123456789012345678901212345678901234567890123456789012123456 (D) yet they remained in their native 34567890123456789012345678901212345678901234567890123456789012123456 7 12 34567890123456789012345678901212345678901234567890123456789012123456 7 12 provincial centers. 34567890123456789012345678901212345678901234567890123456789012123456 7 12 34567890123456789012345678901212345678901234567890123456789012123456 7 12 (E) but remained in the provinces to 7 1234567890123456789012345678901212345678901234567890123456789012123456 2 34567890123456789012345678901212345678901234567890123456789012123456 7 1234567890123456789012345678901212345678901234567890123456789012123456 which they were native. 7 1234567890123456789012345678901212345678901234567890123456789012123456 1234567890123456789012345678901212345678901234567890123456789012123456 77 1234567890123456789012345678901212345678901234567890123456789012123456 7 1 34567890123456789012345678901212345678901234567890123456789012123456 7 12 34567890123456789012345678901212345678901234567890123456789012123456 7 12 34567890123456789012345678901212345678901234567890123456789012123456 7 12 34567890123456789012345678901212345678901234567890123456789012123456 7 12 1234567890123456789012345678901212345678901234567890123456789012123456 7 2 7 1234567890123456789012345678901212345678901234567890123456789012123456 1 34567890123456789012345678901212345678901234567890123456789012123456 7 34567890123456789012345678901212345678901234567890123456789012123456 7 12 1234567890123456789012345678901212345678901234567890123456789012123456 7 34567890123456789012345678901212345678901234567890123456789012123456 7 12 1234567890123456789012345678901212345678901234567890123456789012123456 7 1234567890123456789012345678901212345678901234567890123456789012123456 7 1234567890123456789012345678901212345678901234567890123456789012123456 7 1234567890123456789012345678901212345678901234567890123456789012123456 7 12345678901234567890123456789012123456789012345678901234567890121234567 18

Copyright © 2005 Thomson Peterson’s, a part of The Thomson Corporaton SAT is a registered trademark of the College Entrance Examination Board, which was not involved in the production of and does not endorse this product.

12345678901234567890123456789012123456789012345678901234567890121234567 1234567890123456789012345678901212345678901234567890123456789012123456 7 7 12234567890123456789012345678901212345678901234567890123456789012123456 18. The artwork of the late Renaissance was 20. Philology, the study of words, no longer 7 1 34567890123456789012345678901212345678901234567890123456789012123456 7 12234567890123456789012345678901212345678901234567890123456789012123456 characterized by a deep sympathy for the exists in academia as a distinct discipline 7 1234567890123456789012345678901212345678901234567890123456789012123456 7 1 34567890123456789012345678901212345678901234567890123456789012123456 human subject, often portraying human because it has been subsumed under the 7 12234567890123456789012345678901212345678901234567890123456789012123456 34567890123456789012345678901212345678901234567890123456789012123456 7 1 frailties and failings. study of linguistics. 7 12234567890123456789012345678901212345678901234567890123456789012123456 7 1234567890123456789012345678901212345678901234567890123456789012123456 7 1234567890123456789012345678901212345678901234567890123456789012123456 (A) often portraying human frailties and (A) it has been subsumed under the study 7 1234567890123456789012345678901212345678901234567890123456789012123456 34567890123456789012345678901212345678901234567890123456789012123456 7 1 failings. of linguistics. 7 12234567890123456789012345678901212345678901234567890123456789012123456 7 1234567890123456789012345678901212345678901234567890123456789012123456 (B) and it often portrayed human (B) it was subsumed in the past under 7 1234567890123456789012345678901212345678901234567890123456789012123456 7 1234567890123456789012345678901212345678901234567890123456789012123456 frailties and failings. the study of linguistics. 34567890123456789012345678901212345678901234567890123456789012123456 7 1 7 12234567890123456789012345678901212345678901234567890123456789012123456 (C) human frailties and human failings (C) it has been subsumed with the study 7 1234567890123456789012345678901212345678901234567890123456789012123456 7 1234567890123456789012345678901212345678901234567890123456789012123456 being often portrayed. of linguistics. 7 1234567890123456789012345678901212345678901234567890123456789012123456 34567890123456789012345678901212345678901234567890123456789012123456 7 1 (D) although it often portrayed human (D) linguistics previously having sub7 12234567890123456789012345678901212345678901234567890123456789012123456 7 1234567890123456789012345678901212345678901234567890123456789012123456 frailties and failings. sumed it. 7 1234567890123456789012345678901212345678901234567890123456789012123456 7 1234567890123456789012345678901212345678901234567890123456789012123456 (E) though portraying human frailties (E) it had been subsumed under the 34567890123456789012345678901212345678901234567890123456789012123456 1234567890123456789012345678901212345678901234567890123456789012123456 77 1234567890123456789012345678901212345678901234567890123456789012123456 and failings. study of linguistics. 1234567890123456789012345678901212345678901234567890123456789012123456 77 1234567890123456789012345678901212345678901234567890123456789012123456 7 1234567890123456789012345678901212345678901234567890123456789012123456 19. Pancho Villa’s raid on Columbus, New 7 1234567890123456789012345678901212345678901234567890123456789012123456 7 1 Mexico, which was part of the tumult of 7 12234567890123456789012345678901212345678901234567890123456789012123456 7 1234567890123456789012345678901212345678901234567890123456789012123456 the Mexican revolution, therefore 7 1234567890123456789012345678901212345678901234567890123456789012123456 34567890123456789012345678901212345678901234567890123456789012123456 7 1234567890123456789012345678901212345678901234567890123456789012123456 prompted a retaliatory expedition led by 7 1234567890123456789012345678901212345678901234567890123456789012123456 7 1234567890123456789012345678901212345678901234567890123456789012123456 General Pershing. 1234567890123456789012345678901212345678901234567890123456789012123456 77 1234567890123456789012345678901212345678901234567890123456789012123456 7 1234567890123456789012345678901212345678901234567890123456789012123456 (A) revolution, therefore prompted a 7 1234567890123456789012345678901212345678901234567890123456789012123456 7 1234567890123456789012345678901212345678901234567890123456789012123456 retaliatory expedition led by General 1234567890123456789012345678901212345678901234567890123456789012123456 77 1 Pershing. 7 12234567890123456789012345678901212345678901234567890123456789012123456 7 (B) revolution, thereby prompting a 1234567890123456789012345678901212345678901234567890123456789012123456 7 1 34567890123456789012345678901212345678901234567890123456789012123456 retaliatory expedition led by General 7 1234567890123456789012345678901212345678901234567890123456789012123456 7 12234567890123456789012345678901212345678901234567890123456789012123456 Pershing. 34567890123456789012345678901212345678901234567890123456789012123456 1234567890123456789012345678901212345678901234567890123456789012123456 77 1234567890123456789012345678901212345678901234567890123456789012123456 (C) revolution, had prompted General 7 1 7 12234567890123456789012345678901212345678901234567890123456789012123456 Pershing to lead a retaliatory 7 1234567890123456789012345678901212345678901234567890123456789012123456 34567890123456789012345678901212345678901234567890123456789012123456 7 1234567890123456789012345678901212345678901234567890123456789012123456 expedition. 7 1234567890123456789012345678901212345678901234567890123456789012123456 7 1 (D) revolution; a retaliatory expedition 1234567890123456789012345678901212345678901234567890123456789012123456 7 7 1234567890123456789012345678901212345678901234567890123456789012123456 led by General Pershing thereby 7 12234567890123456789012345678901212345678901234567890123456789012123456 7 1234567890123456789012345678901212345678901234567890123456789012123456 prompted. 7 1234567890123456789012345678901212345678901234567890123456789012123456 7 1234567890123456789012345678901212345678901234567890123456789012123456 (E) revolution, prompted a retaliatory 1 34567890123456789012345678901212345678901234567890123456789012123456 7 2 34567890123456789012345678901212345678901234567890123456789012123456 7 1234567890123456789012345678901212345678901234567890123456789012123456 expedition led by General Pershing. 7 1234567890123456789012345678901212345678901234567890123456789012123456 1234567890123456789012345678901212345678901234567890123456789012123456 77 1234567890123456789012345678901212345678901234567890123456789012123456 7 1 7 12234567890123456789012345678901212345678901234567890123456789012123456 7 1234567890123456789012345678901212345678901234567890123456789012123456 7 1234567890123456789012345678901212345678901234567890123456789012123456 7 1234567890123456789012345678901212345678901234567890123456789012123456 1 34567890123456789012345678901212345678901234567890123456789012123456 7 2 34567890123456789012345678901212345678901234567890123456789012123456 1234567890123456789012345678901212345678901234567890123456789012123456 77 1 7 12234567890123456789012345678901212345678901234567890123456789012123456 1 34567890123456789012345678901212345678901234567890123456789012123456 7 7 12234567890123456789012345678901212345678901234567890123456789012123456 1 34567890123456789012345678901212345678901234567890123456789012123456 7 1234567890123456789012345678901212345678901234567890123456789012123456 7 1234567890123456789012345678901212345678901234567890123456789012123456 7 1234567890123456789012345678901212345678901234567890123456789012123456 7 12345678901234567890123456789012123456789012345678901234567890121234567 Copyright © 2005 Thomson Peterson’s, a part of The Thomson Corporaton SAT is a registered trademark of the College Entrance Examination Board, which was not involved in the production of and does not endorse this product.

19

12345678901234567890123456789012123456789012345678901234567890121234567 1234567890123456789012345678901212345678901234567890123456789012123456 7 34567890123456789012345678901212345678901234567890123456789012123456 7 12 7 1234567890123456789012345678901212345678901234567890123456789012123456 Improving Paragraphs 34567890123456789012345678901212345678901234567890123456789012123456 7 12 34567890123456789012345678901212345678901234567890123456789012123456 7 12 1234567890123456789012345678901212345678901234567890123456789012123456 7 34567890123456789012345678901212345678901234567890123456789012123456 7 12 Directions: 7 1234567890123456789012345678901212345678901234567890123456789012123456 34567890123456789012345678901212345678901234567890123456789012123456 7 12 1. The following questions test your knowledge of paragraph and sentence construction. 34567890123456789012345678901212345678901234567890123456789012123456 7 12 34567890123456789012345678901212345678901234567890123456789012123456 7 12 34567890123456789012345678901212345678901234567890123456789012123456 7 12 2. The following passage is a rough draft of an essay. This rough draft contains various 7 1234567890123456789012345678901212345678901234567890123456789012123456 34567890123456789012345678901212345678901234567890123456789012123456 7 12 errors. 34567890123456789012345678901212345678901234567890123456789012123456 7 12 34567890123456789012345678901212345678901234567890123456789012123456 7 12 34567890123456789012345678901212345678901234567890123456789012123456 7 12 3. Read the rough draft and then answer the questions that follow. Some questions will focus 2 7 1 34567890123456789012345678901212345678901234567890123456789012123456 34567890123456789012345678901212345678901234567890123456789012123456 7 12 on specific sentences and ask if there are any problems with that sentence’s word choice, 34567890123456789012345678901212345678901234567890123456789012123456 7 12 34567890123456789012345678901212345678901234567890123456789012123456 7 12 word usage, or overall structure. Other questions will ask about the paragraph itself. 34567890123456789012345678901212345678901234567890123456789012123456 7 12 2 34567890123456789012345678901212345678901234567890123456789012123456 7 1 These questions will focus on paragraph organization and development. 34567890123456789012345678901212345678901234567890123456789012123456 7 12 34567890123456789012345678901212345678901234567890123456789012123456 7 12 4. Select the answer that best reflects the rules of English grammar and proper essay and 34567890123456789012345678901212345678901234567890123456789012123456 7 12 34567890123456789012345678901212345678901234567890123456789012123456 7 12 paragraph writing. 2 34567890123456789012345678901212345678901234567890123456789012123456 1234567890123456789012345678901212345678901234567890123456789012123456 77 1234567890123456789012345678901212345678901234567890123456789012123456 7 1234567890123456789012345678901212345678901234567890123456789012123456 7 1234567890123456789012345678901212345678901234567890123456789012123456 7 1234567890123456789012345678901212345678901234567890123456789012123456 7 1234567890123456789012345678901212345678901234567890123456789012123456 21. Which of the following offers the best 7 1 Questions 21–25 are based on the following 34567890123456789012345678901212345678901234567890123456789012123456 7 12 passage. combination of sentences 1 and 2 (repro34567890123456789012345678901212345678901234567890123456789012123456 7 12 34567890123456789012345678901212345678901234567890123456789012123456 7 12 duced below)? 2 34567890123456789012345678901212345678901234567890123456789012123456 1234567890123456789012345678901212345678901234567890123456789012123456 77 1234567890123456789012345678901212345678901234567890123456789012123456 (1) An incredible hot-air balloon exhibition 1234567890123456789012345678901212345678901234567890123456789012123456 An incredible hot-air balloon exhibition hap- 7 7 1234567890123456789012345678901212345678901234567890123456789012123456 happened on September 5, 1862. (2) It was 7 1234567890123456789012345678901212345678901234567890123456789012123456 pened on September 5, 1862. It was given 7 1234567890123456789012345678901212345678901234567890123456789012123456 given by Glaisher and Coxwell, two English7 1234567890123456789012345678901212345678901234567890123456789012123456 by Glaisher and Coxwell, two Englishmen. 7 1234567890123456789012345678901212345678901234567890123456789012123456 men. (3) There was no compressed oxygen for 1234567890123456789012345678901212345678901234567890123456789012123456 77 1 them to breathe in those days. (4) They got so (A) An incredible hot-air balloon 34567890123456789012345678901212345678901234567890123456789012123456 7 12 34567890123456789012345678901212345678901234567890123456789012123456 7 12 high that they couldn’t use their limbs. (5) exhibition was given September 5, 1234567890123456789012345678901212345678901234567890123456789012123456 7 7 1234567890123456789012345678901212345678901234567890123456789012123456 Coxwell had to open the descending valve with 1862 by Glaisher and Coxwell, two 34567890123456789012345678901212345678901234567890123456789012123456 7 12 2 34567890123456789012345678901212345678901234567890123456789012123456 7 1234567890123456789012345678901212345678901234567890123456789012123456 his teeth. (6) Before Glaisher passed out, he Englishmen. 7 1234567890123456789012345678901212345678901234567890123456789012123456 (B) An incredibly hot-air balloon 7 1 recorded an elevation of twenty-nine thousand 34567890123456789012345678901212345678901234567890123456789012123456 7 12 feet. (7) Many believe they got eight thousand exhibition happened on September 5, 7 34567890123456789012345678901212345678901234567890123456789012123456 12 2 34567890123456789012345678901212345678901234567890123456789012123456 7 1234567890123456789012345678901212345678901234567890123456789012123456 feet higher before they began to descend, 1862, given by Glaisher and Cox7 1234567890123456789012345678901212345678901234567890123456789012123456 7 1 making their ascent the highest in the ninewell, two Englishmen. 7 1234567890123456789012345678901212345678901234567890123456789012123456 7 1234567890123456789012345678901212345678901234567890123456789012123456 teenth century. (C) Given by Glaisher and Coxwell, two 2 34567890123456789012345678901212345678901234567890123456789012123456 1234567890123456789012345678901212345678901234567890123456789012123456 77 1234567890123456789012345678901212345678901234567890123456789012123456 (8) Now the largest balloon to go up in the Englishmen, an incredible hot-air 7 1234567890123456789012345678901212345678901234567890123456789012123456 7 1234567890123456789012345678901212345678901234567890123456789012123456 nineteenth century was “The Giant.” (9) The balloon exhibition happened on 7 1 2 34567890123456789012345678901212345678901234567890123456789012123456 7 1234567890123456789012345678901212345678901234567890123456789012123456 balloon held 215,000 cubic feet of air and was September 5, 1862. 7 1234567890123456789012345678901212345678901234567890123456789012123456 7 1234567890123456789012345678901212345678901234567890123456789012123456 74 feet wide. (10) It could carry four and a half (D) Glaisher and Coxwell, two English1234567890123456789012345678901212345678901234567890123456789012123456 77 1 tons of cargo. (11) Its flight began in Paris, in men, gave an incredible hot-air 34567890123456789012345678901212345678901234567890123456789012123456 7 12 34567890123456789012345678901212345678901234567890123456789012123456 7 12 1853, with fifteen passengers. (12) All of whom balloon exhibition, happening on 34567890123456789012345678901212345678901234567890123456789012123456 7 12 34567890123456789012345678901212345678901234567890123456789012123456 7 12 returned safely. (13) The successful trip received September 5, 1862. 7 1234567890123456789012345678901212345678901234567890123456789012123456 2 34567890123456789012345678901212345678901234567890123456789012123456 1234567890123456789012345678901212345678901234567890123456789012123456 (E) Two Englishmen, Glaisher and 7 7 1 a great deal of national and international press 34567890123456789012345678901212345678901234567890123456789012123456 12 because many thought the hot-air balloon would Coxwell, gave an incredible hot-air bal- 7 1234567890123456789012345678901212345678901234567890123456789012123456 7 34567890123456789012345678901212345678901234567890123456789012123456 12 become a form of common transportation. loon exhibition on September 5, 1862. 7 7 1234567890123456789012345678901212345678901234567890123456789012123456 1234567890123456789012345678901212345678901234567890123456789012123456 7 1234567890123456789012345678901212345678901234567890123456789012123456 7 1234567890123456789012345678901212345678901234567890123456789012123456 7 12345678901234567890123456789012123456789012345678901234567890121234567 20

Copyright © 2005 Thomson Peterson’s, a part of The Thomson Corporaton SAT is a registered trademark of the College Entrance Examination Board, which was not involved in the production of and does not endorse this product.

12345678901234567890123456789012123456789012345678901234567890121234567 1234567890123456789012345678901212345678901234567890123456789012123456 7 7 12234567890123456789012345678901212345678901234567890123456789012123456 22. Which of the following sentences in the 24. Which of the following is the best way to 7 1 34567890123456789012345678901212345678901234567890123456789012123456 7 12234567890123456789012345678901212345678901234567890123456789012123456 first paragraph appears to be out of order? combine sentences 9 and 10 (reproduced 7 1234567890123456789012345678901212345678901234567890123456789012123456 7 1 34567890123456789012345678901212345678901234567890123456789012123456 below)? 7 12234567890123456789012345678901212345678901234567890123456789012123456 (A) There was no compressed oxygen for 34567890123456789012345678901212345678901234567890123456789012123456 7 1 7 12234567890123456789012345678901212345678901234567890123456789012123456 them to breathe in those days. The balloon held 215,000 cubic feet of air 7 1234567890123456789012345678901212345678901234567890123456789012123456 7 1234567890123456789012345678901212345678901234567890123456789012123456 (B) They got so high that they couldn’t and was 74 feet wide. It could handle four 7 1234567890123456789012345678901212345678901234567890123456789012123456 34567890123456789012345678901212345678901234567890123456789012123456 7 1 use their limbs. and a half tons of cargo. 7 12234567890123456789012345678901212345678901234567890123456789012123456 7 1234567890123456789012345678901212345678901234567890123456789012123456 (C) Coxwell had to open the descending 7 1234567890123456789012345678901212345678901234567890123456789012123456 (A) The balloon held 215,000 cubic feet 7 1234567890123456789012345678901212345678901234567890123456789012123456 valve with his teeth. 34567890123456789012345678901212345678901234567890123456789012123456 7 1 of air and was 74 feet wide, which 7 12234567890123456789012345678901212345678901234567890123456789012123456 (D) Before Glaisher passed out, he 1234567890123456789012345678901212345678901234567890123456789012123456 could handle four and a half tons of 7 7 1234567890123456789012345678901212345678901234567890123456789012123456 recorded an elevation of 29 thousand 7 1234567890123456789012345678901212345678901234567890123456789012123456 cargo. 34567890123456789012345678901212345678901234567890123456789012123456 7 1 feet. 12234567890123456789012345678901212345678901234567890123456789012123456 (B) The balloon held 215,000 cubic feet 7 7 1234567890123456789012345678901212345678901234567890123456789012123456 (E) Many believe they got 8 thousand of air and was 74 feet wide, handling 7 1234567890123456789012345678901212345678901234567890123456789012123456 7 1234567890123456789012345678901212345678901234567890123456789012123456 feet higher before they began to four and a half tons of cargo. 7 1234567890123456789012345678901212345678901234567890123456789012123456 7 1234567890123456789012345678901212345678901234567890123456789012123456 descend. (C) The balloon held 215,000 cubic feet 7 1234567890123456789012345678901212345678901234567890123456789012123456 7 1234567890123456789012345678901212345678901234567890123456789012123456 of air and was 74 feet wide; it could 7 1234567890123456789012345678901212345678901234567890123456789012123456 23. Which of the following is the best revision 34567890123456789012345678901212345678901234567890123456789012123456 7 1234567890123456789012345678901212345678901234567890123456789012123456 handle four and a half tons of cargo. 7 1 for sentence 8 (reproduced below)? 12234567890123456789012345678901212345678901234567890123456789012123456 (D) The balloon held 215,000 cubic feet 7 7 1234567890123456789012345678901212345678901234567890123456789012123456 7 1234567890123456789012345678901212345678901234567890123456789012123456 Now the largest balloon to go up in the of air and was 74 feet wide, and it 34567890123456789012345678901212345678901234567890123456789012123456 1234567890123456789012345678901212345678901234567890123456789012123456 77 1234567890123456789012345678901212345678901234567890123456789012123456 nineteenth century was “The Giant.” could handle four and a half tons of 7 1234567890123456789012345678901212345678901234567890123456789012123456 7 1234567890123456789012345678901212345678901234567890123456789012123456 cargo. 7 1234567890123456789012345678901212345678901234567890123456789012123456 (A) Move “in the nineteenth century” to 1234567890123456789012345678901212345678901234567890123456789012123456 (E) The balloon held 215,000 cubic feet 7 7 1234567890123456789012345678901212345678901234567890123456789012123456 the beginning of the sentence and 7 1234567890123456789012345678901212345678901234567890123456789012123456 of air and was 74 feet wide, but it delete “Now” 7 1234567890123456789012345678901212345678901234567890123456789012123456 7 1 could carry four and a half tons of (B) Add a comma after “Now.” 2 34567890123456789012345678901212345678901234567890123456789012123456 1234567890123456789012345678901212345678901234567890123456789012123456 77 1234567890123456789012345678901212345678901234567890123456789012123456 cargo. (C) Begin the sentence with “Moreover,” 7 1 7 1234567890123456789012345678901212345678901234567890123456789012123456 (D) Delete “now.” 7 12234567890123456789012345678901212345678901234567890123456789012123456 25. Which of the following is the best way to 34567890123456789012345678901212345678901234567890123456789012123456 7 1234567890123456789012345678901212345678901234567890123456789012123456 (E) Replace “to go up” with “exhibition.” 7 1234567890123456789012345678901212345678901234567890123456789012123456 revise sentences 11 and 12 (reproduced 7 1 7 12234567890123456789012345678901212345678901234567890123456789012123456 below)? 7 1234567890123456789012345678901212345678901234567890123456789012123456 34567890123456789012345678901212345678901234567890123456789012123456 1234567890123456789012345678901212345678901234567890123456789012123456 77 1234567890123456789012345678901212345678901234567890123456789012123456 Its flight began in Paris, in 1853, with 7 1 7 1234567890123456789012345678901212345678901234567890123456789012123456 fifteen passengers. All of whom returned 1234567890123456789012345678901212345678901234567890123456789012123456 7 2 34567890123456789012345678901212345678901234567890123456789012123456 7 1234567890123456789012345678901212345678901234567890123456789012123456 safely. 7 1234567890123456789012345678901212345678901234567890123456789012123456 1234567890123456789012345678901212345678901234567890123456789012123456 77 1234567890123456789012345678901212345678901234567890123456789012123456 (A) Replace “whom” with “who.” 7 1 2 34567890123456789012345678901212345678901234567890123456789012123456 7 1234567890123456789012345678901212345678901234567890123456789012123456 (B) Make the second sentence read 7 1234567890123456789012345678901212345678901234567890123456789012123456 7 1234567890123456789012345678901212345678901234567890123456789012123456 “Who all returned safely.” 1234567890123456789012345678901212345678901234567890123456789012123456 77 1 (C) Delete “of” 7 12234567890123456789012345678901212345678901234567890123456789012123456 (D) Replace the period at the end of 7 1234567890123456789012345678901212345678901234567890123456789012123456 7 1234567890123456789012345678901212345678901234567890123456789012123456 sentence 11 with a comma. 7 1234567890123456789012345678901212345678901234567890123456789012123456 7 1 34567890123456789012345678901212345678901234567890123456789012123456 (E) Delete the period at the end of 2 34567890123456789012345678901212345678901234567890123456789012123456 1234567890123456789012345678901212345678901234567890123456789012123456 77 1 sentence 11 and change “returned” 7 12234567890123456789012345678901212345678901234567890123456789012123456 7 1 34567890123456789012345678901212345678901234567890123456789012123456 to “returning” 7 12234567890123456789012345678901212345678901234567890123456789012123456 34567890123456789012345678901212345678901234567890123456789012123456 7 1 1234567890123456789012345678901212345678901234567890123456789012123456 7 1234567890123456789012345678901212345678901234567890123456789012123456 7 1234567890123456789012345678901212345678901234567890123456789012123456 7 12345678901234567890123456789012123456789012345678901234567890121234567 Copyright © 2005 Thomson Peterson’s, a part of The Thomson Corporaton SAT is a registered trademark of the College Entrance Examination Board, which was not involved in the production of and does not endorse this product.

21

12345678901234567890123456789012123456789012345678901234567890121234567 1234567890123456789012345678901212345678901234567890123456789012123456 7 34567890123456789012345678901212345678901234567890123456789012123456 7 12 Questions 26–30 are based on the following 7 26. Which of the following is the revision of 1234567890123456789012345678901212345678901234567890123456789012123456 34567890123456789012345678901212345678901234567890123456789012123456 7 12 passage. sentence 4 (reproduced below)? 34567890123456789012345678901212345678901234567890123456789012123456 7 12 1234567890123456789012345678901212345678901234567890123456789012123456 7 34567890123456789012345678901212345678901234567890123456789012123456 12 One of the officials from the literacy pro- 7 7 1234567890123456789012345678901212345678901234567890123456789012123456 (1) On my nineteenth birthday, I began my trip 34567890123456789012345678901212345678901234567890123456789012123456 7 12 gram I was working was there to meet me. 34567890123456789012345678901212345678901234567890123456789012123456 7 12 to Mali, West Africa. (2) Some 24 hours later I 34567890123456789012345678901212345678901234567890123456789012123456 7 12 34567890123456789012345678901212345678901234567890123456789012123456 7 12 arrived in Bamako, the capital of Mali. (3) The (A) As it is now. 7 1234567890123456789012345678901212345678901234567890123456789012123456 34567890123456789012345678901212345678901234567890123456789012123456 7 12 sun had set and the night was starless. (4) One (B) One of the literacy program I was 34567890123456789012345678901212345678901234567890123456789012123456 7 12 34567890123456789012345678901212345678901234567890123456789012123456 12 of the officials from the literacy program I was working’s officials was there to meet 7 34567890123456789012345678901212345678901234567890123456789012123456 7 12 2 34567890123456789012345678901212345678901234567890123456789012123456 7 1 working was there to meet me. (5) After the me. 34567890123456789012345678901212345678901234567890123456789012123456 7 12 34567890123456789012345678901212345678901234567890123456789012123456 7 12 melee in the baggage claim, we proceeded to his (C) There, was one of the officials from 34567890123456789012345678901212345678901234567890123456789012123456 7 12 34567890123456789012345678901212345678901234567890123456789012123456 7 12 car. (6) Actually, it was a truck. (7) I was soon the literacy program I was working 7 1234567890123456789012345678901212345678901234567890123456789012123456 34567890123456789012345678901212345678901234567890123456789012123456 7 12 to learn that most people in Mali that had to meet me. 34567890123456789012345678901212345678901234567890123456789012123456 7 12 34567890123456789012345678901212345678901234567890123456789012123456 automobiles actually had trucks or SUVs. (8) 12 (D) One of the officials from the literacy 7 34567890123456789012345678901212345678901234567890123456789012123456 7 12 Apparently, there not just a convenience but a program where I worked had been 2 34567890123456789012345678901212345678901234567890123456789012123456 7 1234567890123456789012345678901212345678901234567890123456789012123456 7 1234567890123456789012345678901212345678901234567890123456789012123456 necessity when you live on the edge of the there to meet me. 7 1234567890123456789012345678901212345678901234567890123456789012123456 7 1234567890123456789012345678901212345678901234567890123456789012123456 Sahara. (9) I threw my bags into the bed of the (E) One of the officials from the literacy 1234567890123456789012345678901212345678901234567890123456789012123456 77 1234567890123456789012345678901212345678901234567890123456789012123456 program where I would be working 7 1 truck, and hopped in to the back of the cab. 34567890123456789012345678901212345678901234567890123456789012123456 7 12 (10) Riding to my welcome dinner, I stared out was there to meet me. 34567890123456789012345678901212345678901234567890123456789012123456 7 12 34567890123456789012345678901212345678901234567890123456789012123456 7 12 the windows of the truck and took in the city. 34567890123456789012345678901212345678901234567890123456789012123456 7 12 34567890123456789012345678901212345678901234567890123456789012123456 7 12 27. Which of the following is the best way to (11) It was truly a foreign land to me, and I 34567890123456789012345678901212345678901234567890123456789012123456 7 12 34567890123456789012345678901212345678901234567890123456789012123456 7 12 revise sentence 7 (reproduced below)? knew that I was an alien there. (12) “What am 34567890123456789012345678901212345678901234567890123456789012123456 7 12 2 34567890123456789012345678901212345678901234567890123456789012123456 7 1234567890123456789012345678901212345678901234567890123456789012123456 I was soon to learn that most people in I doing here?” I thought. 7 1234567890123456789012345678901212345678901234567890123456789012123456 7 1234567890123456789012345678901212345678901234567890123456789012123456 Mali that had automobiles actually had (13) It is hard to believe but seven months 1234567890123456789012345678901212345678901234567890123456789012123456 77 1 later I returned to the same airport along the trucks or SUVs. 34567890123456789012345678901212345678901234567890123456789012123456 7 12 34567890123456789012345678901212345678901234567890123456789012123456 7 12 same road that I had traveled on that first night (A) Change “I was soon to learn” to “I 7 1234567890123456789012345678901212345678901234567890123456789012123456 7 1234567890123456789012345678901212345678901234567890123456789012123456 in Bamako, and my perspective on the things was soon learning” 34567890123456789012345678901212345678901234567890123456789012123456 7 12 2 34567890123456789012345678901212345678901234567890123456789012123456 7 1234567890123456789012345678901212345678901234567890123456789012123456 that I saw had completely changed. (14) The (B) Change “that had automobiles” to 7 1234567890123456789012345678901212345678901234567890123456789012123456 7 1 landscape that had once seemed so desolate and “who had automobiles” 34567890123456789012345678901212345678901234567890123456789012123456 7 12 34567890123456789012345678901212345678901234567890123456789012123456 7 12 lifeless now was the homeland of people that I (C) Replace “or” with “and” 34567890123456789012345678901212345678901234567890123456789012123456 7 12 had come to love. (15) When I looked back at 34567890123456789012345678901212345678901234567890123456789012123456 7 12 (D) Add commas after “Mali” and 7 1234567890123456789012345678901212345678901234567890123456789012123456 the capital, Bamako, fast receding on the 7 1234567890123456789012345678901212345678901234567890123456789012123456 “automobiles” 7 1234567890123456789012345678901212345678901234567890123456789012123456 horizon, I did not see a city foreboding and 2 34567890123456789012345678901212345678901234567890123456789012123456 1234567890123456789012345678901212345678901234567890123456789012123456 (E) Add an apostrophe to make “SUVs” 7 7 1234567890123456789012345678901212345678901234567890123456789012123456 wild in its foreignness. (16) I saw the city which 7 1234567890123456789012345678901212345678901234567890123456789012123456 read “SUV’s“ 7 1234567890123456789012345678901212345678901234567890123456789012123456 held so many dear friends. (17) I saw tea7 1 2 34567890123456789012345678901212345678901234567890123456789012123456 7 1234567890123456789012345678901212345678901234567890123456789012123456 drinking sessions going late into the night. 7 1234567890123456789012345678901212345678901234567890123456789012123456 7 1234567890123456789012345678901212345678901234567890123456789012123456 (18) I saw the hospitality and open-heartedness 1234567890123456789012345678901212345678901234567890123456789012123456 77 1 of the people of Mali. (19) The second time, 34567890123456789012345678901212345678901234567890123456789012123456 7 12 34567890123456789012345678901212345678901234567890123456789012123456 7 12 everything looked completely different, and I 34567890123456789012345678901212345678901234567890123456789012123456 7 12 34567890123456789012345678901212345678901234567890123456789012123456 7 12 knew that it was I who had changed and not it. 1234567890123456789012345678901212345678901234567890123456789012123456 7 2 34567890123456789012345678901212345678901234567890123456789012123456 1234567890123456789012345678901212345678901234567890123456789012123456 77 1 34567890123456789012345678901212345678901234567890123456789012123456 7 12 1234567890123456789012345678901212345678901234567890123456789012123456 7 34567890123456789012345678901212345678901234567890123456789012123456 7 12 1234567890123456789012345678901212345678901234567890123456789012123456 7 1234567890123456789012345678901212345678901234567890123456789012123456 7 1234567890123456789012345678901212345678901234567890123456789012123456 7 1234567890123456789012345678901212345678901234567890123456789012123456 7 12345678901234567890123456789012123456789012345678901234567890121234567 22

Copyright © 2005 Thomson Peterson’s, a part of The Thomson Corporaton SAT is a registered trademark of the College Entrance Examination Board, which was not involved in the production of and does not endorse this product.

12345678901234567890123456789012123456789012345678901234567890121234567 1234567890123456789012345678901212345678901234567890123456789012123456 7 7 12234567890123456789012345678901212345678901234567890123456789012123456 28. Sentence 13 (reproduced below) would 7 29. If you were to combine sentences 16–18 1 34567890123456789012345678901212345678901234567890123456789012123456 7 12234567890123456789012345678901212345678901234567890123456789012123456 best be revised to which of the following (reproduced below) into one sentence, 7 1234567890123456789012345678901212345678901234567890123456789012123456 7 1 34567890123456789012345678901212345678901234567890123456789012123456 choices? which of the following would be the best 7 12234567890123456789012345678901212345678901234567890123456789012123456 34567890123456789012345678901212345678901234567890123456789012123456 7 1 choice? 7 12234567890123456789012345678901212345678901234567890123456789012123456 It is hard to believe but seven months later 7 1234567890123456789012345678901212345678901234567890123456789012123456 7 1234567890123456789012345678901212345678901234567890123456789012123456 I returned to the same airport along the I saw the city which held so many dear 7 1234567890123456789012345678901212345678901234567890123456789012123456 34567890123456789012345678901212345678901234567890123456789012123456 1 same road that I had traveled on that first friends. I saw tea-drinking sessions going 7 7 12234567890123456789012345678901212345678901234567890123456789012123456 7 1234567890123456789012345678901212345678901234567890123456789012123456 night in Bamako, and my perspective on late into the night. I saw the hospitality 7 1234567890123456789012345678901212345678901234567890123456789012123456 7 1234567890123456789012345678901212345678901234567890123456789012123456 the things that I saw had completely and open-heartedness of the people of 34567890123456789012345678901212345678901234567890123456789012123456 7 1 7 12234567890123456789012345678901212345678901234567890123456789012123456 changed. Mali. 7 1234567890123456789012345678901212345678901234567890123456789012123456 7 1234567890123456789012345678901212345678901234567890123456789012123456 7 1234567890123456789012345678901212345678901234567890123456789012123456 (A) As it is now. (A) I saw the city which held so many 34567890123456789012345678901212345678901234567890123456789012123456 7 1 7 12234567890123456789012345678901212345678901234567890123456789012123456 (B) It is hard to believe, but seven dear friends; I saw tea-drinking 7 1234567890123456789012345678901212345678901234567890123456789012123456 7 months later I returned to the same 1234567890123456789012345678901212345678901234567890123456789012123456 sessions going late into the night; I 7 1234567890123456789012345678901212345678901234567890123456789012123456 airport along the same road that I saw the hospitality and open34567890123456789012345678901212345678901234567890123456789012123456 1234567890123456789012345678901212345678901234567890123456789012123456 77 1234567890123456789012345678901212345678901234567890123456789012123456 had traveled on that first night in heartedness of the people of Mali. 7 1234567890123456789012345678901212345678901234567890123456789012123456 7 1234567890123456789012345678901212345678901234567890123456789012123456 Bamako: my perspective on the (B) I saw the city which held so many 1234567890123456789012345678901212345678901234567890123456789012123456 77 1234567890123456789012345678901212345678901234567890123456789012123456 things I saw had completely changed. dear friends, drinking tea into late in 7 1 7 12234567890123456789012345678901212345678901234567890123456789012123456 (C) It is hard to believe but seven months the night, and the hospitality and 7 1234567890123456789012345678901212345678901234567890123456789012123456 7 1234567890123456789012345678901212345678901234567890123456789012123456 later I returned to the same airport open-heartedness of the people of 34567890123456789012345678901212345678901234567890123456789012123456 1234567890123456789012345678901212345678901234567890123456789012123456 77 1234567890123456789012345678901212345678901234567890123456789012123456 along the same road that I had Mali. 7 1234567890123456789012345678901212345678901234567890123456789012123456 7 1234567890123456789012345678901212345678901234567890123456789012123456 traveled on that first night in (C) I saw the city which held so many 1234567890123456789012345678901212345678901234567890123456789012123456 77 1234567890123456789012345678901212345678901234567890123456789012123456 Bamako, and my perspective dear friends, I saw tea-drinking 7 1234567890123456789012345678901212345678901234567890123456789012123456 7 1234567890123456789012345678901212345678901234567890123456789012123456 completely changed on the things I sessions going late into the night, I 1234567890123456789012345678901212345678901234567890123456789012123456 77 1 saw. saw the hospitality and open7 12234567890123456789012345678901212345678901234567890123456789012123456 7 1234567890123456789012345678901212345678901234567890123456789012123456 (D) It is hard to believe, but seven heartedness of the people of Mali. 1 34567890123456789012345678901212345678901234567890123456789012123456 7 7 1234567890123456789012345678901212345678901234567890123456789012123456 months later, when I returned to the (D) I saw the city which held so many 7 12234567890123456789012345678901212345678901234567890123456789012123456 34567890123456789012345678901212345678901234567890123456789012123456 7 1234567890123456789012345678901212345678901234567890123456789012123456 same airport along the same road dear friends, tea-drinking sessions 7 1234567890123456789012345678901212345678901234567890123456789012123456 7 1 that I had traveled on that first night going late into the night, the hospi7 12234567890123456789012345678901212345678901234567890123456789012123456 7 1234567890123456789012345678901212345678901234567890123456789012123456 in Bamako, my perspective on the tality and open-heartedness of the 34567890123456789012345678901212345678901234567890123456789012123456 7 1234567890123456789012345678901212345678901234567890123456789012123456 things I saw had completely changed. 7 people of Mali. 1234567890123456789012345678901212345678901234567890123456789012123456 7 1 (E) It is hard to believe, but seven (E) I saw the city which held so many 7 1234567890123456789012345678901212345678901234567890123456789012123456 7 1234567890123456789012345678901212345678901234567890123456789012123456 months later I returned to the same dear friends: tea-drinking sessions 2 34567890123456789012345678901212345678901234567890123456789012123456 1234567890123456789012345678901212345678901234567890123456789012123456 77 1234567890123456789012345678901212345678901234567890123456789012123456 airport along the same road that I going late into the night, the hospi7 1234567890123456789012345678901212345678901234567890123456789012123456 7 1234567890123456789012345678901212345678901234567890123456789012123456 had traveled on that first night in tality and open-heartedness of the 7 1 2 34567890123456789012345678901212345678901234567890123456789012123456 7 1234567890123456789012345678901212345678901234567890123456789012123456 Bamako, and my perspective on the people of Mali. 7 1234567890123456789012345678901212345678901234567890123456789012123456 7 1234567890123456789012345678901212345678901234567890123456789012123456 things that I saw having completely 1234567890123456789012345678901212345678901234567890123456789012123456 77 1 changed. 7 12234567890123456789012345678901212345678901234567890123456789012123456 7 1234567890123456789012345678901212345678901234567890123456789012123456 7 1234567890123456789012345678901212345678901234567890123456789012123456 7 1234567890123456789012345678901212345678901234567890123456789012123456 1 34567890123456789012345678901212345678901234567890123456789012123456 7 2 34567890123456789012345678901212345678901234567890123456789012123456 1234567890123456789012345678901212345678901234567890123456789012123456 77 1 7 12234567890123456789012345678901212345678901234567890123456789012123456 1 34567890123456789012345678901212345678901234567890123456789012123456 7 7 12234567890123456789012345678901212345678901234567890123456789012123456 1 34567890123456789012345678901212345678901234567890123456789012123456 7 1234567890123456789012345678901212345678901234567890123456789012123456 7 1234567890123456789012345678901212345678901234567890123456789012123456 7 1234567890123456789012345678901212345678901234567890123456789012123456 7 12345678901234567890123456789012123456789012345678901234567890121234567 Copyright © 2005 Thomson Peterson’s, a part of The Thomson Corporaton SAT is a registered trademark of the College Entrance Examination Board, which was not involved in the production of and does not endorse this product.

23

12345678901234567890123456789012123456789012345678901234567890121234567 1234567890123456789012345678901212345678901234567890123456789012123456 7 34567890123456789012345678901212345678901234567890123456789012123456 7 12 30. Which of the following must be done to 7 1234567890123456789012345678901212345678901234567890123456789012123456 34567890123456789012345678901212345678901234567890123456789012123456 7 12 sentence 8 (reproduced below) to make it 34567890123456789012345678901212345678901234567890123456789012123456 7 12 7 1234567890123456789012345678901212345678901234567890123456789012123456 conform to the rules of written English? 34567890123456789012345678901212345678901234567890123456789012123456 7 12 2 34567890123456789012345678901212345678901234567890123456789012123456 7 1 34567890123456789012345678901212345678901234567890123456789012123456 7 12 Apparently, there not just a convenience 34567890123456789012345678901212345678901234567890123456789012123456 7 12 34567890123456789012345678901212345678901234567890123456789012123456 7 12 but a necessity when you live on the edge 34567890123456789012345678901212345678901234567890123456789012123456 7 12 2 34567890123456789012345678901212345678901234567890123456789012123456 7 1 of the Sahara. 34567890123456789012345678901212345678901234567890123456789012123456 7 12 34567890123456789012345678901212345678901234567890123456789012123456 7 12 34567890123456789012345678901212345678901234567890123456789012123456 7 12 (A) Eliminate the comma after “Appar34567890123456789012345678901212345678901234567890123456789012123456 7 12 2 34567890123456789012345678901212345678901234567890123456789012123456 7 1 ently” 34567890123456789012345678901212345678901234567890123456789012123456 7 12 34567890123456789012345678901212345678901234567890123456789012123456 7 12 (B) Change “there” to “they are” 34567890123456789012345678901212345678901234567890123456789012123456 7 12 34567890123456789012345678901212345678901234567890123456789012123456 7 12 (C) Add commas after “convenience” 7 1234567890123456789012345678901212345678901234567890123456789012123456 34567890123456789012345678901212345678901234567890123456789012123456 7 12 and “necessity” 34567890123456789012345678901212345678901234567890123456789012123456 7 12 34567890123456789012345678901212345678901234567890123456789012123456 7 (D) Change “you live” to “one lives” 12 34567890123456789012345678901212345678901234567890123456789012123456 7 12 (E) Add “Desert” after “Sahara“ 2 34567890123456789012345678901212345678901234567890123456789012123456 1234567890123456789012345678901212345678901234567890123456789012123456 77 1234567890123456789012345678901212345678901234567890123456789012123456 7 1234567890123456789012345678901212345678901234567890123456789012123456 7 1234567890123456789012345678901212345678901234567890123456789012123456 7 1234567890123456789012345678901212345678901234567890123456789012123456 7 1234567890123456789012345678901212345678901234567890123456789012123456 7 1 34567890123456789012345678901212345678901234567890123456789012123456 7 12 34567890123456789012345678901212345678901234567890123456789012123456 7 12 34567890123456789012345678901212345678901234567890123456789012123456 7 12 2 7 1234567890123456789012345678901212345678901234567890123456789012123456 7 1234567890123456789012345678901212345678901234567890123456789012123456 7 1234567890123456789012345678901212345678901234567890123456789012123456 7 1234567890123456789012345678901212345678901234567890123456789012123456 7 1234567890123456789012345678901212345678901234567890123456789012123456 34567890123456789012345678901212345678901234567890123456789012123456 1234567890123456789012345678901212345678901234567890123456789012123456 77 1234567890123456789012345678901212345678901234567890123456789012123456 7 1234567890123456789012345678901212345678901234567890123456789012123456 7 1234567890123456789012345678901212345678901234567890123456789012123456 7 1 34567890123456789012345678901212345678901234567890123456789012123456 7 12 34567890123456789012345678901212345678901234567890123456789012123456 7 12 1234567890123456789012345678901212345678901234567890123456789012123456 7 1234567890123456789012345678901212345678901234567890123456789012123456 7 34567890123456789012345678901212345678901234567890123456789012123456 7 12 34567890123456789012345678901212345678901234567890123456789012123456 7 12 34567890123456789012345678901212345678901234567890123456789012123456 7 12 1234567890123456789012345678901212345678901234567890123456789012123456 7 34567890123456789012345678901212345678901234567890123456789012123456 7 12 34567890123456789012345678901212345678901234567890123456789012123456 7 12 2 7 1234567890123456789012345678901212345678901234567890123456789012123456 7 1234567890123456789012345678901212345678901234567890123456789012123456 1 34567890123456789012345678901212345678901234567890123456789012123456 7 1234567890123456789012345678901212345678901234567890123456789012123456 7 1234567890123456789012345678901212345678901234567890123456789012123456 7 34567890123456789012345678901212345678901234567890123456789012123456 7 12 34567890123456789012345678901212345678901234567890123456789012123456 7 12 34567890123456789012345678901212345678901234567890123456789012123456 7 12 34567890123456789012345678901212345678901234567890123456789012123456 7 12 1234567890123456789012345678901212345678901234567890123456789012123456 7 2 7 1234567890123456789012345678901212345678901234567890123456789012123456 7 1234567890123456789012345678901212345678901234567890123456789012123456 7 1234567890123456789012345678901212345678901234567890123456789012123456 7 1234567890123456789012345678901212345678901234567890123456789012123456 1 34567890123456789012345678901212345678901234567890123456789012123456 7 2 34567890123456789012345678901212345678901234567890123456789012123456 1234567890123456789012345678901212345678901234567890123456789012123456 77 1234567890123456789012345678901212345678901234567890123456789012123456 7 1234567890123456789012345678901212345678901234567890123456789012123456 7 1234567890123456789012345678901212345678901234567890123456789012123456 7 1 34567890123456789012345678901212345678901234567890123456789012123456 7 12 1234567890123456789012345678901212345678901234567890123456789012123456 7 34567890123456789012345678901212345678901234567890123456789012123456 7 12 1234567890123456789012345678901212345678901234567890123456789012123456 7 34567890123456789012345678901212345678901234567890123456789012123456 7 12 Do not proceed to the next section until time is up. 7 1234567890123456789012345678901212345678901234567890123456789012123456 1234567890123456789012345678901212345678901234567890123456789012123456 7 1234567890123456789012345678901212345678901234567890123456789012123456 7 1234567890123456789012345678901212345678901234567890123456789012123456 7 12345678901234567890123456789012123456789012345678901234567890121234567

STOP

24

Copyright © 2005 Thomson Peterson’s, a part of The Thomson Corporaton SAT is a registered trademark of the College Entrance Examination Board, which was not involved in the production of and does not endorse this product.

Section 4 27 Questions j Time—25 Minutes 12345678901234567890123456789012123456789012345678901234567890121234567 7 12234567890123456789012345678901212345678901234567890123456789012123456 7 1234567890123456789012345678901212345678901234567890123456789012123456 34567890123456789012345678901212345678901234567890123456789012123456 7 1 7 12234567890123456789012345678901212345678901234567890123456789012123456 Directions: Each sentence below has either one or two blanks in it and is followed by five 7 1234567890123456789012345678901212345678901234567890123456789012123456 7 1234567890123456789012345678901212345678901234567890123456789012123456 choices, labeled (A) through (E). These choices represent words or phrases that have been left 7 1234567890123456789012345678901212345678901234567890123456789012123456 34567890123456789012345678901212345678901234567890123456789012123456 7 1234567890123456789012345678901212345678901234567890123456789012123456 out. Choose the word or phrase that, if inserted into the sentence, would best fit the meaning of 7 1234567890123456789012345678901212345678901234567890123456789012123456 7 1234567890123456789012345678901212345678901234567890123456789012123456 the sentence as a whole. 1234567890123456789012345678901212345678901234567890123456789012123456 77 1234567890123456789012345678901212345678901234567890123456789012123456 Example: 7 1234567890123456789012345678901212345678901234567890123456789012123456 7 1 7 12234567890123456789012345678901212345678901234567890123456789012123456 Canine massage is a veterinary technique 7 1234567890123456789012345678901212345678901234567890123456789012123456 7 1234567890123456789012345678901212345678901234567890123456789012123456 for calming dogs that are extremely __________. 34567890123456789012345678901212345678901234567890123456789012123456 1234567890123456789012345678901212345678901234567890123456789012123456 77 1234567890123456789012345678901212345678901234567890123456789012123456 7 1234567890123456789012345678901212345678901234567890123456789012123456 (A) inept 7 1234567890123456789012345678901212345678901234567890123456789012123456 7 1234567890123456789012345678901212345678901234567890123456789012123456 (B) disciplined 1234567890123456789012345678901212345678901234567890123456789012123456 77 1234567890123456789012345678901212345678901234567890123456789012123456 (C) controlled 7 1234567890123456789012345678901212345678901234567890123456789012123456 7 1234567890123456789012345678901212345678901234567890123456789012123456 (D) stressed 7 1 2 34567890123456789012345678901212345678901234567890123456789012123456 7 1234567890123456789012345678901212345678901234567890123456789012123456 (E) restrained 7 1234567890123456789012345678901212345678901234567890123456789012123456 7 1 A B C E O Þ Þ O Þ O Þ O Þ 7 1234567890123456789012345678901212345678901234567890123456789012123456 7 12234567890123456789012345678901212345678901234567890123456789012123456 34567890123456789012345678901212345678901234567890123456789012123456 1234567890123456789012345678901212345678901234567890123456789012123456 77 1234567890123456789012345678901212345678901234567890123456789012123456 7 1 7 12234567890123456789012345678901212345678901234567890123456789012123456 1. The professor’s oldest colleague was 2. The new team member’s __________ was 7 1234567890123456789012345678901212345678901234567890123456789012123456 34567890123456789012345678901212345678901234567890123456789012123456 7 1234567890123456789012345678901212345678901234567890123456789012123456 selected to give the __________ at the an encouragement to the rest of the team, 7 1234567890123456789012345678901212345678901234567890123456789012123456 7 1 funeral. who had become __________ by the string 7 1234567890123456789012345678901212345678901234567890123456789012123456 7 1234567890123456789012345678901212345678901234567890123456789012123456 of defeats. 7 12234567890123456789012345678901212345678901234567890123456789012123456 (A) eulogy 7 1234567890123456789012345678901212345678901234567890123456789012123456 7 1234567890123456789012345678901212345678901234567890123456789012123456 (B) elegy (A) enthusiasm. .elated 7 1234567890123456789012345678901212345678901234567890123456789012123456 7 1 34567890123456789012345678901212345678901234567890123456789012123456 (C) epigraph (B) vigor. .inundated 2 34567890123456789012345678901212345678901234567890123456789012123456 1234567890123456789012345678901212345678901234567890123456789012123456 77 1234567890123456789012345678901212345678901234567890123456789012123456 (D) eponymy (C) ebullience. .dispirited 7 1234567890123456789012345678901212345678901234567890123456789012123456 7 1234567890123456789012345678901212345678901234567890123456789012123456 (E) epitaph (D) dourness. .undone 7 1 2 34567890123456789012345678901212345678901234567890123456789012123456 7 1234567890123456789012345678901212345678901234567890123456789012123456 (E) excessiveness. .downcast 7 1234567890123456789012345678901212345678901234567890123456789012123456 1234567890123456789012345678901212345678901234567890123456789012123456 77 1234567890123456789012345678901212345678901234567890123456789012123456 7 1 7 12234567890123456789012345678901212345678901234567890123456789012123456 1 34567890123456789012345678901212345678901234567890123456789012123456 7 7 12234567890123456789012345678901212345678901234567890123456789012123456 1 34567890123456789012345678901212345678901234567890123456789012123456 7 7 12234567890123456789012345678901212345678901234567890123456789012123456 1 34567890123456789012345678901212345678901234567890123456789012123456 7 1234567890123456789012345678901212345678901234567890123456789012123456 7 1234567890123456789012345678901212345678901234567890123456789012123456 7 1234567890123456789012345678901212345678901234567890123456789012123456 7 12345678901234567890123456789012123456789012345678901234567890121234567

12345678901234567890123456789012123456789012345678901234567890121234567 1234567890123456789012345678901212345678901234567890123456789012123456 7 34567890123456789012345678901212345678901234567890123456789012123456 7 12 3. By the end of the campaign both candi7 7. The outcome of the race seemed 1234567890123456789012345678901212345678901234567890123456789012123456 34567890123456789012345678901212345678901234567890123456789012123456 7 12 dates had resorted to __________ the __________ before the leader’s misstep on 7 34567890123456789012345678901212345678901234567890123456789012123456 12 7 1234567890123456789012345678901212345678901234567890123456789012123456 other. the final leg gave her competitors a(n) 34567890123456789012345678901212345678901234567890123456789012123456 7 12 2 34567890123456789012345678901212345678901234567890123456789012123456 7 1 __________ of winning the title. 34567890123456789012345678901212345678901234567890123456789012123456 7 12 (A) commending 34567890123456789012345678901212345678901234567890123456789012123456 7 12 34567890123456789012345678901212345678901234567890123456789012123456 7 12 (B) denigrating (A) dubious. .prospect 34567890123456789012345678901212345678901234567890123456789012123456 7 12 2 34567890123456789012345678901212345678901234567890123456789012123456 7 1 (C) mollifying (B) inevitable. .hope 34567890123456789012345678901212345678901234567890123456789012123456 7 12 34567890123456789012345678901212345678901234567890123456789012123456 7 12 (D) conceding (C) indubitable. .air 34567890123456789012345678901212345678901234567890123456789012123456 7 12 34567890123456789012345678901212345678901234567890123456789012123456 7 12 (E) swindling (D) assured. .expectation 7 1234567890123456789012345678901212345678901234567890123456789012123456 34567890123456789012345678901212345678901234567890123456789012123456 7 12 (E) partial. .endeavor 34567890123456789012345678901212345678901234567890123456789012123456 7 12 34567890123456789012345678901212345678901234567890123456789012123456 7 4. The cat __________ crept across the lawn, 12 34567890123456789012345678901212345678901234567890123456789012123456 7 12 gracefully __________ the dog. 8. Though the new pharmaceutical regime 2 34567890123456789012345678901212345678901234567890123456789012123456 7 1 34567890123456789012345678901212345678901234567890123456789012123456 7 12 was intended to be beneficial, its actual 34567890123456789012345678901212345678901234567890123456789012123456 7 12 (A) felicitously. .enticing 34567890123456789012345678901212345678901234567890123456789012123456 7 12 effect was __________, a result the 34567890123456789012345678901212345678901234567890123456789012123456 7 12 (B) swiftly. .defeating 2 34567890123456789012345678901212345678901234567890123456789012123456 7 1234567890123456789012345678901212345678901234567890123456789012123456 medical community __________. 7 1234567890123456789012345678901212345678901234567890123456789012123456 (C) acrobatically. .apprehending 1234567890123456789012345678901212345678901234567890123456789012123456 77 1234567890123456789012345678901212345678901234567890123456789012123456 (D) maladroitly. .undermining (A) harmful. .heralded 7 1234567890123456789012345678901212345678901234567890123456789012123456 7 1234567890123456789012345678901212345678901234567890123456789012123456 (E) deftly. .eluding (B) abundant. .castigated 7 1 34567890123456789012345678901212345678901234567890123456789012123456 7 12 (C) fortuitous. .ignored 34567890123456789012345678901212345678901234567890123456789012123456 7 12 34567890123456789012345678901212345678901234567890123456789012123456 7 12 5. The storyteller’s __________ anecdotes (D) detrimental. .lamented 34567890123456789012345678901212345678901234567890123456789012123456 7 12 34567890123456789012345678901212345678901234567890123456789012123456 7 12 earned her the __________ attention of (E) negative. .projected 34567890123456789012345678901212345678901234567890123456789012123456 7 12 the crowd. 34567890123456789012345678901212345678901234567890123456789012123456 7 12 34567890123456789012345678901212345678901234567890123456789012123456 7 12 9. The life of the lightening bug is 2 34567890123456789012345678901212345678901234567890123456789012123456 7 1234567890123456789012345678901212345678901234567890123456789012123456 (A) compelling. .rapt 7 1234567890123456789012345678901212345678901234567890123456789012123456 __________ to human eyes: They live only 7 1234567890123456789012345678901212345678901234567890123456789012123456 (B) pointed. .spellbound 7 1234567890123456789012345678901212345678901234567890123456789012123456 twenty-four hours. 7 1 (C) moribund. .lucid 2 34567890123456789012345678901212345678901234567890123456789012123456 1234567890123456789012345678901212345678901234567890123456789012123456 77 1234567890123456789012345678901212345678901234567890123456789012123456 (D) poignant. .abrasive (A) ludicrous 7 1 7 1234567890123456789012345678901212345678901234567890123456789012123456 (E) meandering. .distracted (B) ephemeral 34567890123456789012345678901212345678901234567890123456789012123456 7 12 2 34567890123456789012345678901212345678901234567890123456789012123456 7 1234567890123456789012345678901212345678901234567890123456789012123456 (C) epic 7 1234567890123456789012345678901212345678901234567890123456789012123456 7 1 6. The bill became bogged down in a(n) (D) ecstatic 34567890123456789012345678901212345678901234567890123456789012123456 7 12 34567890123456789012345678901212345678901234567890123456789012123456 7 12 __________ of contentious issues in a (E) incandescent 34567890123456789012345678901212345678901234567890123456789012123456 7 12 Senate subcommittee. 34567890123456789012345678901212345678901234567890123456789012123456 7 12 7 1234567890123456789012345678901212345678901234567890123456789012123456 10. The kangaroo species __________ in the 7 1234567890123456789012345678901212345678901234567890123456789012123456 (A) marsh 7 1234567890123456789012345678901212345678901234567890123456789012123456 new environment where there was an 2 34567890123456789012345678901212345678901234567890123456789012123456 7 1234567890123456789012345678901212345678901234567890123456789012123456 (B) sequence 7 1234567890123456789012345678901212345678901234567890123456789012123456 abundant supply of food and a(n) 7 1234567890123456789012345678901212345678901234567890123456789012123456 (C) iota 7 1234567890123456789012345678901212345678901234567890123456789012123456 __________ of predators. 7 1 (D) conundrum 2 34567890123456789012345678901212345678901234567890123456789012123456 1234567890123456789012345678901212345678901234567890123456789012123456 77 1234567890123456789012345678901212345678901234567890123456789012123456 (E) quagmire (A) stagnated. .excess 7 1234567890123456789012345678901212345678901234567890123456789012123456 7 1234567890123456789012345678901212345678901234567890123456789012123456 (B) bolstered. .paucity 7 1 2 34567890123456789012345678901212345678901234567890123456789012123456 7 1234567890123456789012345678901212345678901234567890123456789012123456 (C) exploded. .abundance 7 1234567890123456789012345678901212345678901234567890123456789012123456 7 1234567890123456789012345678901212345678901234567890123456789012123456 (D) flagged. .absence 1234567890123456789012345678901212345678901234567890123456789012123456 77 1 (E) flourished. .dearth 34567890123456789012345678901212345678901234567890123456789012123456 7 12 1234567890123456789012345678901212345678901234567890123456789012123456 7 34567890123456789012345678901212345678901234567890123456789012123456 7 12 1234567890123456789012345678901212345678901234567890123456789012123456 7 34567890123456789012345678901212345678901234567890123456789012123456 7 12 1234567890123456789012345678901212345678901234567890123456789012123456 7 1234567890123456789012345678901212345678901234567890123456789012123456 7 1234567890123456789012345678901212345678901234567890123456789012123456 7 1234567890123456789012345678901212345678901234567890123456789012123456 7 12345678901234567890123456789012123456789012345678901234567890121234567 26

Copyright © 2005 Thomson Peterson’s, a part of The Thomson Corporaton SAT is a registered trademark of the College Entrance Examination Board, which was not involved in the production of and does not endorse this product.

12345678901234567890123456789012123456789012345678901234567890121234567 1234567890123456789012345678901212345678901234567890123456789012123456 7 7 12234567890123456789012345678901212345678901234567890123456789012123456 11. With her speech, the politician attempted 7 15. The senior official __________ at the 1 34567890123456789012345678901212345678901234567890123456789012123456 7 12234567890123456789012345678901212345678901234567890123456789012123456 to __________ the fears of the __________ insinuation that his country’s international 7 1234567890123456789012345678901212345678901234567890123456789012123456 7 1 34567890123456789012345678901212345678901234567890123456789012123456 citizens. trade policies were directly __________ the 7 12234567890123456789012345678901212345678901234567890123456789012123456 34567890123456789012345678901212345678901234567890123456789012123456 7 1 region’s economic woes. 7 12234567890123456789012345678901212345678901234567890123456789012123456 (A) intensify. .disingenuous 7 1234567890123456789012345678901212345678901234567890123456789012123456 7 1234567890123456789012345678901212345678901234567890123456789012123456 (B) ignore. .alarmed (A) balked. .responsible for 7 1234567890123456789012345678901212345678901234567890123456789012123456 34567890123456789012345678901212345678901234567890123456789012123456 7 1 (C) assuage. .concerned (B) wrinkled. .at fault for 7 12234567890123456789012345678901212345678901234567890123456789012123456 7 1234567890123456789012345678901212345678901234567890123456789012123456 (D) quell. .disaffected (C) staggered. .inhibiting 7 1234567890123456789012345678901212345678901234567890123456789012123456 7 1234567890123456789012345678901212345678901234567890123456789012123456 (E) exploit. .serene (D) blundered. .implicated in 34567890123456789012345678901212345678901234567890123456789012123456 7 1 7 12234567890123456789012345678901212345678901234567890123456789012123456 (E) riled. .accountable to 7 1234567890123456789012345678901212345678901234567890123456789012123456 7 12. The fencing champion was __________ 1234567890123456789012345678901212345678901234567890123456789012123456 7 1234567890123456789012345678901212345678901234567890123456789012123456 with her rapier, but in most other sports 34567890123456789012345678901212345678901234567890123456789012123456 7 1 Questions 16–27 are based on the following 7 12234567890123456789012345678901212345678901234567890123456789012123456 she was rather __________. 7 1234567890123456789012345678901212345678901234567890123456789012123456 passage. 7 1234567890123456789012345678901212345678901234567890123456789012123456 7 1234567890123456789012345678901212345678901234567890123456789012123456 (A) adroit. .awkward 34567890123456789012345678901212345678901234567890123456789012123456 1234567890123456789012345678901212345678901234567890123456789012123456 The following passage was written by John 7 7 1234567890123456789012345678901212345678901234567890123456789012123456 (B) adept. .lithe 7 1234567890123456789012345678901212345678901234567890123456789012123456 Janovec, an ecologist who has worked in the Los 7 1234567890123456789012345678901212345678901234567890123456789012123456 (C) tenacious. .passable 7 1234567890123456789012345678901212345678901234567890123456789012123456 Amigos watershed in Peru. 7 1234567890123456789012345678901212345678901234567890123456789012123456 (D) incompetent. .clumsy 7 1 7 12234567890123456789012345678901212345678901234567890123456789012123456 Line The Amazonian wilderness harbors the (E) deft. .skillful 7 1234567890123456789012345678901212345678901234567890123456789012123456 1234567890123456789012345678901212345678901234567890123456789012123456 greatest number of species on this planet 7 34567890123456789012345678901212345678901234567890123456789012123456 1234567890123456789012345678901212345678901234567890123456789012123456 77 1234567890123456789012345678901212345678901234567890123456789012123456 and is an irreplaceable resource for 13. Jane Goodall was at first a(n) __________ 7 1234567890123456789012345678901212345678901234567890123456789012123456 present and future generations. Amazoin her field, but since then she has received 7 1234567890123456789012345678901212345678901234567890123456789012123456 7 1234567890123456789012345678901212345678901234567890123456789012123456 (5) nia is crucial for maintaining global many accolades for her work. 1234567890123456789012345678901212345678901234567890123456789012123456 77 1234567890123456789012345678901212345678901234567890123456789012123456 climate and genetic resources, and its 7 1234567890123456789012345678901212345678901234567890123456789012123456 (A) acolyte 7 1234567890123456789012345678901212345678901234567890123456789012123456 forest and rivers provide vital sources of 7 1 (B) maverick 2 34567890123456789012345678901212345678901234567890123456789012123456 7 1234567890123456789012345678901212345678901234567890123456789012123456 food, building materials, pharmaceuti7 1234567890123456789012345678901212345678901234567890123456789012123456 (C) luminary 7 1 cals, and water needed by wildlife and 7 1234567890123456789012345678901212345678901234567890123456789012123456 (D) charlatan 7 12234567890123456789012345678901212345678901234567890123456789012123456 (10) humanity. 34567890123456789012345678901212345678901234567890123456789012123456 7 1234567890123456789012345678901212345678901234567890123456789012123456 (E) miser 7 1234567890123456789012345678901212345678901234567890123456789012123456 The Los Amigos watershed in the 7 1 7 12234567890123456789012345678901212345678901234567890123456789012123456 state of Madre de Dios, southeastern 7 1234567890123456789012345678901212345678901234567890123456789012123456 14. Alston was impressed by the philosopher’s 34567890123456789012345678901212345678901234567890123456789012123456 7 1234567890123456789012345678901212345678901234567890123456789012123456 Peru, is representative of the pristine lecture, but Mario thought the lecture was 7 1234567890123456789012345678901212345678901234567890123456789012123456 7 1 lowland moist forest once found better characterized as __________ than as 7 1234567890123456789012345678901212345678901234567890123456789012123456 7 1234567890123456789012345678901212345678901234567890123456789012123456 (15) throughout most of upper Amazonian erudite. 2 34567890123456789012345678901212345678901234567890123456789012123456 1234567890123456789012345678901212345678901234567890123456789012123456 77 1234567890123456789012345678901212345678901234567890123456789012123456 South America. Threats to tropical 7 1234567890123456789012345678901212345678901234567890123456789012123456 (A) translucent 7 1234567890123456789012345678901212345678901234567890123456789012123456 forests occur in the form of fishing, 7 1 (B) recondite 2 34567890123456789012345678901212345678901234567890123456789012123456 1234567890123456789012345678901212345678901234567890123456789012123456 hunting, gold mining, timber extraction, 7 7 1234567890123456789012345678901212345678901234567890123456789012123456 (C) impeccable 7 1234567890123456789012345678901212345678901234567890123456789012123456 impending road construction, and 7 1234567890123456789012345678901212345678901234567890123456789012123456 (D) specious 7 (20) 1 slash-and-burn agriculture. The Los 2 34567890123456789012345678901212345678901234567890123456789012123456 7 1234567890123456789012345678901212345678901234567890123456789012123456 (E) fictitious Amigos watershed, consisting of 1.6 7 1234567890123456789012345678901212345678901234567890123456789012123456 7 1234567890123456789012345678901212345678901234567890123456789012123456 million hectares (3.95 million acres), still 7 1234567890123456789012345678901212345678901234567890123456789012123456 7 1 offers the increasingly scarce opportunity 7 12234567890123456789012345678901212345678901234567890123456789012123456 7 1 34567890123456789012345678901212345678901234567890123456789012123456 to study rainforest as it was before the 7 12234567890123456789012345678901212345678901234567890123456789012123456 7 1 34567890123456789012345678901212345678901234567890123456789012123456 (25) disruptive encroachment of modern 7 12234567890123456789012345678901212345678901234567890123456789012123456 34567890123456789012345678901212345678901234567890123456789012123456 7 1 human civilization. Because of its 7 1234567890123456789012345678901212345678901234567890123456789012123456 1234567890123456789012345678901212345678901234567890123456789012123456 77 1234567890123456789012345678901212345678901234567890123456789012123456 12345678901234567890123456789012123456789012345678901234567890121234567 Copyright © 2005 Thomson Peterson’s, a part of The Thomson Corporaton SAT is a registered trademark of the College Entrance Examination Board, which was not involved in the production of and does not endorse this product.

27

12345678901234567890123456789012123456789012345678901234567890121234567 1234567890123456789012345678901212345678901234567890123456789012123456 7 34567890123456789012345678901212345678901234567890123456789012123456 7 12 relatively pristine condition and the 7 Amigos area and the Madre de Dios 1234567890123456789012345678901212345678901234567890123456789012123456 34567890123456789012345678901212345678901234567890123456789012123456 7 12 immediate need to justify it as a conser(70) watershed in general. 34567890123456789012345678901212345678901234567890123456789012123456 7 12 7 1234567890123456789012345678901212345678901234567890123456789012123456 vation zone, this area deserves intensive, With support from the Amazon 34567890123456789012345678901212345678901234567890123456789012123456 7 12 2 34567890123456789012345678901212345678901234567890123456789012123456 7 1 (30) long-term projects aimed at botanical Conservation Association, and in 34567890123456789012345678901212345678901234567890123456789012123456 7 12 34567890123456789012345678901212345678901234567890123456789012123456 7 12 training, ecotourism, biological invencollaboration with U.S. and Peruvian 34567890123456789012345678901212345678901234567890123456789012123456 7 12 34567890123456789012345678901212345678901234567890123456789012123456 12 tory, and information synthesis. colleagues, the Botany of the Los Amigos 7 7 1234567890123456789012345678901212345678901234567890123456789012123456 34567890123456789012345678901212345678901234567890123456789012123456 7 12 (75) On July 24, 2001, the government of project has been initiated. At Los 34567890123456789012345678901212345678901234567890123456789012123456 7 12 34567890123456789012345678901212345678901234567890123456789012123456 7 12 Peru and the Amazon Conservation Amigos, we are attempting to develop a 34567890123456789012345678901212345678901234567890123456789012123456 7 12 2 34567890123456789012345678901212345678901234567890123456789012123456 7 1 (35) Association signed a contractual agreesystem of preservation, sustainability, 34567890123456789012345678901212345678901234567890123456789012123456 7 12 34567890123456789012345678901212345678901234567890123456789012123456 7 12 ment creating the first long-term permaand scientific research; a marriage 34567890123456789012345678901212345678901234567890123456789012123456 7 12 34567890123456789012345678901212345678901234567890123456789012123456 12 nently renewable conservation concesbetween various disciplines, from human 7 7 1234567890123456789012345678901212345678901234567890123456789012123456 34567890123456789012345678901212345678901234567890123456789012123456 7 12 (80) sion. To our knowledge this is the first ecology to economic botany, product 34567890123456789012345678901212345678901234567890123456789012123456 7 12 34567890123456789012345678901212345678901234567890123456789012123456 7 12 such agreement to be implemented in the marketing to forest management. The 34567890123456789012345678901212345678901234567890123456789012123456 7 12 2 34567890123456789012345678901212345678901234567890123456789012123456 7 1234567890123456789012345678901212345678901234567890123456789012123456 (40) world. The conservation concession complexity of the ecosystem will best be 7 1234567890123456789012345678901212345678901234567890123456789012123456 7 1234567890123456789012345678901212345678901234567890123456789012123456 protects 340,000 acres of old-growth understood through a multidisciplinary 1234567890123456789012345678901212345678901234567890123456789012123456 77 1234567890123456789012345678901212345678901234567890123456789012123456 Amazonian forest in the Los Amigos approach, and improved understanding 7 1234567890123456789012345678901212345678901234567890123456789012123456 7 1 (85) watershed, which is located in southeastof the complexity will lead to better 34567890123456789012345678901212345678901234567890123456789012123456 7 12 ern Peru. This watershed protects the management. The future of these forests 34567890123456789012345678901212345678901234567890123456789012123456 7 12 34567890123456789012345678901212345678901234567890123456789012123456 7 12 (45) eastern flank of Manu National Park and will depend on sustainable management 2 34567890123456789012345678901212345678901234567890123456789012123456 1234567890123456789012345678901212345678901234567890123456789012123456 77 1234567890123456789012345678901212345678901234567890123456789012123456 is part of the lowland forest corridor that and development of alternative practices 7 1234567890123456789012345678901212345678901234567890123456789012123456 7 1234567890123456789012345678901212345678901234567890123456789012123456 links it to Bahuaja-Sonene National Park. and products that do not require 1234567890123456789012345678901212345678901234567890123456789012123456 77 1234567890123456789012345678901212345678901234567890123456789012123456 (90) The Los Amigos conservation concession irreversible destruction. 7 1234567890123456789012345678901212345678901234567890123456789012123456 7 1234567890123456789012345678901212345678901234567890123456789012123456 will serve as a mechanism for the The botanical project will provide a 1234567890123456789012345678901212345678901234567890123456789012123456 77 1 (50) development of a regional center of foundation of information that is 34567890123456789012345678901212345678901234567890123456789012123456 7 12 34567890123456789012345678901212345678901234567890123456789012123456 7 12 excellence in natural forest management essential to other programs at Los 1234567890123456789012345678901212345678901234567890123456789012123456 7 7 1234567890123456789012345678901212345678901234567890123456789012123456 and biodiversity science. Amigos. By combining botanical studies 34567890123456789012345678901212345678901234567890123456789012123456 7 12 2 34567890123456789012345678901212345678901234567890123456789012123456 7 1234567890123456789012345678901212345678901234567890123456789012123456 (95) Several major projects are being with fisheries and mammology, we will 7 1234567890123456789012345678901212345678901234567890123456789012123456 7 1 implemented at the Los Amigos Conserbetter understand plant/animal interac34567890123456789012345678901212345678901234567890123456789012123456 7 12 34567890123456789012345678901212345678901234567890123456789012123456 7 12 (55) vation Area. Louise Emmons is initiating tions. By providing names, the botanical 34567890123456789012345678901212345678901234567890123456789012123456 7 12 34567890123456789012345678901212345678901234567890123456789012123456 7 12 studies of mammal diversity and ecology program will facilitate accurate commu1234567890123456789012345678901212345678901234567890123456789012123456 7 7 1234567890123456789012345678901212345678901234567890123456789012123456 in the Los Amigos area. Other projects nication about plants and the animals 7 1234567890123456789012345678901212345678901234567890123456789012123456 2 34567890123456789012345678901212345678901234567890123456789012123456 7 1234567890123456789012345678901212345678901234567890123456789012123456 (100) that use them. Included in this scenario involve studies of the diversity of 7 1234567890123456789012345678901212345678901234567890123456789012123456 7 1234567890123456789012345678901212345678901234567890123456789012123456 arthropods, amphibians, reptiles, and are humans, as we will dedicate time to 1234567890123456789012345678901212345678901234567890123456789012123456 77 1 (60) birds. Robin Foster has conducted people-plant interactions in order to 34567890123456789012345678901212345678901234567890123456789012123456 7 12 botanical studies at Los Amigos, resulting learn what plants are used by people in 34567890123456789012345678901212345678901234567890123456789012123456 7 12 34567890123456789012345678901212345678901234567890123456789012123456 7 12 in the labeling of hundreds of plant the Los Amigos area, and what plants 34567890123456789012345678901212345678901234567890123456789012123456 7 12 7 1234567890123456789012345678901212345678901234567890123456789012123456 (105) species along two kilometers of trail in could potentially be used by people. 2 34567890123456789012345678901212345678901234567890123456789012123456 1234567890123456789012345678901212345678901234567890123456789012123456 77 1234567890123456789012345678901212345678901234567890123456789012123456 upland and lowland forest. Michael To be informed, we must develop 7 1234567890123456789012345678901212345678901234567890123456789012123456 7 1234567890123456789012345678901212345678901234567890123456789012123456 (65) Goulding is leading a fisheries and knowledge. To develop knowledge, we 7 1 2 34567890123456789012345678901212345678901234567890123456789012123456 7 1234567890123456789012345678901212345678901234567890123456789012123456 aquatic ecology program, which aims to must collect, organize, and disseminate 7 1 34567890123456789012345678901212345678901234567890123456789012123456 7 12 document the diversity of fish, their information. In this sense, botanical 1234567890123456789012345678901212345678901234567890123456789012123456 7 34567890123456789012345678901212345678901234567890123456789012123456 7 12 (110) information has conservation value. ecologies, and their habitats in the Los 7 1234567890123456789012345678901212345678901234567890123456789012123456 7 1234567890123456789012345678901212345678901234567890123456789012123456 Before we can use plant-based products 1234567890123456789012345678901212345678901234567890123456789012123456 77 1234567890123456789012345678901212345678901234567890123456789012123456 12345678901234567890123456789012123456789012345678901234567890121234567 28

Copyright © 2005 Thomson Peterson’s, a part of The Thomson Corporaton SAT is a registered trademark of the College Entrance Examination Board, which was not involved in the production of and does not endorse this product.

12345678901234567890123456789012123456789012345678901234567890121234567 1234567890123456789012345678901212345678901234567890123456789012123456 7 7 12234567890123456789012345678901212345678901234567890123456789012123456 from the forest, we must know what 7 18. In line 40, “concession” could be re1 34567890123456789012345678901212345678901234567890123456789012123456 7 12234567890123456789012345678901212345678901234567890123456789012123456 species are useful and we must know placed, without changing the meaning, 7 1234567890123456789012345678901212345678901234567890123456789012123456 7 1 34567890123456789012345678901212345678901234567890123456789012123456 their names. We must be able to identify with 7 12234567890123456789012345678901212345678901234567890123456789012123456 34567890123456789012345678901212345678901234567890123456789012123456 7 1 (115) them, to know where they occur in the 7 12234567890123456789012345678901212345678901234567890123456789012123456 (A) grant. 7 1234567890123456789012345678901212345678901234567890123456789012123456 forest, how many of them exist, how they 7 1234567890123456789012345678901212345678901234567890123456789012123456 (B) acknowledgement. 7 1234567890123456789012345678901212345678901234567890123456789012123456 are pollinated and when they produce 34567890123456789012345678901212345678901234567890123456789012123456 7 1 (C) food supply. 7 12234567890123456789012345678901212345678901234567890123456789012123456 fruit (or other useful products). Aside 7 1234567890123456789012345678901212345678901234567890123456789012123456 (D) apology. 7 1234567890123456789012345678901212345678901234567890123456789012123456 from understanding the species as they 7 1234567890123456789012345678901212345678901234567890123456789012123456 (E) compromise. 34567890123456789012345678901212345678901234567890123456789012123456 7 1 (120) occur locally at Los Amigos, we must 7 12234567890123456789012345678901212345678901234567890123456789012123456 7 1234567890123456789012345678901212345678901234567890123456789012123456 have information about their overall 1234567890123456789012345678901212345678901234567890123456789012123456 19. The author implies in paragraph 3 that the 7 7 1234567890123456789012345678901212345678901234567890123456789012123456 distribution in tropical America in order agreement between Peru and the Amazon 7 1 34567890123456789012345678901212345678901234567890123456789012123456 7 12234567890123456789012345678901212345678901234567890123456789012123456 to better understand and manage the Conservation Association is historic 7 1234567890123456789012345678901212345678901234567890123456789012123456 7 1234567890123456789012345678901212345678901234567890123456789012123456 distribution, variation, and viability of primarily because it 7 1234567890123456789012345678901212345678901234567890123456789012123456 34567890123456789012345678901212345678901234567890123456789012123456 7 1234567890123456789012345678901212345678901234567890123456789012123456 (125) their genetic diversity. This involves a 7 1234567890123456789012345678901212345678901234567890123456789012123456 (A) was the first time a South American 7 1234567890123456789012345678901212345678901234567890123456789012123456 more complete understanding of the 7 1234567890123456789012345678901212345678901234567890123456789012123456 government had made an agreement 7 1234567890123456789012345678901212345678901234567890123456789012123456 species through studies in the field and 7 1234567890123456789012345678901212345678901234567890123456789012123456 of any kind with the Amazon 7 1 herbarium. 7 12234567890123456789012345678901212345678901234567890123456789012123456 Conservation Association. 7 1234567890123456789012345678901212345678901234567890123456789012123456 7 1234567890123456789012345678901212345678901234567890123456789012123456 (B) was the first long-term agreement 34567890123456789012345678901212345678901234567890123456789012123456 7 1234567890123456789012345678901212345678901234567890123456789012123456 16. In line 6, “genetic resources” refers to 7 1234567890123456789012345678901212345678901234567890123456789012123456 regarding land in the Amazon 1234567890123456789012345678901212345678901234567890123456789012123456 77 1234567890123456789012345678901212345678901234567890123456789012123456 (A) plant seeds. Rainforest. 7 1234567890123456789012345678901212345678901234567890123456789012123456 7 1234567890123456789012345678901212345678901234567890123456789012123456 (B) different races of people. (C) represented the first time a South 7 1234567890123456789012345678901212345678901234567890123456789012123456 1234567890123456789012345678901212345678901234567890123456789012123456 (C) natural resources, such as oil. American government had agreed to 7 1234567890123456789012345678901212345678901234567890123456789012123456 77 1 (D) diverse species of plants and animals. renew a conservation agreement. 7 12234567890123456789012345678901212345678901234567890123456789012123456 (E) cells that can be used in genetic cures 1234567890123456789012345678901212345678901234567890123456789012123456 (D) is essentially a permanent conserva- 7 7 1 34567890123456789012345678901212345678901234567890123456789012123456 for diseases. tion agreement. 7 1234567890123456789012345678901212345678901234567890123456789012123456 7 12234567890123456789012345678901212345678901234567890123456789012123456 (E) represents the first time such an 34567890123456789012345678901212345678901234567890123456789012123456 1234567890123456789012345678901212345678901234567890123456789012123456 77 1234567890123456789012345678901212345678901234567890123456789012123456 agreement had been made in the 7 1 17. In paragraph 2, the author emphasizes 7 12234567890123456789012345678901212345678901234567890123456789012123456 that the current environmental condition form of a renewable contract. 7 1234567890123456789012345678901212345678901234567890123456789012123456 34567890123456789012345678901212345678901234567890123456789012123456 7 1234567890123456789012345678901212345678901234567890123456789012123456 of Amazonian South America is 7 1234567890123456789012345678901212345678901234567890123456789012123456 7 1 7 1234567890123456789012345678901212345678901234567890123456789012123456 (A) mostly unscathed. 1234567890123456789012345678901212345678901234567890123456789012123456 7 2 34567890123456789012345678901212345678901234567890123456789012123456 7 1234567890123456789012345678901212345678901234567890123456789012123456 (B) largely unknown. 7 1234567890123456789012345678901212345678901234567890123456789012123456 7 1234567890123456789012345678901212345678901234567890123456789012123456 (C) restorable through his project. 1234567890123456789012345678901212345678901234567890123456789012123456 77 1 (D) irredeemable everywhere but in the 7 12234567890123456789012345678901212345678901234567890123456789012123456 Los Amigos watershed. 7 1234567890123456789012345678901212345678901234567890123456789012123456 7 1234567890123456789012345678901212345678901234567890123456789012123456 (E) varying from destroyed to virtually 7 1234567890123456789012345678901212345678901234567890123456789012123456 7 1 34567890123456789012345678901212345678901234567890123456789012123456 pristine. 2 34567890123456789012345678901212345678901234567890123456789012123456 1234567890123456789012345678901212345678901234567890123456789012123456 77 1234567890123456789012345678901212345678901234567890123456789012123456 7 1234567890123456789012345678901212345678901234567890123456789012123456 7 1234567890123456789012345678901212345678901234567890123456789012123456 7 1 7 12234567890123456789012345678901212345678901234567890123456789012123456 1 34567890123456789012345678901212345678901234567890123456789012123456 7 7 12234567890123456789012345678901212345678901234567890123456789012123456 1 34567890123456789012345678901212345678901234567890123456789012123456 7 7 12234567890123456789012345678901212345678901234567890123456789012123456 1 34567890123456789012345678901212345678901234567890123456789012123456 7 1234567890123456789012345678901212345678901234567890123456789012123456 7 1234567890123456789012345678901212345678901234567890123456789012123456 7 1234567890123456789012345678901212345678901234567890123456789012123456 7 12345678901234567890123456789012123456789012345678901234567890121234567 Copyright © 2005 Thomson Peterson’s, a part of The Thomson Corporaton SAT is a registered trademark of the College Entrance Examination Board, which was not involved in the production of and does not endorse this product.

29

12345678901234567890123456789012123456789012345678901234567890121234567 1234567890123456789012345678901212345678901234567890123456789012123456 7 34567890123456789012345678901212345678901234567890123456789012123456 7 12 20. The author’s main purpose in the passage 7 22. The work of Louise Emmons, Robin 1234567890123456789012345678901212345678901234567890123456789012123456 34567890123456789012345678901212345678901234567890123456789012123456 7 12 is to Foster, and Michael Goulding (in the 34567890123456789012345678901212345678901234567890123456789012123456 7 12 7 1234567890123456789012345678901212345678901234567890123456789012123456 fourth paragraph) are employed in the 34567890123456789012345678901212345678901234567890123456789012123456 7 12 (A) demonstrate that conservation efforts 2 34567890123456789012345678901212345678901234567890123456789012123456 7 1 passage as 34567890123456789012345678901212345678901234567890123456789012123456 7 12 have been historically successful and 34567890123456789012345678901212345678901234567890123456789012123456 7 12 34567890123456789012345678901212345678901234567890123456789012123456 7 12 so should be continued. (A) colleagues of the author’s in his 34567890123456789012345678901212345678901234567890123456789012123456 7 12 2 34567890123456789012345678901212345678901234567890123456789012123456 7 1 (B) garner support for opposition to botanical project. 34567890123456789012345678901212345678901234567890123456789012123456 7 12 34567890123456789012345678901212345678901234567890123456789012123456 7 12 destructive activities in the Los (B) examples of the kinds of activities 34567890123456789012345678901212345678901234567890123456789012123456 7 12 34567890123456789012345678901212345678901234567890123456789012123456 7 12 Amigos watershed. the author and his colleagues are 7 1234567890123456789012345678901212345678901234567890123456789012123456 34567890123456789012345678901212345678901234567890123456789012123456 7 12 (C) position the Los Amigos watershed trying to halt. 34567890123456789012345678901212345678901234567890123456789012123456 7 12 34567890123456789012345678901212345678901234567890123456789012123456 12 agreement as a success towards the (C) examples of the influence of interna- 7 34567890123456789012345678901212345678901234567890123456789012123456 7 12 2 34567890123456789012345678901212345678901234567890123456789012123456 7 1 achievement of the vital goal of contional scientists in Peru. 34567890123456789012345678901212345678901234567890123456789012123456 7 12 34567890123456789012345678901212345678901234567890123456789012123456 7 12 serving the Amazonian rainforests. (D) scientists who represent new trends 34567890123456789012345678901212345678901234567890123456789012123456 7 12 34567890123456789012345678901212345678901234567890123456789012123456 7 12 (D) uphold the Peruvian government’s of study in Amazonian botany. 34567890123456789012345678901212345678901234567890123456789012123456 7 12 34567890123456789012345678901212345678901234567890123456789012123456 12 progressive policies on management (E) scientists involved in projects related 7 34567890123456789012345678901212345678901234567890123456789012123456 7 12 of the Los Amigos watershed as an 34567890123456789012345678901212345678901234567890123456789012123456 7 and amenable to the author’s. 12 34567890123456789012345678901212345678901234567890123456789012123456 7 12 example of government policy 2 34567890123456789012345678901212345678901234567890123456789012123456 1234567890123456789012345678901212345678901234567890123456789012123456 77 1 23. The author’s botanical project involves all 7 working toward conservation. 34567890123456789012345678901212345678901234567890123456789012123456 12 34567890123456789012345678901212345678901234567890123456789012123456 7 12 of the following EXCEPT (E) argue that the study of pristine 34567890123456789012345678901212345678901234567890123456789012123456 7 12 2 34567890123456789012345678901212345678901234567890123456789012123456 7 1234567890123456789012345678901212345678901234567890123456789012123456 rainforests is essential for document7 1234567890123456789012345678901212345678901234567890123456789012123456 (A) studying plants in a laboratory. 7 1234567890123456789012345678901212345678901234567890123456789012123456 ing and studying the myriad new 7 1234567890123456789012345678901212345678901234567890123456789012123456 (B) studying how plants are used by 7 1234567890123456789012345678901212345678901234567890123456789012123456 species that the forests contain. 7 1234567890123456789012345678901212345678901234567890123456789012123456 humans and animals. 7 1234567890123456789012345678901212345678901234567890123456789012123456 7 1234567890123456789012345678901212345678901234567890123456789012123456 (C) facilitating pharmaceutical use of 7 1234567890123456789012345678901212345678901234567890123456789012123456 21. The author’s tone in the passage can best 7 1 plants. 2 34567890123456789012345678901212345678901234567890123456789012123456 7 1234567890123456789012345678901212345678901234567890123456789012123456 be described as 7 1234567890123456789012345678901212345678901234567890123456789012123456 (D) providing information on how to 7 1 (A) advocacy for his project over other keep plant species flourishing. 7 1234567890123456789012345678901212345678901234567890123456789012123456 34567890123456789012345678901212345678901234567890123456789012123456 7 12 competing projects. (E) labeling plants in the Los Amigos 2 34567890123456789012345678901212345678901234567890123456789012123456 1234567890123456789012345678901212345678901234567890123456789012123456 77 1234567890123456789012345678901212345678901234567890123456789012123456 (B) general praise for conservation area. 7 1 34567890123456789012345678901212345678901234567890123456789012123456 7 12 projects in Amazonian South 34567890123456789012345678901212345678901234567890123456789012123456 7 12 2 34567890123456789012345678901212345678901234567890123456789012123456 7 1234567890123456789012345678901212345678901234567890123456789012123456 America. 7 1234567890123456789012345678901212345678901234567890123456789012123456 7 1 (C) condemnation for the government of 1234567890123456789012345678901212345678901234567890123456789012123456 7 7 1234567890123456789012345678901212345678901234567890123456789012123456 Peru for allowing destruction of the 34567890123456789012345678901212345678901234567890123456789012123456 7 12 34567890123456789012345678901212345678901234567890123456789012123456 7 12 rainforest. 34567890123456789012345678901212345678901234567890123456789012123456 7 12 34567890123456789012345678901212345678901234567890123456789012123456 7 12 (D) passionate support for his and 7 1234567890123456789012345678901212345678901234567890123456789012123456 2 34567890123456789012345678901212345678901234567890123456789012123456 7 1234567890123456789012345678901212345678901234567890123456789012123456 related projects. 7 1234567890123456789012345678901212345678901234567890123456789012123456 7 1234567890123456789012345678901212345678901234567890123456789012123456 (E) zealous advocacy for his point of view. 1234567890123456789012345678901212345678901234567890123456789012123456 77 1 34567890123456789012345678901212345678901234567890123456789012123456 7 12 34567890123456789012345678901212345678901234567890123456789012123456 7 12 34567890123456789012345678901212345678901234567890123456789012123456 7 12 34567890123456789012345678901212345678901234567890123456789012123456 7 12 1234567890123456789012345678901212345678901234567890123456789012123456 7 2 7 1234567890123456789012345678901212345678901234567890123456789012123456 1 34567890123456789012345678901212345678901234567890123456789012123456 7 34567890123456789012345678901212345678901234567890123456789012123456 7 12 1234567890123456789012345678901212345678901234567890123456789012123456 7 34567890123456789012345678901212345678901234567890123456789012123456 7 12 1234567890123456789012345678901212345678901234567890123456789012123456 7 1234567890123456789012345678901212345678901234567890123456789012123456 7 1234567890123456789012345678901212345678901234567890123456789012123456 7 1234567890123456789012345678901212345678901234567890123456789012123456 7 12345678901234567890123456789012123456789012345678901234567890121234567 30

Copyright © 2005 Thomson Peterson’s, a part of The Thomson Corporaton SAT is a registered trademark of the College Entrance Examination Board, which was not involved in the production of and does not endorse this product.

12345678901234567890123456789012123456789012345678901234567890121234567 1234567890123456789012345678901212345678901234567890123456789012123456 7 7 12234567890123456789012345678901212345678901234567890123456789012123456 24. When the author says that the botanical 7 26. Which of the following issues does the 1 34567890123456789012345678901212345678901234567890123456789012123456 7 12234567890123456789012345678901212345678901234567890123456789012123456 project will “provide names,” (line passage NOT address? 7 1234567890123456789012345678901212345678901234567890123456789012123456 7 1 34567890123456789012345678901212345678901234567890123456789012123456 97–100) he means that the project will 7 12234567890123456789012345678901212345678901234567890123456789012123456 (A) Positive contributions of scientific 34567890123456789012345678901212345678901234567890123456789012123456 7 1 7 12234567890123456789012345678901212345678901234567890123456789012123456 (A) help recognize new species. research for conservation efforts 7 1234567890123456789012345678901212345678901234567890123456789012123456 7 1234567890123456789012345678901212345678901234567890123456789012123456 (B) aid in the standardization of names (B) Pollution of water sources in 7 1234567890123456789012345678901212345678901234567890123456789012123456 34567890123456789012345678901212345678901234567890123456789012123456 7 1 for new species. Amazonian Peru 7 12234567890123456789012345678901212345678901234567890123456789012123456 7 1234567890123456789012345678901212345678901234567890123456789012123456 (C) participate in naming the region’s (C) Economic importance of conserving 7 1234567890123456789012345678901212345678901234567890123456789012123456 7 1234567890123456789012345678901212345678901234567890123456789012123456 different zones. the Amazon rainforests 34567890123456789012345678901212345678901234567890123456789012123456 7 1 7 12234567890123456789012345678901212345678901234567890123456789012123456 (D) publish information for corporations (D) Specific efforts of the Peruvian 7 1234567890123456789012345678901212345678901234567890123456789012123456 7 1234567890123456789012345678901212345678901234567890123456789012123456 and researchers regarding the most government to maintain the integrity 7 1234567890123456789012345678901212345678901234567890123456789012123456 34567890123456789012345678901212345678901234567890123456789012123456 7 1 appropriate names for specific of Peruvian rainforests 7 12234567890123456789012345678901212345678901234567890123456789012123456 7 1234567890123456789012345678901212345678901234567890123456789012123456 plants. (E) Examples of previous scientific 7 1234567890123456789012345678901212345678901234567890123456789012123456 7 1234567890123456789012345678901212345678901234567890123456789012123456 (E) clarify the confusion surrounding the research in Los Amigos 34567890123456789012345678901212345678901234567890123456789012123456 1234567890123456789012345678901212345678901234567890123456789012123456 77 1234567890123456789012345678901212345678901234567890123456789012123456 names of different organizations 1234567890123456789012345678901212345678901234567890123456789012123456 27. The author mentions areas outside the Los 7 working in Amazonia. 7 1234567890123456789012345678901212345678901234567890123456789012123456 7 1234567890123456789012345678901212345678901234567890123456789012123456 Amigos watershed primarily in order to 1234567890123456789012345678901212345678901234567890123456789012123456 77 1 25. When the author says that, “botanical 7 12234567890123456789012345678901212345678901234567890123456789012123456 (A) imply that his future research will 7 1234567890123456789012345678901212345678901234567890123456789012123456 information has conservation value,” 7 1234567890123456789012345678901212345678901234567890123456789012123456 focus on these areas. 34567890123456789012345678901212345678901234567890123456789012123456 7 1234567890123456789012345678901212345678901234567890123456789012123456 (lines 109–110) he means that 7 1234567890123456789012345678901212345678901234567890123456789012123456 (B) draw a comparison between work in 1234567890123456789012345678901212345678901234567890123456789012123456 77 1234567890123456789012345678901212345678901234567890123456789012123456 (A) a robust understanding of conservathose areas and work in the Los 7 1234567890123456789012345678901212345678901234567890123456789012123456 7 1234567890123456789012345678901212345678901234567890123456789012123456 tionism is aided by botanical Amigos area. 7 1234567890123456789012345678901212345678901234567890123456789012123456 1234567890123456789012345678901212345678901234567890123456789012123456 information. (C) underscore the interrelatedness of the 7 1234567890123456789012345678901212345678901234567890123456789012123456 77 1 (B) conservationists should strive to ecosystems. 7 12234567890123456789012345678901212345678901234567890123456789012123456 7 preserve botanical information. 1234567890123456789012345678901212345678901234567890123456789012123456 (D) emphasize that Los Amigos is the 7 1 34567890123456789012345678901212345678901234567890123456789012123456 (C) speciation is important for most pristine locale. 7 1234567890123456789012345678901212345678901234567890123456789012123456 7 12234567890123456789012345678901212345678901234567890123456789012123456 conservation. (E) praise the Peruvian government for its 34567890123456789012345678901212345678901234567890123456789012123456 1234567890123456789012345678901212345678901234567890123456789012123456 77 1234567890123456789012345678901212345678901234567890123456789012123456 (D) political discussions about other conservationist undertakings. 7 1 7 12234567890123456789012345678901212345678901234567890123456789012123456 conservation should use botanical 7 1234567890123456789012345678901212345678901234567890123456789012123456 34567890123456789012345678901212345678901234567890123456789012123456 7 1234567890123456789012345678901212345678901234567890123456789012123456 nomenclature. 7 1234567890123456789012345678901212345678901234567890123456789012123456 7 1 (E) new drugs will be developed in the re1234567890123456789012345678901212345678901234567890123456789012123456 7 7 1234567890123456789012345678901212345678901234567890123456789012123456 gions protected by conservationism. 7 12234567890123456789012345678901212345678901234567890123456789012123456 7 1234567890123456789012345678901212345678901234567890123456789012123456 7 1234567890123456789012345678901212345678901234567890123456789012123456 7 1234567890123456789012345678901212345678901234567890123456789012123456 1 34567890123456789012345678901212345678901234567890123456789012123456 7 2 34567890123456789012345678901212345678901234567890123456789012123456 1234567890123456789012345678901212345678901234567890123456789012123456 77 1234567890123456789012345678901212345678901234567890123456789012123456 7 1234567890123456789012345678901212345678901234567890123456789012123456 7 1234567890123456789012345678901212345678901234567890123456789012123456 7 1 7 12234567890123456789012345678901212345678901234567890123456789012123456 7 1234567890123456789012345678901212345678901234567890123456789012123456 7 1234567890123456789012345678901212345678901234567890123456789012123456 7 1234567890123456789012345678901212345678901234567890123456789012123456 1 34567890123456789012345678901212345678901234567890123456789012123456 7 2 34567890123456789012345678901212345678901234567890123456789012123456 1234567890123456789012345678901212345678901234567890123456789012123456 77 1 7 12234567890123456789012345678901212345678901234567890123456789012123456 1 34567890123456789012345678901212345678901234567890123456789012123456 7 7 12234567890123456789012345678901212345678901234567890123456789012123456 Do not proceed to the next section until time is up. 7 1 34567890123456789012345678901212345678901234567890123456789012123456 1234567890123456789012345678901212345678901234567890123456789012123456 7 1234567890123456789012345678901212345678901234567890123456789012123456 7 1234567890123456789012345678901212345678901234567890123456789012123456 7 12345678901234567890123456789012123456789012345678901234567890121234567

STOP

Copyright © 2005 Thomson Peterson’s, a part of The Thomson Corporaton SAT is a registered trademark of the College Entrance Examination Board, which was not involved in the production of and does not endorse this product.

31

Section 5 21 Questions j Time—25 Minutes

Reference Information

12345678901234567890123456789012123456789012345678901234567890121234567 34567890123456789012345678901212345678901234567890123456789012123456 7 12 34567890123456789012345678901212345678901234567890123456789012123456 7 12 2 1 34567890123456789012345678901212345678901234567890123456789012123456 7 34567890123456789012345678901212345678901234567890123456789012123456 7 12 Directions: Solve the following problems using any available space on the page for scratchwork. 34567890123456789012345678901212345678901234567890123456789012123456 7 12 34567890123456789012345678901212345678901234567890123456789012123456 7 12 Mark the letter of your choice on the answer sheet that best corresponds to the correct answer. 34567890123456789012345678901212345678901234567890123456789012123456 7 12 2 34567890123456789012345678901212345678901234567890123456789012123456 1234567890123456789012345678901212345678901234567890123456789012123456 77 Notes: 1234567890123456789012345678901212345678901234567890123456789012123456 7 1234567890123456789012345678901212345678901234567890123456789012123456 1. You may use a calculator. All of the numbers used are real numbers. 7 1234567890123456789012345678901212345678901234567890123456789012123456 1234567890123456789012345678901212345678901234567890123456789012123456 77 1234567890123456789012345678901212345678901234567890123456789012123456 2. You may use the figures that accompany the problems to help you find the solution. Unless 7 1 34567890123456789012345678901212345678901234567890123456789012123456 7 12 the instructions say that a figure is not drawn to scale, assume that it has been drawn 34567890123456789012345678901212345678901234567890123456789012123456 7 12 34567890123456789012345678901212345678901234567890123456789012123456 7 12 accurately. Each figure lies in a plane unless the instructions say otherwise. 34567890123456789012345678901212345678901234567890123456789012123456 7 12 34567890123456789012345678901212345678901234567890123456789012123456 7 12 34567890123456789012345678901212345678901234567890123456789012123456 7 12 34567890123456789012345678901212345678901234567890123456789012123456 7 12 34567890123456789012345678901212345678901234567890123456789012123456 7 12 2 7 1234567890123456789012345678901212345678901234567890123456789012123456 2s r 2x 7 1234567890123456789012345678901212345678901234567890123456789012123456 w r s x 7 1234567890123456789012345678901212345678901234567890123456789012123456 c h h h b 7 1234567890123456789012345678901212345678901234567890123456789012123456 7 1 34567890123456789012345678901212345678901234567890123456789012123456 w 2 34567890123456789012345678901212345678901234567890123456789012123456 7 1234567890123456789012345678901212345678901234567890123456789012123456 s b 3x a A 5 pr2 7 1234567890123456789012345678901212345678901234567890123456789012123456 1 2 2 2 2 7 1 h c 5 a 1 b Special Right Triangles V 5 ,wh V 5 pr C 5 2pr A 5 ,w bh A 5 7 1234567890123456789012345678901212345678901234567890123456789012123456 2 34567890123456789012345678901212345678901234567890123456789012123456 7 12 2 34567890123456789012345678901212345678901234567890123456789012123456 7 1234567890123456789012345678901212345678901234567890123456789012123456 The number of degrees of arc in a circle is 360. 7 1234567890123456789012345678901212345678901234567890123456789012123456 The measure in degrees of a straight angle is 180. 7 1 34567890123456789012345678901212345678901234567890123456789012123456 7 12 The sum of the measures in degrees of the angles of a triangle is 180. 34567890123456789012345678901212345678901234567890123456789012123456 7 12 2 34567890123456789012345678901212345678901234567890123456789012123456 1234567890123456789012345678901212345678901234567890123456789012123456 77 1234567890123456789012345678901212345678901234567890123456789012123456 7 1 7 1234567890123456789012345678901212345678901234567890123456789012123456 1. Consider two sets of numbers: Set A 2. If four sweaters cost p dollars, and the 7 1234567890123456789012345678901212345678901234567890123456789012123456 2 34567890123456789012345678901212345678901234567890123456789012123456 7 1234567890123456789012345678901212345678901234567890123456789012123456 includes all the positive integers and Set B sweaters go on a half-off sale, how much 7 1234567890123456789012345678901212345678901234567890123456789012123456 7 1234567890123456789012345678901212345678901234567890123456789012123456 includes all the negative integers. Which would 12 sweaters cost in dollars? 1234567890123456789012345678901212345678901234567890123456789012123456 77 1 set has more members? 34567890123456789012345678901212345678901234567890123456789012123456 7 12 p 34567890123456789012345678901212345678901234567890123456789012123456 7 12 (A) 34567890123456789012345678901212345678901234567890123456789012123456 7 12 (A) A 2 34567890123456789012345678901212345678901234567890123456789012123456 7 12 7 1234567890123456789012345678901212345678901234567890123456789012123456 (B) B 3p 34567890123456789012345678901212345678901234567890123456789012123456 7 12 (B) 34567890123456789012345678901212345678901234567890123456789012123456 7 12 (C) They contain an equal number. 2 34567890123456789012345678901212345678901234567890123456789012123456 7 12 (D) Neither. 34567890123456789012345678901212345678901234567890123456789012123456 7 12 7 1234567890123456789012345678901212345678901234567890123456789012123456 (C) 2p (E) It cannot be determined. 2 34567890123456789012345678901212345678901234567890123456789012123456 1234567890123456789012345678901212345678901234567890123456789012123456 77 1 (D) 4p 34567890123456789012345678901212345678901234567890123456789012123456 7 12 1234567890123456789012345678901212345678901234567890123456789012123456 7 34567890123456789012345678901212345678901234567890123456789012123456 7 12 (E) 6p 7 1234567890123456789012345678901212345678901234567890123456789012123456 1234567890123456789012345678901212345678901234567890123456789012123456 7 1234567890123456789012345678901212345678901234567890123456789012123456 7 1234567890123456789012345678901212345678901234567890123456789012123456 7 12345678901234567890123456789012123456789012345678901234567890121234567 32

45

60

30

45

Copyright © 2005 Thomson Peterson’s, a part of The Thomson Corporaton SAT is a registered trademark of the College Entrance Examination Board, which was not involved in the production of and does not endorse this product.

12345678901234567890123456789012123456789012345678901234567890121234567 1234567890123456789012345678901212345678901234567890123456789012123456 7 7 12234567890123456789012345678901212345678901234567890123456789012123456 5. If y 5 mx 1 b where m is a negative 7 1 34567890123456789012345678901212345678901234567890123456789012123456 2 7 12234567890123456789012345678901212345678901234567890123456789012123456 constant and b is a positive constant, 7 1234567890123456789012345678901212345678901234567890123456789012123456 3 7 1 34567890123456789012345678901212345678901234567890123456789012123456 5 3. which of the following could be a possible 7 12234567890123456789012345678901212345678901234567890123456789012123456 4 34567890123456789012345678901212345678901234567890123456789012123456 7 1 graph of y 5 mx 1 b? 7 12234567890123456789012345678901212345678901234567890123456789012123456 5 7 1234567890123456789012345678901212345678901234567890123456789012123456 7 1234567890123456789012345678901212345678901234567890123456789012123456 (A) 1 7 1234567890123456789012345678901212345678901234567890123456789012123456 (A) 34567890123456789012345678901212345678901234567890123456789012123456 7 1 3 7 12234567890123456789012345678901212345678901234567890123456789012123456 7 1234567890123456789012345678901212345678901234567890123456789012123456 7 1234567890123456789012345678901212345678901234567890123456789012123456 8 7 1234567890123456789012345678901212345678901234567890123456789012123456 (B) 34567890123456789012345678901212345678901234567890123456789012123456 7 1 15 7 12234567890123456789012345678901212345678901234567890123456789012123456 7 1234567890123456789012345678901212345678901234567890123456789012123456 5 7 1234567890123456789012345678901212345678901234567890123456789012123456 (C) 7 1234567890123456789012345678901212345678901234567890123456789012123456 6 34567890123456789012345678901212345678901234567890123456789012123456 7 1 7 12234567890123456789012345678901212345678901234567890123456789012123456 15 7 1234567890123456789012345678901212345678901234567890123456789012123456 7 1234567890123456789012345678901212345678901234567890123456789012123456 (D) 7 1234567890123456789012345678901212345678901234567890123456789012123456 8 (B) 34567890123456789012345678901212345678901234567890123456789012123456 1234567890123456789012345678901212345678901234567890123456789012123456 77 1234567890123456789012345678901212345678901234567890123456789012123456 6 7 1234567890123456789012345678901212345678901234567890123456789012123456 (E) 7 1234567890123456789012345678901212345678901234567890123456789012123456 5 1234567890123456789012345678901212345678901234567890123456789012123456 77 1234567890123456789012345678901212345678901234567890123456789012123456 7 1 10 7 12234567890123456789012345678901212345678901234567890123456789012123456 4. If 3y 5 x and y 5 , what is the value of 7 1234567890123456789012345678901212345678901234567890123456789012123456 z 7 1234567890123456789012345678901212345678901234567890123456789012123456 34567890123456789012345678901212345678901234567890123456789012123456 7 1234567890123456789012345678901212345678901234567890123456789012123456 z when x 5 3? 7 1234567890123456789012345678901212345678901234567890123456789012123456 7 1234567890123456789012345678901212345678901234567890123456789012123456 (A) 1 7 1234567890123456789012345678901212345678901234567890123456789012123456 7 1234567890123456789012345678901212345678901234567890123456789012123456 (B) 3 7 1234567890123456789012345678901212345678901234567890123456789012123456 (C) 7 1234567890123456789012345678901212345678901234567890123456789012123456 (C) 5 7 1234567890123456789012345678901212345678901234567890123456789012123456 7 1234567890123456789012345678901212345678901234567890123456789012123456 (D) 7 7 1 2 34567890123456789012345678901212345678901234567890123456789012123456 7 1234567890123456789012345678901212345678901234567890123456789012123456 (E) 10 7 1234567890123456789012345678901212345678901234567890123456789012123456 7 1 1234567890123456789012345678901212345678901234567890123456789012123456 7 7 12234567890123456789012345678901212345678901234567890123456789012123456 7 1234567890123456789012345678901212345678901234567890123456789012123456 7 1234567890123456789012345678901212345678901234567890123456789012123456 1 34567890123456789012345678901212345678901234567890123456789012123456 7 7 12234567890123456789012345678901212345678901234567890123456789012123456 7 1234567890123456789012345678901212345678901234567890123456789012123456 34567890123456789012345678901212345678901234567890123456789012123456 1234567890123456789012345678901212345678901234567890123456789012123456 77 1234567890123456789012345678901212345678901234567890123456789012123456 (D) 7 1 1234567890123456789012345678901212345678901234567890123456789012123456 7 1234567890123456789012345678901212345678901234567890123456789012123456 7 7 12234567890123456789012345678901212345678901234567890123456789012123456 7 1234567890123456789012345678901212345678901234567890123456789012123456 7 1234567890123456789012345678901212345678901234567890123456789012123456 7 1234567890123456789012345678901212345678901234567890123456789012123456 1 34567890123456789012345678901212345678901234567890123456789012123456 7 2 34567890123456789012345678901212345678901234567890123456789012123456 1234567890123456789012345678901212345678901234567890123456789012123456 77 1234567890123456789012345678901212345678901234567890123456789012123456 7 1234567890123456789012345678901212345678901234567890123456789012123456 7 1234567890123456789012345678901212345678901234567890123456789012123456 7 1 7 12234567890123456789012345678901212345678901234567890123456789012123456 7 1234567890123456789012345678901212345678901234567890123456789012123456 7 1234567890123456789012345678901212345678901234567890123456789012123456 7 1234567890123456789012345678901212345678901234567890123456789012123456 1 34567890123456789012345678901212345678901234567890123456789012123456 7 2 34567890123456789012345678901212345678901234567890123456789012123456 1234567890123456789012345678901212345678901234567890123456789012123456 77 1 7 12234567890123456789012345678901212345678901234567890123456789012123456 1 34567890123456789012345678901212345678901234567890123456789012123456 7 7 12234567890123456789012345678901212345678901234567890123456789012123456 1 34567890123456789012345678901212345678901234567890123456789012123456 7 1234567890123456789012345678901212345678901234567890123456789012123456 7 1234567890123456789012345678901212345678901234567890123456789012123456 7 1234567890123456789012345678901212345678901234567890123456789012123456 7 12345678901234567890123456789012123456789012345678901234567890121234567 Copyright © 2005 Thomson Peterson’s, a part of The Thomson Corporaton SAT is a registered trademark of the College Entrance Examination Board, which was not involved in the production of and does not endorse this product.

33

12345678901234567890123456789012123456789012345678901234567890121234567 1234567890123456789012345678901212345678901234567890123456789012123456 7 34567890123456789012345678901212345678901234567890123456789012123456 7 12 Small Medium Large 6. 9. If f(x) 5 x2 is graphed on a standard 7 1234567890123456789012345678901212345678901234567890123456789012123456 34567890123456789012345678901212345678901234567890123456789012123456 7 12 (S) (M) (L) xy-axis and then the graph is rotated 90° 34567890123456789012345678901212345678901234567890123456789012123456 7 12 7 1234567890123456789012345678901212345678901234567890123456789012123456 clockwise, which of the following graphs 34567890123456789012345678901212345678901234567890123456789012123456 7 12 Hats (H) $8 $12 $12 2 34567890123456789012345678901212345678901234567890123456789012123456 7 1 would result? 34567890123456789012345678901212345678901234567890123456789012123456 7 12 Shirts (SH) $12 $12 $14 34567890123456789012345678901212345678901234567890123456789012123456 7 12 34567890123456789012345678901212345678901234567890123456789012123456 7 12 (A) 34567890123456789012345678901212345678901234567890123456789012123456 7 12 Hat & Shirt Prices at Moe’s 2 34567890123456789012345678901212345678901234567890123456789012123456 7 1 34567890123456789012345678901212345678901234567890123456789012123456 7 12 34567890123456789012345678901212345678901234567890123456789012123456 7 12 If Moe’s has a 25% off sale on medium34567890123456789012345678901212345678901234567890123456789012123456 7 12 34567890123456789012345678901212345678901234567890123456789012123456 7 12 sized items, how much would it cost, in 2 34567890123456789012345678901212345678901234567890123456789012123456 7 1 dollars, to order 2 H-M, 2 H-L, and 34567890123456789012345678901212345678901234567890123456789012123456 7 12 34567890123456789012345678901212345678901234567890123456789012123456 7 12 1SH-M? 34567890123456789012345678901212345678901234567890123456789012123456 7 12 34567890123456789012345678901212345678901234567890123456789012123456 7 12 (B) 2 34567890123456789012345678901212345678901234567890123456789012123456 7 1 (A) 51 34567890123456789012345678901212345678901234567890123456789012123456 7 12 34567890123456789012345678901212345678901234567890123456789012123456 7 12 (B) 53 34567890123456789012345678901212345678901234567890123456789012123456 7 12 34567890123456789012345678901212345678901234567890123456789012123456 7 12 (C) 55 34567890123456789012345678901212345678901234567890123456789012123456 7 12 34567890123456789012345678901212345678901234567890123456789012123456 7 12 (D) 56 34567890123456789012345678901212345678901234567890123456789012123456 7 12 34567890123456789012345678901212345678901234567890123456789012123456 7 12 (E) 58 34567890123456789012345678901212345678901234567890123456789012123456 7 12 2 34567890123456789012345678901212345678901234567890123456789012123456 1234567890123456789012345678901212345678901234567890123456789012123456 77 1 34567890123456789012345678901212345678901234567890123456789012123456 7 12 7. If 5=x 1 15 5 30, then x 5 34567890123456789012345678901212345678901234567890123456789012123456 7 12 (C) 34567890123456789012345678901212345678901234567890123456789012123456 7 12 2 34567890123456789012345678901212345678901234567890123456789012123456 7 1234567890123456789012345678901212345678901234567890123456789012123456 (A) 9 7 1234567890123456789012345678901212345678901234567890123456789012123456 7 1234567890123456789012345678901212345678901234567890123456789012123456 (B) 10 7 1234567890123456789012345678901212345678901234567890123456789012123456 (C) 12 7 1234567890123456789012345678901212345678901234567890123456789012123456 7 1234567890123456789012345678901212345678901234567890123456789012123456 (D) 14 7 1234567890123456789012345678901212345678901234567890123456789012123456 7 1234567890123456789012345678901212345678901234567890123456789012123456 (E) 15 7 1234567890123456789012345678901212345678901234567890123456789012123456 7 1 2 34567890123456789012345678901212345678901234567890123456789012123456 7 1234567890123456789012345678901212345678901234567890123456789012123456 2 7 1234567890123456789012345678901212345678901234567890123456789012123456 2 x 2 12 x 7 1 8. 2 5 7 1234567890123456789012345678901212345678901234567890123456789012123456 2x 1 2x 2 12 (D) 34567890123456789012345678901212345678901234567890123456789012123456 7 12 2 34567890123456789012345678901212345678901234567890123456789012123456 7 1234567890123456789012345678901212345678901234567890123456789012123456 x22 7 1234567890123456789012345678901212345678901234567890123456789012123456 (A) 7 1 x24 34567890123456789012345678901212345678901234567890123456789012123456 7 12 34567890123456789012345678901212345678901234567890123456789012123456 7 12 2 34567890123456789012345678901212345678901234567890123456789012123456 7 1234567890123456789012345678901212345678901234567890123456789012123456 3~x 2 2! 7 1234567890123456789012345678901212345678901234567890123456789012123456 (B) 7 1 x 2 4 1234567890123456789012345678901212345678901234567890123456789012123456 7 7 1234567890123456789012345678901212345678901234567890123456789012123456 4~x 2 3! 34567890123456789012345678901212345678901234567890123456789012123456 7 12 (C) 34567890123456789012345678901212345678901234567890123456789012123456 7 12 x22 34567890123456789012345678901212345678901234567890123456789012123456 7 12 34567890123456789012345678901212345678901234567890123456789012123456 7 12 7 1234567890123456789012345678901212345678901234567890123456789012123456 x24 34567890123456789012345678901212345678901234567890123456789012123456 7 12 (D) (E) 34567890123456789012345678901212345678901234567890123456789012123456 7 12 2~x 2 2! 34567890123456789012345678901212345678901234567890123456789012123456 7 12 34567890123456789012345678901212345678901234567890123456789012123456 7 12 4~x 2 3! 7 1234567890123456789012345678901212345678901234567890123456789012123456 (E) 2 34567890123456789012345678901212345678901234567890123456789012123456 7 1234567890123456789012345678901212345678901234567890123456789012123456 3~x 2 2! 7 1234567890123456789012345678901212345678901234567890123456789012123456 1234567890123456789012345678901212345678901234567890123456789012123456 77 1234567890123456789012345678901212345678901234567890123456789012123456 7 1 34567890123456789012345678901212345678901234567890123456789012123456 7 12 1234567890123456789012345678901212345678901234567890123456789012123456 7 34567890123456789012345678901212345678901234567890123456789012123456 7 12 1234567890123456789012345678901212345678901234567890123456789012123456 7 34567890123456789012345678901212345678901234567890123456789012123456 7 12 1234567890123456789012345678901212345678901234567890123456789012123456 7 1234567890123456789012345678901212345678901234567890123456789012123456 7 1234567890123456789012345678901212345678901234567890123456789012123456 7 1234567890123456789012345678901212345678901234567890123456789012123456 7 12345678901234567890123456789012123456789012345678901234567890121234567 34

Copyright © 2005 Thomson Peterson’s, a part of The Thomson Corporaton SAT is a registered trademark of the College Entrance Examination Board, which was not involved in the production of and does not endorse this product.

12345678901234567890123456789012123456789012345678901234567890121234567 1234567890123456789012345678901212345678901234567890123456789012123456 7 7 12234567890123456789012345678901212345678901234567890123456789012123456 10. For which of the following ordered pairs 13. 7 1 34567890123456789012345678901212345678901234567890123456789012123456 7 12234567890123456789012345678901212345678901234567890123456789012123456 (x,y) is x 2 y . 2 and x 1 y . 4? 7 1234567890123456789012345678901212345678901234567890123456789012123456 1 34567890123456789012345678901212345678901234567890123456789012123456 7 7 12234567890123456789012345678901212345678901234567890123456789012123456 (A) (1,3) 7 1 34567890123456789012345678901212345678901234567890123456789012123456 7 12234567890123456789012345678901212345678901234567890123456789012123456 (B) (2,3) 7 1234567890123456789012345678901212345678901234567890123456789012123456 7 1234567890123456789012345678901212345678901234567890123456789012123456 (C) (4,0) 7 1234567890123456789012345678901212345678901234567890123456789012123456 34567890123456789012345678901212345678901234567890123456789012123456 7 1 If the two triangles above are congruent (D) (3,2) 7 12234567890123456789012345678901212345678901234567890123456789012123456 7 1234567890123456789012345678901212345678901234567890123456789012123456 then b 5 (E) (4,1) 7 1234567890123456789012345678901212345678901234567890123456789012123456 7 1234567890123456789012345678901212345678901234567890123456789012123456 34567890123456789012345678901212345678901234567890123456789012123456 7 1 (A) 30 7 12234567890123456789012345678901212345678901234567890123456789012123456 x 1 7 1234567890123456789012345678901212345678901234567890123456789012123456 (B) 40 11. If x and y are positive integers and 5 , 7 1234567890123456789012345678901212345678901234567890123456789012123456 y 2 7 1234567890123456789012345678901212345678901234567890123456789012123456 (C) 45 then x 1 y 5 34567890123456789012345678901212345678901234567890123456789012123456 7 1 7 12234567890123456789012345678901212345678901234567890123456789012123456 (D) 60 7 1234567890123456789012345678901212345678901234567890123456789012123456 (A) 3x (E) It cannot be determined. 7 1234567890123456789012345678901212345678901234567890123456789012123456 7 1234567890123456789012345678901212345678901234567890123456789012123456 (B) 5x 34567890123456789012345678901212345678901234567890123456789012123456 1234567890123456789012345678901212345678901234567890123456789012123456 77 1234567890123456789012345678901212345678901234567890123456789012123456 (C) y 14. 7 1234567890123456789012345678901212345678901234567890123456789012123456 7 1234567890123456789012345678901212345678901234567890123456789012123456 (D) 2y 1234567890123456789012345678901212345678901234567890123456789012123456 77 1234567890123456789012345678901212345678901234567890123456789012123456 (E) 3y 7 1 7 12234567890123456789012345678901212345678901234567890123456789012123456 7 1234567890123456789012345678901212345678901234567890123456789012123456 12. Quentin buys three hot dogs with a 7 1234567890123456789012345678901212345678901234567890123456789012123456 34567890123456789012345678901212345678901234567890123456789012123456 7 1234567890123456789012345678901212345678901234567890123456789012123456 ten-dollar bill and receives seven dollars 7 1234567890123456789012345678901212345678901234567890123456789012123456 7 1234567890123456789012345678901212345678901234567890123456789012123456 and thirty-four cents in change. If the sales 1234567890123456789012345678901212345678901234567890123456789012123456 77 1234567890123456789012345678901212345678901234567890123456789012123456 tax is seven cents per dollar (rounding to 7 1234567890123456789012345678901212345678901234567890123456789012123456 7 1234567890123456789012345678901212345678901234567890123456789012123456 the nearest penny), which of the following 1234567890123456789012345678901212345678901234567890123456789012123456 77 1234567890123456789012345678901212345678901234567890123456789012123456 choices, in cents, is closest to the actual What is the area of the above figure? 7 1 7 12234567890123456789012345678901212345678901234567890123456789012123456 price of a hot dog? 7 1234567890123456789012345678901212345678901234567890123456789012123456 7 1 34567890123456789012345678901212345678901234567890123456789012123456 (A) 2=2 7 1234567890123456789012345678901212345678901234567890123456789012123456 (A) 78 7 12234567890123456789012345678901212345678901234567890123456789012123456 (B) 4 34567890123456789012345678901212345678901234567890123456789012123456 7 1234567890123456789012345678901212345678901234567890123456789012123456 (B) 82 7 1234567890123456789012345678901212345678901234567890123456789012123456 (C) 8 7 1 (C) 86 7 12234567890123456789012345678901212345678901234567890123456789012123456 7 (D) 89 1234567890123456789012345678901212345678901234567890123456789012123456 (D) 9 34567890123456789012345678901212345678901234567890123456789012123456 7 1234567890123456789012345678901212345678901234567890123456789012123456 (E) 92 7 1234567890123456789012345678901212345678901234567890123456789012123456 (E) 12 7 1 1234567890123456789012345678901212345678901234567890123456789012123456 7 1234567890123456789012345678901212345678901234567890123456789012123456 7 7 12234567890123456789012345678901212345678901234567890123456789012123456 7 1234567890123456789012345678901212345678901234567890123456789012123456 7 1234567890123456789012345678901212345678901234567890123456789012123456 7 1234567890123456789012345678901212345678901234567890123456789012123456 1 34567890123456789012345678901212345678901234567890123456789012123456 7 2 34567890123456789012345678901212345678901234567890123456789012123456 1234567890123456789012345678901212345678901234567890123456789012123456 77 1234567890123456789012345678901212345678901234567890123456789012123456 7 1234567890123456789012345678901212345678901234567890123456789012123456 7 1234567890123456789012345678901212345678901234567890123456789012123456 7 1 7 12234567890123456789012345678901212345678901234567890123456789012123456 7 1234567890123456789012345678901212345678901234567890123456789012123456 7 1234567890123456789012345678901212345678901234567890123456789012123456 7 1234567890123456789012345678901212345678901234567890123456789012123456 1 34567890123456789012345678901212345678901234567890123456789012123456 7 2 34567890123456789012345678901212345678901234567890123456789012123456 1234567890123456789012345678901212345678901234567890123456789012123456 77 1 7 12234567890123456789012345678901212345678901234567890123456789012123456 1 34567890123456789012345678901212345678901234567890123456789012123456 7 7 12234567890123456789012345678901212345678901234567890123456789012123456 1 34567890123456789012345678901212345678901234567890123456789012123456 7 1234567890123456789012345678901212345678901234567890123456789012123456 7 1234567890123456789012345678901212345678901234567890123456789012123456 7 1234567890123456789012345678901212345678901234567890123456789012123456 7 12345678901234567890123456789012123456789012345678901234567890121234567 Copyright © 2005 Thomson Peterson’s, a part of The Thomson Corporaton SAT is a registered trademark of the College Entrance Examination Board, which was not involved in the production of and does not endorse this product.

35

12345678901234567890123456789012123456789012345678901234567890121234567 1234567890123456789012345678901212345678901234567890123456789012123456 7 34567890123456789012345678901212345678901234567890123456789012123456 7 12 Questions 15–16 refer to the following chart. 17. If l1i l2, then which of the following pairs 7 1234567890123456789012345678901212345678901234567890123456789012123456 34567890123456789012345678901212345678901234567890123456789012123456 7 12 of angles must be congruent? 34567890123456789012345678901212345678901234567890123456789012123456 7 12 1234567890123456789012345678901212345678901234567890123456789012123456 7 34567890123456789012345678901212345678901234567890123456789012123456 7 12 1234567890123456789012345678901212345678901234567890123456789012123456 7 34567890123456789012345678901212345678901234567890123456789012123456 7 12 34567890123456789012345678901212345678901234567890123456789012123456 7 12 34567890123456789012345678901212345678901234567890123456789012123456 7 12 34567890123456789012345678901212345678901234567890123456789012123456 7 12 2 1 34567890123456789012345678901212345678901234567890123456789012123456 7 34567890123456789012345678901212345678901234567890123456789012123456 7 12 34567890123456789012345678901212345678901234567890123456789012123456 7 12 34567890123456789012345678901212345678901234567890123456789012123456 7 12 34567890123456789012345678901212345678901234567890123456789012123456 7 12 2 1 34567890123456789012345678901212345678901234567890123456789012123456 7 34567890123456789012345678901212345678901234567890123456789012123456 7 12 34567890123456789012345678901212345678901234567890123456789012123456 7 12 34567890123456789012345678901212345678901234567890123456789012123456 7 12 34567890123456789012345678901212345678901234567890123456789012123456 7 12 2 1 34567890123456789012345678901212345678901234567890123456789012123456 7 34567890123456789012345678901212345678901234567890123456789012123456 7 12 34567890123456789012345678901212345678901234567890123456789012123456 7 12 34567890123456789012345678901212345678901234567890123456789012123456 7 12 I. 6 and 12 34567890123456789012345678901212345678901234567890123456789012123456 7 12 2 34567890123456789012345678901212345678901234567890123456789012123456 7 1234567890123456789012345678901212345678901234567890123456789012123456 II. 2 and 9 7 1234567890123456789012345678901212345678901234567890123456789012123456 7 1234567890123456789012345678901212345678901234567890123456789012123456 III. 4 and 10 7 1234567890123456789012345678901212345678901234567890123456789012123456 15. Which year had the least percentage 1234567890123456789012345678901212345678901234567890123456789012123456 77 1234567890123456789012345678901212345678901234567890123456789012123456 (A) I only difference in reported incidence of flu and 7 1 34567890123456789012345678901212345678901234567890123456789012123456 7 (B) II only 12 cold? 34567890123456789012345678901212345678901234567890123456789012123456 7 12 (C) III only 34567890123456789012345678901212345678901234567890123456789012123456 7 12 2 34567890123456789012345678901212345678901234567890123456789012123456 7 1234567890123456789012345678901212345678901234567890123456789012123456 (A) 1960 (D) II and III only 7 1234567890123456789012345678901212345678901234567890123456789012123456 7 1234567890123456789012345678901212345678901234567890123456789012123456 (B) 1970 (E) I and II only 7 1234567890123456789012345678901212345678901234567890123456789012123456 7 1234567890123456789012345678901212345678901234567890123456789012123456 (C) 1980 7 1234567890123456789012345678901212345678901234567890123456789012123456 2b 7 1234567890123456789012345678901212345678901234567890123456789012123456 (D) 1990 is a positive integer, and a, b, 18. If 2a 1234567890123456789012345678901212345678901234567890123456789012123456 77 1234567890123456789012345678901212345678901234567890123456789012123456 (E) 2000 and c are integers, then 7 1 2 34567890123456789012345678901212345678901234567890123456789012123456 1234567890123456789012345678901212345678901234567890123456789012123456 77 1234567890123456789012345678901212345678901234567890123456789012123456 (A) a must be negative. 7 1 16. In percentage terms, in what decade was 7 1234567890123456789012345678901212345678901234567890123456789012123456 (B) b must be negative. the number of reported cold cases about 34567890123456789012345678901212345678901234567890123456789012123456 7 12 2 34567890123456789012345678901212345678901234567890123456789012123456 7 1234567890123456789012345678901212345678901234567890123456789012123456 (C) c must be negative. 25% greater than the number of reported 7 1234567890123456789012345678901212345678901234567890123456789012123456 (D) b must be an even positive integer. 7 1 flu cases? 34567890123456789012345678901212345678901234567890123456789012123456 7 12 (E) None of the above. 34567890123456789012345678901212345678901234567890123456789012123456 7 12 2 34567890123456789012345678901212345678901234567890123456789012123456 7 1234567890123456789012345678901212345678901234567890123456789012123456 (A) 1960 7 1234567890123456789012345678901212345678901234567890123456789012123456 7 1 (B) 1970 1234567890123456789012345678901212345678901234567890123456789012123456 7 7 1234567890123456789012345678901212345678901234567890123456789012123456 (C) 1980 34567890123456789012345678901212345678901234567890123456789012123456 7 12 34567890123456789012345678901212345678901234567890123456789012123456 7 12 (D) 1990 34567890123456789012345678901212345678901234567890123456789012123456 7 12 34567890123456789012345678901212345678901234567890123456789012123456 7 12 (E) 2000 7 1234567890123456789012345678901212345678901234567890123456789012123456 2 34567890123456789012345678901212345678901234567890123456789012123456 1234567890123456789012345678901212345678901234567890123456789012123456 77 1234567890123456789012345678901212345678901234567890123456789012123456 7 1234567890123456789012345678901212345678901234567890123456789012123456 7 1234567890123456789012345678901212345678901234567890123456789012123456 7 1 34567890123456789012345678901212345678901234567890123456789012123456 7 12 34567890123456789012345678901212345678901234567890123456789012123456 7 12 34567890123456789012345678901212345678901234567890123456789012123456 7 12 34567890123456789012345678901212345678901234567890123456789012123456 7 12 1234567890123456789012345678901212345678901234567890123456789012123456 7 2 7 1234567890123456789012345678901212345678901234567890123456789012123456 1 34567890123456789012345678901212345678901234567890123456789012123456 7 34567890123456789012345678901212345678901234567890123456789012123456 7 12 1234567890123456789012345678901212345678901234567890123456789012123456 7 34567890123456789012345678901212345678901234567890123456789012123456 7 12 1234567890123456789012345678901212345678901234567890123456789012123456 7 1234567890123456789012345678901212345678901234567890123456789012123456 7 1234567890123456789012345678901212345678901234567890123456789012123456 7 1234567890123456789012345678901212345678901234567890123456789012123456 7 12345678901234567890123456789012123456789012345678901234567890121234567 2c

36

Copyright © 2005 Thomson Peterson’s, a part of The Thomson Corporaton SAT is a registered trademark of the College Entrance Examination Board, which was not involved in the production of and does not endorse this product.

12345678901234567890123456789012123456789012345678901234567890121234567 1234567890123456789012345678901212345678901234567890123456789012123456 7 7 12234567890123456789012345678901212345678901234567890123456789012123456 19. 21. There are 150 students in math courses at 7 1 34567890123456789012345678901212345678901234567890123456789012123456 7 12234567890123456789012345678901212345678901234567890123456789012123456 Dagmar High School. 73 are in geometry, 7 1234567890123456789012345678901212345678901234567890123456789012123456 7 1 34567890123456789012345678901212345678901234567890123456789012123456 62 are in algebra, and 52 are in neither. 7 12234567890123456789012345678901212345678901234567890123456789012123456 34567890123456789012345678901212345678901234567890123456789012123456 7 1 How many students are in both geometry 7 12234567890123456789012345678901212345678901234567890123456789012123456 7 1234567890123456789012345678901212345678901234567890123456789012123456 and algebra? 7 1234567890123456789012345678901212345678901234567890123456789012123456 7 1234567890123456789012345678901212345678901234567890123456789012123456 34567890123456789012345678901212345678901234567890123456789012123456 7 1 (A) 32 7 12234567890123456789012345678901212345678901234567890123456789012123456 7 1234567890123456789012345678901212345678901234567890123456789012123456 (B) 33 7 1234567890123456789012345678901212345678901234567890123456789012123456 7 1234567890123456789012345678901212345678901234567890123456789012123456 (C) 35 34567890123456789012345678901212345678901234567890123456789012123456 7 1 7 12234567890123456789012345678901212345678901234567890123456789012123456 (D) 36 7 1234567890123456789012345678901212345678901234567890123456789012123456 If all the line segments in the above figure 7 1234567890123456789012345678901212345678901234567890123456789012123456 (E) 37 7 1234567890123456789012345678901212345678901234567890123456789012123456 are congruent, then 34567890123456789012345678901212345678901234567890123456789012123456 7 1 7 12234567890123456789012345678901212345678901234567890123456789012123456 7 1234567890123456789012345678901212345678901234567890123456789012123456 (A) a . b 7 1234567890123456789012345678901212345678901234567890123456789012123456 (B) a , b 7 1234567890123456789012345678901212345678901234567890123456789012123456 34567890123456789012345678901212345678901234567890123456789012123456 7 1234567890123456789012345678901212345678901234567890123456789012123456 (C) 2a 5 b 7 1234567890123456789012345678901212345678901234567890123456789012123456 7 1234567890123456789012345678901212345678901234567890123456789012123456 (D) a 5 b 7 1234567890123456789012345678901212345678901234567890123456789012123456 7 1234567890123456789012345678901212345678901234567890123456789012123456 (E) It cannot be determined. 1234567890123456789012345678901212345678901234567890123456789012123456 77 1 7 12234567890123456789012345678901212345678901234567890123456789012123456 7 1234567890123456789012345678901212345678901234567890123456789012123456 20. Two boys and two girls are assigned to sit 7 1234567890123456789012345678901212345678901234567890123456789012123456 34567890123456789012345678901212345678901234567890123456789012123456 7 1234567890123456789012345678901212345678901234567890123456789012123456 at a five-seat circular table, where the 7 1234567890123456789012345678901212345678901234567890123456789012123456 7 1234567890123456789012345678901212345678901234567890123456789012123456 seats are numbered one through five. If 1234567890123456789012345678901212345678901234567890123456789012123456 77 1234567890123456789012345678901212345678901234567890123456789012123456 neither boy can sit by the open seat, seat 7 1234567890123456789012345678901212345678901234567890123456789012123456 7 1234567890123456789012345678901212345678901234567890123456789012123456 3, how many different seating arrange1234567890123456789012345678901212345678901234567890123456789012123456 77 1234567890123456789012345678901212345678901234567890123456789012123456 ments are possible? 7 1 2 34567890123456789012345678901212345678901234567890123456789012123456 7 1234567890123456789012345678901212345678901234567890123456789012123456 (A) 2 7 1234567890123456789012345678901212345678901234567890123456789012123456 7 1 (B) 3 7 1234567890123456789012345678901212345678901234567890123456789012123456 7 12234567890123456789012345678901212345678901234567890123456789012123456 (C) 4 34567890123456789012345678901212345678901234567890123456789012123456 1234567890123456789012345678901212345678901234567890123456789012123456 77 1234567890123456789012345678901212345678901234567890123456789012123456 (D) 5 7 1 7 12234567890123456789012345678901212345678901234567890123456789012123456 (E) 6 7 1234567890123456789012345678901212345678901234567890123456789012123456 34567890123456789012345678901212345678901234567890123456789012123456 1234567890123456789012345678901212345678901234567890123456789012123456 77 1234567890123456789012345678901212345678901234567890123456789012123456 7 1 1234567890123456789012345678901212345678901234567890123456789012123456 7 1234567890123456789012345678901212345678901234567890123456789012123456 7 7 12234567890123456789012345678901212345678901234567890123456789012123456 7 1234567890123456789012345678901212345678901234567890123456789012123456 7 1234567890123456789012345678901212345678901234567890123456789012123456 7 1234567890123456789012345678901212345678901234567890123456789012123456 1 34567890123456789012345678901212345678901234567890123456789012123456 7 2 34567890123456789012345678901212345678901234567890123456789012123456 1234567890123456789012345678901212345678901234567890123456789012123456 77 1234567890123456789012345678901212345678901234567890123456789012123456 7 1234567890123456789012345678901212345678901234567890123456789012123456 7 1234567890123456789012345678901212345678901234567890123456789012123456 7 1 7 12234567890123456789012345678901212345678901234567890123456789012123456 7 1234567890123456789012345678901212345678901234567890123456789012123456 7 1234567890123456789012345678901212345678901234567890123456789012123456 7 1234567890123456789012345678901212345678901234567890123456789012123456 1 34567890123456789012345678901212345678901234567890123456789012123456 7 2 34567890123456789012345678901212345678901234567890123456789012123456 1234567890123456789012345678901212345678901234567890123456789012123456 77 1 7 12234567890123456789012345678901212345678901234567890123456789012123456 1 34567890123456789012345678901212345678901234567890123456789012123456 7 7 12234567890123456789012345678901212345678901234567890123456789012123456 Do not proceed to the next section until time is up. 7 1 34567890123456789012345678901212345678901234567890123456789012123456 1234567890123456789012345678901212345678901234567890123456789012123456 7 1234567890123456789012345678901212345678901234567890123456789012123456 7 1234567890123456789012345678901212345678901234567890123456789012123456 7 12345678901234567890123456789012123456789012345678901234567890121234567

STOP

Copyright © 2005 Thomson Peterson’s, a part of The Thomson Corporaton SAT is a registered trademark of the College Entrance Examination Board, which was not involved in the production of and does not endorse this product.

37

Section 6 16 Questions j Time—20 Minutes 12345678901234567890123456789012123456789012345678901234567890121234567 34567890123456789012345678901212345678901234567890123456789012123456 7 12 34567890123456789012345678901212345678901234567890123456789012123456 7 12 2 1 34567890123456789012345678901212345678901234567890123456789012123456 7 34567890123456789012345678901212345678901234567890123456789012123456 7 12 Directions: Each passage below is followed by a set of questions. Read each passage, then 34567890123456789012345678901212345678901234567890123456789012123456 7 12 34567890123456789012345678901212345678901234567890123456789012123456 7 12 answer the accompanying questions, basing your answers on what is stated or implied in the 34567890123456789012345678901212345678901234567890123456789012123456 7 12 2 34567890123456789012345678901212345678901234567890123456789012123456 7 1234567890123456789012345678901212345678901234567890123456789012123456 passage and any introductory material provided. Mark the letter of your choice on the answer 7 1234567890123456789012345678901212345678901234567890123456789012123456 7 1234567890123456789012345678901212345678901234567890123456789012123456 sheet that best corresponds to the correct answer. 1234567890123456789012345678901212345678901234567890123456789012123456 77 1234567890123456789012345678901212345678901234567890123456789012123456 7 1234567890123456789012345678901212345678901234567890123456789012123456 7 1 34567890123456789012345678901212345678901234567890123456789012123456 7 12 34567890123456789012345678901212345678901234567890123456789012123456 7 12 Line In 1953, Watson and Crick unlocked the 2. The passage uses the phrase “wet and 34567890123456789012345678901212345678901234567890123456789012123456 7 12 structure of the DNA molecule and set 2 34567890123456789012345678901212345678901234567890123456789012123456 7 1234567890123456789012345678901212345678901234567890123456789012123456 dirty” (line 5) to mean 7 1234567890123456789012345678901212345678901234567890123456789012123456 into motion the modern study of genetics. 7 1234567890123456789012345678901212345678901234567890123456789012123456 (A) haphazard guessing about the genetic 7 1234567890123456789012345678901212345678901234567890123456789012123456 This advance allowed our study of life to 7 1234567890123456789012345678901212345678901234567890123456789012123456 code. 7 1234567890123456789012345678901212345678901234567890123456789012123456 (5) go beyond the so-called wet and dirty 7 1234567890123456789012345678901212345678901234567890123456789012123456 (B) the work of Watson and Crick in 7 1234567890123456789012345678901212345678901234567890123456789012123456 realm of biology, the complicated labora7 1234567890123456789012345678901212345678901234567890123456789012123456 discovering DNA. tory study of proteins, cells, organelles, 7 1 2 34567890123456789012345678901212345678901234567890123456789012123456 7 1234567890123456789012345678901212345678901234567890123456789012123456 (C) information-based biological ions, and lipids. The study of life could 7 1234567890123456789012345678901212345678901234567890123456789012123456 7 1 now be performed with more abstract research. 1234567890123456789012345678901212345678901234567890123456789012123456 7 34567890123456789012345678901212345678901234567890123456789012123456 7 12 (10) methods of analysis. By discovering the (D) the study of the genetic code. 34567890123456789012345678901212345678901234567890123456789012123456 7 12 34567890123456789012345678901212345678901234567890123456789012123456 7 12 basic structure of DNA, we had received (E) involved laboratory practices in 7 1234567890123456789012345678901212345678901234567890123456789012123456 our first glance into the information-based 34567890123456789012345678901212345678901234567890123456789012123456 7 12 studying basic biological entities. 34567890123456789012345678901212345678901234567890123456789012123456 7 12 realm locked inside the genetic code. 2 34567890123456789012345678901212345678901234567890123456789012123456 1234567890123456789012345678901212345678901234567890123456789012123456 77 1234567890123456789012345678901212345678901234567890123456789012123456 Although little-known today in the United 7 1 7 1234567890123456789012345678901212345678901234567890123456789012123456 1. Which of the following does the passage States, Clark Saunders (1859–1941) cast a large 7 1234567890123456789012345678901212345678901234567890123456789012123456 2 34567890123456789012345678901212345678901234567890123456789012123456 7 1234567890123456789012345678901212345678901234567890123456789012123456 discuss as a change that the discovery of shadow in the first several decades of the 7 1234567890123456789012345678901212345678901234567890123456789012123456 7 1234567890123456789012345678901212345678901234567890123456789012123456 DNA brought to the study of life? twentieth century, writing many widely read 1234567890123456789012345678901212345678901234567890123456789012123456 77 1 (A) The study of lipids and proteins books on Native American, Spanish, and Anglo 7 34567890123456789012345678901212345678901234567890123456789012123456 12 34567890123456789012345678901212345678901234567890123456789012123456 7 12 became irrelevant. folklore. He also wrote extensively on the 34567890123456789012345678901212345678901234567890123456789012123456 7 12 (B) New and more abstract methods of 34567890123456789012345678901212345678901234567890123456789012123456 7 12 different cultures of California, the Sierras, and 7 1234567890123456789012345678901212345678901234567890123456789012123456 study were possible. 2 34567890123456789012345678901212345678901234567890123456789012123456 7 1234567890123456789012345678901212345678901234567890123456789012123456 the Southwest. He was a major and influential 7 1234567890123456789012345678901212345678901234567890123456789012123456 (C) Biology could then focus on mol7 1234567890123456789012345678901212345678901234567890123456789012123456 contributor to Sunset Magazine in its early 7 1234567890123456789012345678901212345678901234567890123456789012123456 ecules rather than cells. 7 1 years. In his day, Saunders was important for 2 34567890123456789012345678901212345678901234567890123456789012123456 7 1234567890123456789012345678901212345678901234567890123456789012123456 (D) Modern genetics matured past its 7 1 introducing much of the American public to a Mendelian roots. 34567890123456789012345678901212345678901234567890123456789012123456 7 12 7 1234567890123456789012345678901212345678901234567890123456789012123456 person-sized understanding of the “Old West.” (E) Information-based study of genes 34567890123456789012345678901212345678901234567890123456789012123456 7 12 2 34567890123456789012345678901212345678901234567890123456789012123456 7 1 became obsolete. 7 1234567890123456789012345678901212345678901234567890123456789012123456 1234567890123456789012345678901212345678901234567890123456789012123456 77 1234567890123456789012345678901212345678901234567890123456789012123456 12345678901234567890123456789012123456789012345678901234567890121234567 38

Copyright © 2005 Thomson Peterson’s, a part of The Thomson Corporaton SAT is a registered trademark of the College Entrance Examination Board, which was not involved in the production of and does not endorse this product.

12345678901234567890123456789012123456789012345678901234567890121234567 1234567890123456789012345678901212345678901234567890123456789012123456 7 7 12234567890123456789012345678901212345678901234567890123456789012123456 3. The passage presents Saunders as a(n) The following two passages deal with the 7 1 34567890123456789012345678901212345678901234567890123456789012123456 7 12234567890123456789012345678901212345678901234567890123456789012123456 political movements working for the woman’s 7 1234567890123456789012345678901212345678901234567890123456789012123456 (A) influential contemporary western 7 1 34567890123456789012345678901212345678901234567890123456789012123456 vote in America. 7 12234567890123456789012345678901212345678901234567890123456789012123456 writer. 34567890123456789012345678901212345678901234567890123456789012123456 7 1 7 12234567890123456789012345678901212345678901234567890123456789012123456 (B) important historian of the West. 7 1234567890123456789012345678901212345678901234567890123456789012123456 Passage 1 7 1234567890123456789012345678901212345678901234567890123456789012123456 (C) a specialist of Native American 7 1234567890123456789012345678901212345678901234567890123456789012123456 34567890123456789012345678901212345678901234567890123456789012123456 7 1 Line The first organized assertion of woman’s studies. 7 12234567890123456789012345678901212345678901234567890123456789012123456 1234567890123456789012345678901212345678901234567890123456789012123456 rights in the United States was made at the 7 (D) widely read author in his own day. 7 1234567890123456789012345678901212345678901234567890123456789012123456 7 1234567890123456789012345678901212345678901234567890123456789012123456 Seneca Falls convention in 1848. The (E) the first editor of Sunset Magazine. 34567890123456789012345678901212345678901234567890123456789012123456 7 1 12234567890123456789012345678901212345678901234567890123456789012123456 convention, though, had little immediate 7 7 1234567890123456789012345678901212345678901234567890123456789012123456 (5) impact because of the national issues that 7 The history of rock and roll is inseparable from 1234567890123456789012345678901212345678901234567890123456789012123456 7 1234567890123456789012345678901212345678901234567890123456789012123456 would soon embroil the country. The the development of blues and gospel music in 7 1 34567890123456789012345678901212345678901234567890123456789012123456 7 12234567890123456789012345678901212345678901234567890123456789012123456 contentious debates involving slavery and 7 the southeastern United States. Though the 1234567890123456789012345678901212345678901234567890123456789012123456 7 1234567890123456789012345678901212345678901234567890123456789012123456 state’s rights that preceded the Civil War genre gained mass appeal through legendary 7 1234567890123456789012345678901212345678901234567890123456789012123456 34567890123456789012345678901212345678901234567890123456789012123456 7 1234567890123456789012345678901212345678901234567890123456789012123456 soon took center stage in national debates. 7 figures such as Elvis Presley or the wildly 1234567890123456789012345678901212345678901234567890123456789012123456 1234567890123456789012345678901212345678901234567890123456789012123456 (10) Thus woman’s rights issues would have to 7 popular Beatles, the musical roots of rock and 7 1234567890123456789012345678901212345678901234567890123456789012123456 7 1234567890123456789012345678901212345678901234567890123456789012123456 wait until the war and its antecedent roll extend far before such groups. In fact, 1234567890123456789012345678901212345678901234567890123456789012123456 77 1 many of the groups who popularized rock and problems had been addressed before they 7 12234567890123456789012345678901212345678901234567890123456789012123456 7 1234567890123456789012345678901212345678901234567890123456789012123456 would be addressed. roll were consciously attempting to emulate the 7 1234567890123456789012345678901212345678901234567890123456789012123456 34567890123456789012345678901212345678901234567890123456789012123456 7 1234567890123456789012345678901212345678901234567890123456789012123456 In 1869, two organizations were work of blues greats such as B. B. King or 7 1234567890123456789012345678901212345678901234567890123456789012123456 7 1234567890123456789012345678901212345678901234567890123456789012123456 (15) formed that would play important roles in Muddy Waters. The Rolling Stones are a good 1234567890123456789012345678901212345678901234567890123456789012123456 77 1234567890123456789012345678901212345678901234567890123456789012123456 securing the woman’s right to vote. The example of this trend, which developed in the 7 1234567890123456789012345678901212345678901234567890123456789012123456 7 1234567890123456789012345678901212345678901234567890123456789012123456 first was the American Woman’s Suffrage late fifties and early sixties. The Rolling Stones, 1234567890123456789012345678901212345678901234567890123456789012123456 77 1234567890123456789012345678901212345678901234567890123456789012123456 Association (AWSA). Leaving federal and 7 1 both then and now, have always explicitly 2 34567890123456789012345678901212345678901234567890123456789012123456 7 1234567890123456789012345678901212345678901234567890123456789012123456 constitutional issues aside, the AWSA stated their admiration and imitation of blues 7 1234567890123456789012345678901212345678901234567890123456789012123456 7 1 greats. (20) focused their attention on state-level 1234567890123456789012345678901212345678901234567890123456789012123456 7 7 12234567890123456789012345678901212345678901234567890123456789012123456 politics. They also restricted their ambi7 1234567890123456789012345678901212345678901234567890123456789012123456 tions to securing the woman’s vote and 7 1234567890123456789012345678901212345678901234567890123456789012123456 4. B. B. King is used in this passage as an 7 1 34567890123456789012345678901212345678901234567890123456789012123456 downplayed discussion of women’s full 7 12234567890123456789012345678901212345678901234567890123456789012123456 example of a 7 1234567890123456789012345678901212345678901234567890123456789012123456 equality. Taking a different track, the 34567890123456789012345678901212345678901234567890123456789012123456 1234567890123456789012345678901212345678901234567890123456789012123456 77 1234567890123456789012345678901212345678901234567890123456789012123456 (A) blues artist who was emulated by (25) National Woman’s Suffrage Association 7 1 7 1234567890123456789012345678901212345678901234567890123456789012123456 early rock bands. (NWSA), led by Elizabeth Stanton and 1234567890123456789012345678901212345678901234567890123456789012123456 7 2 34567890123456789012345678901212345678901234567890123456789012123456 7 1234567890123456789012345678901212345678901234567890123456789012123456 (B) musical artist influenced by Elvis Susan B. Anthony, believed that the only 7 1234567890123456789012345678901212345678901234567890123456789012123456 7 1234567890123456789012345678901212345678901234567890123456789012123456 Presley. way to assure the long-term security of the 1234567890123456789012345678901212345678901234567890123456789012123456 77 1 (C) musician who incorporated aspects woman’s vote was to ground it in the 7 12234567890123456789012345678901212345678901234567890123456789012123456 of rock and roll. 7 1234567890123456789012345678901212345678901234567890123456789012123456 (30) constitution. The NWSA challenged the 7 1234567890123456789012345678901212345678901234567890123456789012123456 (D) musician who often played with 7 1234567890123456789012345678901212345678901234567890123456789012123456 exclusion of woman from the Fifteenth 7 1 34567890123456789012345678901212345678901234567890123456789012123456 Muddy Waters. 2 34567890123456789012345678901212345678901234567890123456789012123456 7 1234567890123456789012345678901212345678901234567890123456789012123456 Amendment, the amendment that 7 1234567890123456789012345678901212345678901234567890123456789012123456 (E) gospel singer who influenced the 7 1234567890123456789012345678901212345678901234567890123456789012123456 extended the vote to African-American 7 1234567890123456789012345678901212345678901234567890123456789012123456 Rolling Stones. 7 1 men. Furthermore, the NWSA linked the 2 34567890123456789012345678901212345678901234567890123456789012123456 1234567890123456789012345678901212345678901234567890123456789012123456 77 1 (35) fight for suffrage with other inequalities 7 12234567890123456789012345678901212345678901234567890123456789012123456 7 1 34567890123456789012345678901212345678901234567890123456789012123456 faced by woman, such as marriage laws, 7 12234567890123456789012345678901212345678901234567890123456789012123456 34567890123456789012345678901212345678901234567890123456789012123456 7 1 which greatly disadvantaged women. 7 1234567890123456789012345678901212345678901234567890123456789012123456 1234567890123456789012345678901212345678901234567890123456789012123456 77 1234567890123456789012345678901212345678901234567890123456789012123456 12345678901234567890123456789012123456789012345678901234567890121234567 Copyright © 2005 Thomson Peterson’s, a part of The Thomson Corporaton SAT is a registered trademark of the College Entrance Examination Board, which was not involved in the production of and does not endorse this product.

39

12345678901234567890123456789012123456789012345678901234567890121234567 1234567890123456789012345678901212345678901234567890123456789012123456 7 34567890123456789012345678901212345678901234567890123456789012123456 7 12 By the late 1880s the differences that 5. The word “antecedent” in line 11 can best 7 1234567890123456789012345678901212345678901234567890123456789012123456 34567890123456789012345678901212345678901234567890123456789012123456 7 12 separated the two organizations had be replaced by 34567890123456789012345678901212345678901234567890123456789012123456 7 12 7 1234567890123456789012345678901212345678901234567890123456789012123456 (40) receded in importance as the women’s 34567890123456789012345678901212345678901234567890123456789012123456 7 12 (A) antebellum. 2 34567890123456789012345678901212345678901234567890123456789012123456 7 1 movement had become a substantial and 34567890123456789012345678901212345678901234567890123456789012123456 7 12 (B) referent. 34567890123456789012345678901212345678901234567890123456789012123456 7 12 broad-based political force in the country. 34567890123456789012345678901212345678901234567890123456789012123456 7 12 (C) causal. 34567890123456789012345678901212345678901234567890123456789012123456 7 12 In 1890, the two organizations joined 2 34567890123456789012345678901212345678901234567890123456789012123456 7 1 (D) subsequent. 34567890123456789012345678901212345678901234567890123456789012123456 7 12 forces under the title of the National 34567890123456789012345678901212345678901234567890123456789012123456 7 12 (E) abolitionist. 34567890123456789012345678901212345678901234567890123456789012123456 7 12 (45) American Woman’s Suffrage Association 34567890123456789012345678901212345678901234567890123456789012123456 7 12 2 34567890123456789012345678901212345678901234567890123456789012123456 7 1 (NAWSA). The NAWSA would go on to 34567890123456789012345678901212345678901234567890123456789012123456 7 12 6. Which of the following does the first 34567890123456789012345678901212345678901234567890123456789012123456 7 12 play a vital role in the further fight to 34567890123456789012345678901212345678901234567890123456789012123456 7 12 passage say was the first organized push 34567890123456789012345678901212345678901234567890123456789012123456 7 12 achieve the woman’s vote. for woman’s suffrage? 2 34567890123456789012345678901212345678901234567890123456789012123456 7 1 34567890123456789012345678901212345678901234567890123456789012123456 7 12 34567890123456789012345678901212345678901234567890123456789012123456 7 12 (A) formation of the National Woman’s 34567890123456789012345678901212345678901234567890123456789012123456 7 Passage 2 12 34567890123456789012345678901212345678901234567890123456789012123456 7 12 Suffrage Association 2 34567890123456789012345678901212345678901234567890123456789012123456 7 1234567890123456789012345678901212345678901234567890123456789012123456 In 1920, when Tennessee became the 7 1234567890123456789012345678901212345678901234567890123456789012123456 (B) formation of the American Woman’s 7 1234567890123456789012345678901212345678901234567890123456789012123456 (50) thirty-eighth state to approve the constitu7 1234567890123456789012345678901212345678901234567890123456789012123456 Suffrage Association 7 1234567890123456789012345678901212345678901234567890123456789012123456 tional amendment securing the woman’s 7 1234567890123456789012345678901212345678901234567890123456789012123456 (C) convening of the Seneca Falls 7 1 right to vote, woman’s suffrage became 34567890123456789012345678901212345678901234567890123456789012123456 7 12 convention 34567890123456789012345678901212345678901234567890123456789012123456 7 12 enshrined in the constitution. But wom34567890123456789012345678901212345678901234567890123456789012123456 7 12 (D) Tennessee passing the Twenty-Second 2 34567890123456789012345678901212345678901234567890123456789012123456 7 1234567890123456789012345678901212345678901234567890123456789012123456 an’s suffrage did not happen in one fell 7 1234567890123456789012345678901212345678901234567890123456789012123456 Amendment 7 1234567890123456789012345678901212345678901234567890123456789012123456 (55) swoop. The success of the woman’s 7 1234567890123456789012345678901212345678901234567890123456789012123456 (E) “partial suffrages” of local woman’s 7 1234567890123456789012345678901212345678901234567890123456789012123456 suffrage movement was the story of a 7 1234567890123456789012345678901212345678901234567890123456789012123456 suffrage efforts 7 1234567890123456789012345678901212345678901234567890123456789012123456 number of partial victories that led to the 1234567890123456789012345678901212345678901234567890123456789012123456 77 1234567890123456789012345678901212345678901234567890123456789012123456 explicit endorsement of the woman’s right 7. What national event does the first passage 7 1 2 34567890123456789012345678901212345678901234567890123456789012123456 7 1234567890123456789012345678901212345678901234567890123456789012123456 to vote in the constitution. cite as pushing woman’s voting rights to 7 1234567890123456789012345678901212345678901234567890123456789012123456 7 1 (60) As early as the 1870s and 1880s, the background of the national conscious1234567890123456789012345678901212345678901234567890123456789012123456 7 34567890123456789012345678901212345678901234567890123456789012123456 7 12 women had begun to win the right to vote ness? 34567890123456789012345678901212345678901234567890123456789012123456 7 12 34567890123456789012345678901212345678901234567890123456789012123456 7 in local affairs such as municipal elections, 12 7 1234567890123456789012345678901212345678901234567890123456789012123456 (A) Civil War school board elections, or prohibition 34567890123456789012345678901212345678901234567890123456789012123456 7 12 34567890123456789012345678901212345678901234567890123456789012123456 7 12 (B) Suffrage movement measures. These “partial suffrages” 2 34567890123456789012345678901212345678901234567890123456789012123456 1234567890123456789012345678901212345678901234567890123456789012123456 77 1234567890123456789012345678901212345678901234567890123456789012123456 (C) Prohibition 7 1 (65) demonstrated that women could in fact 1234567890123456789012345678901212345678901234567890123456789012123456 (D) Passage of the Fifteenth Amendment 7 responsibly and reasonably participate in 1234567890123456789012345678901212345678901234567890123456789012123456 7 34567890123456789012345678901212345678901234567890123456789012123456 7 12 (E) World War I a representative democracy (at least as 34567890123456789012345678901212345678901234567890123456789012123456 7 12 34567890123456789012345678901212345678901234567890123456789012123456 7 12 voters). Once such successes were 34567890123456789012345678901212345678901234567890123456789012123456 7 12 8. According to the first passage, the 7 1234567890123456789012345678901212345678901234567890123456789012123456 achieved and maintained over a period of 2 34567890123456789012345678901212345678901234567890123456789012123456 7 1234567890123456789012345678901212345678901234567890123456789012123456 National Woman’s Suffrage Association 7 1234567890123456789012345678901212345678901234567890123456789012123456 (70) time, restricting the full voting rights of 7 1234567890123456789012345678901212345678901234567890123456789012123456 focused their efforts on 7 1234567890123456789012345678901212345678901234567890123456789012123456 woman became more and more suspect. If 7 1 2 34567890123456789012345678901212345678901234567890123456789012123456 7 1234567890123456789012345678901212345678901234567890123456789012123456 women were helping decide who was on (A) local elections. 7 1234567890123456789012345678901212345678901234567890123456789012123456 7 1234567890123456789012345678901212345678901234567890123456789012123456 the local school board, why should they (B) constitutional issues. 1234567890123456789012345678901212345678901234567890123456789012123456 77 1 not also have a voice in deciding who was (C) prohibition efforts. 34567890123456789012345678901212345678901234567890123456789012123456 7 12 7 1234567890123456789012345678901212345678901234567890123456789012123456 (75) president of the country? Such questions (D) school board elections. 34567890123456789012345678901212345678901234567890123456789012123456 7 12 7 1234567890123456789012345678901212345678901234567890123456789012123456 became more difficult for non-suffragists (E) state elections. 34567890123456789012345678901212345678901234567890123456789012123456 7 12 2 34567890123456789012345678901212345678901234567890123456789012123456 7 1 to answer, and thus the logic of restricting 7 1234567890123456789012345678901212345678901234567890123456789012123456 7 1234567890123456789012345678901212345678901234567890123456789012123456 the woman’s vote began to crumble. 1234567890123456789012345678901212345678901234567890123456789012123456 7 12345678901234567890123456789012123456789012345678901234567890121234567 40

Copyright © 2005 Thomson Peterson’s, a part of The Thomson Corporaton SAT is a registered trademark of the College Entrance Examination Board, which was not involved in the production of and does not endorse this product.

12345678901234567890123456789012123456789012345678901234567890121234567 1234567890123456789012345678901212345678901234567890123456789012123456 7 7 12234567890123456789012345678901212345678901234567890123456789012123456 9. The differences between the AWSA and 7 12. When is the earliest success of the 1 34567890123456789012345678901212345678901234567890123456789012123456 7 12234567890123456789012345678901212345678901234567890123456789012123456 the NWSA were ultimately resolved when woman’s suffrage movement that the 7 1234567890123456789012345678901212345678901234567890123456789012123456 7 1 34567890123456789012345678901212345678901234567890123456789012123456 second passage points to? 7 12234567890123456789012345678901212345678901234567890123456789012123456 (A) the Twenty-Second Amendment 34567890123456789012345678901212345678901234567890123456789012123456 7 1 7 12234567890123456789012345678901212345678901234567890123456789012123456 passed. (A) 1848 7 1234567890123456789012345678901212345678901234567890123456789012123456 7 1234567890123456789012345678901212345678901234567890123456789012123456 (B) the two organizations were com(B) 1869 7 1234567890123456789012345678901212345678901234567890123456789012123456 34567890123456789012345678901212345678901234567890123456789012123456 7 1 bined to form the NAWSA. (C) 1870s 7 12234567890123456789012345678901212345678901234567890123456789012123456 7 1234567890123456789012345678901212345678901234567890123456789012123456 (C) the Civil War ended. (D) 1880s 7 1234567890123456789012345678901212345678901234567890123456789012123456 7 1234567890123456789012345678901212345678901234567890123456789012123456 (D) prohibition passed. (E) 1920 34567890123456789012345678901212345678901234567890123456789012123456 7 1 7 12234567890123456789012345678901212345678901234567890123456789012123456 (E) woman’s suffragists won significant 7 1234567890123456789012345678901212345678901234567890123456789012123456 7 1234567890123456789012345678901212345678901234567890123456789012123456 13. Which of the following is NOT an victories in the 1890 general election. 7 1234567890123456789012345678901212345678901234567890123456789012123456 example of a “partial suffrage” as 34567890123456789012345678901212345678901234567890123456789012123456 7 1 7 12234567890123456789012345678901212345678901234567890123456789012123456 10. In Passage 1, the author’s attitude toward described in the second passage? 7 1234567890123456789012345678901212345678901234567890123456789012123456 7 1234567890123456789012345678901212345678901234567890123456789012123456 the subject matter is 7 1234567890123456789012345678901212345678901234567890123456789012123456 (A) A mayoral election 34567890123456789012345678901212345678901234567890123456789012123456 1234567890123456789012345678901212345678901234567890123456789012123456 77 1234567890123456789012345678901212345678901234567890123456789012123456 (A) intense scrutiny. (B) A school board measure 7 1234567890123456789012345678901212345678901234567890123456789012123456 7 1234567890123456789012345678901212345678901234567890123456789012123456 (B) distanced suspicion. (C) Passage of the Fifteenth Amendment 1234567890123456789012345678901212345678901234567890123456789012123456 77 1234567890123456789012345678901212345678901234567890123456789012123456 (C) mild censure. (D) A state prohibition referendum 7 1 7 12234567890123456789012345678901212345678901234567890123456789012123456 (D) appreciative description. (E) Impeaching a city council member 7 1234567890123456789012345678901212345678901234567890123456789012123456 7 1234567890123456789012345678901212345678901234567890123456789012123456 (E) enthusiastic support. 34567890123456789012345678901212345678901234567890123456789012123456 1234567890123456789012345678901212345678901234567890123456789012123456 77 1234567890123456789012345678901212345678901234567890123456789012123456 14. The author of the second passage argues 7 1234567890123456789012345678901212345678901234567890123456789012123456 11. Passage 2 locates the ultimate victory of that the “partial suffrages” were most 7 1234567890123456789012345678901212345678901234567890123456789012123456 7 1234567890123456789012345678901212345678901234567890123456789012123456 the woman’s suffrage movement with effective in bringing full voting rights for 1234567890123456789012345678901212345678901234567890123456789012123456 77 1234567890123456789012345678901212345678901234567890123456789012123456 which of the following events? woman because 7 1234567890123456789012345678901212345678901234567890123456789012123456 1234567890123456789012345678901212345678901234567890123456789012123456 77 1 (A) Tennessee approving the woman’s (A) through them woman were able to 7 12234567890123456789012345678901212345678901234567890123456789012123456 7 1234567890123456789012345678901212345678901234567890123456789012123456 voting rights amendment elect prosuffrage representatives. 1 34567890123456789012345678901212345678901234567890123456789012123456 7 7 1234567890123456789012345678901212345678901234567890123456789012123456 (B) Congress passing the Twenty-Second (B) they showed women voting ably. 7 12234567890123456789012345678901212345678901234567890123456789012123456 34567890123456789012345678901212345678901234567890123456789012123456 1234567890123456789012345678901212345678901234567890123456789012123456 Amendment (C) they demonstrated that woman could 7 7 1234567890123456789012345678901212345678901234567890123456789012123456 7 1 (C) The combination of AWSA and participate in a full democracy. 7 12234567890123456789012345678901212345678901234567890123456789012123456 7 1234567890123456789012345678901212345678901234567890123456789012123456 NWSA into NAWSA (D) they demonstrated that woman could 34567890123456789012345678901212345678901234567890123456789012123456 1234567890123456789012345678901212345678901234567890123456789012123456 77 1234567890123456789012345678901212345678901234567890123456789012123456 (D) Woman earning the full vote in handle the intricacies of foreign 7 1 7 1234567890123456789012345678901212345678901234567890123456789012123456 Wyoming policy. 1234567890123456789012345678901212345678901234567890123456789012123456 7 2 34567890123456789012345678901212345678901234567890123456789012123456 7 1234567890123456789012345678901212345678901234567890123456789012123456 (E) Women’s fruitful participation in (E) they established the power of the 7 1234567890123456789012345678901212345678901234567890123456789012123456 7 1234567890123456789012345678901212345678901234567890123456789012123456 local elections woman voter. 1234567890123456789012345678901212345678901234567890123456789012123456 77 1 7 12234567890123456789012345678901212345678901234567890123456789012123456 7 1234567890123456789012345678901212345678901234567890123456789012123456 7 1234567890123456789012345678901212345678901234567890123456789012123456 7 1234567890123456789012345678901212345678901234567890123456789012123456 1 34567890123456789012345678901212345678901234567890123456789012123456 7 2 34567890123456789012345678901212345678901234567890123456789012123456 1234567890123456789012345678901212345678901234567890123456789012123456 77 1234567890123456789012345678901212345678901234567890123456789012123456 7 1234567890123456789012345678901212345678901234567890123456789012123456 7 1234567890123456789012345678901212345678901234567890123456789012123456 7 1 7 12234567890123456789012345678901212345678901234567890123456789012123456 1 34567890123456789012345678901212345678901234567890123456789012123456 7 7 12234567890123456789012345678901212345678901234567890123456789012123456 1 34567890123456789012345678901212345678901234567890123456789012123456 7 7 12234567890123456789012345678901212345678901234567890123456789012123456 1 34567890123456789012345678901212345678901234567890123456789012123456 7 1234567890123456789012345678901212345678901234567890123456789012123456 7 1234567890123456789012345678901212345678901234567890123456789012123456 7 1234567890123456789012345678901212345678901234567890123456789012123456 7 12345678901234567890123456789012123456789012345678901234567890121234567 Copyright © 2005 Thomson Peterson’s, a part of The Thomson Corporaton SAT is a registered trademark of the College Entrance Examination Board, which was not involved in the production of and does not endorse this product.

41

12345678901234567890123456789012123456789012345678901234567890121234567 1234567890123456789012345678901212345678901234567890123456789012123456 7 34567890123456789012345678901212345678901234567890123456789012123456 7 12 15. Which of the following questions is NOT 16. The author of the second passage would 7 1234567890123456789012345678901212345678901234567890123456789012123456 34567890123456789012345678901212345678901234567890123456789012123456 7 12 addressed in either passage? most likely see the work of the 34567890123456789012345678901212345678901234567890123456789012123456 7 12 1234567890123456789012345678901212345678901234567890123456789012123456 7 34567890123456789012345678901212345678901234567890123456789012123456 7 12 (A) When did the woman’s right to vote (A) AWSA as crucial for the ultimate 7 1234567890123456789012345678901212345678901234567890123456789012123456 34567890123456789012345678901212345678901234567890123456789012123456 7 12 become a constitutional amendment? success of the suffrage movement. 34567890123456789012345678901212345678901234567890123456789012123456 7 12 34567890123456789012345678901212345678901234567890123456789012123456 7 12 (B) What effect did the Civil War have (B) NWSA as indispensable for “partial 34567890123456789012345678901212345678901234567890123456789012123456 7 12 2 34567890123456789012345678901212345678901234567890123456789012123456 7 1 on the woman’s suffrage movement? suffrages.” 34567890123456789012345678901212345678901234567890123456789012123456 7 12 34567890123456789012345678901212345678901234567890123456789012123456 7 12 (C) What are the names of two leaders of (C) NWSA as unimportant for the 34567890123456789012345678901212345678901234567890123456789012123456 7 12 34567890123456789012345678901212345678901234567890123456789012123456 12 the National Woman’s Suffrage passage of the woman’s voting rights 7 7 1234567890123456789012345678901212345678901234567890123456789012123456 34567890123456789012345678901212345678901234567890123456789012123456 7 12 Association? amendment. 34567890123456789012345678901212345678901234567890123456789012123456 7 12 34567890123456789012345678901212345678901234567890123456789012123456 7 12 (D) What are “partial suffrages?” (D) Seneca Falls convention as the most 34567890123456789012345678901212345678901234567890123456789012123456 7 12 2 34567890123456789012345678901212345678901234567890123456789012123456 7 1 (E) Which constitutional amendment important single event in the 34567890123456789012345678901212345678901234567890123456789012123456 7 12 34567890123456789012345678901212345678901234567890123456789012123456 7 12 gave women the vote? women’s suffrage movement. 34567890123456789012345678901212345678901234567890123456789012123456 7 12 34567890123456789012345678901212345678901234567890123456789012123456 7 12 (E) the NAWSA as important for the 34567890123456789012345678901212345678901234567890123456789012123456 7 12 34567890123456789012345678901212345678901234567890123456789012123456 7 12 unity of the woman’s suffrage 34567890123456789012345678901212345678901234567890123456789012123456 7 12 movement. 34567890123456789012345678901212345678901234567890123456789012123456 7 12 34567890123456789012345678901212345678901234567890123456789012123456 7 12 2 34567890123456789012345678901212345678901234567890123456789012123456 1234567890123456789012345678901212345678901234567890123456789012123456 77 1 34567890123456789012345678901212345678901234567890123456789012123456 7 12 34567890123456789012345678901212345678901234567890123456789012123456 7 12 34567890123456789012345678901212345678901234567890123456789012123456 7 12 2 7 1234567890123456789012345678901212345678901234567890123456789012123456 7 1234567890123456789012345678901212345678901234567890123456789012123456 7 1234567890123456789012345678901212345678901234567890123456789012123456 7 1234567890123456789012345678901212345678901234567890123456789012123456 7 1234567890123456789012345678901212345678901234567890123456789012123456 34567890123456789012345678901212345678901234567890123456789012123456 1234567890123456789012345678901212345678901234567890123456789012123456 77 1234567890123456789012345678901212345678901234567890123456789012123456 7 1234567890123456789012345678901212345678901234567890123456789012123456 7 1234567890123456789012345678901212345678901234567890123456789012123456 7 1 34567890123456789012345678901212345678901234567890123456789012123456 7 12 34567890123456789012345678901212345678901234567890123456789012123456 7 12 1234567890123456789012345678901212345678901234567890123456789012123456 7 1234567890123456789012345678901212345678901234567890123456789012123456 7 34567890123456789012345678901212345678901234567890123456789012123456 7 12 34567890123456789012345678901212345678901234567890123456789012123456 7 12 34567890123456789012345678901212345678901234567890123456789012123456 7 12 1234567890123456789012345678901212345678901234567890123456789012123456 7 34567890123456789012345678901212345678901234567890123456789012123456 7 12 34567890123456789012345678901212345678901234567890123456789012123456 7 12 2 7 1234567890123456789012345678901212345678901234567890123456789012123456 7 1234567890123456789012345678901212345678901234567890123456789012123456 1 34567890123456789012345678901212345678901234567890123456789012123456 7 1234567890123456789012345678901212345678901234567890123456789012123456 7 1234567890123456789012345678901212345678901234567890123456789012123456 7 34567890123456789012345678901212345678901234567890123456789012123456 7 12 34567890123456789012345678901212345678901234567890123456789012123456 7 12 34567890123456789012345678901212345678901234567890123456789012123456 7 12 34567890123456789012345678901212345678901234567890123456789012123456 7 12 1234567890123456789012345678901212345678901234567890123456789012123456 7 2 7 1234567890123456789012345678901212345678901234567890123456789012123456 7 1234567890123456789012345678901212345678901234567890123456789012123456 7 1234567890123456789012345678901212345678901234567890123456789012123456 7 1234567890123456789012345678901212345678901234567890123456789012123456 1 34567890123456789012345678901212345678901234567890123456789012123456 7 2 34567890123456789012345678901212345678901234567890123456789012123456 1234567890123456789012345678901212345678901234567890123456789012123456 77 1234567890123456789012345678901212345678901234567890123456789012123456 7 1234567890123456789012345678901212345678901234567890123456789012123456 7 1234567890123456789012345678901212345678901234567890123456789012123456 7 1 34567890123456789012345678901212345678901234567890123456789012123456 7 12 1234567890123456789012345678901212345678901234567890123456789012123456 7 34567890123456789012345678901212345678901234567890123456789012123456 7 12 1234567890123456789012345678901212345678901234567890123456789012123456 7 34567890123456789012345678901212345678901234567890123456789012123456 7 12 Do not proceed to the next section until time is up. 7 1234567890123456789012345678901212345678901234567890123456789012123456 1234567890123456789012345678901212345678901234567890123456789012123456 7 1234567890123456789012345678901212345678901234567890123456789012123456 7 1234567890123456789012345678901212345678901234567890123456789012123456 7 12345678901234567890123456789012123456789012345678901234567890121234567

STOP

42

Copyright © 2005 Thomson Peterson’s, a part of The Thomson Corporaton SAT is a registered trademark of the College Entrance Examination Board, which was not involved in the production of and does not endorse this product.

Section 7 13 Questions j Time—20 Minutes 12345678901234567890123456789012123456789012345678901234567890121234567 7 12234567890123456789012345678901212345678901234567890123456789012123456 7 1234567890123456789012345678901212345678901234567890123456789012123456 34567890123456789012345678901212345678901234567890123456789012123456 7 1 7 12234567890123456789012345678901212345678901234567890123456789012123456 Notes: 7 1234567890123456789012345678901212345678901234567890123456789012123456 7 1234567890123456789012345678901212345678901234567890123456789012123456 7 1234567890123456789012345678901212345678901234567890123456789012123456 1. All numbers used are real numbers. 34567890123456789012345678901212345678901234567890123456789012123456 1234567890123456789012345678901212345678901234567890123456789012123456 77 1234567890123456789012345678901212345678901234567890123456789012123456 2. All angle measurements can be assumed to be positive unless otherwise noted. 7 1234567890123456789012345678901212345678901234567890123456789012123456 1234567890123456789012345678901212345678901234567890123456789012123456 77 1234567890123456789012345678901212345678901234567890123456789012123456 3. All figures lie in the same plane unless otherwise noted. 7 1234567890123456789012345678901212345678901234567890123456789012123456 7 1 7 12234567890123456789012345678901212345678901234567890123456789012123456 4. Drawings that accompany questions are intended to provide information useful in 7 1234567890123456789012345678901212345678901234567890123456789012123456 7 1234567890123456789012345678901212345678901234567890123456789012123456 answering the question. The figures are drawn closely to scale unless otherwise noted. 34567890123456789012345678901212345678901234567890123456789012123456 1234567890123456789012345678901212345678901234567890123456789012123456 77 1234567890123456789012345678901212345678901234567890123456789012123456 7 1234567890123456789012345678901212345678901234567890123456789012123456 7 1234567890123456789012345678901212345678901234567890123456789012123456 7 1234567890123456789012345678901212345678901234567890123456789012123456 Directions for Student Produced Responses 7 1234567890123456789012345678901212345678901234567890123456789012123456 1234567890123456789012345678901212345678901234567890123456789012123456 77 1234567890123456789012345678901212345678901234567890123456789012123456 Enter your responses to questions 1–13 in the Answer: 1.4 7 1234567890123456789012345678901212345678901234567890123456789012123456 7 1 special grids provided on your answer sheet. Either position is correct. 2 34567890123456789012345678901212345678901234567890123456789012123456 1234567890123456789012345678901212345678901234567890123456789012123456 77 1234567890123456789012345678901212345678901234567890123456789012123456 7 1 Input your answers as indicated in the directions 1 . 4 1 . 4 7 1234567890123456789012345678901212345678901234567890123456789012123456 below. 7 12234567890123456789012345678901212345678901234567890123456789012123456 O O O O 34567890123456789012345678901212345678901234567890123456789012123456 7 1234567890123456789012345678901212345678901234567890123456789012123456 O O Þ O O Þ O O 4 Decimal → 7 1234567890123456789012345678901212345678901234567890123456789012123456 Answer: or 4/9 7 1 O O O point O O O 9 7 12234567890123456789012345678901212345678901234567890123456789012123456 O Þ O O Þ O O O 7 1234567890123456789012345678901212345678901234567890123456789012123456 34567890123456789012345678901212345678901234567890123456789012123456 7 1234567890123456789012345678901212345678901234567890123456789012123456 O O O O O O O O Write answer → 4 / 9 7 1234567890123456789012345678901212345678901234567890123456789012123456 O O O O O O O O 7 1 in boxes. O O O Þ O O Þ O Þ O ← Fraction line 7 1234567890123456789012345678901212345678901234567890123456789012123456 7 1234567890123456789012345678901212345678901234567890123456789012123456 O O O O 2 34567890123456789012345678901212345678901234567890123456789012123456 7 1234567890123456789012345678901212345678901234567890123456789012123456 O O O Note: You may begin your answer in any col- 7 1234567890123456789012345678901212345678901234567890123456789012123456 7 1234567890123456789012345678901212345678901234567890123456789012123456 O O O ⎧O umn, space permitting. Leave blank any columns 7 1234567890123456789012345678901212345678901234567890123456789012123456 O O O O 7 1 O O O O ⎪Þ not needed. 7 12234567890123456789012345678901212345678901234567890123456789012123456 O O O 7 1234567890123456789012345678901212345678901234567890123456789012123456 Grid in → 7 1234567890123456789012345678901212345678901234567890123456789012123456 O O O O ⎨ • Writing your answer in the boxes at the top result. 7 1234567890123456789012345678901212345678901234567890123456789012123456 O O O O 7 1 34567890123456789012345678901212345678901234567890123456789012123456 of the columns will help you accurately grid O O O O ⎪ 2 34567890123456789012345678901212345678901234567890123456789012123456 7 1234567890123456789012345678901212345678901234567890123456789012123456 O O O O 7 1234567890123456789012345678901212345678901234567890123456789012123456 your answer, but it is not required. You will 7 1234567890123456789012345678901212345678901234567890123456789012123456 O O Þ O ⎩ 7 1234567890123456789012345678901212345678901234567890123456789012123456 only receive credit for an answer if the ovals 7 1 2 34567890123456789012345678901212345678901234567890123456789012123456 7 1234567890123456789012345678901212345678901234567890123456789012123456 are filled in properly. 7 1 7 12234567890123456789012345678901212345678901234567890123456789012123456 7 1 34567890123456789012345678901212345678901234567890123456789012123456 • Only fill in one oval in each column. 7 12234567890123456789012345678901212345678901234567890123456789012123456 34567890123456789012345678901212345678901234567890123456789012123456 7 1 1234567890123456789012345678901212345678901234567890123456789012123456 7 1234567890123456789012345678901212345678901234567890123456789012123456 7 1234567890123456789012345678901212345678901234567890123456789012123456 7 12345678901234567890123456789012123456789012345678901234567890121234567 /

.

/

.

.

.

.

0

0

0

1

1

1

1

2

2

2

2

3

3

3

3

4

4

4

5

5

5

5

6

6

6

6

7

7

7

7

8

8

8

8

9

9

9

/

/

.

.

.

0

0

0

1

1

1

1

2

2

2

2

2

3

3

3

3

4

4

4

/

.

0

.

0

0

1

1

2

2

2

3

3

3

3

4

4

4

12345678901234567890123456789012123456789012345678901234567890121234567 1234567890123456789012345678901212345678901234567890123456789012123456 7 34567890123456789012345678901212345678901234567890123456789012123456 7 12 • If a problem has several correct answers, Decimal Accuracy 7 1234567890123456789012345678901212345678901234567890123456789012123456 34567890123456789012345678901212345678901234567890123456789012123456 7 12 just grid in one of them. 34567890123456789012345678901212345678901234567890123456789012123456 7 12 Decimal answers must be gridded as accu1234567890123456789012345678901212345678901234567890123456789012123456 7 34567890123456789012345678901212345678901234567890123456789012123456 12 • There are no negative answers. rately as possible. The answer 0.3333 . . . 7 7 1234567890123456789012345678901212345678901234567890123456789012123456 34567890123456789012345678901212345678901234567890123456789012123456 7 12 must be gridded as .333. 34567890123456789012345678901212345678901234567890123456789012123456 7 12 • Never grid in mixed numbers. The answer 34567890123456789012345678901212345678901234567890123456789012123456 7 12 34567890123456789012345678901212345678901234567890123456789012123456 12 1 Less accurate values, such as .33 are not 7 7 1234567890123456789012345678901212345678901234567890123456789012123456 3 must be gridded as 16/5 or 3.2. If 34567890123456789012345678901212345678901234567890123456789012123456 7 12 acceptable. 5 34567890123456789012345678901212345678901234567890123456789012123456 7 12 is gridded, it will be read 34567890123456789012345678901212345678901234567890123456789012123456 7 12 3 1 / 5 34567890123456789012345678901212345678901234567890123456789012123456 7 12 1 1 31 2 34567890123456789012345678901212345678901234567890123456789012123456 7 1 Acceptable ways to grid 5 .3333 . . . O Þ as , not 3 . 34567890123456789012345678901212345678901234567890123456789012123456 7 12 3 O O O O 5 5 34567890123456789012345678901212345678901234567890123456789012123456 7 12 34567890123456789012345678901212345678901234567890123456789012123456 7 12 O O O 34567890123456789012345678901212345678901234567890123456789012123456 7 12 . 3 3 3 1 / 3 O Þ O O 2 34567890123456789012345678901212345678901234567890123456789012123456 7 1 34567890123456789012345678901212345678901234567890123456789012123456 7 12 O O O O O O Þ O 34567890123456789012345678901212345678901234567890123456789012123456 7 12 Þ O O O Þ O O O O O O O 34567890123456789012345678901212345678901234567890123456789012123456 7 12 O O O O 34567890123456789012345678901212345678901234567890123456789012123456 7 12 O O O O O O O O O Þ 2 34567890123456789012345678901212345678901234567890123456789012123456 7 1234567890123456789012345678901212345678901234567890123456789012123456 O O O O Þ O O O 7 1234567890123456789012345678901212345678901234567890123456789012123456 O O O O O O O O 7 1234567890123456789012345678901212345678901234567890123456789012123456 O Þ Þ Þ O O Þ O 7 1234567890123456789012345678901212345678901234567890123456789012123456 7 1234567890123456789012345678901212345678901234567890123456789012123456 O O O O O O O O 7 1234567890123456789012345678901212345678901234567890123456789012123456 O O O O O O O O 7 1 O O O O O O O O 34567890123456789012345678901212345678901234567890123456789012123456 7 12 34567890123456789012345678901212345678901234567890123456789012123456 7 12 34567890123456789012345678901212345678901234567890123456789012123456 7 12 2 7 1234567890123456789012345678901212345678901234567890123456789012123456 7 1234567890123456789012345678901212345678901234567890123456789012123456 7 1234567890123456789012345678901212345678901234567890123456789012123456 7 1234567890123456789012345678901212345678901234567890123456789012123456 2 2 2 7 1234567890123456789012345678901212345678901234567890123456789012123456 1. [2(3 ) 1 (4 2 3(4)) ] 5 7. Given P(3, 2) and Q(3, 6), what is the 34567890123456789012345678901212345678901234567890123456789012123456 1234567890123456789012345678901212345678901234567890123456789012123456 77 1234567890123456789012345678901212345678901234567890123456789012123456 y-coordinate of the midpoint of the line 1234567890123456789012345678901212345678901234567890123456789012123456 77 1234567890123456789012345678901212345678901234567890123456789012123456 that 22 2 3x . 13, what is a segment defined by these two points? 7 1 2. Given positive integer that is less than x? 2 34567890123456789012345678901212345678901234567890123456789012123456 1234567890123456789012345678901212345678901234567890123456789012123456 77 1234567890123456789012345678901212345678901234567890123456789012123456 8. 7 1 7 1234567890123456789012345678901212345678901234567890123456789012123456 3. A phone company charges 40 cents for a 34567890123456789012345678901212345678901234567890123456789012123456 7 12 2 34567890123456789012345678901212345678901234567890123456789012123456 7 1234567890123456789012345678901212345678901234567890123456789012123456 completed long-distance phone call and 6 7 1234567890123456789012345678901212345678901234567890123456789012123456 7 1 cents per minute on top of the initial fee. 34567890123456789012345678901212345678901234567890123456789012123456 7 12 34567890123456789012345678901212345678901234567890123456789012123456 7 12 How much, in dollars, would a 30-minute 34567890123456789012345678901212345678901234567890123456789012123456 7 12 34567890123456789012345678901212345678901234567890123456789012123456 7 12 long-distance phone call cost? 1234567890123456789012345678901212345678901234567890123456789012123456 7 1234567890123456789012345678901212345678901234567890123456789012123456 7 7 1234567890123456789012345678901212345678901234567890123456789012123456 4. What is the least common multiple of 18 34567890123456789012345678901212345678901234567890123456789012123456 7 12 34567890123456789012345678901212345678901234567890123456789012123456 7 12 and 24? 34567890123456789012345678901212345678901234567890123456789012123456 7 12 34567890123456789012345678901212345678901234567890123456789012123456 7 12 1234567890123456789012345678901212345678901234567890123456789012123456 7 34567890123456789012345678901212345678901234567890123456789012123456 7 12 5. If |x 2 3| . 2, then what is one integer 34567890123456789012345678901212345678901234567890123456789012123456 7 12 34567890123456789012345678901212345678901234567890123456789012123456 7 12 value that x cannot equal? 34567890123456789012345678901212345678901234567890123456789012123456 7 12 7 1234567890123456789012345678901212345678901234567890123456789012123456 What is the length of the hypotenuse? 34567890123456789012345678901212345678901234567890123456789012123456 7 12 34567890123456789012345678901212345678901234567890123456789012123456 7 12 6. If the kth term in a series is generated by 34567890123456789012345678901212345678901234567890123456789012123456 7 12 9. What is the difference between the mean 10~k 1 1! 34567890123456789012345678901212345678901234567890123456789012123456 7 12 7 1234567890123456789012345678901212345678901234567890123456789012123456 , what is the the equation n 5 and the median of the number set {7, 4, 3, 2 34567890123456789012345678901212345678901234567890123456789012123456 7 1234567890123456789012345678901212345678901234567890123456789012123456 k21 7 1 eleventh term in the series? 6, 2, 2}? 34567890123456789012345678901212345678901234567890123456789012123456 7 12 1234567890123456789012345678901212345678901234567890123456789012123456 7 34567890123456789012345678901212345678901234567890123456789012123456 7 12 1234567890123456789012345678901212345678901234567890123456789012123456 7 1234567890123456789012345678901212345678901234567890123456789012123456 7 1234567890123456789012345678901212345678901234567890123456789012123456 7 1234567890123456789012345678901212345678901234567890123456789012123456 7 12345678901234567890123456789012123456789012345678901234567890121234567 /

.

.

.

.

0

0

0

1

1

2

2

2

3

3

3

4

4

4

4

5

5

5

1 2

44

/

/

.

.

.

0

0

0

1

1

1

2

2

2

4

4

4

4

5

5

5

5

6

6

6

6

/

.

.

.

.

0

0

0

1

1

1

1

2

2

2

2

2

3

3

3

3

4

4

4

4

5

5

5

5

6

6

6

6

Copyright © 2005 Thomson Peterson’s, a part of The Thomson Corporaton SAT is a registered trademark of the College Entrance Examination Board, which was not involved in the production of and does not endorse this product.

12345678901234567890123456789012123456789012345678901234567890121234567 1234567890123456789012345678901212345678901234567890123456789012123456 7 7 12234567890123456789012345678901212345678901234567890123456789012123456 10. If line a and line b are perpendicular, line 13. What is the one integer that is between 7 1 34567890123456789012345678901212345678901234567890123456789012123456 7 12234567890123456789012345678901212345678901234567890123456789012123456 b and line c are perpendicular, and the 999 and 1199 and satisfies these two 7 1234567890123456789012345678901212345678901234567890123456789012123456 7 1 34567890123456789012345678901212345678901234567890123456789012123456 2 conditions: 7 12234567890123456789012345678901212345678901234567890123456789012123456 slope of line a is , what is the slope of 34567890123456789012345678901212345678901234567890123456789012123456 7 1 Its digits sum to seventeen. 5 7 12234567890123456789012345678901212345678901234567890123456789012123456 7 1234567890123456789012345678901212345678901234567890123456789012123456 line c? 7 1234567890123456789012345678901212345678901234567890123456789012123456 Its tens and units digits are the same. 7 1234567890123456789012345678901212345678901234567890123456789012123456 34567890123456789012345678901212345678901234567890123456789012123456 7 1 7 12234567890123456789012345678901212345678901234567890123456789012123456 11. If f(x) 5 x2 1 3 and x 5 y 1 2, when 7 1234567890123456789012345678901212345678901234567890123456789012123456 y 5 3, f(x) 5 7 1234567890123456789012345678901212345678901234567890123456789012123456 7 1234567890123456789012345678901212345678901234567890123456789012123456 34567890123456789012345678901212345678901234567890123456789012123456 7 1 7 12234567890123456789012345678901212345678901234567890123456789012123456 12. If a square is inscribed in a circle and the 7 1234567890123456789012345678901212345678901234567890123456789012123456 7 1234567890123456789012345678901212345678901234567890123456789012123456 area of the square is eight, what is the 7 1234567890123456789012345678901212345678901234567890123456789012123456 34567890123456789012345678901212345678901234567890123456789012123456 7 1 longest distance that a straight line could 7 12234567890123456789012345678901212345678901234567890123456789012123456 7 1234567890123456789012345678901212345678901234567890123456789012123456 be that originated and terminated on the 7 1234567890123456789012345678901212345678901234567890123456789012123456 7 1234567890123456789012345678901212345678901234567890123456789012123456 circumference of the circle? 34567890123456789012345678901212345678901234567890123456789012123456 1234567890123456789012345678901212345678901234567890123456789012123456 77 1234567890123456789012345678901212345678901234567890123456789012123456 7 1234567890123456789012345678901212345678901234567890123456789012123456 7 1234567890123456789012345678901212345678901234567890123456789012123456 7 1234567890123456789012345678901212345678901234567890123456789012123456 7 1234567890123456789012345678901212345678901234567890123456789012123456 7 1 7 12234567890123456789012345678901212345678901234567890123456789012123456 7 1234567890123456789012345678901212345678901234567890123456789012123456 7 1234567890123456789012345678901212345678901234567890123456789012123456 34567890123456789012345678901212345678901234567890123456789012123456 1234567890123456789012345678901212345678901234567890123456789012123456 77 1234567890123456789012345678901212345678901234567890123456789012123456 7 1234567890123456789012345678901212345678901234567890123456789012123456 7 1234567890123456789012345678901212345678901234567890123456789012123456 7 1234567890123456789012345678901212345678901234567890123456789012123456 7 1234567890123456789012345678901212345678901234567890123456789012123456 7 1234567890123456789012345678901212345678901234567890123456789012123456 7 1234567890123456789012345678901212345678901234567890123456789012123456 7 1234567890123456789012345678901212345678901234567890123456789012123456 7 1 7 12234567890123456789012345678901212345678901234567890123456789012123456 7 1234567890123456789012345678901212345678901234567890123456789012123456 1 34567890123456789012345678901212345678901234567890123456789012123456 7 1234567890123456789012345678901212345678901234567890123456789012123456 7 7 12234567890123456789012345678901212345678901234567890123456789012123456 7 1234567890123456789012345678901212345678901234567890123456789012123456 7 1234567890123456789012345678901212345678901234567890123456789012123456 1 34567890123456789012345678901212345678901234567890123456789012123456 7 7 12234567890123456789012345678901212345678901234567890123456789012123456 7 1234567890123456789012345678901212345678901234567890123456789012123456 34567890123456789012345678901212345678901234567890123456789012123456 1234567890123456789012345678901212345678901234567890123456789012123456 77 1234567890123456789012345678901212345678901234567890123456789012123456 7 1 1234567890123456789012345678901212345678901234567890123456789012123456 7 1234567890123456789012345678901212345678901234567890123456789012123456 7 7 12234567890123456789012345678901212345678901234567890123456789012123456 7 1234567890123456789012345678901212345678901234567890123456789012123456 7 1234567890123456789012345678901212345678901234567890123456789012123456 7 1234567890123456789012345678901212345678901234567890123456789012123456 1 34567890123456789012345678901212345678901234567890123456789012123456 7 2 34567890123456789012345678901212345678901234567890123456789012123456 1234567890123456789012345678901212345678901234567890123456789012123456 77 1234567890123456789012345678901212345678901234567890123456789012123456 7 1234567890123456789012345678901212345678901234567890123456789012123456 7 1234567890123456789012345678901212345678901234567890123456789012123456 7 1 7 12234567890123456789012345678901212345678901234567890123456789012123456 7 1234567890123456789012345678901212345678901234567890123456789012123456 7 1234567890123456789012345678901212345678901234567890123456789012123456 7 1234567890123456789012345678901212345678901234567890123456789012123456 1 34567890123456789012345678901212345678901234567890123456789012123456 7 2 34567890123456789012345678901212345678901234567890123456789012123456 1234567890123456789012345678901212345678901234567890123456789012123456 77 1 7 12234567890123456789012345678901212345678901234567890123456789012123456 1 34567890123456789012345678901212345678901234567890123456789012123456 7 7 12234567890123456789012345678901212345678901234567890123456789012123456 Do not proceed to the next section until time is up. 7 1 34567890123456789012345678901212345678901234567890123456789012123456 1234567890123456789012345678901212345678901234567890123456789012123456 7 1234567890123456789012345678901212345678901234567890123456789012123456 7 1234567890123456789012345678901212345678901234567890123456789012123456 7 12345678901234567890123456789012123456789012345678901234567890121234567

STOP

Copyright © 2005 Thomson Peterson’s, a part of The Thomson Corporaton SAT is a registered trademark of the College Entrance Examination Board, which was not involved in the production of and does not endorse this product.

45

Section 8 Time—25 Minutes 12345678901234567890123456789012123456789012345678901234567890121234567 34567890123456789012345678901212345678901234567890123456789012123456 7 12 34567890123456789012345678901212345678901234567890123456789012123456 7 12 2 1 34567890123456789012345678901212345678901234567890123456789012123456 7 34567890123456789012345678901212345678901234567890123456789012123456 7 12 Directions: Think carefully about the statement below and the assignment that follows it. 34567890123456789012345678901212345678901234567890123456789012123456 7 12 34567890123456789012345678901212345678901234567890123456789012123456 7 12 34567890123456789012345678901212345678901234567890123456789012123456 7 12 34567890123456789012345678901212345678901234567890123456789012123456 7 12 Consider the following statement. 34567890123456789012345678901212345678901234567890123456789012123456 7 12 34567890123456789012345678901212345678901234567890123456789012123456 7 12 34567890123456789012345678901212345678901234567890123456789012123456 7 12 Some people think that war is justified under certain circumstances. 34567890123456789012345678901212345678901234567890123456789012123456 7 12 2 34567890123456789012345678901212345678901234567890123456789012123456 1234567890123456789012345678901212345678901234567890123456789012123456 77 1 Assignment: Write an essay in which you agree or disagree with the preceding statement. Develop your 34567890123456789012345678901212345678901234567890123456789012123456 7 12 34567890123456789012345678901212345678901234567890123456789012123456 7 12 point of view on this statement and be sure to support your stance with sufficient details. 34567890123456789012345678901212345678901234567890123456789012123456 7 12 2 34567890123456789012345678901212345678901234567890123456789012123456 1234567890123456789012345678901212345678901234567890123456789012123456 77 1234567890123456789012345678901212345678901234567890123456789012123456 7 1234567890123456789012345678901212345678901234567890123456789012123456 7 1234567890123456789012345678901212345678901234567890123456789012123456 7 1234567890123456789012345678901212345678901234567890123456789012123456 7 1234567890123456789012345678901212345678901234567890123456789012123456 7 1234567890123456789012345678901212345678901234567890123456789012123456 7 1234567890123456789012345678901212345678901234567890123456789012123456 7 1234567890123456789012345678901212345678901234567890123456789012123456 7 1 34567890123456789012345678901212345678901234567890123456789012123456 7 12 34567890123456789012345678901212345678901234567890123456789012123456 7 12 1234567890123456789012345678901212345678901234567890123456789012123456 7 1234567890123456789012345678901212345678901234567890123456789012123456 7 34567890123456789012345678901212345678901234567890123456789012123456 7 12 34567890123456789012345678901212345678901234567890123456789012123456 7 12 34567890123456789012345678901212345678901234567890123456789012123456 7 12 1234567890123456789012345678901212345678901234567890123456789012123456 7 34567890123456789012345678901212345678901234567890123456789012123456 7 12 34567890123456789012345678901212345678901234567890123456789012123456 7 12 2 7 1234567890123456789012345678901212345678901234567890123456789012123456 7 1234567890123456789012345678901212345678901234567890123456789012123456 1 34567890123456789012345678901212345678901234567890123456789012123456 7 1234567890123456789012345678901212345678901234567890123456789012123456 7 1234567890123456789012345678901212345678901234567890123456789012123456 7 34567890123456789012345678901212345678901234567890123456789012123456 7 12 34567890123456789012345678901212345678901234567890123456789012123456 7 12 34567890123456789012345678901212345678901234567890123456789012123456 7 12 34567890123456789012345678901212345678901234567890123456789012123456 7 12 1234567890123456789012345678901212345678901234567890123456789012123456 7 2 7 1234567890123456789012345678901212345678901234567890123456789012123456 7 1234567890123456789012345678901212345678901234567890123456789012123456 7 1234567890123456789012345678901212345678901234567890123456789012123456 7 1234567890123456789012345678901212345678901234567890123456789012123456 1 34567890123456789012345678901212345678901234567890123456789012123456 7 2 34567890123456789012345678901212345678901234567890123456789012123456 1234567890123456789012345678901212345678901234567890123456789012123456 77 1234567890123456789012345678901212345678901234567890123456789012123456 7 1234567890123456789012345678901212345678901234567890123456789012123456 7 1234567890123456789012345678901212345678901234567890123456789012123456 7 1 34567890123456789012345678901212345678901234567890123456789012123456 7 12 1234567890123456789012345678901212345678901234567890123456789012123456 7 34567890123456789012345678901212345678901234567890123456789012123456 7 12 1234567890123456789012345678901212345678901234567890123456789012123456 7 34567890123456789012345678901212345678901234567890123456789012123456 7 12 1234567890123456789012345678901212345678901234567890123456789012123456 7 1234567890123456789012345678901212345678901234567890123456789012123456 7 1234567890123456789012345678901212345678901234567890123456789012123456 7 1234567890123456789012345678901212345678901234567890123456789012123456 7 12345678901234567890123456789012123456789012345678901234567890121234567 46

Copyright © 2005 Thomson Peterson’s, a part of The Thomson Corporaton SAT is a registered trademark of the College Entrance Examination Board, which was not involved in the production of and does not endorse this product.

12345678901234567890123456789012123456789012345678901234567890121234567 1234567890123456789012345678901212345678901234567890123456789012123456 7 7 12234567890123456789012345678901212345678901234567890123456789012123456 1 34567890123456789012345678901212345678901234567890123456789012123456 7 7 12234567890123456789012345678901212345678901234567890123456789012123456 7 1234567890123456789012345678901212345678901234567890123456789012123456 1 34567890123456789012345678901212345678901234567890123456789012123456 7 7 12234567890123456789012345678901212345678901234567890123456789012123456 1 34567890123456789012345678901212345678901234567890123456789012123456 7 7 12234567890123456789012345678901212345678901234567890123456789012123456 7 1234567890123456789012345678901212345678901234567890123456789012123456 7 1234567890123456789012345678901212345678901234567890123456789012123456 7 1234567890123456789012345678901212345678901234567890123456789012123456 34567890123456789012345678901212345678901234567890123456789012123456 7 1 7 12234567890123456789012345678901212345678901234567890123456789012123456 7 1234567890123456789012345678901212345678901234567890123456789012123456 7 1234567890123456789012345678901212345678901234567890123456789012123456 7 1234567890123456789012345678901212345678901234567890123456789012123456 34567890123456789012345678901212345678901234567890123456789012123456 7 1 7 12234567890123456789012345678901212345678901234567890123456789012123456 7 1234567890123456789012345678901212345678901234567890123456789012123456 7 1234567890123456789012345678901212345678901234567890123456789012123456 7 1234567890123456789012345678901212345678901234567890123456789012123456 34567890123456789012345678901212345678901234567890123456789012123456 7 1 7 12234567890123456789012345678901212345678901234567890123456789012123456 7 1234567890123456789012345678901212345678901234567890123456789012123456 7 1234567890123456789012345678901212345678901234567890123456789012123456 7 1234567890123456789012345678901212345678901234567890123456789012123456 34567890123456789012345678901212345678901234567890123456789012123456 1234567890123456789012345678901212345678901234567890123456789012123456 77 1234567890123456789012345678901212345678901234567890123456789012123456 7 1234567890123456789012345678901212345678901234567890123456789012123456 7 1234567890123456789012345678901212345678901234567890123456789012123456 7 1234567890123456789012345678901212345678901234567890123456789012123456 7 1234567890123456789012345678901212345678901234567890123456789012123456 7 1 7 12234567890123456789012345678901212345678901234567890123456789012123456 7 1234567890123456789012345678901212345678901234567890123456789012123456 7 1234567890123456789012345678901212345678901234567890123456789012123456 34567890123456789012345678901212345678901234567890123456789012123456 1234567890123456789012345678901212345678901234567890123456789012123456 77 1234567890123456789012345678901212345678901234567890123456789012123456 7 1234567890123456789012345678901212345678901234567890123456789012123456 7 1234567890123456789012345678901212345678901234567890123456789012123456 7 1234567890123456789012345678901212345678901234567890123456789012123456 7 1234567890123456789012345678901212345678901234567890123456789012123456 7 1234567890123456789012345678901212345678901234567890123456789012123456 7 1234567890123456789012345678901212345678901234567890123456789012123456 7 1234567890123456789012345678901212345678901234567890123456789012123456 7 1 7 12234567890123456789012345678901212345678901234567890123456789012123456 7 1234567890123456789012345678901212345678901234567890123456789012123456 1 34567890123456789012345678901212345678901234567890123456789012123456 7 1234567890123456789012345678901212345678901234567890123456789012123456 7 7 12234567890123456789012345678901212345678901234567890123456789012123456 7 1234567890123456789012345678901212345678901234567890123456789012123456 7 1234567890123456789012345678901212345678901234567890123456789012123456 1 34567890123456789012345678901212345678901234567890123456789012123456 7 7 12234567890123456789012345678901212345678901234567890123456789012123456 7 1234567890123456789012345678901212345678901234567890123456789012123456 34567890123456789012345678901212345678901234567890123456789012123456 1234567890123456789012345678901212345678901234567890123456789012123456 77 1234567890123456789012345678901212345678901234567890123456789012123456 7 1 1234567890123456789012345678901212345678901234567890123456789012123456 7 1234567890123456789012345678901212345678901234567890123456789012123456 7 7 12234567890123456789012345678901212345678901234567890123456789012123456 7 1234567890123456789012345678901212345678901234567890123456789012123456 7 1234567890123456789012345678901212345678901234567890123456789012123456 7 1234567890123456789012345678901212345678901234567890123456789012123456 1 34567890123456789012345678901212345678901234567890123456789012123456 7 2 34567890123456789012345678901212345678901234567890123456789012123456 1234567890123456789012345678901212345678901234567890123456789012123456 77 1234567890123456789012345678901212345678901234567890123456789012123456 7 1234567890123456789012345678901212345678901234567890123456789012123456 7 1234567890123456789012345678901212345678901234567890123456789012123456 7 1 7 12234567890123456789012345678901212345678901234567890123456789012123456 7 1234567890123456789012345678901212345678901234567890123456789012123456 7 1234567890123456789012345678901212345678901234567890123456789012123456 7 1234567890123456789012345678901212345678901234567890123456789012123456 1 34567890123456789012345678901212345678901234567890123456789012123456 7 2 34567890123456789012345678901212345678901234567890123456789012123456 1234567890123456789012345678901212345678901234567890123456789012123456 77 1 7 12234567890123456789012345678901212345678901234567890123456789012123456 1 34567890123456789012345678901212345678901234567890123456789012123456 7 7 12234567890123456789012345678901212345678901234567890123456789012123456 1 34567890123456789012345678901212345678901234567890123456789012123456 7 1234567890123456789012345678901212345678901234567890123456789012123456 7 1234567890123456789012345678901212345678901234567890123456789012123456 7 1234567890123456789012345678901212345678901234567890123456789012123456 7 12345678901234567890123456789012123456789012345678901234567890121234567 Copyright © 2005 Thomson Peterson’s, a part of The Thomson Corporaton SAT is a registered trademark of the College Entrance Examination Board, which was not involved in the production of and does not endorse this product.

47

12345678901234567890123456789012123456789012345678901234567890121234567 1234567890123456789012345678901212345678901234567890123456789012123456 7 34567890123456789012345678901212345678901234567890123456789012123456 7 12 1234567890123456789012345678901212345678901234567890123456789012123456 7 34567890123456789012345678901212345678901234567890123456789012123456 7 12 34567890123456789012345678901212345678901234567890123456789012123456 7 12 1234567890123456789012345678901212345678901234567890123456789012123456 7 34567890123456789012345678901212345678901234567890123456789012123456 7 12 1234567890123456789012345678901212345678901234567890123456789012123456 7 34567890123456789012345678901212345678901234567890123456789012123456 7 12 34567890123456789012345678901212345678901234567890123456789012123456 7 12 34567890123456789012345678901212345678901234567890123456789012123456 7 12 34567890123456789012345678901212345678901234567890123456789012123456 7 12 2 1 34567890123456789012345678901212345678901234567890123456789012123456 7 34567890123456789012345678901212345678901234567890123456789012123456 7 12 34567890123456789012345678901212345678901234567890123456789012123456 7 12 34567890123456789012345678901212345678901234567890123456789012123456 7 12 34567890123456789012345678901212345678901234567890123456789012123456 7 12 2 1 34567890123456789012345678901212345678901234567890123456789012123456 7 34567890123456789012345678901212345678901234567890123456789012123456 7 12 34567890123456789012345678901212345678901234567890123456789012123456 7 12 34567890123456789012345678901212345678901234567890123456789012123456 7 12 34567890123456789012345678901212345678901234567890123456789012123456 7 12 2 1 34567890123456789012345678901212345678901234567890123456789012123456 7 34567890123456789012345678901212345678901234567890123456789012123456 7 12 34567890123456789012345678901212345678901234567890123456789012123456 7 12 34567890123456789012345678901212345678901234567890123456789012123456 7 12 34567890123456789012345678901212345678901234567890123456789012123456 7 12 2 7 1234567890123456789012345678901212345678901234567890123456789012123456 7 1234567890123456789012345678901212345678901234567890123456789012123456 7 1234567890123456789012345678901212345678901234567890123456789012123456 7 1234567890123456789012345678901212345678901234567890123456789012123456 7 1234567890123456789012345678901212345678901234567890123456789012123456 34567890123456789012345678901212345678901234567890123456789012123456 1234567890123456789012345678901212345678901234567890123456789012123456 77 1 34567890123456789012345678901212345678901234567890123456789012123456 7 12 34567890123456789012345678901212345678901234567890123456789012123456 7 12 34567890123456789012345678901212345678901234567890123456789012123456 7 12 2 7 1234567890123456789012345678901212345678901234567890123456789012123456 7 1234567890123456789012345678901212345678901234567890123456789012123456 7 1234567890123456789012345678901212345678901234567890123456789012123456 7 1234567890123456789012345678901212345678901234567890123456789012123456 7 1234567890123456789012345678901212345678901234567890123456789012123456 34567890123456789012345678901212345678901234567890123456789012123456 1234567890123456789012345678901212345678901234567890123456789012123456 77 1234567890123456789012345678901212345678901234567890123456789012123456 7 1234567890123456789012345678901212345678901234567890123456789012123456 7 1234567890123456789012345678901212345678901234567890123456789012123456 7 1 34567890123456789012345678901212345678901234567890123456789012123456 7 12 34567890123456789012345678901212345678901234567890123456789012123456 7 12 1234567890123456789012345678901212345678901234567890123456789012123456 7 1234567890123456789012345678901212345678901234567890123456789012123456 7 34567890123456789012345678901212345678901234567890123456789012123456 7 12 34567890123456789012345678901212345678901234567890123456789012123456 7 12 34567890123456789012345678901212345678901234567890123456789012123456 7 12 1234567890123456789012345678901212345678901234567890123456789012123456 7 34567890123456789012345678901212345678901234567890123456789012123456 7 12 34567890123456789012345678901212345678901234567890123456789012123456 7 12 2 7 1234567890123456789012345678901212345678901234567890123456789012123456 7 1234567890123456789012345678901212345678901234567890123456789012123456 1 34567890123456789012345678901212345678901234567890123456789012123456 7 1234567890123456789012345678901212345678901234567890123456789012123456 7 1234567890123456789012345678901212345678901234567890123456789012123456 7 34567890123456789012345678901212345678901234567890123456789012123456 7 12 34567890123456789012345678901212345678901234567890123456789012123456 7 12 34567890123456789012345678901212345678901234567890123456789012123456 7 12 34567890123456789012345678901212345678901234567890123456789012123456 7 12 1234567890123456789012345678901212345678901234567890123456789012123456 7 2 7 1234567890123456789012345678901212345678901234567890123456789012123456 7 1234567890123456789012345678901212345678901234567890123456789012123456 7 1234567890123456789012345678901212345678901234567890123456789012123456 7 1234567890123456789012345678901212345678901234567890123456789012123456 1 34567890123456789012345678901212345678901234567890123456789012123456 7 2 34567890123456789012345678901212345678901234567890123456789012123456 1234567890123456789012345678901212345678901234567890123456789012123456 77 1234567890123456789012345678901212345678901234567890123456789012123456 7 1234567890123456789012345678901212345678901234567890123456789012123456 7 1234567890123456789012345678901212345678901234567890123456789012123456 7 1 34567890123456789012345678901212345678901234567890123456789012123456 7 12 1234567890123456789012345678901212345678901234567890123456789012123456 7 34567890123456789012345678901212345678901234567890123456789012123456 7 12 1234567890123456789012345678901212345678901234567890123456789012123456 7 34567890123456789012345678901212345678901234567890123456789012123456 7 12 When you are finished with your essay put your pencil down until the 7 1234567890123456789012345678901212345678901234567890123456789012123456 7 1234567890123456789012345678901212345678901234567890123456789012123456 time allotted is over. 1234567890123456789012345678901212345678901234567890123456789012123456 77 1234567890123456789012345678901212345678901234567890123456789012123456 12345678901234567890123456789012123456789012345678901234567890121234567

STOP

48

Copyright © 2005 Thomson Peterson’s, a part of The Thomson Corporaton SAT is a registered trademark of the College Entrance Examination Board, which was not involved in the production of and does not endorse this product.

Answers and Explanations 1234567890123456789012345678901212345678901234567890123456789 9 123456789012345678901234567890121234567890123456789012345678 9 123456789012345678901234567890121234567890123456789012345678 9 123456789012345678901234567890121234567890123456789012345678 Section 1 9 123456789012345678901234567890121234567890123456789012345678 9 123456789012345678901234567890121234567890123456789012345678 23456789012345678901234567890121234567890123456789012345678 9 123456789012345678901234567890121234567890123456789012345678 1. C The passage begins, “Musical notes, like all sounds, are a result of the 9 1 9 123456789012345678901234567890121234567890123456789012345678 sound waves created by movement.” The author then goes on to talk 9 123456789012345678901234567890121234567890123456789012345678 9 123456789012345678901234567890121234567890123456789012345678 about musical notes and how they illustrate properties of sound waves. 23456789012345678901234567890121234567890123456789012345678 123456789012345678901234567890121234567890123456789012345678 99 123456789012345678901234567890121234567890123456789012345678 Choice (C) captures this idea. 9 123456789012345678901234567890121234567890123456789012345678 9 123456789012345678901234567890121234567890123456789012345678 2. A Pitch is determined by the frequency of the sound wave. This eliminates 9 123456789012345678901234567890121234567890123456789012345678 9 123456789012345678901234567890121234567890123456789012345678 (B) and (E). Choice (C) seems to refer more to the intensity, so eliminate 9 123456789012345678901234567890121234567890123456789012345678 9 123456789012345678901234567890121234567890123456789012345678 it too. The final sentence says that pitch can be described either in 9 123456789012345678901234567890121234567890123456789012345678 9 1 numbers or in letters, so eliminate (D). That leaves (A), the cor23456789012345678901234567890121234567890123456789012345678 123456789012345678901234567890121234567890123456789012345678 99 123456789012345678901234567890121234567890123456789012345678 rect answer. 9 1 123456789012345678901234567890121234567890123456789012345678 9 9 123456789012345678901234567890121234567890123456789012345678 3. B The passage states that Langston Hughes “persuaded her to continue 9 123456789012345678901234567890121234567890123456789012345678 9 123456789012345678901234567890121234567890123456789012345678 her education in the North.” And the passage uses this fact to explain 123456789012345678901234567890121234567890123456789012345678 9 9 123456789012345678901234567890121234567890123456789012345678 her transfer to Northwestern. This is what (B), the correct an9 123456789012345678901234567890121234567890123456789012345678 23456789012345678901234567890121234567890123456789012345678 9 123456789012345678901234567890121234567890123456789012345678 swer, suggests. 9 123456789012345678901234567890121234567890123456789012345678 9 1 9 123456789012345678901234567890121234567890123456789012345678 4. C The passage doesn’t specifically say that Walker was writing poetry 123456789012345678901234567890121234567890123456789012345678 9 23456789012345678901234567890121234567890123456789012345678 before she entered New Orleans University. Eliminate (A). Hughes 123456789012345678901234567890121234567890123456789012345678 99 123456789012345678901234567890121234567890123456789012345678 recognized her talent, but he didn’t create it, so eliminate (B). Hughes 9 123456789012345678901234567890121234567890123456789012345678 9 123456789012345678901234567890121234567890123456789012345678 recognized her talent before she transferred to Northwestern, so 9 1 23456789012345678901234567890121234567890123456789012345678 9 123456789012345678901234567890121234567890123456789012345678 eliminate (D). The passage, if anything, implies that Walker wrote 9 123456789012345678901234567890121234567890123456789012345678 9 123456789012345678901234567890121234567890123456789012345678 poetry for some time before publishing anything, so eliminate (E). The 123456789012345678901234567890121234567890123456789012345678 99 1 passage makes reference to her parents’ occupations and encourage9 123456789012345678901234567890121234567890123456789012345678 9 123456789012345678901234567890121234567890123456789012345678 ment, implying that they had an influence on her decision to become 9 123456789012345678901234567890121234567890123456789012345678 9 123456789012345678901234567890121234567890123456789012345678 a poet. 123456789012345678901234567890121234567890123456789012345678 9 23456789012345678901234567890121234567890123456789012345678 123456789012345678901234567890121234567890123456789012345678 99 1 5. C The author is poking a bit of fun at the Ungers, so eliminate (A), (B), 9 123456789012345678901234567890121234567890123456789012345678 9 123456789012345678901234567890121234567890123456789012345678 and (E). His tone is more playful than downtrodden, so the answer 9 123456789012345678901234567890121234567890123456789012345678 23456789012345678901234567890121234567890123456789012345678 9 1 is (C). 9 123456789012345678901234567890121234567890123456789012345678 123456789012345678901234567890121234567890123456789012345678 99 123456789012345678901234567890121234567890123456789012345678 1234567890123456789012345678901212345678901234567890123456789 Copyright © 2005 Thomson Peterson’s, a part of The Thomson Corporaton SAT is a registered trademark of the College Entrance Examination Board, which was not involved in the production of and does not endorse this product.

49

1234567890123456789012345678901212345678901234567890123456789 123456789012345678901234567890121234567890123456789012345678 9 9 123456789012345678901234567890121234567890123456789012345678 123456789012345678901234567890121234567890123456789012345678 9 9 123456789012345678901234567890121234567890123456789012345678 6. D The “Chicago beef-princess” suggests the wider high-class social world 9 123456789012345678901234567890121234567890123456789012345678 9 123456789012345678901234567890121234567890123456789012345678 in Chicago. When one thing stands in for another, it is a metaphor. The 9 123456789012345678901234567890121234567890123456789012345678 23456789012345678901234567890121234567890123456789012345678 9 1 answer is (D). 9 123456789012345678901234567890121234567890123456789012345678 9 123456789012345678901234567890121234567890123456789012345678 9 123456789012345678901234567890121234567890123456789012345678 7. A Even if you do not know the definition of fatuity you can still get this 9 123456789012345678901234567890121234567890123456789012345678 23456789012345678901234567890121234567890123456789012345678 9 1 question. John is going from Hades, which we can assume is hot, to 9 123456789012345678901234567890121234567890123456789012345678 9 123456789012345678901234567890121234567890123456789012345678 Boston. He will probably not need the light suits and fans. The answer 9 123456789012345678901234567890121234567890123456789012345678 9 123456789012345678901234567890121234567890123456789012345678 is (A). 23456789012345678901234567890121234567890123456789012345678 9 1 9 123456789012345678901234567890121234567890123456789012345678 8. B We know that John does not feel rejected, because he says he knows he 9 123456789012345678901234567890121234567890123456789012345678 9 123456789012345678901234567890121234567890123456789012345678 will always be welcome at home. Eliminate (A). On the other hand, he 9 123456789012345678901234567890121234567890123456789012345678 23456789012345678901234567890121234567890123456789012345678 9 1 does feel something negative, or he wouldn’t cry. Eliminate (C) and (E). 9 123456789012345678901234567890121234567890123456789012345678 9 123456789012345678901234567890121234567890123456789012345678 The handshake and the fact that John’s tears are not mentioned until he 9 123456789012345678901234567890121234567890123456789012345678 9 123456789012345678901234567890121234567890123456789012345678 has turned away from his father suggest that he is composed. The best 23456789012345678901234567890121234567890123456789012345678 123456789012345678901234567890121234567890123456789012345678 9 9 123456789012345678901234567890121234567890123456789012345678 answer is (B). 9 123456789012345678901234567890121234567890123456789012345678 123456789012345678901234567890121234567890123456789012345678 9 9 123456789012345678901234567890121234567890123456789012345678 9. C If you were leaving home (and you were crying), why would you stop 9 123456789012345678901234567890121234567890123456789012345678 9 1 and look back? Most likely you would do so because you were sad to 9 123456789012345678901234567890121234567890123456789012345678 9 123456789012345678901234567890121234567890123456789012345678 leave and wanted to get one last look before you went. Which of the 9 123456789012345678901234567890121234567890123456789012345678 23456789012345678901234567890121234567890123456789012345678 9 123456789012345678901234567890121234567890123456789012345678 answer choices matches this sentiment? Choice (C) does. The medita9 123456789012345678901234567890121234567890123456789012345678 9 123456789012345678901234567890121234567890123456789012345678 tion on what the sign says serves to emphasize the quaintness of the 123456789012345678901234567890121234567890123456789012345678 9 9 123456789012345678901234567890121234567890123456789012345678 town, of which John will no longer be a part. The other answers rely on 9 123456789012345678901234567890121234567890123456789012345678 9 123456789012345678901234567890121234567890123456789012345678 your being distracted from the main emotions of the story. 123456789012345678901234567890121234567890123456789012345678 9 9 123456789012345678901234567890121234567890123456789012345678 9 1 10. E Hades is hell in Greco-Roman mythology. Midas represents wealth. 23456789012345678901234567890121234567890123456789012345678 9 123456789012345678901234567890121234567890123456789012345678 Unger resonates with the hunger the family feels for the wealth and 9 123456789012345678901234567890121234567890123456789012345678 9 1 prestige of the North. In other words, the names suggest that the story 123456789012345678901234567890121234567890123456789012345678 9 9 123456789012345678901234567890121234567890123456789012345678 uses the experiences of this one family to represent a larger situation. It 9 123456789012345678901234567890121234567890123456789012345678 9 123456789012345678901234567890121234567890123456789012345678 is an allegory, choice (E). 123456789012345678901234567890121234567890123456789012345678 9 9 123456789012345678901234567890121234567890123456789012345678 9 123456789012345678901234567890121234567890123456789012345678 11. C Infused is used to mean that his work was filled with the experiences he 23456789012345678901234567890121234567890123456789012345678 123456789012345678901234567890121234567890123456789012345678 9 9 123456789012345678901234567890121234567890123456789012345678 had in Manchuria. Eliminate all but (B) and (C). Saturated has 9 1 9 123456789012345678901234567890121234567890123456789012345678 something of a negative tone, and the author praises Abe’s work, so 123456789012345678901234567890121234567890123456789012345678 9 23456789012345678901234567890121234567890123456789012345678 9 123456789012345678901234567890121234567890123456789012345678 eliminate (B). The answer is (C). 9 123456789012345678901234567890121234567890123456789012345678 9 123456789012345678901234567890121234567890123456789012345678 12. E The metaphorical use of orbit and gravitational pull is used in 9 123456789012345678901234567890121234567890123456789012345678 9 1 conjunction with the negative words “controlled” and “oppressive.” 23456789012345678901234567890121234567890123456789012345678 123456789012345678901234567890121234567890123456789012345678 9 9 123456789012345678901234567890121234567890123456789012345678 Abe’s work is not controlled by oppressive forces. Eliminate (B), (C), 9 123456789012345678901234567890121234567890123456789012345678 9 123456789012345678901234567890121234567890123456789012345678 and (D). Choices (A) and (E) are similar answers, but (E) better captures 9 1 23456789012345678901234567890121234567890123456789012345678 9 123456789012345678901234567890121234567890123456789012345678 the author’s intent. 9 123456789012345678901234567890121234567890123456789012345678 123456789012345678901234567890121234567890123456789012345678 9 9 123456789012345678901234567890121234567890123456789012345678 9 1 13. D Abe forged a medical certificate, so we know he was not actually sick. 23456789012345678901234567890121234567890123456789012345678 9 123456789012345678901234567890121234567890123456789012345678 You can eliminate (A) and (B). The passage makes no reference to Abe 9 1 9 123456789012345678901234567890121234567890123456789012345678 helping the sick and injured, so eliminate (C). The sentence in the 123456789012345678901234567890121234567890123456789012345678 9 9 123456789012345678901234567890121234567890123456789012345678 passage says that the forged medical certificate allowed him to avoid 9 123456789012345678901234567890121234567890123456789012345678 123456789012345678901234567890121234567890123456789012345678 9 123456789012345678901234567890121234567890123456789012345678 9 123456789012345678901234567890121234567890123456789012345678 9 1234567890123456789012345678901212345678901234567890123456789 50

Copyright © 2005 Thomson Peterson’s, a part of The Thomson Corporaton SAT is a registered trademark of the College Entrance Examination Board, which was not involved in the production of and does not endorse this product.

1234567890123456789012345678901212345678901234567890123456789 123456789012345678901234567890121234567890123456789012345678 9 9 123456789012345678901234567890121234567890123456789012345678 123456789012345678901234567890121234567890123456789012345678 9 9 123456789012345678901234567890121234567890123456789012345678 fighting. Choice (D) corresponds with that meaning. Choice (E) can be 9 123456789012345678901234567890121234567890123456789012345678 9 123456789012345678901234567890121234567890123456789012345678 eliminated because you don’t know what his intentions were for after 9 123456789012345678901234567890121234567890123456789012345678 23456789012345678901234567890121234567890123456789012345678 9 1 the war. 9 123456789012345678901234567890121234567890123456789012345678 9 123456789012345678901234567890121234567890123456789012345678 9 123456789012345678901234567890121234567890123456789012345678 14. E Even if you don’t know the defintion of apocalyptic, you probably 9 123456789012345678901234567890121234567890123456789012345678 23456789012345678901234567890121234567890123456789012345678 9 1 know that it is a negative word. Eliminate (D) (exhilarating is a positive 9 123456789012345678901234567890121234567890123456789012345678 9 123456789012345678901234567890121234567890123456789012345678 word). There is no reference to nuclear weapons in the passage, so 9 123456789012345678901234567890121234567890123456789012345678 9 123456789012345678901234567890121234567890123456789012345678 eliminate (A). There was famine, and Abe seems to have been strongly 23456789012345678901234567890121234567890123456789012345678 9 1 9 123456789012345678901234567890121234567890123456789012345678 affected by the loss of his father, but neither of these answers is specific 9 123456789012345678901234567890121234567890123456789012345678 9 enough. Only (E) expressly answers the question. 123456789012345678901234567890121234567890123456789012345678 9 123456789012345678901234567890121234567890123456789012345678 23456789012345678901234567890121234567890123456789012345678 1 15. A This question is a little bit more difficult than some vocabulary 9 9 123456789012345678901234567890121234567890123456789012345678 9 123456789012345678901234567890121234567890123456789012345678 questions because you have to look in a few different places. The third 9 123456789012345678901234567890121234567890123456789012345678 9 123456789012345678901234567890121234567890123456789012345678 paragraph, where the word appears, tells you that the avant-garde 23456789012345678901234567890121234567890123456789012345678 123456789012345678901234567890121234567890123456789012345678 99 123456789012345678901234567890121234567890123456789012345678 group was political and that Abe worked in various genres. The fourth 9 123456789012345678901234567890121234567890123456789012345678 9 123456789012345678901234567890121234567890123456789012345678 paragraph refers to his earlier work, which was the work in the third 123456789012345678901234567890121234567890123456789012345678 99 123456789012345678901234567890121234567890123456789012345678 paragraph, as “experimental and heavily political.” Since one of these 9 1 9 123456789012345678901234567890121234567890123456789012345678 words is an answer choice (A), it is the best answer. 9 123456789012345678901234567890121234567890123456789012345678 9 123456789012345678901234567890121234567890123456789012345678 23456789012345678901234567890121234567890123456789012345678 9 123456789012345678901234567890121234567890123456789012345678 16. C This question basically asks you to distinguish between the author’s 9 123456789012345678901234567890121234567890123456789012345678 9 123456789012345678901234567890121234567890123456789012345678 opinion and the basic facts of Abe’s career. Choices (A), (B), and (E) all 123456789012345678901234567890121234567890123456789012345678 99 123456789012345678901234567890121234567890123456789012345678 contain evaluative opinions, so eliminate them. The author expresses 9 123456789012345678901234567890121234567890123456789012345678 9 123456789012345678901234567890121234567890123456789012345678 strong opinions about the themes furusato and the emperor, but never 9 123456789012345678901234567890121234567890123456789012345678 presents any facts about their influence on Japanese literature in the 9 123456789012345678901234567890121234567890123456789012345678 9 1 world. The best answer is (C). The author presents it as a known fact 23456789012345678901234567890121234567890123456789012345678 123456789012345678901234567890121234567890123456789012345678 99 123456789012345678901234567890121234567890123456789012345678 that young Japanese artists after World War II were interested in 9 1 9 123456789012345678901234567890121234567890123456789012345678 Marxism. 9 123456789012345678901234567890121234567890123456789012345678 23456789012345678901234567890121234567890123456789012345678 123456789012345678901234567890121234567890123456789012345678 99 123456789012345678901234567890121234567890123456789012345678 1 17. B As always, go back to the passage to look for the context of the phrase. 9 9 123456789012345678901234567890121234567890123456789012345678 Shortly after the phrase appears, the author says that readers have 9 123456789012345678901234567890121234567890123456789012345678 23456789012345678901234567890121234567890123456789012345678 9 123456789012345678901234567890121234567890123456789012345678 wrongly decided that Woman in the Dunes was Abe’s masterpiece. The 9 123456789012345678901234567890121234567890123456789012345678 9 1 author also refers to the lack of translations of Abe’s earlier works. The 123456789012345678901234567890121234567890123456789012345678 9 9 123456789012345678901234567890121234567890123456789012345678 answer that best summarizes these two things is (B). 9 123456789012345678901234567890121234567890123456789012345678 9 123456789012345678901234567890121234567890123456789012345678 9 123456789012345678901234567890121234567890123456789012345678 18. D The author’s purpose in paragraph 4 is to suggest that too much 9 123456789012345678901234567890121234567890123456789012345678 9 123456789012345678901234567890121234567890123456789012345678 attention has been given to Abe’s later work, as you just determined in 23456789012345678901234567890121234567890123456789012345678 9 123456789012345678901234567890121234567890123456789012345678 question 17. So the answer cannot be (A). There is only a brief 9 123456789012345678901234567890121234567890123456789012345678 9 123456789012345678901234567890121234567890123456789012345678 comparison to Abe’s contemporaries, so (B) is too specific. (E) is not 9 123456789012345678901234567890121234567890123456789012345678 9 1 factually correct, since most of the work the passage discusses was 23456789012345678901234567890121234567890123456789012345678 123456789012345678901234567890121234567890123456789012345678 99 123456789012345678901234567890121234567890123456789012345678 produced in Japan. You are left with (C) and (D). (C) is too neutral; this 9 123456789012345678901234567890121234567890123456789012345678 9 123456789012345678901234567890121234567890123456789012345678 author is opinionated. She/he does not suggest that Abe’s later work is 9 1 23456789012345678901234567890121234567890123456789012345678 9 123456789012345678901234567890121234567890123456789012345678 bad, but rather that his early work also deserves attention. Choice (D) is 9 1 9 123456789012345678901234567890121234567890123456789012345678 the best answer. 123456789012345678901234567890121234567890123456789012345678 9 9 123456789012345678901234567890121234567890123456789012345678 123456789012345678901234567890121234567890123456789012345678 9 123456789012345678901234567890121234567890123456789012345678 9 123456789012345678901234567890121234567890123456789012345678 9 123456789012345678901234567890121234567890123456789012345678 9 1234567890123456789012345678901212345678901234567890123456789 Copyright © 2005 Thomson Peterson’s, a part of The Thomson Corporaton SAT is a registered trademark of the College Entrance Examination Board, which was not involved in the production of and does not endorse this product.

51

1234567890123456789012345678901212345678901234567890123456789 123456789012345678901234567890121234567890123456789012345678 9 9 123456789012345678901234567890121234567890123456789012345678 123456789012345678901234567890121234567890123456789012345678 9 9 123456789012345678901234567890121234567890123456789012345678 19. B The author is most interested in literary works. There is no reason to 9 123456789012345678901234567890121234567890123456789012345678 9 123456789012345678901234567890121234567890123456789012345678 suspect that the author is an artist or writer. The tone is critical and 9 123456789012345678901234567890121234567890123456789012345678 23456789012345678901234567890121234567890123456789012345678 9 1 scholarly. (B) is the best answer. 9 123456789012345678901234567890121234567890123456789012345678 9 123456789012345678901234567890121234567890123456789012345678 9 123456789012345678901234567890121234567890123456789012345678 20. D The author of this passage does express many strong opinions, but not 9 123456789012345678901234567890121234567890123456789012345678 23456789012345678901234567890121234567890123456789012345678 9 1 in regard to Marxism. You can therefore eliminate both (A) and (B). If 9 123456789012345678901234567890121234567890123456789012345678 9 123456789012345678901234567890121234567890123456789012345678 anything, she/he is more positive than negative about the influence of 9 123456789012345678901234567890121234567890123456789012345678 9 123456789012345678901234567890121234567890123456789012345678 Marxism on Abe’s work. Eliminate (C) and (E), which imply a negative 23456789012345678901234567890121234567890123456789012345678 9 1 9 123456789012345678901234567890121234567890123456789012345678 bias. The answer is (D). 9 123456789012345678901234567890121234567890123456789012345678 9 123456789012345678901234567890121234567890123456789012345678 9 123456789012345678901234567890121234567890123456789012345678 23456789012345678901234567890121234567890123456789012345678 9 1 Section 2 9 123456789012345678901234567890121234567890123456789012345678 9 123456789012345678901234567890121234567890123456789012345678 9 123456789012345678901234567890121234567890123456789012345678 1. C With a calculator this problem is straightforward enough, but you do not 9 123456789012345678901234567890121234567890123456789012345678 23456789012345678901234567890121234567890123456789012345678 9 123456789012345678901234567890121234567890123456789012345678 need a calculator to solve this problem. The wording is a bit tricky, but to 9 123456789012345678901234567890121234567890123456789012345678 9 123456789012345678901234567890121234567890123456789012345678 find the percentage of 75 that 12 represents, you would place 12 over 75. 123456789012345678901234567890121234567890123456789012345678 9 9 123456789012345678901234567890121234567890123456789012345678 4 12 12/3 9 123456789012345678901234567890121234567890123456789012345678 5 5 . Percent means “of 100,” so if you This can be simplified 9 1 75 75/3 25 9 123456789012345678901234567890121234567890123456789012345678 9 123456789012345678901234567890121234567890123456789012345678 change the 25 in the denominator to 100, you’ll have your percentage in 9 123456789012345678901234567890121234567890123456789012345678 4 3 4 16 4 23456789012345678901234567890121234567890123456789012345678 123456789012345678901234567890121234567890123456789012345678 9 9 123456789012345678901234567890121234567890123456789012345678 5 5 . This is answer (C). the numerator. 9 123456789012345678901234567890121234567890123456789012345678 25 25 3 4 100 123456789012345678901234567890121234567890123456789012345678 9 9 123456789012345678901234567890121234567890123456789012345678 You could also have set up an algebraic equation, and then cross9 123456789012345678901234567890121234567890123456789012345678 9 123456789012345678901234567890121234567890123456789012345678 multiplied to find the answer. 123456789012345678901234567890121234567890123456789012345678 9 123456789012345678901234567890121234567890123456789012345678 9 9 1 12 n 9 123456789012345678901234567890121234567890123456789012345678 5 9 123456789012345678901234567890121234567890123456789012345678 75 100 123456789012345678901234567890121234567890123456789012345678 9 9 123456789012345678901234567890121234567890123456789012345678 ~12!~100! 5 ~75!~n! 9 123456789012345678901234567890121234567890123456789012345678 23456789012345678901234567890121234567890123456789012345678 123456789012345678901234567890121234567890123456789012345678 9 9 123456789012345678901234567890121234567890123456789012345678 1200 5 75n 9 1 9 123456789012345678901234567890121234567890123456789012345678 1200 75n 9 123456789012345678901234567890121234567890123456789012345678 5 23456789012345678901234567890121234567890123456789012345678 9 123456789012345678901234567890121234567890123456789012345678 75 75 9 123456789012345678901234567890121234567890123456789012345678 9 1 16 5 n 9 123456789012345678901234567890121234567890123456789012345678 123456789012345678901234567890121234567890123456789012345678 9 23456789012345678901234567890121234567890123456789012345678 9 123456789012345678901234567890121234567890123456789012345678 2. D If a circle is inscribed in a square, then the circle is inside the square. 9 123456789012345678901234567890121234567890123456789012345678 9 123456789012345678901234567890121234567890123456789012345678 You can find the length of the square’s sides using the area formula for 123456789012345678901234567890121234567890123456789012345678 9 9 1 a square: 9 123456789012345678901234567890121234567890123456789012345678 9 123456789012345678901234567890121234567890123456789012345678 2 9 123456789012345678901234567890121234567890123456789012345678 A 5 s 9 123456789012345678901234567890121234567890123456789012345678 9 123456789012345678901234567890121234567890123456789012345678 2 36 5 s 23456789012345678901234567890121234567890123456789012345678 123456789012345678901234567890121234567890123456789012345678 9 123456789012345678901234567890121234567890123456789012345678 9 9 123456789012345678901234567890121234567890123456789012345678 65s 9 123456789012345678901234567890121234567890123456789012345678 9 1 It might help if you draw a circle inside a square to visualize the next 23456789012345678901234567890121234567890123456789012345678 123456789012345678901234567890121234567890123456789012345678 9 9 1 part. The side length of the square is the same as the diameter of the 9 123456789012345678901234567890121234567890123456789012345678 9 123456789012345678901234567890121234567890123456789012345678 circle. Draw a diameter and you’ll see that it’s the same length as a side. 9 123456789012345678901234567890121234567890123456789012345678 23456789012345678901234567890121234567890123456789012345678 9 1 123456789012345678901234567890121234567890123456789012345678 9 123456789012345678901234567890121234567890123456789012345678 9 123456789012345678901234567890121234567890123456789012345678 9 1234567890123456789012345678901212345678901234567890123456789 52

Copyright © 2005 Thomson Peterson’s, a part of The Thomson Corporaton SAT is a registered trademark of the College Entrance Examination Board, which was not involved in the production of and does not endorse this product.

1234567890123456789012345678901212345678901234567890123456789 123456789012345678901234567890121234567890123456789012345678 9 9 123456789012345678901234567890121234567890123456789012345678 123456789012345678901234567890121234567890123456789012345678 9 9 123456789012345678901234567890121234567890123456789012345678 This is a key relationship that the two figures share. Once you know the 9 123456789012345678901234567890121234567890123456789012345678 9 123456789012345678901234567890121234567890123456789012345678 circle’s diameter is 6, its radius must be half that, 3. This radius can be 9 123456789012345678901234567890121234567890123456789012345678 23456789012345678901234567890121234567890123456789012345678 9 1 placed into the area formula for a circle: 9 123456789012345678901234567890121234567890123456789012345678 9 123456789012345678901234567890121234567890123456789012345678 2 9 123456789012345678901234567890121234567890123456789012345678 A5 pr 9 123456789012345678901234567890121234567890123456789012345678 23456789012345678901234567890121234567890123456789012345678 9 1 A 5 p~3!2 9 123456789012345678901234567890121234567890123456789012345678 9 123456789012345678901234567890121234567890123456789012345678 9 123456789012345678901234567890121234567890123456789012345678 A 5 9p 9 123456789012345678901234567890121234567890123456789012345678 23456789012345678901234567890121234567890123456789012345678 9 1 9 Choice (D) is correct. 123456789012345678901234567890121234567890123456789012345678 9 123456789012345678901234567890121234567890123456789012345678 9 123456789012345678901234567890121234567890123456789012345678 3. B First, you are looking for a line with a positive slope, which means that it 9 123456789012345678901234567890121234567890123456789012345678 23456789012345678901234567890121234567890123456789012345678 9 1 rises as you go from left to right. This eliminates choices (A), (D), and (E). 9 123456789012345678901234567890121234567890123456789012345678 9 123456789012345678901234567890121234567890123456789012345678 Second, a slope of 1 means that it rises as much as it runs (it goes up at the 9 123456789012345678901234567890121234567890123456789012345678 9 123456789012345678901234567890121234567890123456789012345678 same rate that it goes over). A line with a slope of 1 will be halfway 23456789012345678901234567890121234567890123456789012345678 123456789012345678901234567890121234567890123456789012345678 99 123456789012345678901234567890121234567890123456789012345678 between a line that is completely horizontal and a line that is completely 9 123456789012345678901234567890121234567890123456789012345678 9 123456789012345678901234567890121234567890123456789012345678 vertical. Choice (B) is that line, since the rise in choice (C) is too gradual. 123456789012345678901234567890121234567890123456789012345678 99 123456789012345678901234567890121234567890123456789012345678 9 1 4. C To answer this question, you have to determine how the series is 9 123456789012345678901234567890121234567890123456789012345678 9 123456789012345678901234567890121234567890123456789012345678 generated. The numbers are increasing, so it is very unlikely that either 9 123456789012345678901234567890121234567890123456789012345678 23456789012345678901234567890121234567890123456789012345678 9 123456789012345678901234567890121234567890123456789012345678 subtraction or division is involved. The numbers increase, and note 9 123456789012345678901234567890121234567890123456789012345678 9 123456789012345678901234567890121234567890123456789012345678 how fast the increase is. Addition can be ruled out since the increase 123456789012345678901234567890121234567890123456789012345678 99 123456789012345678901234567890121234567890123456789012345678 from one term to the next is too great; simple multiplication is also 9 123456789012345678901234567890121234567890123456789012345678 unlikely. Look at the four terms closely, and you’ll notice that each 9 123456789012345678901234567890121234567890123456789012345678 9 123456789012345678901234567890121234567890123456789012345678 number is the square of an ascending integer. The series is one squared, 9 123456789012345678901234567890121234567890123456789012345678 9 1 then two squared, then three squared, and so forth. This series then is 23456789012345678901234567890121234567890123456789012345678 123456789012345678901234567890121234567890123456789012345678 99 123456789012345678901234567890121234567890123456789012345678 generated by squaring the integers. Therefore, the eighth term is eight 9 1 9 123456789012345678901234567890121234567890123456789012345678 squared, 64. Choice (C) is correct. 9 123456789012345678901234567890121234567890123456789012345678 23456789012345678901234567890121234567890123456789012345678 123456789012345678901234567890121234567890123456789012345678 99 123456789012345678901234567890121234567890123456789012345678 5. B The area of a parallelogram is the height times the base. You do not 9 1 9 123456789012345678901234567890121234567890123456789012345678 know the height, but you can determine it by using the geometry of a 9 123456789012345678901234567890121234567890123456789012345678 23456789012345678901234567890121234567890123456789012345678 9 123456789012345678901234567890121234567890123456789012345678 triangle. If you drop a perpendicular from the top left corner to the 9 123456789012345678901234567890121234567890123456789012345678 9 1 opposite side (which you will call the base), then you have a triangle 123456789012345678901234567890121234567890123456789012345678 9 9 123456789012345678901234567890121234567890123456789012345678 whose height is the height of the parallelogram. Measures of adjacent 9 123456789012345678901234567890121234567890123456789012345678 9 123456789012345678901234567890121234567890123456789012345678 angles of parallelograms sum to 180, and so the bottom left-hand angle 9 123456789012345678901234567890121234567890123456789012345678 9 123456789012345678901234567890121234567890123456789012345678 measures 60° (this is because the bottom right interior angle is 120°, 123456789012345678901234567890121234567890123456789012345678 9 23456789012345678901234567890121234567890123456789012345678 9 123456789012345678901234567890121234567890123456789012345678 and 180 2 120 5 60). Surprise! This gives you a 30-60-90 triangle, and 9 123456789012345678901234567890121234567890123456789012345678 9 123456789012345678901234567890121234567890123456789012345678 3, the height you can determine the height. Since the hypotenuse is 2 = 123456789012345678901234567890121234567890123456789012345678 99 1 is 3. The base is 6, and so the area is: 9 123456789012345678901234567890121234567890123456789012345678 9 123456789012345678901234567890121234567890123456789012345678 9 123456789012345678901234567890121234567890123456789012345678 A 5 bh 9 123456789012345678901234567890121234567890123456789012345678 9 123456789012345678901234567890121234567890123456789012345678 6 3 A 5 ~ !~ ! 23456789012345678901234567890121234567890123456789012345678 123456789012345678901234567890121234567890123456789012345678 99 1 9 123456789012345678901234567890121234567890123456789012345678 A 5 18 9 123456789012345678901234567890121234567890123456789012345678 9 123456789012345678901234567890121234567890123456789012345678 That’s choice (B). 23456789012345678901234567890121234567890123456789012345678 9 1 123456789012345678901234567890121234567890123456789012345678 9 123456789012345678901234567890121234567890123456789012345678 9 123456789012345678901234567890121234567890123456789012345678 9 1234567890123456789012345678901212345678901234567890123456789 Copyright © 2005 Thomson Peterson’s, a part of The Thomson Corporaton SAT is a registered trademark of the College Entrance Examination Board, which was not involved in the production of and does not endorse this product.

53

1234567890123456789012345678901212345678901234567890123456789 123456789012345678901234567890121234567890123456789012345678 9 9 123456789012345678901234567890121234567890123456789012345678 123456789012345678901234567890121234567890123456789012345678 9 9 123456789012345678901234567890121234567890123456789012345678 6. A To find the reciprocal, switch the numerator and the denominator. 9 123456789012345678901234567890121234567890123456789012345678 9 123456789012345678901234567890121234567890123456789012345678 Once you’ve flipped the fractions over, look for a fraction where the 9 123456789012345678901234567890121234567890123456789012345678 23456789012345678901234567890121234567890123456789012345678 9 1 numerator is greater than the denominator and where the difference 9 123456789012345678901234567890121234567890123456789012345678 9 123456789012345678901234567890121234567890123456789012345678 between the two is greatest. Choices (A) and (E) look like the best 9 123456789012345678901234567890121234567890123456789012345678 9 123456789012345678901234567890121234567890123456789012345678 9 1 1 23456789012345678901234567890121234567890123456789012345678 9 1 candidates. The reciprocal of A is 4 5 4 , and the reciprocal of 9 123456789012345678901234567890121234567890123456789012345678 2 2 2 9 123456789012345678901234567890121234567890123456789012345678 1 13 1 9 123456789012345678901234567890121234567890123456789012345678 9 123456789012345678901234567890121234567890123456789012345678 (E) is 4 5 4 . Choice (A) is the greatest. 23456789012345678901234567890121234567890123456789012345678 9 1 3 3 3 9 123456789012345678901234567890121234567890123456789012345678 9 123456789012345678901234567890121234567890123456789012345678 7. E Another sketch with more of the angles numbered is helpful here. 9 123456789012345678901234567890121234567890123456789012345678 9 123456789012345678901234567890121234567890123456789012345678 23456789012345678901234567890121234567890123456789012345678 9 1 9 123456789012345678901234567890121234567890123456789012345678 9 123456789012345678901234567890121234567890123456789012345678 9 123456789012345678901234567890121234567890123456789012345678 9 123456789012345678901234567890121234567890123456789012345678 23456789012345678901234567890121234567890123456789012345678 123456789012345678901234567890121234567890123456789012345678 9 123456789012345678901234567890121234567890123456789012345678 9 123456789012345678901234567890121234567890123456789012345678 9 123456789012345678901234567890121234567890123456789012345678 9 123456789012345678901234567890121234567890123456789012345678 9 123456789012345678901234567890121234567890123456789012345678 9 9 1 9 123456789012345678901234567890121234567890123456789012345678 9 123456789012345678901234567890121234567890123456789012345678 9 123456789012345678901234567890121234567890123456789012345678 23456789012345678901234567890121234567890123456789012345678 123456789012345678901234567890121234567890123456789012345678 9 123456789012345678901234567890121234567890123456789012345678 9 123456789012345678901234567890121234567890123456789012345678 9 123456789012345678901234567890121234567890123456789012345678 9 9 123456789012345678901234567890121234567890123456789012345678 Angle 1 is 90° because its measure and the measure of the angle 9 123456789012345678901234567890121234567890123456789012345678 9 adjacent to it must sum to 180. Couple this fact with the given 123456789012345678901234567890121234567890123456789012345678 9 123456789012345678901234567890121234567890123456789012345678 information a 5 60, and it means that the measure of angle 2 is 30° 9 123456789012345678901234567890121234567890123456789012345678 9 1 since the measures of the angles in that triangle must sum to 180. 23456789012345678901234567890121234567890123456789012345678 123456789012345678901234567890121234567890123456789012345678 9 123456789012345678901234567890121234567890123456789012345678 9 9 1 Angle 3 is also 30° since it is vertical with angle 2. If you can find angle 9 123456789012345678901234567890121234567890123456789012345678 9 123456789012345678901234567890121234567890123456789012345678 4, you could figure out angle x since you will have two of the three 23456789012345678901234567890121234567890123456789012345678 123456789012345678901234567890121234567890123456789012345678 9 9 123456789012345678901234567890121234567890123456789012345678 angles on that small triangle. To find the measure of angle 4, look at the 9 1 9 123456789012345678901234567890121234567890123456789012345678 big triangle with angles 4, c, and the right angle. Since a right angle is 9 123456789012345678901234567890121234567890123456789012345678 23456789012345678901234567890121234567890123456789012345678 9 123456789012345678901234567890121234567890123456789012345678 90° and c 5 50, the measure of angle 4 is 40° since the measures of the 9 123456789012345678901234567890121234567890123456789012345678 9 1 angles in the larger triangle must sum to 180. If the measure of angle 123456789012345678901234567890121234567890123456789012345678 9 9 123456789012345678901234567890121234567890123456789012345678 4 5 40, and the measure of the angle 3 5 30, then 9 123456789012345678901234567890121234567890123456789012345678 9 123456789012345678901234567890121234567890123456789012345678 9 123456789012345678901234567890121234567890123456789012345678 180 5 m∠3 1 m∠4 1 m∠x 9 123456789012345678901234567890121234567890123456789012345678 9 123456789012345678901234567890121234567890123456789012345678 180 5 30 1 40 1 x 23456789012345678901234567890121234567890123456789012345678 123456789012345678901234567890121234567890123456789012345678 9 9 123456789012345678901234567890121234567890123456789012345678 180 5 70 1 x 9 123456789012345678901234567890121234567890123456789012345678 123456789012345678901234567890121234567890123456789012345678 9 9 1 110 5 x 9 123456789012345678901234567890121234567890123456789012345678 9 123456789012345678901234567890121234567890123456789012345678 Choice (E) is correct. 9 123456789012345678901234567890121234567890123456789012345678 9 123456789012345678901234567890121234567890123456789012345678 9 If you had no idea how to answer this question, you might have noticed 123456789012345678901234567890121234567890123456789012345678 23456789012345678901234567890121234567890123456789012345678 9 123456789012345678901234567890121234567890123456789012345678 that the figures were drawn to scale. Looking at x, it certainly looks 9 1 9 123456789012345678901234567890121234567890123456789012345678 greater than 90°. Choice (A) is highly unlikely as an answer, and (E) 9 123456789012345678901234567890121234567890123456789012345678 9 123456789012345678901234567890121234567890123456789012345678 would be your best guess since it’s the only choice greater than 90°. 23456789012345678901234567890121234567890123456789012345678 9 1 123456789012345678901234567890121234567890123456789012345678 9 123456789012345678901234567890121234567890123456789012345678 9 123456789012345678901234567890121234567890123456789012345678 9 1234567890123456789012345678901212345678901234567890123456789

S

54

D

S

D

Copyright © 2005 Thomson Peterson’s, a part of The Thomson Corporaton SAT is a registered trademark of the College Entrance Examination Board, which was not involved in the production of and does not endorse this product.

1234567890123456789012345678901212345678901234567890123456789 123456789012345678901234567890121234567890123456789012345678 9 9 123456789012345678901234567890121234567890123456789012345678 123456789012345678901234567890121234567890123456789012345678 9 9 123456789012345678901234567890121234567890123456789012345678 8. E A problem like this takes a careful step-by-step execution, but 9 123456789012345678901234567890121234567890123456789012345678 9 123456789012345678901234567890121234567890123456789012345678 fortunately nothing else is needed. 9 123456789012345678901234567890121234567890123456789012345678 23456789012345678901234567890121234567890123456789012345678 9 1 22 9 123456789012345678901234567890121234567890123456789012345678 1 1 2 2 21 22 21 22 9 123456789012345678901234567890121234567890123456789012345678 5 64, choice (E). 4 1 4 2 1 2 5 5 5 ~ ! ~ ! # @ # @ 9 123456789012345678901234567890121234567890123456789012345678 8 1 9 123456789012345678901234567890121234567890123456789012345678 23456789012345678901234567890121234567890123456789012345678 9 1 64 9 123456789012345678901234567890121234567890123456789012345678 9 123456789012345678901234567890121234567890123456789012345678 9. C Here you need to take care to read the chart correctly. Each complete 9 123456789012345678901234567890121234567890123456789012345678 9 123456789012345678901234567890121234567890123456789012345678 box represents 150,000 widgets. Company B made one-and-a-half 23456789012345678901234567890121234567890123456789012345678 9 1 9 123456789012345678901234567890121234567890123456789012345678 more boxes. That last box is not a full box, as you can see by its size and 9 123456789012345678901234567890121234567890123456789012345678 9 123456789012345678901234567890121234567890123456789012345678 by the dashed lines on the right end. Choice (B) represents one box 9 123456789012345678901234567890121234567890123456789012345678 23456789012345678901234567890121234567890123456789012345678 9 1 worth of difference (150,000 widgets), while choice (D)represents two 9 123456789012345678901234567890121234567890123456789012345678 9 123456789012345678901234567890121234567890123456789012345678 complete boxes of widgets as the difference. Answer (C) is correct, since 9 123456789012345678901234567890121234567890123456789012345678 9 123456789012345678901234567890121234567890123456789012345678 it shows a difference in production of one-and-a-half boxes (150,000 23456789012345678901234567890121234567890123456789012345678 123456789012345678901234567890121234567890123456789012345678 99 123456789012345678901234567890121234567890123456789012345678 widgets 1 75,000 widgets 5 225,000). 9 123456789012345678901234567890121234567890123456789012345678 123456789012345678901234567890121234567890123456789012345678 99 123456789012345678901234567890121234567890123456789012345678 10. A Since you are told that x is odd, you should suspect that the answer 9 123456789012345678901234567890121234567890123456789012345678 9 1 would have something to do with being odd or even or neither. That 9 123456789012345678901234567890121234567890123456789012345678 9 123456789012345678901234567890121234567890123456789012345678 makes choices (A) or (B) the prime suspects. x cannot be even because 9 123456789012345678901234567890121234567890123456789012345678 23456789012345678901234567890121234567890123456789012345678 9 123456789012345678901234567890121234567890123456789012345678 an even number raised to an even power must be even. Try giving x an 9 123456789012345678901234567890121234567890123456789012345678 x 3 odd value, like 3. x 5 3 5 27. This satisfies the facts given in the 9 123456789012345678901234567890121234567890123456789012345678 9 123456789012345678901234567890121234567890123456789012345678 problem, since 27 is an odd number, so choice (A) is the answer. 9 123456789012345678901234567890121234567890123456789012345678 123456789012345678901234567890121234567890123456789012345678 99 123456789012345678901234567890121234567890123456789012345678 11. E Do not be unnerved by the newness of this concept. All that you need to 9 123456789012345678901234567890121234567890123456789012345678 9 123456789012345678901234567890121234567890123456789012345678 know about factorials is provided in the explanation. So be a good 9 1 23456789012345678901234567890121234567890123456789012345678 9 123456789012345678901234567890121234567890123456789012345678 test-taking robot: Take the numbers they give you and feed them into 9 123456789012345678901234567890121234567890123456789012345678 9 1 the formula. 123456789012345678901234567890121234567890123456789012345678 9 9 123456789012345678901234567890121234567890123456789012345678 9 123456789012345678901234567890121234567890123456789012345678 6! 6 3 5 3 4 3 3 3 2 3 1 9 123456789012345678901234567890121234567890123456789012345678 5 5 6 3 5 3 4 5 120, choice (E). 9 123456789012345678901234567890121234567890123456789012345678 3! 3 3 2 3 1 9 123456789012345678901234567890121234567890123456789012345678 9 123456789012345678901234567890121234567890123456789012345678 If you don’t get flustered (and no self-respecting test-taking robot ever 23456789012345678901234567890121234567890123456789012345678 123456789012345678901234567890121234567890123456789012345678 99 123456789012345678901234567890121234567890123456789012345678 does), the problem is quite straightforward. You just apply the concept 9 1 9 123456789012345678901234567890121234567890123456789012345678 of factorial and then multiply to find the answer. 123456789012345678901234567890121234567890123456789012345678 9 23456789012345678901234567890121234567890123456789012345678 123456789012345678901234567890121234567890123456789012345678 99 123456789012345678901234567890121234567890123456789012345678 12. B Here replace x with three, and then solve: 9 123456789012345678901234567890121234567890123456789012345678 9 123456789012345678901234567890121234567890123456789012345678 2 2 2 9 1 f(3) 5 (3!) 5 (3 3 2 3 1) 5 (6) 5 36, answer (B). 23456789012345678901234567890121234567890123456789012345678 123456789012345678901234567890121234567890123456789012345678 99 123456789012345678901234567890121234567890123456789012345678 9 123456789012345678901234567890121234567890123456789012345678 9 123456789012345678901234567890121234567890123456789012345678 9 1 9 123456789012345678901234567890121234567890123456789012345678 9 123456789012345678901234567890121234567890123456789012345678 9 123456789012345678901234567890121234567890123456789012345678 9 123456789012345678901234567890121234567890123456789012345678 123456789012345678901234567890121234567890123456789012345678 9 23456789012345678901234567890121234567890123456789012345678 123456789012345678901234567890121234567890123456789012345678 99 1 9 123456789012345678901234567890121234567890123456789012345678 123456789012345678901234567890121234567890123456789012345678 9 9 123456789012345678901234567890121234567890123456789012345678 123456789012345678901234567890121234567890123456789012345678 9 123456789012345678901234567890121234567890123456789012345678 9 123456789012345678901234567890121234567890123456789012345678 9 123456789012345678901234567890121234567890123456789012345678 9 1234567890123456789012345678901212345678901234567890123456789

FG

Copyright © 2005 Thomson Peterson’s, a part of The Thomson Corporaton SAT is a registered trademark of the College Entrance Examination Board, which was not involved in the production of and does not endorse this product.

55

1234567890123456789012345678901212345678901234567890123456789 123456789012345678901234567890121234567890123456789012345678 9 9 123456789012345678901234567890121234567890123456789012345678 123456789012345678901234567890121234567890123456789012345678 9 9 123456789012345678901234567890121234567890123456789012345678 13. A This one’s the toughest factorial problem, but a clue is provided by the 9 123456789012345678901234567890121234567890123456789012345678 9 123456789012345678901234567890121234567890123456789012345678 answer choices. Most of the answers are in terms of y. Therefore, you 9 123456789012345678901234567890121234567890123456789012345678 23456789012345678901234567890121234567890123456789012345678 9 1 y! 9 123456789012345678901234567890121234567890123456789012345678 are probably going to have to manipulate so that only y’s remain in 9 123456789012345678901234567890121234567890123456789012345678 x! 9 123456789012345678901234567890121234567890123456789012345678 9 123456789012345678901234567890121234567890123456789012345678 the expression. 23456789012345678901234567890121234567890123456789012345678 9 1 9 123456789012345678901234567890121234567890123456789012345678 To do this, let’s replace the x’s with y’s. Since, and y and x are integers, 9 123456789012345678901234567890121234567890123456789012345678 9 123456789012345678901234567890121234567890123456789012345678 x! 5 (y 1 2)!. Substituting this into the problem: 9 123456789012345678901234567890121234567890123456789012345678 23456789012345678901234567890121234567890123456789012345678 9 1 9 123456789012345678901234567890121234567890123456789012345678 y! y! 1 y! 9 123456789012345678901234567890121234567890123456789012345678 9 123456789012345678901234567890121234567890123456789012345678 5 5 5 9 123456789012345678901234567890121234567890123456789012345678 x! ~y 1 2!! ~y 1 2!~y 1 1!y! ~y 1 2!~y 1 1! 23456789012345678901234567890121234567890123456789012345678 9 1 9 123456789012345678901234567890121234567890123456789012345678 This is choice (A). You could also solve the problem by picking values 9 123456789012345678901234567890121234567890123456789012345678 9 123456789012345678901234567890121234567890123456789012345678 for the two variables that are consistent with y 1 2 5 x, and then 9 123456789012345678901234567890121234567890123456789012345678 23456789012345678901234567890121234567890123456789012345678 9 123456789012345678901234567890121234567890123456789012345678 plugging the values into the factorial fraction and also into the answer 9 123456789012345678901234567890121234567890123456789012345678 9 123456789012345678901234567890121234567890123456789012345678 choices. If only one answer choice matches the factorial fraction, then 123456789012345678901234567890121234567890123456789012345678 9 9 123456789012345678901234567890121234567890123456789012345678 you have the right answer. If two answers match, then pick another set 9 123456789012345678901234567890121234567890123456789012345678 9 1 of values for the variables and repeat the process. 9 123456789012345678901234567890121234567890123456789012345678 9 123456789012345678901234567890121234567890123456789012345678 9 123456789012345678901234567890121234567890123456789012345678 14. A You will probably recall that the determinant of a matrix is found by 23456789012345678901234567890121234567890123456789012345678 123456789012345678901234567890121234567890123456789012345678 9 9 123456789012345678901234567890121234567890123456789012345678 cross-multiplying. Since you know the end result is 26, all you need is 9 123456789012345678901234567890121234567890123456789012345678 9 123456789012345678901234567890121234567890123456789012345678 the right set-up and the proper computation: 123456789012345678901234567890121234567890123456789012345678 9 123456789012345678901234567890121234567890123456789012345678 9 9 123456789012345678901234567890121234567890123456789012345678 n 4 9 123456789012345678901234567890121234567890123456789012345678 5 26 9 123456789012345678901234567890121234567890123456789012345678 5 27 9 1 23456789012345678901234567890121234567890123456789012345678 9 123456789012345678901234567890121234567890123456789012345678 ~27!~n! 2 ~4!~5! 5 26 9 123456789012345678901234567890121234567890123456789012345678 9 1 9 123456789012345678901234567890121234567890123456789012345678 27n 2 20 5 26 9 123456789012345678901234567890121234567890123456789012345678 23456789012345678901234567890121234567890123456789012345678 9 123456789012345678901234567890121234567890123456789012345678 27n 2 20 1 20 5 26 1 20 9 123456789012345678901234567890121234567890123456789012345678 9 1 27n 514 9 123456789012345678901234567890121234567890123456789012345678 9 123456789012345678901234567890121234567890123456789012345678 23456789012345678901234567890121234567890123456789012345678 9 123456789012345678901234567890121234567890123456789012345678 27n 14 9 123456789012345678901234567890121234567890123456789012345678 5 9 1 27 27 123456789012345678901234567890121234567890123456789012345678 9 9 123456789012345678901234567890121234567890123456789012345678 n 5 22 9 123456789012345678901234567890121234567890123456789012345678 9 123456789012345678901234567890121234567890123456789012345678 9 123456789012345678901234567890121234567890123456789012345678 15. E First note that the question asks which statements must be true. Some 9 123456789012345678901234567890121234567890123456789012345678 9 123456789012345678901234567890121234567890123456789012345678 statements could be true under the right conditions, but if they are not 23456789012345678901234567890121234567890123456789012345678 123456789012345678901234567890121234567890123456789012345678 9 9 123456789012345678901234567890121234567890123456789012345678 always true, they are not going to be the right answer for this problem. 123456789012345678901234567890121234567890123456789012345678 9 9 123456789012345678901234567890121234567890123456789012345678 Now, from p 1 q 5 2q 1 6, you can determine: p 5 q 1 6 by 9 1 23456789012345678901234567890121234567890123456789012345678 9 123456789012345678901234567890121234567890123456789012345678 subtracting a q from both sides. If p is odd then q is odd, and if p is even 9 123456789012345678901234567890121234567890123456789012345678 9 123456789012345678901234567890121234567890123456789012345678 then q is even (since an odd plus an even is odd and an even plus an even 123456789012345678901234567890121234567890123456789012345678 9 9 1 is even). But neither of them has to be even or odd. Thus I and II are not 9 123456789012345678901234567890121234567890123456789012345678 123456789012345678901234567890121234567890123456789012345678 9 9 123456789012345678901234567890121234567890123456789012345678 123456789012345678901234567890121234567890123456789012345678 9 9 123456789012345678901234567890121234567890123456789012345678 123456789012345678901234567890121234567890123456789012345678 9 123456789012345678901234567890121234567890123456789012345678 9 123456789012345678901234567890121234567890123456789012345678 9 123456789012345678901234567890121234567890123456789012345678 9 1234567890123456789012345678901212345678901234567890123456789

S D

56

Copyright © 2005 Thomson Peterson’s, a part of The Thomson Corporaton SAT is a registered trademark of the College Entrance Examination Board, which was not involved in the production of and does not endorse this product.

1234567890123456789012345678901212345678901234567890123456789 123456789012345678901234567890121234567890123456789012345678 9 9 123456789012345678901234567890121234567890123456789012345678 123456789012345678901234567890121234567890123456789012345678 9 9 123456789012345678901234567890121234567890123456789012345678 necessarily true. The same is true for III, since p and q could both be 9 123456789012345678901234567890121234567890123456789012345678 9 123456789012345678901234567890121234567890123456789012345678 odd, which would make their product odd. Choice (E) is the answer, 9 123456789012345678901234567890121234567890123456789012345678 23456789012345678901234567890121234567890123456789012345678 9 1 since it’s the only choice left standing. 9 123456789012345678901234567890121234567890123456789012345678 9 123456789012345678901234567890121234567890123456789012345678 9 123456789012345678901234567890121234567890123456789012345678 One last point: this question should have been answered on the second 9 123456789012345678901234567890121234567890123456789012345678 23456789012345678901234567890121234567890123456789012345678 9 1 pass. Once you saw the Roman numerals, you should have realized that 9 123456789012345678901234567890121234567890123456789012345678 9 123456789012345678901234567890121234567890123456789012345678 it would take some time to answer, and that waiting until the second 9 123456789012345678901234567890121234567890123456789012345678 9 123456789012345678901234567890121234567890123456789012345678 pass would allow you more time to get to questions that might take 23456789012345678901234567890121234567890123456789012345678 9 1 9 123456789012345678901234567890121234567890123456789012345678 less time. 9 123456789012345678901234567890121234567890123456789012345678 9 123456789012345678901234567890121234567890123456789012345678 16. B Look at the graph and you’ll see that there are three distinct regions in 9 123456789012345678901234567890121234567890123456789012345678 23456789012345678901234567890121234567890123456789012345678 9 1 which x is greater than zero: 9 123456789012345678901234567890121234567890123456789012345678 9 123456789012345678901234567890121234567890123456789012345678 9 123456789012345678901234567890121234567890123456789012345678 (1) x to the left of negative three 9 123456789012345678901234567890121234567890123456789012345678 23456789012345678901234567890121234567890123456789012345678 9 123456789012345678901234567890121234567890123456789012345678 (2) x between negative one and one 9 123456789012345678901234567890121234567890123456789012345678 9 123456789012345678901234567890121234567890123456789012345678 (3) x to the right of three 123456789012345678901234567890121234567890123456789012345678 99 123456789012345678901234567890121234567890123456789012345678 9 123456789012345678901234567890121234567890123456789012345678 Once your eyes give you that information, it’s up to your brain to 9 1 9 123456789012345678901234567890121234567890123456789012345678 decipher the wilderness of greater than/less than signs and find the 9 123456789012345678901234567890121234567890123456789012345678 9 123456789012345678901234567890121234567890123456789012345678 answer that describes these three regions correctly. Choice (B) does this. 23456789012345678901234567890121234567890123456789012345678 123456789012345678901234567890121234567890123456789012345678 99 123456789012345678901234567890121234567890123456789012345678 17. E Since the question asks which number must be in Box E, the conditions 9 123456789012345678901234567890121234567890123456789012345678 9 123456789012345678901234567890121234567890123456789012345678 must only allow for one number to be in Box E. There is only one prime 123456789012345678901234567890121234567890123456789012345678 99 123456789012345678901234567890121234567890123456789012345678 number, 11, and so it goes in Box B. 9, 12, and 15 are evenly divisible by 9 123456789012345678901234567890121234567890123456789012345678 9 123456789012345678901234567890121234567890123456789012345678 three, and so they go in A, C, and D. You don’t know what order, they 123456789012345678901234567890121234567890123456789012345678 99 1 are in, but it doesn’t matter. That leaves 16 for Box E, which means 9 123456789012345678901234567890121234567890123456789012345678 9 123456789012345678901234567890121234567890123456789012345678 choice (E) is the answer. 123456789012345678901234567890121234567890123456789012345678 9 123456789012345678901234567890121234567890123456789012345678 9 9 123456789012345678901234567890121234567890123456789012345678 18. D Since B, D, E, and H are all midpoints on their respective lines, the 9 123456789012345678901234567890121234567890123456789012345678 9 123456789012345678901234567890121234567890123456789012345678 CG divides rectangle is divided into four equal rectangles. The diagonal 123456789012345678901234567890121234567890123456789012345678 9 9 123456789012345678901234567890121234567890123456789012345678 the two smaller rectangles it traverses into halves. One-half of each of 9 123456789012345678901234567890121234567890123456789012345678 23456789012345678901234567890121234567890123456789012345678 9 123456789012345678901234567890121234567890123456789012345678 these two rectangles is shaded, so a total of one of the smaller rectangles 9 123456789012345678901234567890121234567890123456789012345678 9 1 is shaded. Since the smaller rectangles are equal in size and there are 123456789012345678901234567890121234567890123456789012345678 9 9 123456789012345678901234567890121234567890123456789012345678 four of them, one-fourth of the larger rectangle is shaded or 25%, 9 123456789012345678901234567890121234567890123456789012345678 9 123456789012345678901234567890121234567890123456789012345678 choice (D). 9 123456789012345678901234567890121234567890123456789012345678 9 123456789012345678901234567890121234567890123456789012345678 2p 19. D Since n is the same thing as n1, you are looking to raise n n so that the 9 123456789012345678901234567890121234567890123456789012345678 23456789012345678901234567890121234567890123456789012345678 123456789012345678901234567890121234567890123456789012345678 99 123456789012345678901234567890121234567890123456789012345678 resulting exponent is 1. Remember that when you raise a number to an 9 123456789012345678901234567890121234567890123456789012345678 9 123456789012345678901234567890121234567890123456789012345678 2p 9 1 exponent, you multiply the exponents. What multiplied with yields 23456789012345678901234567890121234567890123456789012345678 9 123456789012345678901234567890121234567890123456789012345678 n 9 123456789012345678901234567890121234567890123456789012345678 9 123456789012345678901234567890121234567890123456789012345678 2n 9 123456789012345678901234567890121234567890123456789012345678 . one? The answer is its reciprocal, 9 1 p 23456789012345678901234567890121234567890123456789012345678 9 123456789012345678901234567890121234567890123456789012345678 2n 9 1 2p 2p 2n 9 123456789012345678901234567890121234567890123456789012345678 p DS D S 1 ~ n n ! 5 n n p 5 n . So (D) is the answer. 9 123456789012345678901234567890121234567890123456789012345678 9 123456789012345678901234567890121234567890123456789012345678 23456789012345678901234567890121234567890123456789012345678 9 1 123456789012345678901234567890121234567890123456789012345678 9 123456789012345678901234567890121234567890123456789012345678 9 123456789012345678901234567890121234567890123456789012345678 9 1234567890123456789012345678901212345678901234567890123456789 Copyright © 2005 Thomson Peterson’s, a part of The Thomson Corporaton SAT is a registered trademark of the College Entrance Examination Board, which was not involved in the production of and does not endorse this product.

57

1234567890123456789012345678901212345678901234567890123456789 123456789012345678901234567890121234567890123456789012345678 9 9 123456789012345678901234567890121234567890123456789012345678 123456789012345678901234567890121234567890123456789012345678 9 9 123456789012345678901234567890121234567890123456789012345678 20. A A straightforward way to start this problem is simply to start with the 9 123456789012345678901234567890121234567890123456789012345678 9 123456789012345678901234567890121234567890123456789012345678 first prime number. 2 and 3 cannot be n, but 5 could since 3, 5, and 7 9 123456789012345678901234567890121234567890123456789012345678 23456789012345678901234567890121234567890123456789012345678 9 1 could form a prime triplet. Continuing on with the prime numbers to 9 123456789012345678901234567890121234567890123456789012345678 9 123456789012345678901234567890121234567890123456789012345678 fifty, there is not another prime number whose closest primes are two 9 123456789012345678901234567890121234567890123456789012345678 9 123456789012345678901234567890121234567890123456789012345678 more and two less than itself. Thus the answer is (A). 23456789012345678901234567890121234567890123456789012345678 9 1 9 123456789012345678901234567890121234567890123456789012345678 9 123456789012345678901234567890121234567890123456789012345678 21. B If a geometry figure is described but not drawn, it’s always a good idea 9 123456789012345678901234567890121234567890123456789012345678 9 123456789012345678901234567890121234567890123456789012345678 to sketch the figure yourself. Draw a circle and then one tangent line. 23456789012345678901234567890121234567890123456789012345678 9 1 9 123456789012345678901234567890121234567890123456789012345678 Next draw a second tangent line that is perpendicular to the first. Now 9 123456789012345678901234567890121234567890123456789012345678 9 draw a third tangent that also is perpendicular to the first. Your sketch 123456789012345678901234567890121234567890123456789012345678 9 123456789012345678901234567890121234567890123456789012345678 can only look like this: 23456789012345678901234567890121234567890123456789012345678 9 1 9 123456789012345678901234567890121234567890123456789012345678 9 123456789012345678901234567890121234567890123456789012345678 9 123456789012345678901234567890121234567890123456789012345678 9 123456789012345678901234567890121234567890123456789012345678 23456789012345678901234567890121234567890123456789012345678 123456789012345678901234567890121234567890123456789012345678 9 123456789012345678901234567890121234567890123456789012345678 9 123456789012345678901234567890121234567890123456789012345678 9 123456789012345678901234567890121234567890123456789012345678 9 123456789012345678901234567890121234567890123456789012345678 9 123456789012345678901234567890121234567890123456789012345678 9 9 1 9 123456789012345678901234567890121234567890123456789012345678 9 123456789012345678901234567890121234567890123456789012345678 9 123456789012345678901234567890121234567890123456789012345678 23456789012345678901234567890121234567890123456789012345678 123456789012345678901234567890121234567890123456789012345678 9 123456789012345678901234567890121234567890123456789012345678 9 123456789012345678901234567890121234567890123456789012345678 9 123456789012345678901234567890121234567890123456789012345678 9 123456789012345678901234567890121234567890123456789012345678 9 123456789012345678901234567890121234567890123456789012345678 9 123456789012345678901234567890121234567890123456789012345678 9 123456789012345678901234567890121234567890123456789012345678 9 123456789012345678901234567890121234567890123456789012345678 9 9 1 9 123456789012345678901234567890121234567890123456789012345678 9 123456789012345678901234567890121234567890123456789012345678 9 123456789012345678901234567890121234567890123456789012345678 Your lines could be rotated around, but the interrelationships of the 9 123456789012345678901234567890121234567890123456789012345678 9 123456789012345678901234567890121234567890123456789012345678 tangents must be the same. The only place to draw a fourth tangent line 23456789012345678901234567890121234567890123456789012345678 123456789012345678901234567890121234567890123456789012345678 9 9 123456789012345678901234567890121234567890123456789012345678 that will be perpendicular to two other tangent lines is at the bottom of 9 1 9 123456789012345678901234567890121234567890123456789012345678 the circle. 9 123456789012345678901234567890121234567890123456789012345678 23456789012345678901234567890121234567890123456789012345678 123456789012345678901234567890121234567890123456789012345678 9 9 123456789012345678901234567890121234567890123456789012345678 Does this figure look familiar? It should, since it has the same 9 1 9 123456789012345678901234567890121234567890123456789012345678 appearance as the first geometry problem in this section, question 2. 123456789012345678901234567890121234567890123456789012345678 9 23456789012345678901234567890121234567890123456789012345678 9 123456789012345678901234567890121234567890123456789012345678 You have drawn a circle inscribed in a square. The fact that it is a 9 123456789012345678901234567890121234567890123456789012345678 9 123456789012345678901234567890121234567890123456789012345678 square could clue you in to the fact that there are two sets of parallel 123456789012345678901234567890121234567890123456789012345678 9 9 1 lines. You might also recall from your geometry class that two lines that 9 123456789012345678901234567890121234567890123456789012345678 9 123456789012345678901234567890121234567890123456789012345678 are perpendicular to the same line are parallel to each other. This means 9 123456789012345678901234567890121234567890123456789012345678 9 123456789012345678901234567890121234567890123456789012345678 that both sets of tangent lines across the circle are parallel. Either way, 123456789012345678901234567890121234567890123456789012345678 9 23456789012345678901234567890121234567890123456789012345678 9 123456789012345678901234567890121234567890123456789012345678 the answer is (B). 9 123456789012345678901234567890121234567890123456789012345678 9 123456789012345678901234567890121234567890123456789012345678 Choice (D) is close, but the circle is inscribed in the square, not the other 9 123456789012345678901234567890121234567890123456789012345678 9 1 way around. 23456789012345678901234567890121234567890123456789012345678 123456789012345678901234567890121234567890123456789012345678 9 9 1 9 123456789012345678901234567890121234567890123456789012345678 123456789012345678901234567890121234567890123456789012345678 9 9 123456789012345678901234567890121234567890123456789012345678 123456789012345678901234567890121234567890123456789012345678 9 123456789012345678901234567890121234567890123456789012345678 9 123456789012345678901234567890121234567890123456789012345678 9 123456789012345678901234567890121234567890123456789012345678 9 1234567890123456789012345678901212345678901234567890123456789 58

Copyright © 2005 Thomson Peterson’s, a part of The Thomson Corporaton SAT is a registered trademark of the College Entrance Examination Board, which was not involved in the production of and does not endorse this product.

1234567890123456789012345678901212345678901234567890123456789 123456789012345678901234567890121234567890123456789012345678 9 9 123456789012345678901234567890121234567890123456789012345678 123456789012345678901234567890121234567890123456789012345678 9 9 123456789012345678901234567890121234567890123456789012345678 The fact that questions 2 and 21 employ the same figure shows you 9 123456789012345678901234567890121234567890123456789012345678 9 123456789012345678901234567890121234567890123456789012345678 how two questions can use the same figure but vary greatly in terms of 9 123456789012345678901234567890121234567890123456789012345678 23456789012345678901234567890121234567890123456789012345678 9 1 difficulty. A point to ponder is, “How did the test-takers make the last 9 123456789012345678901234567890121234567890123456789012345678 9 123456789012345678901234567890121234567890123456789012345678 question harder?” First and foremost, no figure was included. This 9 123456789012345678901234567890121234567890123456789012345678 9 123456789012345678901234567890121234567890123456789012345678 should spur you to draw any figure that is not given to you, and make a 23456789012345678901234567890121234567890123456789012345678 9 1 9 123456789012345678901234567890121234567890123456789012345678 hard problem a lot easier to visualize and answer. 9 123456789012345678901234567890121234567890123456789012345678 9 123456789012345678901234567890121234567890123456789012345678 9 123456789012345678901234567890121234567890123456789012345678 23456789012345678901234567890121234567890123456789012345678 9 1 9 123456789012345678901234567890121234567890123456789012345678 Section 3 9 123456789012345678901234567890121234567890123456789012345678 9 123456789012345678901234567890121234567890123456789012345678 1. E You should always take a close look at underlined pronouns in 9 123456789012345678901234567890121234567890123456789012345678 23456789012345678901234567890121234567890123456789012345678 9 1 identifying sentence error questions. “Registered” in this sentence is an 9 123456789012345678901234567890121234567890123456789012345678 9 123456789012345678901234567890121234567890123456789012345678 adjective, and “who” is a pronoun that represents “voters.” Because it 9 123456789012345678901234567890121234567890123456789012345678 is the subject of the verb, “who,” rather than “whom,” is correct. In 9 123456789012345678901234567890121234567890123456789012345678 23456789012345678901234567890121234567890123456789012345678 9 123456789012345678901234567890121234567890123456789012345678 addition, there are no other errors in this sentence. So, the answer is (E). 9 123456789012345678901234567890121234567890123456789012345678 123456789012345678901234567890121234567890123456789012345678 99 123456789012345678901234567890121234567890123456789012345678 2. C This sentence lacks parallel structure. “Debating” in the first part 9 123456789012345678901234567890121234567890123456789012345678 9 123456789012345678901234567890121234567890123456789012345678 should be matched by “setting” in the second part. The answer is 9 1 9 123456789012345678901234567890121234567890123456789012345678 therefore (C). 9 123456789012345678901234567890121234567890123456789012345678 9 123456789012345678901234567890121234567890123456789012345678 23456789012345678901234567890121234567890123456789012345678 9 123456789012345678901234567890121234567890123456789012345678 3. B There’s an error in diction here. “Affects” is a verb; “effects” is the 9 123456789012345678901234567890121234567890123456789012345678 9 123456789012345678901234567890121234567890123456789012345678 noun form. “Side effects” is the desired phrase. Choice (B) is the 123456789012345678901234567890121234567890123456789012345678 99 123456789012345678901234567890121234567890123456789012345678 answer. 9 123456789012345678901234567890121234567890123456789012345678 9 123456789012345678901234567890121234567890123456789012345678 4. B The error here is difficult to detect. The phrase over the last decade 9 123456789012345678901234567890121234567890123456789012345678 9 123456789012345678901234567890121234567890123456789012345678 indicates that the growth is still continuing. This is also indicated in the 9 1 23456789012345678901234567890121234567890123456789012345678 9 123456789012345678901234567890121234567890123456789012345678 present tense form of employs (D). Thus the first verb should be has 9 123456789012345678901234567890121234567890123456789012345678 9 1 grown instead of had grown. Choice (B) is the answer. 9 123456789012345678901234567890121234567890123456789012345678 9 123456789012345678901234567890121234567890123456789012345678 23456789012345678901234567890121234567890123456789012345678 9 123456789012345678901234567890121234567890123456789012345678 5. C Here is another case of missing parallel structure. This is a common 9 123456789012345678901234567890121234567890123456789012345678 9 1 mistake in this section. Reading widely and making herself need to be 9 123456789012345678901234567890121234567890123456789012345678 9 123456789012345678901234567890121234567890123456789012345678 matched by another gerund in part C, such as conducting sophisticated 23456789012345678901234567890121234567890123456789012345678 123456789012345678901234567890121234567890123456789012345678 99 123456789012345678901234567890121234567890123456789012345678 research. The answer is (C). 9 1 123456789012345678901234567890121234567890123456789012345678 9 9 123456789012345678901234567890121234567890123456789012345678 6. E Using “one” might sound awkward to one, but it is not incorrect. Nor 9 123456789012345678901234567890121234567890123456789012345678 9 123456789012345678901234567890121234567890123456789012345678 are there any other errors. The answer is (E). 9 123456789012345678901234567890121234567890123456789012345678 9 123456789012345678901234567890121234567890123456789012345678 7. D The tenses of the verbs in this sentence are not consistent; this is another 123456789012345678901234567890121234567890123456789012345678 9 23456789012345678901234567890121234567890123456789012345678 9 123456789012345678901234567890121234567890123456789012345678 favorite error in this type of question. Has undergone should be 9 123456789012345678901234567890121234567890123456789012345678 9 123456789012345678901234567890121234567890123456789012345678 matched with has exploded. The answer is (D). 123456789012345678901234567890121234567890123456789012345678 99 1 9 123456789012345678901234567890121234567890123456789012345678 8. D In comparing two things (films in this case), one should use the 9 123456789012345678901234567890121234567890123456789012345678 9 123456789012345678901234567890121234567890123456789012345678 comparative case of an adjective and not the superlative. Best, choice 9 123456789012345678901234567890121234567890123456789012345678 9 123456789012345678901234567890121234567890123456789012345678 (D), is superlative and not comparative (better would be correct), and 23456789012345678901234567890121234567890123456789012345678 123456789012345678901234567890121234567890123456789012345678 99 1 so it is incorrect. 9 123456789012345678901234567890121234567890123456789012345678 123456789012345678901234567890121234567890123456789012345678 9 9 123456789012345678901234567890121234567890123456789012345678 123456789012345678901234567890121234567890123456789012345678 9 123456789012345678901234567890121234567890123456789012345678 9 123456789012345678901234567890121234567890123456789012345678 9 123456789012345678901234567890121234567890123456789012345678 9 1234567890123456789012345678901212345678901234567890123456789 Copyright © 2005 Thomson Peterson’s, a part of The Thomson Corporaton SAT is a registered trademark of the College Entrance Examination Board, which was not involved in the production of and does not endorse this product.

59

1234567890123456789012345678901212345678901234567890123456789 123456789012345678901234567890121234567890123456789012345678 9 9 123456789012345678901234567890121234567890123456789012345678 123456789012345678901234567890121234567890123456789012345678 9 9 123456789012345678901234567890121234567890123456789012345678 9. C You agree with a noun, and that a phrase. (A) is idiomatically correct. 9 123456789012345678901234567890121234567890123456789012345678 9 123456789012345678901234567890121234567890123456789012345678 There is no error in (B). You may use reasonable for with the 9 123456789012345678901234567890121234567890123456789012345678 23456789012345678901234567890121234567890123456789012345678 9 1 construction here (subject 1 infinitive). No error there. To not 9 123456789012345678901234567890121234567890123456789012345678 9 123456789012345678901234567890121234567890123456789012345678 acquiesce is acceptable for spoken speech, but in written language it 9 123456789012345678901234567890121234567890123456789012345678 9 123456789012345678901234567890121234567890123456789012345678 should be not to acquiesce. There’s the rub. The answer is (C). 23456789012345678901234567890121234567890123456789012345678 9 1 9 123456789012345678901234567890121234567890123456789012345678 9 123456789012345678901234567890121234567890123456789012345678 10. C A country (Canada) requires the pronoun it, not they. The problem is 9 123456789012345678901234567890121234567890123456789012345678 9 123456789012345678901234567890121234567890123456789012345678 therefore in choice (C). 23456789012345678901234567890121234567890123456789012345678 9 1 9 123456789012345678901234567890121234567890123456789012345678 11. E This sentence is unnecessarily wordy. Having published her first novel, 9 123456789012345678901234567890121234567890123456789012345678 9 123456789012345678901234567890121234567890123456789012345678 the author is a lot snappier and more direct. The answer is (E). 9 123456789012345678901234567890121234567890123456789012345678 23456789012345678901234567890121234567890123456789012345678 9 1 9 123456789012345678901234567890121234567890123456789012345678 12. D Here, despite is not used in a grammatically correct way. Despite the 9 123456789012345678901234567890121234567890123456789012345678 9 123456789012345678901234567890121234567890123456789012345678 fact that is grammatically correct, but it is unnecessarily wordy and it is 9 123456789012345678901234567890121234567890123456789012345678 23456789012345678901234567890121234567890123456789012345678 9 123456789012345678901234567890121234567890123456789012345678 not appropriately joined with a semicolon. How about just but? That’s 9 123456789012345678901234567890121234567890123456789012345678 9 123456789012345678901234567890121234567890123456789012345678 the answer (D). 123456789012345678901234567890121234567890123456789012345678 9 123456789012345678901234567890121234567890123456789012345678 9 9 123456789012345678901234567890121234567890123456789012345678 9 1 13. C This sentence might have sounded okay to you, but it has a misplaced 9 123456789012345678901234567890121234567890123456789012345678 modifier. These are very popular mistakes in the sentence correction 9 123456789012345678901234567890121234567890123456789012345678 9 123456789012345678901234567890121234567890123456789012345678 questions. The phrase after he had worked on his serve for a few days 23456789012345678901234567890121234567890123456789012345678 123456789012345678901234567890121234567890123456789012345678 9 9 123456789012345678901234567890121234567890123456789012345678 modifies the noun player, so player must be right next to it. Only (B) 9 123456789012345678901234567890121234567890123456789012345678 and (C) accomplish this. Choice (B), however, changes the sense of the 9 123456789012345678901234567890121234567890123456789012345678 9 123456789012345678901234567890121234567890123456789012345678 original sentence. The answer is (C). 123456789012345678901234567890121234567890123456789012345678 9 123456789012345678901234567890121234567890123456789012345678 9 9 123456789012345678901234567890121234567890123456789012345678 14. C In sentence construction parallel parts of a sentence should have 9 123456789012345678901234567890121234567890123456789012345678 9 1 parallel forms. In this sentence the verbs portray and reveal are parallel, 23456789012345678901234567890121234567890123456789012345678 123456789012345678901234567890121234567890123456789012345678 9 9 123456789012345678901234567890121234567890123456789012345678 but in the original sentence construction they are not in parallel form. 9 1 9 123456789012345678901234567890121234567890123456789012345678 Choice (C) corrects this mistake. 9 123456789012345678901234567890121234567890123456789012345678 23456789012345678901234567890121234567890123456789012345678 123456789012345678901234567890121234567890123456789012345678 9 9 123456789012345678901234567890121234567890123456789012345678 9 1 15. B This is another favorite of the test writers: Any time you have “not 9 123456789012345678901234567890121234567890123456789012345678 only” you need to signal the contrasting phrase with “but also.” The 9 123456789012345678901234567890121234567890123456789012345678 23456789012345678901234567890121234567890123456789012345678 9 123456789012345678901234567890121234567890123456789012345678 answer here is therefore (B). 9 123456789012345678901234567890121234567890123456789012345678 9 1 16. A There is no error here. The answer is (A). 123456789012345678901234567890121234567890123456789012345678 9 123456789012345678901234567890121234567890123456789012345678 9 9 123456789012345678901234567890121234567890123456789012345678 17. A There is not a grammatical error in the underlined portion, and none of 9 123456789012345678901234567890121234567890123456789012345678 9 123456789012345678901234567890121234567890123456789012345678 the alternatives improve upon the original. Because you do not repeat 9 123456789012345678901234567890121234567890123456789012345678 9 123456789012345678901234567890121234567890123456789012345678 the subject, there is no comma needed before the underlined portion. 23456789012345678901234567890121234567890123456789012345678 123456789012345678901234567890121234567890123456789012345678 9 9 123456789012345678901234567890121234567890123456789012345678 Were you to choose answer (C), you would need to add a comma. 9 123456789012345678901234567890121234567890123456789012345678 9 123456789012345678901234567890121234567890123456789012345678 Choice (A) is the answer. 9 1 23456789012345678901234567890121234567890123456789012345678 123456789012345678901234567890121234567890123456789012345678 9 9 123456789012345678901234567890121234567890123456789012345678 18. D Here the error is logical rather than grammatical per se. The first part of 9 123456789012345678901234567890121234567890123456789012345678 9 123456789012345678901234567890121234567890123456789012345678 the sentence emphasizes sympathy, but the second part focuses on 9 1 23456789012345678901234567890121234567890123456789012345678 9 123456789012345678901234567890121234567890123456789012345678 negative human qualities. You need a contrasting coordinator. That 9 1 9 123456789012345678901234567890121234567890123456789012345678 leaves (D) and (E). (D) is the better choice. 123456789012345678901234567890121234567890123456789012345678 9 9 123456789012345678901234567890121234567890123456789012345678 123456789012345678901234567890121234567890123456789012345678 9 123456789012345678901234567890121234567890123456789012345678 9 123456789012345678901234567890121234567890123456789012345678 9 123456789012345678901234567890121234567890123456789012345678 9 1234567890123456789012345678901212345678901234567890123456789 60

Copyright © 2005 Thomson Peterson’s, a part of The Thomson Corporaton SAT is a registered trademark of the College Entrance Examination Board, which was not involved in the production of and does not endorse this product.

1234567890123456789012345678901212345678901234567890123456789 123456789012345678901234567890121234567890123456789012345678 9 9 123456789012345678901234567890121234567890123456789012345678 123456789012345678901234567890121234567890123456789012345678 9 9 123456789012345678901234567890121234567890123456789012345678 19. E It was not because Pancho Villa’s raid was part of the tumult of the 9 123456789012345678901234567890121234567890123456789012345678 9 123456789012345678901234567890121234567890123456789012345678 Mexican revolution that it prompted a retaliatory expedition. There9 123456789012345678901234567890121234567890123456789012345678 23456789012345678901234567890121234567890123456789012345678 9 1 fore is misused. Simply delete it and you have a correct sentence. The 9 123456789012345678901234567890121234567890123456789012345678 9 123456789012345678901234567890121234567890123456789012345678 answer is (E). 9 123456789012345678901234567890121234567890123456789012345678 9 123456789012345678901234567890121234567890123456789012345678 23456789012345678901234567890121234567890123456789012345678 1 20. A There is nothing grammatically wrong with the underlined portion, and 9 9 123456789012345678901234567890121234567890123456789012345678 9 123456789012345678901234567890121234567890123456789012345678 none of the other choices improve on the wording of the sentence. (A) is 9 123456789012345678901234567890121234567890123456789012345678 9 123456789012345678901234567890121234567890123456789012345678 the best choice. 23456789012345678901234567890121234567890123456789012345678 9 1 9 123456789012345678901234567890121234567890123456789012345678 21. E Right away, you should notice two sentences in passive voice and think 9 123456789012345678901234567890121234567890123456789012345678 9 123456789012345678901234567890121234567890123456789012345678 about making them active. Only (D) and (E) do that. (D) includes an 9 123456789012345678901234567890121234567890123456789012345678 23456789012345678901234567890121234567890123456789012345678 9 1 imprecise 2ing verb. The test writers love to throw these around. 9 123456789012345678901234567890121234567890123456789012345678 9 123456789012345678901234567890121234567890123456789012345678 Sometimes they are the right answer, but you should always scrutinize 9 123456789012345678901234567890121234567890123456789012345678 9 123456789012345678901234567890121234567890123456789012345678 them. Here (E) is a much sharper sentence. 23456789012345678901234567890121234567890123456789012345678 123456789012345678901234567890121234567890123456789012345678 99 123456789012345678901234567890121234567890123456789012345678 9 123456789012345678901234567890121234567890123456789012345678 22. A Doesn’t sentence 2 seem too specific? It is really an explanation for why 9 123456789012345678901234567890121234567890123456789012345678 9 123456789012345678901234567890121234567890123456789012345678 the men couldn’t use their limbs. It should therefore follow sentence 4. 123456789012345678901234567890121234567890123456789012345678 99 1 The answer is (A). 9 123456789012345678901234567890121234567890123456789012345678 9 123456789012345678901234567890121234567890123456789012345678 9 123456789012345678901234567890121234567890123456789012345678 23. D What is the logical connection between the two sentences? The first 23456789012345678901234567890121234567890123456789012345678 123456789012345678901234567890121234567890123456789012345678 99 123456789012345678901234567890121234567890123456789012345678 deals with the highest trip. The second deals with the largest balloon. 9 123456789012345678901234567890121234567890123456789012345678 Now has nothing to do with that. Neither does in the nineteenth 9 123456789012345678901234567890121234567890123456789012345678 9 123456789012345678901234567890121234567890123456789012345678 century. You don’t want to begin with either of these. Moreover 123456789012345678901234567890121234567890123456789012345678 99 123456789012345678901234567890121234567890123456789012345678 represents paragraph 2 as an extension of the ideas in paragraph 1, 9 123456789012345678901234567890121234567890123456789012345678 9 123456789012345678901234567890121234567890123456789012345678 which is also inaccurate. The easiest thing to do is simply get rid of 9 1 23456789012345678901234567890121234567890123456789012345678 9 123456789012345678901234567890121234567890123456789012345678 now, (D). 9 123456789012345678901234567890121234567890123456789012345678 9 1 9 123456789012345678901234567890121234567890123456789012345678 24. C Sentence combination is huge in this section. This example is trickier 9 123456789012345678901234567890121234567890123456789012345678 23456789012345678901234567890121234567890123456789012345678 9 123456789012345678901234567890121234567890123456789012345678 than most. It already has an and in the first sentence, so if you use and 9 123456789012345678901234567890121234567890123456789012345678 9 1 again your sentence will start to sound like a run-on. Here, too, the 9 123456789012345678901234567890121234567890123456789012345678 9 123456789012345678901234567890121234567890123456789012345678 2ing verb is imprecise. Which should really go very close to the noun it 23456789012345678901234567890121234567890123456789012345678 123456789012345678901234567890121234567890123456789012345678 99 123456789012345678901234567890121234567890123456789012345678 modifies, so eliminate (A). But implies a contrast, when all of these 9 1 9 123456789012345678901234567890121234567890123456789012345678 ideas are similar, so you can eliminate (E). Go with the semicolon (C). 123456789012345678901234567890121234567890123456789012345678 9 23456789012345678901234567890121234567890123456789012345678 9 123456789012345678901234567890121234567890123456789012345678 25. D All of whom returned safely is not a complete sentence. It modifies 9 123456789012345678901234567890121234567890123456789012345678 9 123456789012345678901234567890121234567890123456789012345678 “passengers” in the preceding sentence. Only (D) addresses that major 9 123456789012345678901234567890121234567890123456789012345678 9 1 problem! 23456789012345678901234567890121234567890123456789012345678 123456789012345678901234567890121234567890123456789012345678 99 123456789012345678901234567890121234567890123456789012345678 9 123456789012345678901234567890121234567890123456789012345678 26. E What’s missing in this sentence is where. As it stands now, it implies 9 123456789012345678901234567890121234567890123456789012345678 9 1 that literacy program is the direct object of working. Choices (D) and 23456789012345678901234567890121234567890123456789012345678 123456789012345678901234567890121234567890123456789012345678 99 123456789012345678901234567890121234567890123456789012345678 (E) correct the error, but (D) makes undesirable changes to the verb 9 123456789012345678901234567890121234567890123456789012345678 9 123456789012345678901234567890121234567890123456789012345678 tenses. (E) is the best answer. 9 1 23456789012345678901234567890121234567890123456789012345678 123456789012345678901234567890121234567890123456789012345678 99 1 9 123456789012345678901234567890121234567890123456789012345678 123456789012345678901234567890121234567890123456789012345678 9 9 123456789012345678901234567890121234567890123456789012345678 123456789012345678901234567890121234567890123456789012345678 9 123456789012345678901234567890121234567890123456789012345678 9 123456789012345678901234567890121234567890123456789012345678 9 123456789012345678901234567890121234567890123456789012345678 9 1234567890123456789012345678901212345678901234567890123456789 Copyright © 2005 Thomson Peterson’s, a part of The Thomson Corporaton SAT is a registered trademark of the College Entrance Examination Board, which was not involved in the production of and does not endorse this product.

61

1234567890123456789012345678901212345678901234567890123456789 123456789012345678901234567890121234567890123456789012345678 9 9 123456789012345678901234567890121234567890123456789012345678 123456789012345678901234567890121234567890123456789012345678 9 9 123456789012345678901234567890121234567890123456789012345678 27. B That had automobiles should not be separated by commas because it is 9 123456789012345678901234567890121234567890123456789012345678 9 123456789012345678901234567890121234567890123456789012345678 an integral part of the category being described, not an added 9 123456789012345678901234567890121234567890123456789012345678 23456789012345678901234567890121234567890123456789012345678 9 1 description. But it isn’t correct in written English to write people that. It 9 123456789012345678901234567890121234567890123456789012345678 9 123456789012345678901234567890121234567890123456789012345678 has to be people who (or people whom if what follows positions the 9 123456789012345678901234567890121234567890123456789012345678 9 123456789012345678901234567890121234567890123456789012345678 people as the object of a verb). The answer is (B). 23456789012345678901234567890121234567890123456789012345678 9 1 9 123456789012345678901234567890121234567890123456789012345678 9 123456789012345678901234567890121234567890123456789012345678 28. D The sentence as it stands is a bit of a disaster. It sounds like a run-on: it 9 123456789012345678901234567890121234567890123456789012345678 9 123456789012345678901234567890121234567890123456789012345678 just goes on and on like the Energizer Bunny. So what you will want to 23456789012345678901234567890121234567890123456789012345678 9 1 9 123456789012345678901234567890121234567890123456789012345678 do is make it more direct, showcasing the important parts and 9 123456789012345678901234567890121234567890123456789012345678 9 subordinating the descriptions that are really secondary. You also need 123456789012345678901234567890121234567890123456789012345678 9 123456789012345678901234567890121234567890123456789012345678 a comma after It is hard to believe. Start with the easiest thing, and 23456789012345678901234567890121234567890123456789012345678 9 1 9 123456789012345678901234567890121234567890123456789012345678 eliminate (A) and (C) off the bat. Which of (B), (D), and (E) makes the 9 123456789012345678901234567890121234567890123456789012345678 9 123456789012345678901234567890121234567890123456789012345678 sentence more direct? Definitely not (E). Choice (B) gets rid of the 9 123456789012345678901234567890121234567890123456789012345678 23456789012345678901234567890121234567890123456789012345678 9 123456789012345678901234567890121234567890123456789012345678 second comma/and combination, which could be good. But is a colon 9 123456789012345678901234567890121234567890123456789012345678 9 123456789012345678901234567890121234567890123456789012345678 really in order here? No. The best answer is (D). 123456789012345678901234567890121234567890123456789012345678 9 123456789012345678901234567890121234567890123456789012345678 9 9 123456789012345678901234567890121234567890123456789012345678 9 1 29. A This is a little tricky because the repetition here does serve a purpose; it 9 123456789012345678901234567890121234567890123456789012345678 isn’t just extra wordage that got in the author’s way. Basically, the 9 123456789012345678901234567890121234567890123456789012345678 9 123456789012345678901234567890121234567890123456789012345678 sentences are a list. When you have clauses that form a list (or other 23456789012345678901234567890121234567890123456789012345678 123456789012345678901234567890121234567890123456789012345678 9 9 123456789012345678901234567890121234567890123456789012345678 things requiring lots of words and/or punctuation), you separate them 9 123456789012345678901234567890121234567890123456789012345678 9 123456789012345678901234567890121234567890123456789012345678 with semicolons rather than commas. (A) looks good. All of the other 123456789012345678901234567890121234567890123456789012345678 9 9 123456789012345678901234567890121234567890123456789012345678 answers, except (D), change the sense of the original ever so slightly. (D) 9 123456789012345678901234567890121234567890123456789012345678 9 123456789012345678901234567890121234567890123456789012345678 could be possible if it had and before the last clause, but (A) is still 123456789012345678901234567890121234567890123456789012345678 9 9 1 better. 9 123456789012345678901234567890121234567890123456789012345678 9 123456789012345678901234567890121234567890123456789012345678 30. B While it would be possible to add a comma after convenience, it doesn’t 9 123456789012345678901234567890121234567890123456789012345678 9 123456789012345678901234567890121234567890123456789012345678 make much sense to add one after necessity. Changing you live to one 9 123456789012345678901234567890121234567890123456789012345678 23456789012345678901234567890121234567890123456789012345678 9 123456789012345678901234567890121234567890123456789012345678 lives is possible, but not required. So is adding Desert. The comma after 9 123456789012345678901234567890121234567890123456789012345678 9 1 apparently isn’t strictly required, but it is desirable. The only absolutely 9 123456789012345678901234567890121234567890123456789012345678 9 123456789012345678901234567890121234567890123456789012345678 necessary change is to replace “there” with “they are” (choice B). 23456789012345678901234567890121234567890123456789012345678 123456789012345678901234567890121234567890123456789012345678 9 9 123456789012345678901234567890121234567890123456789012345678 “There are” might have been more difficult to rule against (though still 9 1 9 123456789012345678901234567890121234567890123456789012345678 incorrect), but the sentence doesn’t even say there are; it just says there. 123456789012345678901234567890121234567890123456789012345678 9 23456789012345678901234567890121234567890123456789012345678 123456789012345678901234567890121234567890123456789012345678 9 123456789012345678901234567890121234567890123456789012345678 9 123456789012345678901234567890121234567890123456789012345678 9 123456789012345678901234567890121234567890123456789012345678 9 9 1 9 123456789012345678901234567890121234567890123456789012345678 9 123456789012345678901234567890121234567890123456789012345678 9 123456789012345678901234567890121234567890123456789012345678 9 123456789012345678901234567890121234567890123456789012345678 123456789012345678901234567890121234567890123456789012345678 9 23456789012345678901234567890121234567890123456789012345678 123456789012345678901234567890121234567890123456789012345678 9 123456789012345678901234567890121234567890123456789012345678 9 123456789012345678901234567890121234567890123456789012345678 9 123456789012345678901234567890121234567890123456789012345678 9 9 1 9 123456789012345678901234567890121234567890123456789012345678 123456789012345678901234567890121234567890123456789012345678 9 9 123456789012345678901234567890121234567890123456789012345678 123456789012345678901234567890121234567890123456789012345678 9 9 123456789012345678901234567890121234567890123456789012345678 123456789012345678901234567890121234567890123456789012345678 9 123456789012345678901234567890121234567890123456789012345678 9 123456789012345678901234567890121234567890123456789012345678 9 123456789012345678901234567890121234567890123456789012345678 9 1234567890123456789012345678901212345678901234567890123456789 62

Copyright © 2005 Thomson Peterson’s, a part of The Thomson Corporaton SAT is a registered trademark of the College Entrance Examination Board, which was not involved in the production of and does not endorse this product.

1234567890123456789012345678901212345678901234567890123456789 123456789012345678901234567890121234567890123456789012345678 9 9 123456789012345678901234567890121234567890123456789012345678 123456789012345678901234567890121234567890123456789012345678 9 9 123456789012345678901234567890121234567890123456789012345678 9 123456789012345678901234567890121234567890123456789012345678 Section 4 123456789012345678901234567890121234567890123456789012345678 9 9 1. A What is the name of the kind of talk that is delivered at a funeral? 123456789012345678901234567890121234567890123456789012345678 9 123456789012345678901234567890121234567890123456789012345678 Eulogy. If you know this, the answer pops out at you. If you did not 9 123456789012345678901234567890121234567890123456789012345678 9 123456789012345678901234567890121234567890123456789012345678 know it, consider each of the choices in their turn. Epigraph is a quote 9 123456789012345678901234567890121234567890123456789012345678 9 123456789012345678901234567890121234567890123456789012345678 at the beginning of a piece of writing. Eponymy is something with the 23456789012345678901234567890121234567890123456789012345678 9 1 9 123456789012345678901234567890121234567890123456789012345678 same name as something else. Epitaph is what is written on a 9 123456789012345678901234567890121234567890123456789012345678 9 123456789012345678901234567890121234567890123456789012345678 gravestone. That leaves (A) and (B). Elegy is a poem written in memory. 9 123456789012345678901234567890121234567890123456789012345678 23456789012345678901234567890121234567890123456789012345678 9 1 You don’t “give” a poem. That leaves (A), the correct answer. 9 123456789012345678901234567890121234567890123456789012345678 9 123456789012345678901234567890121234567890123456789012345678 9 123456789012345678901234567890121234567890123456789012345678 2. C On this dual-blank sentence, let’s do the first blank first since we know 9 123456789012345678901234567890121234567890123456789012345678 23456789012345678901234567890121234567890123456789012345678 9 1 that the blank was an encouragement to the rest of the team. Good 9 123456789012345678901234567890121234567890123456789012345678 9 123456789012345678901234567890121234567890123456789012345678 spirits would be an encouragement to the rest of the team. You can 9 123456789012345678901234567890121234567890123456789012345678 9 123456789012345678901234567890121234567890123456789012345678 eliminate (D) and (E). As for the second blank, what does a string of 23456789012345678901234567890121234567890123456789012345678 123456789012345678901234567890121234567890123456789012345678 99 123456789012345678901234567890121234567890123456789012345678 defeats do to a team? It discourages them. (A), elated, does not match 9 123456789012345678901234567890121234567890123456789012345678 9 123456789012345678901234567890121234567890123456789012345678 this. Nor does (B), inundated. But (C), dispirited, fits well and you’ve 123456789012345678901234567890121234567890123456789012345678 99 123456789012345678901234567890121234567890123456789012345678 already eliminated (D) and (E). Choice (C) is the best answer. 9 1 9 123456789012345678901234567890121234567890123456789012345678 3. B You might not know what resorted means, but if you know it’s a 9 123456789012345678901234567890121234567890123456789012345678 9 123456789012345678901234567890121234567890123456789012345678 negative word, you can make an educated guess. Which of the answer 23456789012345678901234567890121234567890123456789012345678 123456789012345678901234567890121234567890123456789012345678 99 123456789012345678901234567890121234567890123456789012345678 choices is also a negative verb? (A), (C), and (D) are not. (E) is not a 9 123456789012345678901234567890121234567890123456789012345678 9 123456789012345678901234567890121234567890123456789012345678 good answer because swindling has nothing to do with campaigning. 123456789012345678901234567890121234567890123456789012345678 99 123456789012345678901234567890121234567890123456789012345678 Choice (B) is the best choice. 9 123456789012345678901234567890121234567890123456789012345678 123456789012345678901234567890121234567890123456789012345678 99 123456789012345678901234567890121234567890123456789012345678 4. E The second half of the sentence gives more clues, so you ought to start 9 1 9 123456789012345678901234567890121234567890123456789012345678 there. What are cats most likely to do to dogs? Avoid them, 9 123456789012345678901234567890121234567890123456789012345678 9 123456789012345678901234567890121234567890123456789012345678 probably—which will lead you to (E), the correct answer. But for good 123456789012345678901234567890121234567890123456789012345678 9 9 123456789012345678901234567890121234567890123456789012345678 measure, let’s eliminate the other possibilities. For a cat to undermine a 9 123456789012345678901234567890121234567890123456789012345678 9 123456789012345678901234567890121234567890123456789012345678 dog isn’t logical. Being undermined is something that happens to 123456789012345678901234567890121234567890123456789012345678 9 9 123456789012345678901234567890121234567890123456789012345678 humans or projects, so you can definitely eliminate (D). One could say 9 123456789012345678901234567890121234567890123456789012345678 23456789012345678901234567890121234567890123456789012345678 9 123456789012345678901234567890121234567890123456789012345678 that a cat enticed a dog to do something, but it isn’t good usage simply 9 123456789012345678901234567890121234567890123456789012345678 to say that the cat enticed the dog. Eliminate (A). Is it likely for a cat to 9 1 9 123456789012345678901234567890121234567890123456789012345678 gracefully apprehend a dog? No. Eliminate (C). The only possibilities 9 123456789012345678901234567890121234567890123456789012345678 23456789012345678901234567890121234567890123456789012345678 9 123456789012345678901234567890121234567890123456789012345678 left are (B) and (E). A cat might possibly defeat a dog in battle, but use 9 123456789012345678901234567890121234567890123456789012345678 9 123456789012345678901234567890121234567890123456789012345678 the other clues. Defeat and “gracefully” don’t go well together, and it 123456789012345678901234567890121234567890123456789012345678 99 1 doesn’t make sense for a battle to happen while the cat is creeping 9 123456789012345678901234567890121234567890123456789012345678 9 123456789012345678901234567890121234567890123456789012345678 across the lawn. Eliminate (B). The answer is (E). 9 123456789012345678901234567890121234567890123456789012345678 9 123456789012345678901234567890121234567890123456789012345678 9 123456789012345678901234567890121234567890123456789012345678 5. A Attack the second blank first. The most likely adjective to describe 23456789012345678901234567890121234567890123456789012345678 123456789012345678901234567890121234567890123456789012345678 99 123456789012345678901234567890121234567890123456789012345678 attention will be something like undivided or rapt. (A), which includes 9 123456789012345678901234567890121234567890123456789012345678 9 123456789012345678901234567890121234567890123456789012345678 rapt, is the answer. (B), which includes spellbound, is also possible. But 9 1 23456789012345678901234567890121234567890123456789012345678 9 123456789012345678901234567890121234567890123456789012345678 pointed anecdotes doesn’t make sense, so the answer is (A). 9 1 9 123456789012345678901234567890121234567890123456789012345678 123456789012345678901234567890121234567890123456789012345678 9 9 123456789012345678901234567890121234567890123456789012345678 123456789012345678901234567890121234567890123456789012345678 9 123456789012345678901234567890121234567890123456789012345678 9 123456789012345678901234567890121234567890123456789012345678 9 123456789012345678901234567890121234567890123456789012345678 9 1234567890123456789012345678901212345678901234567890123456789 Copyright © 2005 Thomson Peterson’s, a part of The Thomson Corporaton SAT is a registered trademark of the College Entrance Examination Board, which was not involved in the production of and does not endorse this product.

63

1234567890123456789012345678901212345678901234567890123456789 123456789012345678901234567890121234567890123456789012345678 9 9 123456789012345678901234567890121234567890123456789012345678 123456789012345678901234567890121234567890123456789012345678 9 9 123456789012345678901234567890121234567890123456789012345678 6. E You know the word is going to be negative: both bogged and 9 123456789012345678901234567890121234567890123456789012345678 9 123456789012345678901234567890121234567890123456789012345678 contentious tell you so. Eliminate (B). Now think that the word is 9 123456789012345678901234567890121234567890123456789012345678 23456789012345678901234567890121234567890123456789012345678 9 1 basically going to mean mess. You can eliminate (A) and (C). 9 123456789012345678901234567890121234567890123456789012345678 9 123456789012345678901234567890121234567890123456789012345678 Conundrum is a confusing problem, not really a messy situation. (E) is 9 123456789012345678901234567890121234567890123456789012345678 9 123456789012345678901234567890121234567890123456789012345678 the best answer. 23456789012345678901234567890121234567890123456789012345678 9 1 9 123456789012345678901234567890121234567890123456789012345678 9 123456789012345678901234567890121234567890123456789012345678 7. B Here the first blank seems more approachable. The reference to a clear 9 123456789012345678901234567890121234567890123456789012345678 9 123456789012345678901234567890121234567890123456789012345678 leader indicates that the outcome was known. Eliminate (A) and (E). 23456789012345678901234567890121234567890123456789012345678 9 1 9 123456789012345678901234567890121234567890123456789012345678 For the second blank, the clue is that the leader misstepped and so rest 9 123456789012345678901234567890121234567890123456789012345678 9 of the competitors must have gotten a chance at the title, but they 123456789012345678901234567890121234567890123456789012345678 9 123456789012345678901234567890121234567890123456789012345678 weren’t assured a victory. Eliminate (D) and (C). The answer is (B). 23456789012345678901234567890121234567890123456789012345678 9 1 9 123456789012345678901234567890121234567890123456789012345678 9 123456789012345678901234567890121234567890123456789012345678 8. D Consider the first blank. The word though indicates that the drug was 9 123456789012345678901234567890121234567890123456789012345678 9 123456789012345678901234567890121234567890123456789012345678 intended to be beneficial but ultimately was not. Do any of the answer 23456789012345678901234567890121234567890123456789012345678 123456789012345678901234567890121234567890123456789012345678 9 9 123456789012345678901234567890121234567890123456789012345678 choices mean not beneficial? (A), (D), and (E) do. How would the 9 123456789012345678901234567890121234567890123456789012345678 9 123456789012345678901234567890121234567890123456789012345678 medical community respond to a bad result? Ostensibly they would 123456789012345678901234567890121234567890123456789012345678 9 9 123456789012345678901234567890121234567890123456789012345678 think that a bad result was bad. That eliminates (A) and (E). This 9 1 9 123456789012345678901234567890121234567890123456789012345678 leaves (D). 9 123456789012345678901234567890121234567890123456789012345678 9 123456789012345678901234567890121234567890123456789012345678 23456789012345678901234567890121234567890123456789012345678 9 123456789012345678901234567890121234567890123456789012345678 9. B A life that only lasts 24 hours is what in comparison to a normal human 9 123456789012345678901234567890121234567890123456789012345678 9 123456789012345678901234567890121234567890123456789012345678 life? It is short. Which of the answer choices contains the notion of 123456789012345678901234567890121234567890123456789012345678 9 9 123456789012345678901234567890121234567890123456789012345678 shortness in its meaning? (B), ephemeral, does. 9 123456789012345678901234567890121234567890123456789012345678 9 123456789012345678901234567890121234567890123456789012345678 10. E What is a species likely to do in an environment? It either grows in 9 123456789012345678901234567890121234567890123456789012345678 9 123456789012345678901234567890121234567890123456789012345678 number or diminishes in number. Each of the first words, except in (B), 9 1 23456789012345678901234567890121234567890123456789012345678 9 123456789012345678901234567890121234567890123456789012345678 could mean one of those things. Eliminate (B). When you discover that 9 123456789012345678901234567890121234567890123456789012345678 9 1 there is an abundance of food, you know that the first word will suggest 123456789012345678901234567890121234567890123456789012345678 9 9 123456789012345678901234567890121234567890123456789012345678 that the kangaroos increased in numbers. Eliminate (A) and (D). Now 9 123456789012345678901234567890121234567890123456789012345678 9 123456789012345678901234567890121234567890123456789012345678 you need the second part of the sentence. To grow in numbers, the 123456789012345678901234567890121234567890123456789012345678 9 9 123456789012345678901234567890121234567890123456789012345678 kangaroos will need an absence or near absence of predators. Eliminate 9 123456789012345678901234567890121234567890123456789012345678 23456789012345678901234567890121234567890123456789012345678 9 123456789012345678901234567890121234567890123456789012345678 (C). You are left with (E). 9 123456789012345678901234567890121234567890123456789012345678 9 1 9 123456789012345678901234567890121234567890123456789012345678 11. C The best clue in this sentence is “fears.” Citizens with fears can only be 123456789012345678901234567890121234567890123456789012345678 9 23456789012345678901234567890121234567890123456789012345678 9 123456789012345678901234567890121234567890123456789012345678 concerned or alarmed. That leaves (B) and (C). It’s not particularly 9 123456789012345678901234567890121234567890123456789012345678 9 123456789012345678901234567890121234567890123456789012345678 logical to say that a speech is designed to ignore something. On the 123456789012345678901234567890121234567890123456789012345678 9 9 1 other hand, it is common to use assuage with fears. The best answer 9 123456789012345678901234567890121234567890123456789012345678 is C. 9 123456789012345678901234567890121234567890123456789012345678 9 123456789012345678901234567890121234567890123456789012345678 9 123456789012345678901234567890121234567890123456789012345678 12. A Let’s attack the first blank. If the female is a fencing champion then she 123456789012345678901234567890121234567890123456789012345678 9 23456789012345678901234567890121234567890123456789012345678 9 123456789012345678901234567890121234567890123456789012345678 must be skillful with her rapier (her sword). Which of the first answer 9 123456789012345678901234567890121234567890123456789012345678 9 123456789012345678901234567890121234567890123456789012345678 choices matches skillful? Choices (A), (B), and (E) do. (C) is possible but 123456789012345678901234567890121234567890123456789012345678 9 9 1 not likely. As for the second blank, the conjunction but indicates that 9 123456789012345678901234567890121234567890123456789012345678 9 123456789012345678901234567890121234567890123456789012345678 her skillfulness in fencing is in contrast to her lack of skill in other 9 123456789012345678901234567890121234567890123456789012345678 9 123456789012345678901234567890121234567890123456789012345678 sports. Which of the remaining second answer choices matches with 9 123456789012345678901234567890121234567890123456789012345678 23456789012345678901234567890121234567890123456789012345678 9 1 this pre-guess? Only awkward, choice (A), does. 9 123456789012345678901234567890121234567890123456789012345678 123456789012345678901234567890121234567890123456789012345678 9 123456789012345678901234567890121234567890123456789012345678 9 1234567890123456789012345678901212345678901234567890123456789 64

Copyright © 2005 Thomson Peterson’s, a part of The Thomson Corporaton SAT is a registered trademark of the College Entrance Examination Board, which was not involved in the production of and does not endorse this product.

1234567890123456789012345678901212345678901234567890123456789 123456789012345678901234567890121234567890123456789012345678 9 9 123456789012345678901234567890121234567890123456789012345678 123456789012345678901234567890121234567890123456789012345678 9 9 123456789012345678901234567890121234567890123456789012345678 13. B There is a contrast drawn in the sentence between receiving accolades— 9 123456789012345678901234567890121234567890123456789012345678 9 123456789012345678901234567890121234567890123456789012345678 praise, awards—and Jane Goodall’s initial standing in her field. She 9 123456789012345678901234567890121234567890123456789012345678 23456789012345678901234567890121234567890123456789012345678 9 1 must have met with a lack of support or outright disapproval. Eliminate 9 123456789012345678901234567890121234567890123456789012345678 9 123456789012345678901234567890121234567890123456789012345678 (E) because it is illogical. Eliminate (C) because it goes with, rather than 9 123456789012345678901234567890121234567890123456789012345678 9 123456789012345678901234567890121234567890123456789012345678 against, accolades. An acolyte is someone who assists a clergyman, so 23456789012345678901234567890121234567890123456789012345678 9 1 9 123456789012345678901234567890121234567890123456789012345678 you can eliminate (A). You are left with (B) and (D). A charlatan is a 9 123456789012345678901234567890121234567890123456789012345678 9 123456789012345678901234567890121234567890123456789012345678 fake, an incompetent. If the sentence said, “Some people thought she 9 123456789012345678901234567890121234567890123456789012345678 23456789012345678901234567890121234567890123456789012345678 9 1 was a ——,” charlatan might work, but it says she actually was “a —-.” 9 123456789012345678901234567890121234567890123456789012345678 9 123456789012345678901234567890121234567890123456789012345678 She couldn’t have been a fake and later gotten awards. Eliminate (D). 9 123456789012345678901234567890121234567890123456789012345678 9 123456789012345678901234567890121234567890123456789012345678 You are left with (B), a maverick, an independent thinker, a dissenter, 23456789012345678901234567890121234567890123456789012345678 9 1 9 123456789012345678901234567890121234567890123456789012345678 a pioneer. 9 123456789012345678901234567890121234567890123456789012345678 9 123456789012345678901234567890121234567890123456789012345678 14. D This sentence is contrasting the views of Alston and Mario (the conjunc9 123456789012345678901234567890121234567890123456789012345678 23456789012345678901234567890121234567890123456789012345678 9 123456789012345678901234567890121234567890123456789012345678 tion but clues you into this fact). Alston thinks that the lecture was 9 123456789012345678901234567890121234567890123456789012345678 9 123456789012345678901234567890121234567890123456789012345678 impressive, which probably means smart, accurate, logical. Mario’s view 123456789012345678901234567890121234567890123456789012345678 99 123456789012345678901234567890121234567890123456789012345678 is in contrast to this. You can eliminate (A) and (C). Recondite is not 9 123456789012345678901234567890121234567890123456789012345678 9 1 likely to be a word to describe a lecture, so eliminate (B). You are left 9 123456789012345678901234567890121234567890123456789012345678 9 123456789012345678901234567890121234567890123456789012345678 with specious or fictitious. Specious means logically false; fictitious comes 9 123456789012345678901234567890121234567890123456789012345678 23456789012345678901234567890121234567890123456789012345678 9 123456789012345678901234567890121234567890123456789012345678 from fiction, and presumably the philosopher didn’t tell a story but 9 123456789012345678901234567890121234567890123456789012345678 9 123456789012345678901234567890121234567890123456789012345678 rather made an argument. Choice (D) is the best answer. 123456789012345678901234567890121234567890123456789012345678 99 123456789012345678901234567890121234567890123456789012345678 9 123456789012345678901234567890121234567890123456789012345678 15. A If you know that insinuation is a negative word, you can guess that the 9 123456789012345678901234567890121234567890123456789012345678 9 123456789012345678901234567890121234567890123456789012345678 first blank will describe a logical response to a negative thing. Balk is a 123456789012345678901234567890121234567890123456789012345678 99 1 common word in this situation, but if you don’t know that use the 9 123456789012345678901234567890121234567890123456789012345678 9 process of elimination. You can eliminate (B) and probably (D) because 123456789012345678901234567890121234567890123456789012345678 9 123456789012345678901234567890121234567890123456789012345678 they are not negative words. Move to the next blank. If the official’s 9 123456789012345678901234567890121234567890123456789012345678 9 123456789012345678901234567890121234567890123456789012345678 response is negative, it’s most logical that he is accused of having 23456789012345678901234567890121234567890123456789012345678 123456789012345678901234567890121234567890123456789012345678 99 123456789012345678901234567890121234567890123456789012345678 something to do with the economic woes. Eliminate (C). That leaves (A) 9 1 9 123456789012345678901234567890121234567890123456789012345678 and (E) as the most likely answers. But you don’t “rile” at something; 9 123456789012345678901234567890121234567890123456789012345678 23456789012345678901234567890121234567890123456789012345678 9 123456789012345678901234567890121234567890123456789012345678 it’s not good usage. Eliminate (E) and you are left with (A). 9 123456789012345678901234567890121234567890123456789012345678 9 1 9 123456789012345678901234567890121234567890123456789012345678 16. D The author is speaking generally in this first paragraph. Global 123456789012345678901234567890121234567890123456789012345678 9 23456789012345678901234567890121234567890123456789012345678 9 123456789012345678901234567890121234567890123456789012345678 warming and species extinction are two big, general problems; he refers 9 123456789012345678901234567890121234567890123456789012345678 9 123456789012345678901234567890121234567890123456789012345678 to them in a positive light at “maintaining global climate and genetic 123456789012345678901234567890121234567890123456789012345678 99 1 resources.” “Genetic resources” refers diverse species of plants and 9 123456789012345678901234567890121234567890123456789012345678 9 123456789012345678901234567890121234567890123456789012345678 animals, choice (D). 9 123456789012345678901234567890121234567890123456789012345678 9 123456789012345678901234567890121234567890123456789012345678 9 123456789012345678901234567890121234567890123456789012345678 17. E The author mentions that Los Amigos is relatively pristine, and that the 23456789012345678901234567890121234567890123456789012345678 9 123456789012345678901234567890121234567890123456789012345678 rainforest is facing threats. Eliminate (A) and (B). He isn’t talking in the 9 123456789012345678901234567890121234567890123456789012345678 9 123456789012345678901234567890121234567890123456789012345678 passage about restoring the rainforest, but preventing future damage. 9 123456789012345678901234567890121234567890123456789012345678 9 1 Eliminate (C). He does not say that every other part of the rainforest is 23456789012345678901234567890121234567890123456789012345678 123456789012345678901234567890121234567890123456789012345678 99 1 already destroyed beyond repair. Your logic should tell you that. 9 123456789012345678901234567890121234567890123456789012345678 9 123456789012345678901234567890121234567890123456789012345678 Eliminate (D) and you are left with (E), the correct answer. 9 123456789012345678901234567890121234567890123456789012345678 23456789012345678901234567890121234567890123456789012345678 9 1 123456789012345678901234567890121234567890123456789012345678 9 123456789012345678901234567890121234567890123456789012345678 9 123456789012345678901234567890121234567890123456789012345678 9 1234567890123456789012345678901212345678901234567890123456789 Copyright © 2005 Thomson Peterson’s, a part of The Thomson Corporaton SAT is a registered trademark of the College Entrance Examination Board, which was not involved in the production of and does not endorse this product.

65

1234567890123456789012345678901212345678901234567890123456789 123456789012345678901234567890121234567890123456789012345678 9 9 123456789012345678901234567890121234567890123456789012345678 123456789012345678901234567890121234567890123456789012345678 9 9 123456789012345678901234567890121234567890123456789012345678 18. A First go back and get the context of the use of this phrase. It refers to 9 123456789012345678901234567890121234567890123456789012345678 9 123456789012345678901234567890121234567890123456789012345678 land being set aside for conservation use. The only possibility is (A). 9 123456789012345678901234567890121234567890123456789012345678 23456789012345678901234567890121234567890123456789012345678 9 1 9 123456789012345678901234567890121234567890123456789012345678 19. D This is a difficult question because it requires you to infer the answer. 9 123456789012345678901234567890121234567890123456789012345678 9 123456789012345678901234567890121234567890123456789012345678 The best way to do that is to eliminate the least likely answers and then 9 123456789012345678901234567890121234567890123456789012345678 23456789012345678901234567890121234567890123456789012345678 9 1 see what’s left. The passage tells you that the agreement was “the first 9 123456789012345678901234567890121234567890123456789012345678 9 123456789012345678901234567890121234567890123456789012345678 long-term permanently renewable conservation concession.” There are 9 123456789012345678901234567890121234567890123456789012345678 9 123456789012345678901234567890121234567890123456789012345678 two references to time in this sentence, so the answer must have to do 23456789012345678901234567890121234567890123456789012345678 9 1 9 123456789012345678901234567890121234567890123456789012345678 with time—that leaves (B), (D), and (E). The author isn’t really 9 123456789012345678901234567890121234567890123456789012345678 9 interested in the legal aspects, though, so eliminate (E). Because he 123456789012345678901234567890121234567890123456789012345678 9 123456789012345678901234567890121234567890123456789012345678 includes both “long-term” and “renewable,” the agreement probably 23456789012345678901234567890121234567890123456789012345678 9 1 9 123456789012345678901234567890121234567890123456789012345678 wasn’t the first contract that was simply one or the other. Eliminate (B). 9 123456789012345678901234567890121234567890123456789012345678 9 123456789012345678901234567890121234567890123456789012345678 That leaves you with (D), the correct answer. 9 123456789012345678901234567890121234567890123456789012345678 23456789012345678901234567890121234567890123456789012345678 123456789012345678901234567890121234567890123456789012345678 9 9 123456789012345678901234567890121234567890123456789012345678 20. C This is a question that you should be asking yourself as you read through 9 123456789012345678901234567890121234567890123456789012345678 9 123456789012345678901234567890121234567890123456789012345678 the passage. The passage begins by discussing the importance of conser123456789012345678901234567890121234567890123456789012345678 9 9 123456789012345678901234567890121234567890123456789012345678 vation efforts in Amazonia and then links the work at the Los Amigos 9 1 9 123456789012345678901234567890121234567890123456789012345678 watershed with this goal. The correct answer will contain both of these 9 123456789012345678901234567890121234567890123456789012345678 9 123456789012345678901234567890121234567890123456789012345678 things. (A) is too general. (B) isn’t accurate—he doesn’t focus on elimi23456789012345678901234567890121234567890123456789012345678 123456789012345678901234567890121234567890123456789012345678 9 9 123456789012345678901234567890121234567890123456789012345678 nating bad things but on continuing good things. (C) sounds good. (D) 9 123456789012345678901234567890121234567890123456789012345678 9 123456789012345678901234567890121234567890123456789012345678 is incorrect because the passage is not primarily about the Peruvian gov123456789012345678901234567890121234567890123456789012345678 9 9 123456789012345678901234567890121234567890123456789012345678 ernment. (E) points to one issue that the passage discusses but lacks 9 123456789012345678901234567890121234567890123456789012345678 9 123456789012345678901234567890121234567890123456789012345678 many of the other issues the passage discusses. (C) is the best answer. 123456789012345678901234567890121234567890123456789012345678 9 9 1 21. D This question calls for a little nuance. He does advocate for his project, 9 123456789012345678901234567890121234567890123456789012345678 9 123456789012345678901234567890121234567890123456789012345678 but does not position it against other projects. Eliminate (A). (B) is too 9 123456789012345678901234567890121234567890123456789012345678 9 123456789012345678901234567890121234567890123456789012345678 general. (C) is not accurate—he does not condemn the government. (D) 9 123456789012345678901234567890121234567890123456789012345678 23456789012345678901234567890121234567890123456789012345678 9 123456789012345678901234567890121234567890123456789012345678 sounds good. (E) uses language that is too strong—he is not a zealot, 9 123456789012345678901234567890121234567890123456789012345678 9 1 but a scientist making his case in calm, rational language. (D) is correct. 9 123456789012345678901234567890121234567890123456789012345678 9 123456789012345678901234567890121234567890123456789012345678 23456789012345678901234567890121234567890123456789012345678 9 123456789012345678901234567890121234567890123456789012345678 22. E The author positions his project as complementary to other projects. These 9 123456789012345678901234567890121234567890123456789012345678 9 1 scientists are examples of the other amenable projects. The answer is (E). 123456789012345678901234567890121234567890123456789012345678 9 123456789012345678901234567890121234567890123456789012345678 9 9 123456789012345678901234567890121234567890123456789012345678 23. E This is a tricky answer because the right choice is the one you’d least 9 123456789012345678901234567890121234567890123456789012345678 9 123456789012345678901234567890121234567890123456789012345678 expect. The author focuses on working with plants in the watershed, but 9 123456789012345678901234567890121234567890123456789012345678 9 123456789012345678901234567890121234567890123456789012345678 in the last word of the passage mentions an “herbarium,” which through 23456789012345678901234567890121234567890123456789012345678 9 123456789012345678901234567890121234567890123456789012345678 context clues and word study, you can guess means a laboratory where 9 123456789012345678901234567890121234567890123456789012345678 9 123456789012345678901234567890121234567890123456789012345678 plants are grown. Eliminate (A). The author mentions studying “human9 123456789012345678901234567890121234567890123456789012345678 9 1 plant” interactions in paragraph 6. Eliminate (B). Somewhat surprisingly, 23456789012345678901234567890121234567890123456789012345678 123456789012345678901234567890121234567890123456789012345678 9 9 123456789012345678901234567890121234567890123456789012345678 the author is in favor of pharmaceutical use of Amazon plants, as he 9 123456789012345678901234567890121234567890123456789012345678 9 123456789012345678901234567890121234567890123456789012345678 indicates in paragraphs 1 and 5 and implies in paragraph 6. Eliminate (C). 9 1 23456789012345678901234567890121234567890123456789012345678 9 123456789012345678901234567890121234567890123456789012345678 (D) is obviously not the answer. You might think that because he focuses 9 1 9 123456789012345678901234567890121234567890123456789012345678 on naming, he means labeling, but in fact it is a scientist on another project, 123456789012345678901234567890121234567890123456789012345678 9 9 123456789012345678901234567890121234567890123456789012345678 Robin Foster, who actually labeled plants. (E) is the answer. 9 123456789012345678901234567890121234567890123456789012345678 123456789012345678901234567890121234567890123456789012345678 9 123456789012345678901234567890121234567890123456789012345678 9 123456789012345678901234567890121234567890123456789012345678 9 1234567890123456789012345678901212345678901234567890123456789 66

Copyright © 2005 Thomson Peterson’s, a part of The Thomson Corporaton SAT is a registered trademark of the College Entrance Examination Board, which was not involved in the production of and does not endorse this product.

1234567890123456789012345678901212345678901234567890123456789 123456789012345678901234567890121234567890123456789012345678 9 9 123456789012345678901234567890121234567890123456789012345678 123456789012345678901234567890121234567890123456789012345678 9 9 123456789012345678901234567890121234567890123456789012345678 24. B As always, first go back and read the section cited in the question. The 9 123456789012345678901234567890121234567890123456789012345678 9 123456789012345678901234567890121234567890123456789012345678 sentence in which “providing names” occurs, mentions communication 9 123456789012345678901234567890121234567890123456789012345678 23456789012345678901234567890121234567890123456789012345678 9 1 about plants and the animals that use them. You will recall that earlier 9 123456789012345678901234567890121234567890123456789012345678 9 123456789012345678901234567890121234567890123456789012345678 in the passage, it was stated that one of the major projects in studying 9 123456789012345678901234567890121234567890123456789012345678 9 123456789012345678901234567890121234567890123456789012345678 Amazonia was discovering new species. One hurdle for communication 23456789012345678901234567890121234567890123456789012345678 9 1 9 123456789012345678901234567890121234567890123456789012345678 among scientists once a species is discovered is standardizing the name 9 123456789012345678901234567890121234567890123456789012345678 9 123456789012345678901234567890121234567890123456789012345678 of the species. This is how “providing names” will facilitate communi9 123456789012345678901234567890121234567890123456789012345678 23456789012345678901234567890121234567890123456789012345678 9 1 cation. Choice (B) correctly points this out. (If you had difficulty with 9 123456789012345678901234567890121234567890123456789012345678 9 123456789012345678901234567890121234567890123456789012345678 this question, notice that all the other choices mention issues not 9 123456789012345678901234567890121234567890123456789012345678 9 123456789012345678901234567890121234567890123456789012345678 directly addressed in the passage. That is a strong indicator that an 23456789012345678901234567890121234567890123456789012345678 9 1 9 123456789012345678901234567890121234567890123456789012345678 answer is incorrect.) 9 123456789012345678901234567890121234567890123456789012345678 9 123456789012345678901234567890121234567890123456789012345678 25. A The author’s full argument goes, “To be informed, we must develop 9 123456789012345678901234567890121234567890123456789012345678 23456789012345678901234567890121234567890123456789012345678 9 123456789012345678901234567890121234567890123456789012345678 knowledge. To develop knowledge, we must collect, organize, and 9 123456789012345678901234567890121234567890123456789012345678 9 123456789012345678901234567890121234567890123456789012345678 disseminate information. In this sense, botanical information has 123456789012345678901234567890121234567890123456789012345678 99 123456789012345678901234567890121234567890123456789012345678 conservation value.” The author is arguing that being informed is 9 123456789012345678901234567890121234567890123456789012345678 9 1 essential for conservationism, and so in this sense botanical informa9 123456789012345678901234567890121234567890123456789012345678 9 123456789012345678901234567890121234567890123456789012345678 tion has conservation value. So even though (B), (C), (D), and (E) are all 9 123456789012345678901234567890121234567890123456789012345678 23456789012345678901234567890121234567890123456789012345678 9 123456789012345678901234567890121234567890123456789012345678 things the author might agree with, only (A) captures the meaning of 9 123456789012345678901234567890121234567890123456789012345678 9 123456789012345678901234567890121234567890123456789012345678 the argument made here. 123456789012345678901234567890121234567890123456789012345678 99 123456789012345678901234567890121234567890123456789012345678 9 123456789012345678901234567890121234567890123456789012345678 26. B You might confuse repeated use of the word watershed with an actual 9 123456789012345678901234567890121234567890123456789012345678 9 123456789012345678901234567890121234567890123456789012345678 discussion of water pollution, but the author doesn’t mention water 123456789012345678901234567890121234567890123456789012345678 99 1 pollution explicitly. The answer is (B). If you don’t get this right away, 9 123456789012345678901234567890121234567890123456789012345678 9 you can arrive at it by eliminating the others. He does clearly mention 123456789012345678901234567890121234567890123456789012345678 9 123456789012345678901234567890121234567890123456789012345678 all of the other choices. 9 123456789012345678901234567890121234567890123456789012345678 9 123456789012345678901234567890121234567890123456789012345678 23456789012345678901234567890121234567890123456789012345678 9 123456789012345678901234567890121234567890123456789012345678 27. C The author is talking about how his work at Los Amigos relates to 9 123456789012345678901234567890121234567890123456789012345678 9 1 other conservation projects, and how the Los Amigos area is related to 9 123456789012345678901234567890121234567890123456789012345678 9 123456789012345678901234567890121234567890123456789012345678 other environmentally protected areas. Only (C) captures that meaning. 23456789012345678901234567890121234567890123456789012345678 123456789012345678901234567890121234567890123456789012345678 99 123456789012345678901234567890121234567890123456789012345678 9 1 123456789012345678901234567890121234567890123456789012345678 9 9 123456789012345678901234567890121234567890123456789012345678 Section 5 9 123456789012345678901234567890121234567890123456789012345678 9 123456789012345678901234567890121234567890123456789012345678 1. C For every positive integer, there is a negative integer the same distance 9 123456789012345678901234567890121234567890123456789012345678 9 123456789012345678901234567890121234567890123456789012345678 from zero. This means that there are an equal number of positive and 123456789012345678901234567890121234567890123456789012345678 9 23456789012345678901234567890121234567890123456789012345678 9 123456789012345678901234567890121234567890123456789012345678 negative integers. Therefore each set contains the same number of 9 123456789012345678901234567890121234567890123456789012345678 9 123456789012345678901234567890121234567890123456789012345678 members, which is choice (C). 123456789012345678901234567890121234567890123456789012345678 99 1 2. B Four sweaters cost p dollars, and so 12 sweaters at the regular price 9 123456789012345678901234567890121234567890123456789012345678 9 123456789012345678901234567890121234567890123456789012345678 would cost 3p (4 3 3 5 12). Since the sweaters are half-off, the 12 9 123456789012345678901234567890121234567890123456789012345678 9 123456789012345678901234567890121234567890123456789012345678 3p 9 123456789012345678901234567890121234567890123456789012345678 , choice (B). sweaters only cost half as much, 23456789012345678901234567890121234567890123456789012345678 9 123456789012345678901234567890121234567890123456789012345678 2 9 1 9 123456789012345678901234567890121234567890123456789012345678 123456789012345678901234567890121234567890123456789012345678 9 9 123456789012345678901234567890121234567890123456789012345678 123456789012345678901234567890121234567890123456789012345678 9 123456789012345678901234567890121234567890123456789012345678 9 123456789012345678901234567890121234567890123456789012345678 9 123456789012345678901234567890121234567890123456789012345678 9 1234567890123456789012345678901212345678901234567890123456789 Copyright © 2005 Thomson Peterson’s, a part of The Thomson Corporaton SAT is a registered trademark of the College Entrance Examination Board, which was not involved in the production of and does not endorse this product.

67

1234567890123456789012345678901212345678901234567890123456789 123456789012345678901234567890121234567890123456789012345678 9 9 123456789012345678901234567890121234567890123456789012345678 123456789012345678901234567890121234567890123456789012345678 9 9 123456789012345678901234567890121234567890123456789012345678 3. C This fraction looks complicated, but realize that the numerator and de9 123456789012345678901234567890121234567890123456789012345678 9 123456789012345678901234567890121234567890123456789012345678 nominator are both fractions. You might then see that the numerator 9 123456789012345678901234567890121234567890123456789012345678 23456789012345678901234567890121234567890123456789012345678 9 1 2 4 9 123456789012345678901234567890121234567890123456789012345678 is less than the denominator , and so this fraction is less than 9 123456789012345678901234567890121234567890123456789012345678 3 5 9 123456789012345678901234567890121234567890123456789012345678 9 123456789012345678901234567890121234567890123456789012345678 one. That eliminates (D) and (E) as possible answer choices. You have to 23456789012345678901234567890121234567890123456789012345678 9 1 9 123456789012345678901234567890121234567890123456789012345678 simplify the fraction to find the answer, and since you’re dividing frac9 123456789012345678901234567890121234567890123456789012345678 tions, remember to multiply by the reciprocal of the divisor: 9 123456789012345678901234567890121234567890123456789012345678 9 123456789012345678901234567890121234567890123456789012345678 23456789012345678901234567890121234567890123456789012345678 9 1 2 9 123456789012345678901234567890121234567890123456789012345678 9 123456789012345678901234567890121234567890123456789012345678 3 2 4 2 5 10 5 9 123456789012345678901234567890121234567890123456789012345678 5 4 5 3 5 5 . (C) is the answer. 9 123456789012345678901234567890121234567890123456789012345678 3 5 3 4 12 6 23456789012345678901234567890121234567890123456789012345678 9 4 1 9 123456789012345678901234567890121234567890123456789012345678 9 123456789012345678901234567890121234567890123456789012345678 5 9 123456789012345678901234567890121234567890123456789012345678 9 123456789012345678901234567890121234567890123456789012345678 4. E Here you just need to plug in and be careful. If x 5 3 then y 5 1 because 23456789012345678901234567890121234567890123456789012345678 123456789012345678901234567890121234567890123456789012345678 9 9 123456789012345678901234567890121234567890123456789012345678 3y 5 x 9 123456789012345678901234567890121234567890123456789012345678 123456789012345678901234567890121234567890123456789012345678 9 9 123456789012345678901234567890121234567890123456789012345678 3y 5 3 9 123456789012345678901234567890121234567890123456789012345678 9 1 3y 3 9 123456789012345678901234567890121234567890123456789012345678 5 9 123456789012345678901234567890121234567890123456789012345678 3 3 9 123456789012345678901234567890121234567890123456789012345678 23456789012345678901234567890121234567890123456789012345678 9 123456789012345678901234567890121234567890123456789012345678 y51 9 123456789012345678901234567890121234567890123456789012345678 123456789012345678901234567890121234567890123456789012345678 9 9 123456789012345678901234567890121234567890123456789012345678 If y 5 1, then 9 123456789012345678901234567890121234567890123456789012345678 9 123456789012345678901234567890121234567890123456789012345678 10 9 123456789012345678901234567890121234567890123456789012345678 y5 9 123456789012345678901234567890121234567890123456789012345678 z 9 123456789012345678901234567890121234567890123456789012345678 9 1 23456789012345678901234567890121234567890123456789012345678 9 123456789012345678901234567890121234567890123456789012345678 10 9 123456789012345678901234567890121234567890123456789012345678 1 5 9 1 z 123456789012345678901234567890121234567890123456789012345678 9 9 123456789012345678901234567890121234567890123456789012345678 10 9 123456789012345678901234567890121234567890123456789012345678 ~z!1 5 z ~z! 9 123456789012345678901234567890121234567890123456789012345678 123456789012345678901234567890121234567890123456789012345678 9 9 123456789012345678901234567890121234567890123456789012345678 9 z 5 10 123456789012345678901234567890121234567890123456789012345678 23456789012345678901234567890121234567890123456789012345678 123456789012345678901234567890121234567890123456789012345678 9 9 123456789012345678901234567890121234567890123456789012345678 This is choice (E). 9 1 123456789012345678901234567890121234567890123456789012345678 9 9 123456789012345678901234567890121234567890123456789012345678 5. C y 5 mx 1 b is the equation for a line, so the wiggly lines of choices (D) 9 123456789012345678901234567890121234567890123456789012345678 9 123456789012345678901234567890121234567890123456789012345678 and (E) cannot be correct. A negative m means the slope is negative, so 9 123456789012345678901234567890121234567890123456789012345678 9 123456789012345678901234567890121234567890123456789012345678 the line must slant down when viewed from left to right. That 123456789012345678901234567890121234567890123456789012345678 9 23456789012345678901234567890121234567890123456789012345678 9 123456789012345678901234567890121234567890123456789012345678 eliminates (A). The y-intercept b is positive, so the line must cross the 9 123456789012345678901234567890121234567890123456789012345678 9 123456789012345678901234567890121234567890123456789012345678 y-axis above the x-axis. This is true of (C), so it’s the answer. 123456789012345678901234567890121234567890123456789012345678 9 9 1 9 123456789012345678901234567890121234567890123456789012345678 6. A This problem requires a little careful decoding, but all the steps are 9 123456789012345678901234567890121234567890123456789012345678 9 straightforward. The first part of the order is 2 H-M, which are two 123456789012345678901234567890121234567890123456789012345678 9 123456789012345678901234567890121234567890123456789012345678 medium hats. Usually the hats are $12 each, so two would cost $24, but 9 123456789012345678901234567890121234567890123456789012345678 23456789012345678901234567890121234567890123456789012345678 9 123456789012345678901234567890121234567890123456789012345678 since all medium-sized items are 25% off, the price is reduced. Twenty9 1 9 123456789012345678901234567890121234567890123456789012345678 five percent of 24 is 6, so together the two medium hats cost $18 ($24 9 123456789012345678901234567890121234567890123456789012345678 9 123456789012345678901234567890121234567890123456789012345678 minus $6). The next part is 2 H-L, which is two large hats, and together 23456789012345678901234567890121234567890123456789012345678 9 1 123456789012345678901234567890121234567890123456789012345678 9 123456789012345678901234567890121234567890123456789012345678 9 123456789012345678901234567890121234567890123456789012345678 9 1234567890123456789012345678901212345678901234567890123456789

SD

SD

S D

68

Copyright © 2005 Thomson Peterson’s, a part of The Thomson Corporaton SAT is a registered trademark of the College Entrance Examination Board, which was not involved in the production of and does not endorse this product.

1234567890123456789012345678901212345678901234567890123456789 123456789012345678901234567890121234567890123456789012345678 9 9 123456789012345678901234567890121234567890123456789012345678 123456789012345678901234567890121234567890123456789012345678 9 9 123456789012345678901234567890121234567890123456789012345678 they cost $24. The last part of the order is 1 SH-M, one medium shirt, 9 123456789012345678901234567890121234567890123456789012345678 9 123456789012345678901234567890121234567890123456789012345678 which costs $9 since it is 25% off ($3 is 25% of $12, the normal cost of 9 123456789012345678901234567890121234567890123456789012345678 23456789012345678901234567890121234567890123456789012345678 9 1 the medium shirt). Adding up 18 1 24 1 9 5 51, answer (A). 9 123456789012345678901234567890121234567890123456789012345678 9 123456789012345678901234567890121234567890123456789012345678 9 123456789012345678901234567890121234567890123456789012345678 7. A The task here is to solve for x. The test makers are betting that the 9 123456789012345678901234567890121234567890123456789012345678 23456789012345678901234567890121234567890123456789012345678 9 1 square root symbol will throw you off a bit. 9 123456789012345678901234567890121234567890123456789012345678 9 123456789012345678901234567890121234567890123456789012345678 9 123456789012345678901234567890121234567890123456789012345678 x 1 15 5 30 5 = 9 123456789012345678901234567890121234567890123456789012345678 23456789012345678901234567890121234567890123456789012345678 9 1 9 123456789012345678901234567890121234567890123456789012345678 5 x 1 15 2 15 5 30 2 15 = 9 123456789012345678901234567890121234567890123456789012345678 9 123456789012345678901234567890121234567890123456789012345678 9 123456789012345678901234567890121234567890123456789012345678 5 x 5 15 = 23456789012345678901234567890121234567890123456789012345678 9 1 9 123456789012345678901234567890121234567890123456789012345678 9 123456789012345678901234567890121234567890123456789012345678 5=x 15 9 123456789012345678901234567890121234567890123456789012345678 5 9 123456789012345678901234567890121234567890123456789012345678 5 5 23456789012345678901234567890121234567890123456789012345678 123456789012345678901234567890121234567890123456789012345678 99 123456789012345678901234567890121234567890123456789012345678 =x 5 3 9 123456789012345678901234567890121234567890123456789012345678 123456789012345678901234567890121234567890123456789012345678 99 123456789012345678901234567890121234567890123456789012345678 x59 9 123456789012345678901234567890121234567890123456789012345678 9 1 Answer (A). 9 123456789012345678901234567890121234567890123456789012345678 9 123456789012345678901234567890121234567890123456789012345678 8. D Here there is no shortcut. We have to factor and then simplify. 9 123456789012345678901234567890121234567890123456789012345678 23456789012345678901234567890121234567890123456789012345678 9 123456789012345678901234567890121234567890123456789012345678 2 9 123456789012345678901234567890121234567890123456789012345678 x 2 x 2 12 x 2 4 x 1 3 x -4 x 1 3 ~ !~ ! 5 ~ !~ ! 5 9 123456789012345678901234567890121234567890123456789012345678 5 9 123456789012345678901234567890121234567890123456789012345678 2 2 2x 1 2x 2 12 2 x 1 x 2 6 2 x 1 3 x 2 2 ~ ! ~ !~ ! 123456789012345678901234567890121234567890123456789012345678 99 123456789012345678901234567890121234567890123456789012345678 x24 9 123456789012345678901234567890121234567890123456789012345678 . (D) is the answer. 9 123456789012345678901234567890121234567890123456789012345678 2 x 2 2 ~ ! 123456789012345678901234567890121234567890123456789012345678 99 1 9. C The first thing to do is to have it clearly in mind what the graph of 9 123456789012345678901234567890121234567890123456789012345678 2 9 123456789012345678901234567890121234567890123456789012345678 looks like. f(x) 5 x 123456789012345678901234567890121234567890123456789012345678 9 123456789012345678901234567890121234567890123456789012345678 9 9 123456789012345678901234567890121234567890123456789012345678 9 123456789012345678901234567890121234567890123456789012345678 9 123456789012345678901234567890121234567890123456789012345678 123456789012345678901234567890121234567890123456789012345678 9 9 123456789012345678901234567890121234567890123456789012345678 9 123456789012345678901234567890121234567890123456789012345678 23456789012345678901234567890121234567890123456789012345678 123456789012345678901234567890121234567890123456789012345678 99 123456789012345678901234567890121234567890123456789012345678 9 1 123456789012345678901234567890121234567890123456789012345678 9 123456789012345678901234567890121234567890123456789012345678 9 9 123456789012345678901234567890121234567890123456789012345678 9 123456789012345678901234567890121234567890123456789012345678 9 123456789012345678901234567890121234567890123456789012345678 9 123456789012345678901234567890121234567890123456789012345678 123456789012345678901234567890121234567890123456789012345678 9 23456789012345678901234567890121234567890123456789012345678 123456789012345678901234567890121234567890123456789012345678 99 123456789012345678901234567890121234567890123456789012345678 9 123456789012345678901234567890121234567890123456789012345678 9 123456789012345678901234567890121234567890123456789012345678 9 1 9 123456789012345678901234567890121234567890123456789012345678 9 123456789012345678901234567890121234567890123456789012345678 9 123456789012345678901234567890121234567890123456789012345678 9 123456789012345678901234567890121234567890123456789012345678 123456789012345678901234567890121234567890123456789012345678 9 23456789012345678901234567890121234567890123456789012345678 123456789012345678901234567890121234567890123456789012345678 99 1 9 123456789012345678901234567890121234567890123456789012345678 123456789012345678901234567890121234567890123456789012345678 9 9 123456789012345678901234567890121234567890123456789012345678 123456789012345678901234567890121234567890123456789012345678 9 123456789012345678901234567890121234567890123456789012345678 9 123456789012345678901234567890121234567890123456789012345678 9 123456789012345678901234567890121234567890123456789012345678 9 1234567890123456789012345678901212345678901234567890123456789 Copyright © 2005 Thomson Peterson’s, a part of The Thomson Corporaton SAT is a registered trademark of the College Entrance Examination Board, which was not involved in the production of and does not endorse this product.

69

1234567890123456789012345678901212345678901234567890123456789 123456789012345678901234567890121234567890123456789012345678 9 9 123456789012345678901234567890121234567890123456789012345678 9 123456789012345678901234567890121234567890123456789012345678 2 2 9 123456789012345678901234567890121234567890123456789012345678 This shape comes about as a result of the x term. When f(x) 5 x , the 9 123456789012345678901234567890121234567890123456789012345678 9 123456789012345678901234567890121234567890123456789012345678 fact that all x values are squared makes every y value positive (keep in 9 123456789012345678901234567890121234567890123456789012345678 23456789012345678901234567890121234567890123456789012345678 9 1 mind that f(x) acts as the y value). This graph has no negative y values, 9 123456789012345678901234567890121234567890123456789012345678 9 123456789012345678901234567890121234567890123456789012345678 but to find the right answer, it must now be rotated clockwise. This 9 123456789012345678901234567890121234567890123456789012345678 9 123456789012345678901234567890121234567890123456789012345678 means rotating to the right. 23456789012345678901234567890121234567890123456789012345678 9 1 9 123456789012345678901234567890121234567890123456789012345678 9 123456789012345678901234567890121234567890123456789012345678 Here’s a good way to visualize this. Hold up your left hand, and make a 9 123456789012345678901234567890121234567890123456789012345678 9 123456789012345678901234567890121234567890123456789012345678 bowl-shape with your fingers on one side and your thumb on the other. 23456789012345678901234567890121234567890123456789012345678 9 1 9 123456789012345678901234567890121234567890123456789012345678 The tips of your fingers and thumbs should all be point towards the ceil9 123456789012345678901234567890121234567890123456789012345678 9 ing. This is your graph before rotation. Now take your left hand and shift 123456789012345678901234567890121234567890123456789012345678 9 123456789012345678901234567890121234567890123456789012345678 it 90° to the right; in essence, snap your wrist down. The tips of your 23456789012345678901234567890121234567890123456789012345678 9 1 9 123456789012345678901234567890121234567890123456789012345678 fingers and thumb should now be pointing to the right. Find the answer 9 123456789012345678901234567890121234567890123456789012345678 9 123456789012345678901234567890121234567890123456789012345678 choice that looks like the position of your left hand. (C) is the answer. 9 123456789012345678901234567890121234567890123456789012345678 23456789012345678901234567890121234567890123456789012345678 123456789012345678901234567890121234567890123456789012345678 9 9 123456789012345678901234567890121234567890123456789012345678 10. E The correct ordered pair has to satisfy two conditions, x 2 y . 2 and 9 123456789012345678901234567890121234567890123456789012345678 9 123456789012345678901234567890121234567890123456789012345678 x 1 y . 4. Let’s start with the first condition and take it from there. In 123456789012345678901234567890121234567890123456789012345678 9 9 123456789012345678901234567890121234567890123456789012345678 order for x 2 y . 2, x must be greater than y by more than two. This 9 1 9 123456789012345678901234567890121234567890123456789012345678 eliminates choices (A), (B), and (D). Now you have only two choices left 9 123456789012345678901234567890121234567890123456789012345678 9 123456789012345678901234567890121234567890123456789012345678 to check the second condition, x 1 y . 4. If you add the x and y values 23456789012345678901234567890121234567890123456789012345678 123456789012345678901234567890121234567890123456789012345678 9 9 123456789012345678901234567890121234567890123456789012345678 of choice (C) together you get 4, but the value must be greater than 4. 9 123456789012345678901234567890121234567890123456789012345678 9 123456789012345678901234567890121234567890123456789012345678 Choice (E) has to be the correct answer. 123456789012345678901234567890121234567890123456789012345678 9 123456789012345678901234567890121234567890123456789012345678 9 9 123456789012345678901234567890121234567890123456789012345678 11. A The best way to approach a problem like this is to use one of the 9 123456789012345678901234567890121234567890123456789012345678 equations and solve for one of the variables in terms of the other. Then 9 123456789012345678901234567890121234567890123456789012345678 9 1 you can place that answer into the second equation. 23456789012345678901234567890121234567890123456789012345678 123456789012345678901234567890121234567890123456789012345678 9 123456789012345678901234567890121234567890123456789012345678 9 9 1 This problem succumbs quite nicely to this strategy. Take the first 9 123456789012345678901234567890121234567890123456789012345678 9 123456789012345678901234567890121234567890123456789012345678 equation and solve for y in terms of x. Cross-multiplication makes 23456789012345678901234567890121234567890123456789012345678 123456789012345678901234567890121234567890123456789012345678 9 9 123456789012345678901234567890121234567890123456789012345678 this simple. 9 1 9 123456789012345678901234567890121234567890123456789012345678 9 123456789012345678901234567890121234567890123456789012345678 1 x 23456789012345678901234567890121234567890123456789012345678 9 123456789012345678901234567890121234567890123456789012345678 5 9 123456789012345678901234567890121234567890123456789012345678 y 2 9 1 123456789012345678901234567890121234567890123456789012345678 9 9 123456789012345678901234567890121234567890123456789012345678 2x 5 y 9 123456789012345678901234567890121234567890123456789012345678 9 123456789012345678901234567890121234567890123456789012345678 You can now substitute this information into x 1 y. Subbing in for y: 9 123456789012345678901234567890121234567890123456789012345678 9 123456789012345678901234567890121234567890123456789012345678 x 1 y 5 x 1 2x 5 3x, which is (A). 123456789012345678901234567890121234567890123456789012345678 9 23456789012345678901234567890121234567890123456789012345678 123456789012345678901234567890121234567890123456789012345678 9 9 123456789012345678901234567890121234567890123456789012345678 12. B There are two main ways to approach this problem. You can come up 9 123456789012345678901234567890121234567890123456789012345678 9 123456789012345678901234567890121234567890123456789012345678 with a snazzy algebraic formula, or you can go to the answer choices 9 1 23456789012345678901234567890121234567890123456789012345678 9 123456789012345678901234567890121234567890123456789012345678 and start cranking in numbers. Let’s use the crank method. 9 123456789012345678901234567890121234567890123456789012345678 123456789012345678901234567890121234567890123456789012345678 9 9 123456789012345678901234567890121234567890123456789012345678 Whichever method you use, you must first determine how much 9 1 23456789012345678901234567890121234567890123456789012345678 9 123456789012345678901234567890121234567890123456789012345678 Quentin paid for the three hot dogs. This is done by subtracting $7.34 9 1 9 123456789012345678901234567890121234567890123456789012345678 from $10, giving you $2.66. This is the price, including tax, of three 123456789012345678901234567890121234567890123456789012345678 9 9 123456789012345678901234567890121234567890123456789012345678 hot dogs. 123456789012345678901234567890121234567890123456789012345678 9 123456789012345678901234567890121234567890123456789012345678 9 123456789012345678901234567890121234567890123456789012345678 9 123456789012345678901234567890121234567890123456789012345678 9 1234567890123456789012345678901212345678901234567890123456789 70

Copyright © 2005 Thomson Peterson’s, a part of The Thomson Corporaton SAT is a registered trademark of the College Entrance Examination Board, which was not involved in the production of and does not endorse this product.

Tip

1234567890123456789012345678901212345678901234567890123456789 123456789012345678901234567890121234567890123456789012345678 9 9 123456789012345678901234567890121234567890123456789012345678 123456789012345678901234567890121234567890123456789012345678 9 9 123456789012345678901234567890121234567890123456789012345678 With the crank method, starting with the middle answer choice (C) 9 123456789012345678901234567890121234567890123456789012345678 9 123456789012345678901234567890121234567890123456789012345678 often yields a clue, even if it’s incorrect. Choice (C) has a hot dog at 86 9 123456789012345678901234567890121234567890123456789012345678 23456789012345678901234567890121234567890123456789012345678 9 1 cents. Add 6 or 7 cents sales tax to this, and then multiply by three. 9 123456789012345678901234567890121234567890123456789012345678 9 123456789012345678901234567890121234567890123456789012345678 You get: 9 123456789012345678901234567890121234567890123456789012345678 9 123456789012345678901234567890121234567890123456789012345678 23456789012345678901234567890121234567890123456789012345678 9 1 3(86 1 7) 5 3(93)5279 9 123456789012345678901234567890121234567890123456789012345678 9 123456789012345678901234567890121234567890123456789012345678 9 123456789012345678901234567890121234567890123456789012345678 9 123456789012345678901234567890121234567890123456789012345678 You can add the entire amount of sales tax per dollar, 7 cents, or you 23456789012345678901234567890121234567890123456789012345678 9 1 9 123456789012345678901234567890121234567890123456789012345678 can take into account the fact that 86 cents is not quite an entire 9 123456789012345678901234567890121234567890123456789012345678 9 123456789012345678901234567890121234567890123456789012345678 dollar, so Quentin might not owe the entire 7 cents sales tax. This 9 123456789012345678901234567890121234567890123456789012345678 might seem vague, but the question does allow for some wiggle room 23456789012345678901234567890121234567890123456789012345678 9 1 9 123456789012345678901234567890121234567890123456789012345678 since it asks “which is closest to the price . . .” Whether you use 6 or 9 123456789012345678901234567890121234567890123456789012345678 9 123456789012345678901234567890121234567890123456789012345678 7 cents sales tax, the same answer works best both ways. 9 123456789012345678901234567890121234567890123456789012345678 23456789012345678901234567890121234567890123456789012345678 123456789012345678901234567890121234567890123456789012345678 99 123456789012345678901234567890121234567890123456789012345678 9 123456789012345678901234567890121234567890123456789012345678 You know that Quentin spent $2.66, or 266 cents. Answer choice (C) 9 123456789012345678901234567890121234567890123456789012345678 9 123456789012345678901234567890121234567890123456789012345678 comes out at 279 cents, which is 13 cents too many. This means that (C) 123456789012345678901234567890121234567890123456789012345678 99 1 is incorrect because it’s too great. If (C) is too great, then (D) and (E) are 9 123456789012345678901234567890121234567890123456789012345678 9 123456789012345678901234567890121234567890123456789012345678 even greater, so they must also be incorrect. This leaves (A) and (B). Try 9 123456789012345678901234567890121234567890123456789012345678 23456789012345678901234567890121234567890123456789012345678 9 123456789012345678901234567890121234567890123456789012345678 (B). If it also too great, then the answer must be (A). Here’s (B). 9 123456789012345678901234567890121234567890123456789012345678 123456789012345678901234567890121234567890123456789012345678 99 123456789012345678901234567890121234567890123456789012345678 3(82 1 7) 5 3(89) 5 267 9 123456789012345678901234567890121234567890123456789012345678 9 123456789012345678901234567890121234567890123456789012345678 This is only one penny off. If you use a 6-cent sales tax, it is only 2 cents 9 123456789012345678901234567890121234567890123456789012345678 9 123456789012345678901234567890121234567890123456789012345678 off. (B) is closest to the price of the hot dog, so it’s the correct answer. 9 123456789012345678901234567890121234567890123456789012345678 9 1 23456789012345678901234567890121234567890123456789012345678 123456789012345678901234567890121234567890123456789012345678 99 123456789012345678901234567890121234567890123456789012345678 9 1 123456789012345678901234567890121234567890123456789012345678 9 9 123456789012345678901234567890121234567890123456789012345678 9 123456789012345678901234567890121234567890123456789012345678 9 123456789012345678901234567890121234567890123456789012345678 123456789012345678901234567890121234567890123456789012345678 9 9 123456789012345678901234567890121234567890123456789012345678 9 123456789012345678901234567890121234567890123456789012345678 23456789012345678901234567890121234567890123456789012345678 123456789012345678901234567890121234567890123456789012345678 99 123456789012345678901234567890121234567890123456789012345678 9 1 123456789012345678901234567890121234567890123456789012345678 9 123456789012345678901234567890121234567890123456789012345678 9 9 123456789012345678901234567890121234567890123456789012345678 9 123456789012345678901234567890121234567890123456789012345678 9 123456789012345678901234567890121234567890123456789012345678 9 123456789012345678901234567890121234567890123456789012345678 123456789012345678901234567890121234567890123456789012345678 9 23456789012345678901234567890121234567890123456789012345678 123456789012345678901234567890121234567890123456789012345678 99 123456789012345678901234567890121234567890123456789012345678 9 123456789012345678901234567890121234567890123456789012345678 9 123456789012345678901234567890121234567890123456789012345678 9 1 9 123456789012345678901234567890121234567890123456789012345678 9 123456789012345678901234567890121234567890123456789012345678 9 123456789012345678901234567890121234567890123456789012345678 9 123456789012345678901234567890121234567890123456789012345678 123456789012345678901234567890121234567890123456789012345678 9 23456789012345678901234567890121234567890123456789012345678 123456789012345678901234567890121234567890123456789012345678 99 1 9 123456789012345678901234567890121234567890123456789012345678 123456789012345678901234567890121234567890123456789012345678 9 9 123456789012345678901234567890121234567890123456789012345678 123456789012345678901234567890121234567890123456789012345678 9 123456789012345678901234567890121234567890123456789012345678 9 123456789012345678901234567890121234567890123456789012345678 9 123456789012345678901234567890121234567890123456789012345678 9 1234567890123456789012345678901212345678901234567890123456789 Copyright © 2005 Thomson Peterson’s, a part of The Thomson Corporaton SAT is a registered trademark of the College Entrance Examination Board, which was not involved in the production of and does not endorse this product.

71

1234567890123456789012345678901212345678901234567890123456789 123456789012345678901234567890121234567890123456789012345678 9 9 123456789012345678901234567890121234567890123456789012345678 123456789012345678901234567890121234567890123456789012345678 9 9 123456789012345678901234567890121234567890123456789012345678 13. D Since the triangles are congruent, you know that the bottom right angle 9 123456789012345678901234567890121234567890123456789012345678 9 123456789012345678901234567890121234567890123456789012345678 on the left triangle is a right angle. You also know that the side opposite 9 123456789012345678901234567890121234567890123456789012345678 23456789012345678901234567890121234567890123456789012345678 9 1 x=3 9 123456789012345678901234567890121234567890123456789012345678 9 123456789012345678901234567890121234567890123456789012345678 of b is , and from this you can deduce that the side adjacent to b 9 123456789012345678901234567890121234567890123456789012345678 2 9 123456789012345678901234567890121234567890123456789012345678 x 23456789012345678901234567890121234567890123456789012345678 9 1 is by using the Pythagorean theorem. Here’s what the computation 9 123456789012345678901234567890121234567890123456789012345678 2 9 123456789012345678901234567890121234567890123456789012345678 9 123456789012345678901234567890121234567890123456789012345678 would look like: 9 123456789012345678901234567890121234567890123456789012345678 23456789012345678901234567890121234567890123456789012345678 9 1 2 2 2 9 123456789012345678901234567890121234567890123456789012345678 1 b 5 c a 9 123456789012345678901234567890121234567890123456789012345678 2 9 123456789012345678901234567890121234567890123456789012345678 9 123456789012345678901234567890121234567890123456789012345678 3x = 2 2 23456789012345678901234567890121234567890123456789012345678 9 1 1b 5x 9 123456789012345678901234567890121234567890123456789012345678 2 9 123456789012345678901234567890121234567890123456789012345678 9 123456789012345678901234567890121234567890123456789012345678 3 2 2 2 9 123456789012345678901234567890121234567890123456789012345678 1 b 5 x x 23456789012345678901234567890121234567890123456789012345678 9 123456789012345678901234567890121234567890123456789012345678 4 9 123456789012345678901234567890121234567890123456789012345678 9 123456789012345678901234567890121234567890123456789012345678 3 2 3 2 3 2 9 123456789012345678901234567890121234567890123456789012345678 2 2 x 2 x 1b 5x 2 x 9 123456789012345678901234567890121234567890123456789012345678 4 4 4 123456789012345678901234567890121234567890123456789012345678 9 9 1 9 123456789012345678901234567890121234567890123456789012345678 x2 2 9 123456789012345678901234567890121234567890123456789012345678 b 5 9 123456789012345678901234567890121234567890123456789012345678 4 23456789012345678901234567890121234567890123456789012345678 123456789012345678901234567890121234567890123456789012345678 9 9 123456789012345678901234567890121234567890123456789012345678 x 9 123456789012345678901234567890121234567890123456789012345678 b 5 9 123456789012345678901234567890121234567890123456789012345678 2 123456789012345678901234567890121234567890123456789012345678 9 123456789012345678901234567890121234567890123456789012345678 9 9 123456789012345678901234567890121234567890123456789012345678 The sides of this right triangle, then, are in the proportions of a 9 123456789012345678901234567890121234567890123456789012345678 9 123456789012345678901234567890121234567890123456789012345678 30-60-90 triangle. And since angle b is opposite the second largest side, 9 1 23456789012345678901234567890121234567890123456789012345678 9 123456789012345678901234567890121234567890123456789012345678 its measure is 60°, choice (D). 9 123456789012345678901234567890121234567890123456789012345678 9 1 9 123456789012345678901234567890121234567890123456789012345678 14. C You can infer that the figure is a square since all the sides must be 9 123456789012345678901234567890121234567890123456789012345678 23456789012345678901234567890121234567890123456789012345678 9 123456789012345678901234567890121234567890123456789012345678 congruent because they are all formed by points equidistant from zero. 9 123456789012345678901234567890121234567890123456789012345678 9 1 To find the area of the square, you only need the length of one side. You 9 123456789012345678901234567890121234567890123456789012345678 can get this two ways: you can use the distance formula for two points 9 123456789012345678901234567890121234567890123456789012345678 23456789012345678901234567890121234567890123456789012345678 9 123456789012345678901234567890121234567890123456789012345678 or you can take a shortcut and realize that the square is cut into four 9 123456789012345678901234567890121234567890123456789012345678 9 1 45-45-90 triangles by the y- and x-axes. The length of each non9 123456789012345678901234567890121234567890123456789012345678 9 123456789012345678901234567890121234567890123456789012345678 hypotenuse side of these triangles is 2, so the hypotenuse must be 23456789012345678901234567890121234567890123456789012345678 123456789012345678901234567890121234567890123456789012345678 9 123456789012345678901234567890121234567890123456789012345678 9 9 123456789012345678901234567890121234567890123456789012345678 2=2. This hypotenuse is also the length of the square’s side, so place 9 123456789012345678901234567890121234567890123456789012345678 9 1 this value into the area of a square formula: 23456789012345678901234567890121234567890123456789012345678 123456789012345678901234567890121234567890123456789012345678 9 9 123456789012345678901234567890121234567890123456789012345678 9 123456789012345678901234567890121234567890123456789012345678 A 5 s2 9 123456789012345678901234567890121234567890123456789012345678 2 9 1 ~ ! A 5 2 2 = 23456789012345678901234567890121234567890123456789012345678 123456789012345678901234567890121234567890123456789012345678 9 123456789012345678901234567890121234567890123456789012345678 9 9 123456789012345678901234567890121234567890123456789012345678 A543258 9 123456789012345678901234567890121234567890123456789012345678 9 1 23456789012345678901234567890121234567890123456789012345678 Choice (C) is the answer. 123456789012345678901234567890121234567890123456789012345678 9 9 1 9 123456789012345678901234567890121234567890123456789012345678 123456789012345678901234567890121234567890123456789012345678 9 9 123456789012345678901234567890121234567890123456789012345678 123456789012345678901234567890121234567890123456789012345678 9 123456789012345678901234567890121234567890123456789012345678 9 123456789012345678901234567890121234567890123456789012345678 9 123456789012345678901234567890121234567890123456789012345678 9 1234567890123456789012345678901212345678901234567890123456789

S D

72

Copyright © 2005 Thomson Peterson’s, a part of The Thomson Corporaton SAT is a registered trademark of the College Entrance Examination Board, which was not involved in the production of and does not endorse this product.

1234567890123456789012345678901212345678901234567890123456789 123456789012345678901234567890121234567890123456789012345678 9 9 123456789012345678901234567890121234567890123456789012345678 123456789012345678901234567890121234567890123456789012345678 9 9 123456789012345678901234567890121234567890123456789012345678 15. E The smallest percentage difference is going to be the year where the flu 9 123456789012345678901234567890121234567890123456789012345678 9 123456789012345678901234567890121234567890123456789012345678 column and the cold column are closest. (The converse is not true for 9 123456789012345678901234567890121234567890123456789012345678 23456789012345678901234567890121234567890123456789012345678 9 1 the greatest percentage difference.) Eyeing the chart is the fastest way to 9 123456789012345678901234567890121234567890123456789012345678 9 123456789012345678901234567890121234567890123456789012345678 determine the answer. The values in the columns for 2000 look and are 9 123456789012345678901234567890121234567890123456789012345678 9 123456789012345678901234567890121234567890123456789012345678 the closest, and so the percentage difference is the least. (E) is 23456789012345678901234567890121234567890123456789012345678 9 1 9 123456789012345678901234567890121234567890123456789012345678 the answer. 9 123456789012345678901234567890121234567890123456789012345678 9 123456789012345678901234567890121234567890123456789012345678 9 123456789012345678901234567890121234567890123456789012345678 16. D Cold cases (white bar) have to be greater than flu cases (shaded bar), so 23456789012345678901234567890121234567890123456789012345678 9 1 9 123456789012345678901234567890121234567890123456789012345678 there are only two decades that could be the correct answer, 1970 and 9 123456789012345678901234567890121234567890123456789012345678 9 1990, choices (B) and (D). In 1970, the number of cold cases is about 123456789012345678901234567890121234567890123456789012345678 9 123456789012345678901234567890121234567890123456789012345678 twice as many as the flu (2,000 to 1,000), so the percentage difference 23456789012345678901234567890121234567890123456789012345678 9 1 9 123456789012345678901234567890121234567890123456789012345678 would be 100%. This makes 1990, choice (D), the answer by default, 9 123456789012345678901234567890121234567890123456789012345678 9 123456789012345678901234567890121234567890123456789012345678 but you can see that the percentage difference works out to about 25% 9 123456789012345678901234567890121234567890123456789012345678 23456789012345678901234567890121234567890123456789012345678 9 123456789012345678901234567890121234567890123456789012345678 since there were roughly 4,000 cold cases and 3,000 flu cases. 9 123456789012345678901234567890121234567890123456789012345678 123456789012345678901234567890121234567890123456789012345678 99 123456789012345678901234567890121234567890123456789012345678 17. A You may want to save Roman numeral questions for the second pass! A 9 123456789012345678901234567890121234567890123456789012345678 9 123456789012345678901234567890121234567890123456789012345678 judicious use of POE will help you out on this one, though. Looking at 9 1 9 123456789012345678901234567890121234567890123456789012345678 I, angles 6 and 12 are both formed by the same intersecting line, and 9 123456789012345678901234567890121234567890123456789012345678 9 123456789012345678901234567890121234567890123456789012345678 they are congruent. Since I must be true, you can take this information 23456789012345678901234567890121234567890123456789012345678 123456789012345678901234567890121234567890123456789012345678 99 123456789012345678901234567890121234567890123456789012345678 and cross out any answer choices that do not have I in them. 9 123456789012345678901234567890121234567890123456789012345678 9 123456789012345678901234567890121234567890123456789012345678 Surprisingly, this gets rid (B), (C), and (D). The answer is either (A) (I 123456789012345678901234567890121234567890123456789012345678 99 123456789012345678901234567890121234567890123456789012345678 only) or (E) (I and II only). Since these are the only choices, you do not 9 123456789012345678901234567890121234567890123456789012345678 9 123456789012345678901234567890121234567890123456789012345678 even have to worry about III. Check II, and if it must be true, then the 123456789012345678901234567890121234567890123456789012345678 99 1 answer is (E). If it isn’t true, the answer must be (A). As it turns out, 2 9 123456789012345678901234567890121234567890123456789012345678 9 and 9 are not congruent, so the answer is choice (A). 123456789012345678901234567890121234567890123456789012345678 123456789012345678901234567890121234567890123456789012345678 9 9 123456789012345678901234567890121234567890123456789012345678 18. A This problem looks rather complicated, but the negative in front of the 9 123456789012345678901234567890121234567890123456789012345678 23456789012345678901234567890121234567890123456789012345678 9 123456789012345678901234567890121234567890123456789012345678 a simplifies it. If 2a2b is positive, then a must be negative and raised 9 123456789012345678901234567890121234567890123456789012345678 9 1 to an odd power. To get your head around this, consider that if a were 9 123456789012345678901234567890121234567890123456789012345678 9 123456789012345678901234567890121234567890123456789012345678 positive, the entire expression could not be positive. So (A) must 23456789012345678901234567890121234567890123456789012345678 123456789012345678901234567890121234567890123456789012345678 99 123456789012345678901234567890121234567890123456789012345678 be true. 9 1 123456789012345678901234567890121234567890123456789012345678 9 9 123456789012345678901234567890121234567890123456789012345678 19. D The first thing to recognize here is that the figure is composed of three 9 123456789012345678901234567890121234567890123456789012345678 9 123456789012345678901234567890121234567890123456789012345678 triangles, and since all the line segments are congruent, all the triangles 9 123456789012345678901234567890121234567890123456789012345678 9 123456789012345678901234567890121234567890123456789012345678 are equilateral and congruent to each other. Equilateral triangles are all 123456789012345678901234567890121234567890123456789012345678 9 23456789012345678901234567890121234567890123456789012345678 9 123456789012345678901234567890121234567890123456789012345678 60-60-60 triangles, and so both a and b are 60. Choice (D) is true. 9 123456789012345678901234567890121234567890123456789012345678 9 123456789012345678901234567890121234567890123456789012345678 20. C Since the boys cannot sit by seat 3, both girls always have to be sitting in 9 123456789012345678901234567890121234567890123456789012345678 9 1 seat 2 and 4. The boys have to be in seats 1 and 5. 23456789012345678901234567890121234567890123456789012345678 123456789012345678901234567890121234567890123456789012345678 99 123456789012345678901234567890121234567890123456789012345678 9 123456789012345678901234567890121234567890123456789012345678 If the first girl is in seat 2, then the second is in seat 4. If the first boy is 9 123456789012345678901234567890121234567890123456789012345678 9 1 in seat 1 the second boy is in seat 5. That is one arrangement. The boys 23456789012345678901234567890121234567890123456789012345678 123456789012345678901234567890121234567890123456789012345678 99 1 could switch seats, and that is a second arrangement. The girls could 9 123456789012345678901234567890121234567890123456789012345678 9 123456789012345678901234567890121234567890123456789012345678 also switch seats, and this would yield another two possible arrange9 123456789012345678901234567890121234567890123456789012345678 23456789012345678901234567890121234567890123456789012345678 9 1 ments. That gives four total, (C). 9 123456789012345678901234567890121234567890123456789012345678 123456789012345678901234567890121234567890123456789012345678 99 123456789012345678901234567890121234567890123456789012345678 1234567890123456789012345678901212345678901234567890123456789 2c

Copyright © 2005 Thomson Peterson’s, a part of The Thomson Corporaton SAT is a registered trademark of the College Entrance Examination Board, which was not involved in the production of and does not endorse this product.

73

1234567890123456789012345678901212345678901234567890123456789 123456789012345678901234567890121234567890123456789012345678 9 9 123456789012345678901234567890121234567890123456789012345678 123456789012345678901234567890121234567890123456789012345678 9 9 123456789012345678901234567890121234567890123456789012345678 21. E There is a little formula that can be used, and you can draw up a Venn 9 123456789012345678901234567890121234567890123456789012345678 9 123456789012345678901234567890121234567890123456789012345678 diagram if you like, but if you keep your head you can reason your way 9 123456789012345678901234567890121234567890123456789012345678 23456789012345678901234567890121234567890123456789012345678 9 1 through this problem. If 52 students are in neither class, then 98 9 123456789012345678901234567890121234567890123456789012345678 9 123456789012345678901234567890121234567890123456789012345678 students are in algebra or geometry. The question states that “73 are in 9 123456789012345678901234567890121234567890123456789012345678 9 123456789012345678901234567890121234567890123456789012345678 geometry and 62 are in algebra,” so that would be a total of 73 1 62 5 23456789012345678901234567890121234567890123456789012345678 9 1 9 123456789012345678901234567890121234567890123456789012345678 135 students. Since this number is greater than the number (98) of 9 123456789012345678901234567890121234567890123456789012345678 9 123456789012345678901234567890121234567890123456789012345678 students you found earlier, the difference between 135 and 98 must be 9 123456789012345678901234567890121234567890123456789012345678 23456789012345678901234567890121234567890123456789012345678 9 1 the number of students enrolled in both algebra and geometry. This 9 123456789012345678901234567890121234567890123456789012345678 9 123456789012345678901234567890121234567890123456789012345678 difference is 37 (135 2 98 5 37), choice (E). 9 123456789012345678901234567890121234567890123456789012345678 9 123456789012345678901234567890121234567890123456789012345678 23456789012345678901234567890121234567890123456789012345678 9 1 9 123456789012345678901234567890121234567890123456789012345678 Section 6 9 123456789012345678901234567890121234567890123456789012345678 9 123456789012345678901234567890121234567890123456789012345678 1. B In reference to the discovery of DNA, the passage states, the study of 9 123456789012345678901234567890121234567890123456789012345678 23456789012345678901234567890121234567890123456789012345678 9 123456789012345678901234567890121234567890123456789012345678 life could now be performed with more abstract methods of analysis. 9 123456789012345678901234567890121234567890123456789012345678 9 123456789012345678901234567890121234567890123456789012345678 (B) makes the same point, using the same key word, while all the other 9 123456789012345678901234567890121234567890123456789012345678 9 123456789012345678901234567890121234567890123456789012345678 choices either go beyond what the passage actually states (e.g. (A), the 123456789012345678901234567890121234567890123456789012345678 9 9 1 passage does not say that the study of lipids and proteins became 9 123456789012345678901234567890121234567890123456789012345678 9 123456789012345678901234567890121234567890123456789012345678 irrelevant; (C) basically says the same thing) or bring in topics not 9 123456789012345678901234567890121234567890123456789012345678 23456789012345678901234567890121234567890123456789012345678 9 123456789012345678901234567890121234567890123456789012345678 mentioned in the passage (e.g. (D), Mendelian genetics is not mentioned 9 123456789012345678901234567890121234567890123456789012345678 9 123456789012345678901234567890121234567890123456789012345678 in the passage). 123456789012345678901234567890121234567890123456789012345678 9 123456789012345678901234567890121234567890123456789012345678 9 9 123456789012345678901234567890121234567890123456789012345678 2. E The passage contrasts “wet and dirty“ study of lipids and proteins with 9 123456789012345678901234567890121234567890123456789012345678 9 123456789012345678901234567890121234567890123456789012345678 the information-based study of DNA. In this sense “wet and dirty“ 123456789012345678901234567890121234567890123456789012345678 9 9 1 involves intensive laboratory work with things like lipids and proteins. 9 123456789012345678901234567890121234567890123456789012345678 9 123456789012345678901234567890121234567890123456789012345678 It is not haphazard guessing (A). Choice (E) states this correctly, also 123456789012345678901234567890121234567890123456789012345678 9 9 123456789012345678901234567890121234567890123456789012345678 drawing on language in the passage (information-based). 9 123456789012345678901234567890121234567890123456789012345678 23456789012345678901234567890121234567890123456789012345678 9 123456789012345678901234567890121234567890123456789012345678 3. D The passage says that Saunders was a writer on Western topics who was 9 123456789012345678901234567890121234567890123456789012345678 9 1 widely read in the past. (D) fits with this. (A) does not. The passage does 9 123456789012345678901234567890121234567890123456789012345678 9 123456789012345678901234567890121234567890123456789012345678 not say anything like (B), (C), or (E). Thus (D) is the best choice. 23456789012345678901234567890121234567890123456789012345678 123456789012345678901234567890121234567890123456789012345678 9 123456789012345678901234567890121234567890123456789012345678 9 9 1 4. A The passage reads, many of the groups who popularized rock and roll 9 123456789012345678901234567890121234567890123456789012345678 9 123456789012345678901234567890121234567890123456789012345678 consciously were attempting to emulate the work of blues greats such as 23456789012345678901234567890121234567890123456789012345678 123456789012345678901234567890121234567890123456789012345678 9 9 123456789012345678901234567890121234567890123456789012345678 B. B. King. Choice (A) is an accurate paraphrase of the information 9 123456789012345678901234567890121234567890123456789012345678 9 123456789012345678901234567890121234567890123456789012345678 given in the passage. 9 1 23456789012345678901234567890121234567890123456789012345678 123456789012345678901234567890121234567890123456789012345678 9 9 123456789012345678901234567890121234567890123456789012345678 5. C Antecedent means coming before. Even if you don’t know the word, 123456789012345678901234567890121234567890123456789012345678 9 9 123456789012345678901234567890121234567890123456789012345678 you can use word analysis to figure out that it has something to do with 9 1 before. Don’t confuse antewith anti(against). You can eliminate (D) 23456789012345678901234567890121234567890123456789012345678 123456789012345678901234567890121234567890123456789012345678 9 9 123456789012345678901234567890121234567890123456789012345678 and (E). Referent is a synonym for antecedent only in grammatical 9 123456789012345678901234567890121234567890123456789012345678 9 123456789012345678901234567890121234567890123456789012345678 usage. Eliminate (B). You are left with antebellum, meaning before the 9 1 23456789012345678901234567890121234567890123456789012345678 9 123456789012345678901234567890121234567890123456789012345678 war, or causal, meaning causing. Causal is more specific and more 9 1 9 123456789012345678901234567890121234567890123456789012345678 logical. (C) is the answer. 123456789012345678901234567890121234567890123456789012345678 9 9 123456789012345678901234567890121234567890123456789012345678 123456789012345678901234567890121234567890123456789012345678 9 123456789012345678901234567890121234567890123456789012345678 9 123456789012345678901234567890121234567890123456789012345678 9 123456789012345678901234567890121234567890123456789012345678 9 1234567890123456789012345678901212345678901234567890123456789 74

Copyright © 2005 Thomson Peterson’s, a part of The Thomson Corporaton SAT is a registered trademark of the College Entrance Examination Board, which was not involved in the production of and does not endorse this product.

1234567890123456789012345678901212345678901234567890123456789 123456789012345678901234567890121234567890123456789012345678 9 9 123456789012345678901234567890121234567890123456789012345678 123456789012345678901234567890121234567890123456789012345678 9 9 123456789012345678901234567890121234567890123456789012345678 6. C The passage begins by describing the Seneca Falls convention as the first 9 123456789012345678901234567890121234567890123456789012345678 9 123456789012345678901234567890121234567890123456789012345678 organized attempt for woman’s voting rights (read the first sentences of 9 123456789012345678901234567890121234567890123456789012345678 23456789012345678901234567890121234567890123456789012345678 9 1 the passage to see this). AWSA and NWSA came after Seneca Falls. (D) 9 123456789012345678901234567890121234567890123456789012345678 9 123456789012345678901234567890121234567890123456789012345678 and (E) refer to information provided in Passage 2. Choice (C) is 9 123456789012345678901234567890121234567890123456789012345678 9 123456789012345678901234567890121234567890123456789012345678 the answer. 23456789012345678901234567890121234567890123456789012345678 9 1 9 123456789012345678901234567890121234567890123456789012345678 9 123456789012345678901234567890121234567890123456789012345678 7. A The passage states that the Seneca Falls conference did not have an 9 123456789012345678901234567890121234567890123456789012345678 9 123456789012345678901234567890121234567890123456789012345678 immediate effect because the nation became embroiled in issues related 23456789012345678901234567890121234567890123456789012345678 9 1 9 123456789012345678901234567890121234567890123456789012345678 to the coming Civil War. Knowledge of the dates of the Civil War will 9 123456789012345678901234567890121234567890123456789012345678 9 help you avoid confusing it with World War I. It was the Civil War, (A), 123456789012345678901234567890121234567890123456789012345678 9 123456789012345678901234567890121234567890123456789012345678 that pushed the woman’s voting rights movement to the background of 23456789012345678901234567890121234567890123456789012345678 9 1 9 123456789012345678901234567890121234567890123456789012345678 the national consciousness. 9 123456789012345678901234567890121234567890123456789012345678 9 123456789012345678901234567890121234567890123456789012345678 9 123456789012345678901234567890121234567890123456789012345678 8. B Here it is best to go back to the passage and clarify in your own mind 23456789012345678901234567890121234567890123456789012345678 123456789012345678901234567890121234567890123456789012345678 99 123456789012345678901234567890121234567890123456789012345678 the distinctions drawn between the NWSA and the AWSA. The NWSA, 9 123456789012345678901234567890121234567890123456789012345678 9 123456789012345678901234567890121234567890123456789012345678 which the question asks about, focused their efforts on federal and 123456789012345678901234567890121234567890123456789012345678 99 123456789012345678901234567890121234567890123456789012345678 constitutional issues, whereas the AWSA focused on state-level issues. 9 1 9 123456789012345678901234567890121234567890123456789012345678 So (B) is the correct answer. 9 123456789012345678901234567890121234567890123456789012345678 9 123456789012345678901234567890121234567890123456789012345678 23456789012345678901234567890121234567890123456789012345678 9 123456789012345678901234567890121234567890123456789012345678 9. B Don’t be distracted by the wrong answers. Without even reading the 9 123456789012345678901234567890121234567890123456789012345678 9 123456789012345678901234567890121234567890123456789012345678 passage, you could guess that the conflict between two organizations 123456789012345678901234567890121234567890123456789012345678 99 123456789012345678901234567890121234567890123456789012345678 was resolved when they combined. The answer is (B). 9 123456789012345678901234567890121234567890123456789012345678 9 123456789012345678901234567890121234567890123456789012345678 10. D The author of Passage 1 doesn’t have a very strong opinion, so you can 9 123456789012345678901234567890121234567890123456789012345678 9 123456789012345678901234567890121234567890123456789012345678 eliminate (A) and (E). Now you need to decide if the author’s opinion, 9 1 23456789012345678901234567890121234567890123456789012345678 9 123456789012345678901234567890121234567890123456789012345678 however subtle, is positive or negative. It seems positive. For example, 9 123456789012345678901234567890121234567890123456789012345678 9 1 the author describes the work of the NAWSA as important. Thus the 123456789012345678901234567890121234567890123456789012345678 9 9 123456789012345678901234567890121234567890123456789012345678 correct answer will be positive. That eliminates (B) and (C). The answer 9 123456789012345678901234567890121234567890123456789012345678 9 123456789012345678901234567890121234567890123456789012345678 is (D). 123456789012345678901234567890121234567890123456789012345678 9 9 123456789012345678901234567890121234567890123456789012345678 9 123456789012345678901234567890121234567890123456789012345678 11. A The first question is to determine what the “ultimate victory of the 23456789012345678901234567890121234567890123456789012345678 123456789012345678901234567890121234567890123456789012345678 99 123456789012345678901234567890121234567890123456789012345678 woman’s suffrage movement“ is. The first paragraph of the second 9 1 9 123456789012345678901234567890121234567890123456789012345678 passage makes it clear the author views the passing of the amendment 123456789012345678901234567890121234567890123456789012345678 9 23456789012345678901234567890121234567890123456789012345678 9 123456789012345678901234567890121234567890123456789012345678 to the constitution as the “ultimate victory,” and this occurred with 9 123456789012345678901234567890121234567890123456789012345678 9 123456789012345678901234567890121234567890123456789012345678 Tennessee approving the amendment. So (A) is the answer. 123456789012345678901234567890121234567890123456789012345678 99 1 12. C The earliest time that the second passage points to is the 1870s (the first 9 123456789012345678901234567890121234567890123456789012345678 9 123456789012345678901234567890121234567890123456789012345678 passage refers to the Seneca Falls convention in 1848), and so (C) is 9 123456789012345678901234567890121234567890123456789012345678 9 123456789012345678901234567890121234567890123456789012345678 the answer. 123456789012345678901234567890121234567890123456789012345678 9 23456789012345678901234567890121234567890123456789012345678 123456789012345678901234567890121234567890123456789012345678 99 123456789012345678901234567890121234567890123456789012345678 13. C The second passage describes “partial suffrage“ as the right to, “vote in 9 123456789012345678901234567890121234567890123456789012345678 9 123456789012345678901234567890121234567890123456789012345678 local affairs such as municipal elections, school board elections, or 9 1 23456789012345678901234567890121234567890123456789012345678 9 123456789012345678901234567890121234567890123456789012345678 prohibition measures.” All of the examples but (C) refer to local affairs. 9 1 9 123456789012345678901234567890121234567890123456789012345678 Choice (C) refers to something mentioned only in Passage 1 (which was, 123456789012345678901234567890121234567890123456789012345678 9 9 123456789012345678901234567890121234567890123456789012345678 moreover, before the 1848 Seneca Falls convention). 9 123456789012345678901234567890121234567890123456789012345678 123456789012345678901234567890121234567890123456789012345678 9 123456789012345678901234567890121234567890123456789012345678 9 123456789012345678901234567890121234567890123456789012345678 9 1234567890123456789012345678901212345678901234567890123456789 Copyright © 2005 Thomson Peterson’s, a part of The Thomson Corporaton SAT is a registered trademark of the College Entrance Examination Board, which was not involved in the production of and does not endorse this product.

75

1234567890123456789012345678901212345678901234567890123456789 123456789012345678901234567890121234567890123456789012345678 9 9 123456789012345678901234567890121234567890123456789012345678 123456789012345678901234567890121234567890123456789012345678 9 9 123456789012345678901234567890121234567890123456789012345678 14. B The second passage argues that the “partial suffrages“ showed that 9 123456789012345678901234567890121234567890123456789012345678 9 123456789012345678901234567890121234567890123456789012345678 woman could “responsibly and reasonably participate in a representa9 123456789012345678901234567890121234567890123456789012345678 23456789012345678901234567890121234567890123456789012345678 9 1 tive democracy.” These examples made the reasoning of nonsuffragists, 9 123456789012345678901234567890121234567890123456789012345678 9 123456789012345678901234567890121234567890123456789012345678 that woman were not fit to vote, difficult to maintain. Choice (B) 9 123456789012345678901234567890121234567890123456789012345678 9 123456789012345678901234567890121234567890123456789012345678 correctly points to this same idea. 23456789012345678901234567890121234567890123456789012345678 9 1 9 123456789012345678901234567890121234567890123456789012345678 9 123456789012345678901234567890121234567890123456789012345678 15. E To answer a question like this you have to clearly have in mind which 9 123456789012345678901234567890121234567890123456789012345678 9 123456789012345678901234567890121234567890123456789012345678 passage discusses what. Neither passage mentions the number of the 23456789012345678901234567890121234567890123456789012345678 9 1 9 123456789012345678901234567890121234567890123456789012345678 amendment that gave women the right to vote. The second passage 9 123456789012345678901234567890121234567890123456789012345678 9 mentions when the amendment became law in the first paragraph, so 123456789012345678901234567890121234567890123456789012345678 9 123456789012345678901234567890121234567890123456789012345678 you can eliminate (A). The first passage mentions the Civil War in the 23456789012345678901234567890121234567890123456789012345678 9 1 9 123456789012345678901234567890121234567890123456789012345678 first paragraph. The first passage mentions the leaders of NWSA in the 9 123456789012345678901234567890121234567890123456789012345678 9 123456789012345678901234567890121234567890123456789012345678 second paragraph. The second passage is all about “partial suffrages,” 9 123456789012345678901234567890121234567890123456789012345678 23456789012345678901234567890121234567890123456789012345678 9 123456789012345678901234567890121234567890123456789012345678 and you’ve confirmed this in question 14. The only possible answer 9 123456789012345678901234567890121234567890123456789012345678 9 123456789012345678901234567890121234567890123456789012345678 is (E). 123456789012345678901234567890121234567890123456789012345678 9 123456789012345678901234567890121234567890123456789012345678 9 9 123456789012345678901234567890121234567890123456789012345678 9 1 16. A Recall that the author of the second passage saw the “partial 9 123456789012345678901234567890121234567890123456789012345678 suffrages,” the local successes of the woman’s movement in the 1870s 9 123456789012345678901234567890121234567890123456789012345678 9 123456789012345678901234567890121234567890123456789012345678 and 1880s, as crucial to the ultimate success of the movement. The first 23456789012345678901234567890121234567890123456789012345678 123456789012345678901234567890121234567890123456789012345678 9 9 123456789012345678901234567890121234567890123456789012345678 passage says that AWSA focused on state-level issues. Choice (A) states 9 123456789012345678901234567890121234567890123456789012345678 9 123456789012345678901234567890121234567890123456789012345678 as much and is the best choice. NWSA was specifically interested in 123456789012345678901234567890121234567890123456789012345678 9 9 123456789012345678901234567890121234567890123456789012345678 federal and constitutional issues, so (B) is factually incorrect. The 9 123456789012345678901234567890121234567890123456789012345678 9 123456789012345678901234567890121234567890123456789012345678 author of passage two agrees that the constitutional amendment was 123456789012345678901234567890121234567890123456789012345678 9 9 1 the ultimate success of the suffrage movement, so he or she would not 9 123456789012345678901234567890121234567890123456789012345678 9 agree with (C). (D) also contradicts that view. (E) is more difficult to 123456789012345678901234567890121234567890123456789012345678 9 123456789012345678901234567890121234567890123456789012345678 eliminate, but the author of Passage 2 doesn’t mention any differences 9 123456789012345678901234567890121234567890123456789012345678 9 123456789012345678901234567890121234567890123456789012345678 within the suffrage movement. Choice (A) is the best answer. 23456789012345678901234567890121234567890123456789012345678 123456789012345678901234567890121234567890123456789012345678 9 123456789012345678901234567890121234567890123456789012345678 9 9 1 9 123456789012345678901234567890121234567890123456789012345678 9 123456789012345678901234567890121234567890123456789012345678 23456789012345678901234567890121234567890123456789012345678 123456789012345678901234567890121234567890123456789012345678 9 123456789012345678901234567890121234567890123456789012345678 9 9 1 123456789012345678901234567890121234567890123456789012345678 9 123456789012345678901234567890121234567890123456789012345678 9 9 123456789012345678901234567890121234567890123456789012345678 9 123456789012345678901234567890121234567890123456789012345678 9 123456789012345678901234567890121234567890123456789012345678 9 123456789012345678901234567890121234567890123456789012345678 123456789012345678901234567890121234567890123456789012345678 9 23456789012345678901234567890121234567890123456789012345678 123456789012345678901234567890121234567890123456789012345678 9 123456789012345678901234567890121234567890123456789012345678 9 123456789012345678901234567890121234567890123456789012345678 9 123456789012345678901234567890121234567890123456789012345678 9 9 1 9 123456789012345678901234567890121234567890123456789012345678 9 123456789012345678901234567890121234567890123456789012345678 9 123456789012345678901234567890121234567890123456789012345678 9 123456789012345678901234567890121234567890123456789012345678 123456789012345678901234567890121234567890123456789012345678 9 23456789012345678901234567890121234567890123456789012345678 123456789012345678901234567890121234567890123456789012345678 9 9 1 9 123456789012345678901234567890121234567890123456789012345678 123456789012345678901234567890121234567890123456789012345678 9 9 123456789012345678901234567890121234567890123456789012345678 123456789012345678901234567890121234567890123456789012345678 9 123456789012345678901234567890121234567890123456789012345678 9 123456789012345678901234567890121234567890123456789012345678 9 123456789012345678901234567890121234567890123456789012345678 9 1234567890123456789012345678901212345678901234567890123456789 76

Copyright © 2005 Thomson Peterson’s, a part of The Thomson Corporaton SAT is a registered trademark of the College Entrance Examination Board, which was not involved in the production of and does not endorse this product.

1234567890123456789012345678901212345678901234567890123456789 123456789012345678901234567890121234567890123456789012345678 9 9 123456789012345678901234567890121234567890123456789012345678 123456789012345678901234567890121234567890123456789012345678 9 9 123456789012345678901234567890121234567890123456789012345678 9 123456789012345678901234567890121234567890123456789012345678 Section 7 123456789012345678901234567890121234567890123456789012345678 9 9 1. This one is all about PEMDAS. 123456789012345678901234567890121234567890123456789012345678 123456789012345678901234567890121234567890123456789012345678 9 9 123456789012345678901234567890121234567890123456789012345678 9 123456789012345678901234567890121234567890123456789012345678 2~32!2 1 ~4 2 3~4!!2# 5 2~9!2 1 ~4 2 12!2 @ 9 123456789012345678901234567890121234567890123456789012345678 2 9 123456789012345678901234567890121234567890123456789012345678 5 2 81 1 28 162 1 64 ~ ! ~ ! @ # 23456789012345678901234567890121234567890123456789012345678 9 1 9 123456789012345678901234567890121234567890123456789012345678 5 162 1 64 9 123456789012345678901234567890121234567890123456789012345678 9 123456789012345678901234567890121234567890123456789012345678 9 123456789012345678901234567890121234567890123456789012345678 5 226 23456789012345678901234567890121234567890123456789012345678 9 1 9 123456789012345678901234567890121234567890123456789012345678 2. Remember when you multiply or divide an inequality by a negative, you 9 123456789012345678901234567890121234567890123456789012345678 9 123456789012345678901234567890121234567890123456789012345678 switch the direction of the inequality sign: 9 123456789012345678901234567890121234567890123456789012345678 23456789012345678901234567890121234567890123456789012345678 9 1 9 123456789012345678901234567890121234567890123456789012345678 22 2 3x . 13 9 123456789012345678901234567890121234567890123456789012345678 9 123456789012345678901234567890121234567890123456789012345678 9 123456789012345678901234567890121234567890123456789012345678 22 2 22 2 3x . 13 2 22 23456789012345678901234567890121234567890123456789012345678 123456789012345678901234567890121234567890123456789012345678 99 123456789012345678901234567890121234567890123456789012345678 23x . 29 9 123456789012345678901234567890121234567890123456789012345678 9 123456789012345678901234567890121234567890123456789012345678 23x 29 9 123456789012345678901234567890121234567890123456789012345678 , 9 123456789012345678901234567890121234567890123456789012345678 23 23 9 1 9 123456789012345678901234567890121234567890123456789012345678 9 123456789012345678901234567890121234567890123456789012345678 x,3 9 123456789012345678901234567890121234567890123456789012345678 23456789012345678901234567890121234567890123456789012345678 9 123456789012345678901234567890121234567890123456789012345678 The only two positive integers less than 3 are 2 or 1. Either one is an 9 123456789012345678901234567890121234567890123456789012345678 9 123456789012345678901234567890121234567890123456789012345678 acceptable answer. 9 123456789012345678901234567890121234567890123456789012345678 123456789012345678901234567890121234567890123456789012345678 99 123456789012345678901234567890121234567890123456789012345678 3. The cost will include the initial charge and the minute rate times thirty, 0.40 9 123456789012345678901234567890121234567890123456789012345678 9 123456789012345678901234567890121234567890123456789012345678 1 0.06(30) 5 0.40 1 1.80 5 2.20. 123456789012345678901234567890121234567890123456789012345678 99 1 9 123456789012345678901234567890121234567890123456789012345678 4. A good, no-nonsense way to find the least common multiple (LCM) is to 9 123456789012345678901234567890121234567890123456789012345678 9 123456789012345678901234567890121234567890123456789012345678 multiply both numbers by increasing positive integers until both multiply to 123456789012345678901234567890121234567890123456789012345678 9 9 123456789012345678901234567890121234567890123456789012345678 the same number. 9 123456789012345678901234567890121234567890123456789012345678 9 123456789012345678901234567890121234567890123456789012345678 18 3 2 5 36 24 3 2 5 48 123456789012345678901234567890121234567890123456789012345678 9 9 123456789012345678901234567890121234567890123456789012345678 18 3 3 5 54 24 3 3 5 72 9 123456789012345678901234567890121234567890123456789012345678 23456789012345678901234567890121234567890123456789012345678 9 123456789012345678901234567890121234567890123456789012345678 18 3 4 5 72 24 3 4 5 96 9 123456789012345678901234567890121234567890123456789012345678 9 1 18 3 5 5 90 24 3 6 5 120 123456789012345678901234567890121234567890123456789012345678 9 9 123456789012345678901234567890121234567890123456789012345678 18 3 6 5 108 24 3 6 5 144 9 123456789012345678901234567890121234567890123456789012345678 9 123456789012345678901234567890121234567890123456789012345678 18 3 7 5 126 9 123456789012345678901234567890121234567890123456789012345678 9 123456789012345678901234567890121234567890123456789012345678 18 3 8 5 144 123456789012345678901234567890121234567890123456789012345678 9 23456789012345678901234567890121234567890123456789012345678 123456789012345678901234567890121234567890123456789012345678 99 123456789012345678901234567890121234567890123456789012345678 Both numbers multiply out to 72, so this is the answer. Creating these two 9 123456789012345678901234567890121234567890123456789012345678 9 tables would take a lot of time if you didn’t have a calculator, but since you 123456789012345678901234567890121234567890123456789012345678 9 1 do, make the most of it. 23456789012345678901234567890121234567890123456789012345678 123456789012345678901234567890121234567890123456789012345678 99 123456789012345678901234567890121234567890123456789012345678 9 123456789012345678901234567890121234567890123456789012345678 9 123456789012345678901234567890121234567890123456789012345678 9 1 9 123456789012345678901234567890121234567890123456789012345678 123456789012345678901234567890121234567890123456789012345678 9 9 123456789012345678901234567890121234567890123456789012345678 123456789012345678901234567890121234567890123456789012345678 9 9 123456789012345678901234567890121234567890123456789012345678 123456789012345678901234567890121234567890123456789012345678 9 123456789012345678901234567890121234567890123456789012345678 9 123456789012345678901234567890121234567890123456789012345678 9 123456789012345678901234567890121234567890123456789012345678 9 1234567890123456789012345678901212345678901234567890123456789 Copyright © 2005 Thomson Peterson’s, a part of The Thomson Corporaton SAT is a registered trademark of the College Entrance Examination Board, which was not involved in the production of and does not endorse this product.

77

1234567890123456789012345678901212345678901234567890123456789 123456789012345678901234567890121234567890123456789012345678 9 9 123456789012345678901234567890121234567890123456789012345678 123456789012345678901234567890121234567890123456789012345678 9 9 123456789012345678901234567890121234567890123456789012345678 5. If x is any negative number, then the inequality will hold true because the 9 123456789012345678901234567890121234567890123456789012345678 9 123456789012345678901234567890121234567890123456789012345678 absolute value bars will knock the value up well over 2. If you don’t see this, 9 123456789012345678901234567890121234567890123456789012345678 23456789012345678901234567890121234567890123456789012345678 9 1 plug in some negative numbers. 9 123456789012345678901234567890121234567890123456789012345678 9 123456789012345678901234567890121234567890123456789012345678 9 123456789012345678901234567890121234567890123456789012345678 Thinking positively, if x . 5 then the inequality is also true. The only 9 123456789012345678901234567890121234567890123456789012345678 23456789012345678901234567890121234567890123456789012345678 9 1 remaining integers are 0–5. Plugging zero into the inequality, it still holds 9 123456789012345678901234567890121234567890123456789012345678 9 123456789012345678901234567890121234567890123456789012345678 true. But if you plug 1–5 in, the inequality is not true. Thus any integer 1, 2, 9 123456789012345678901234567890121234567890123456789012345678 9 123456789012345678901234567890121234567890123456789012345678 3, 4, 5 is correct. 23456789012345678901234567890121234567890123456789012345678 9 1 9 123456789012345678901234567890121234567890123456789012345678 6. This problem works very much like a function machine. Take the number 9 123456789012345678901234567890121234567890123456789012345678 9 123456789012345678901234567890121234567890123456789012345678 they give you and place it into the equation, then solve. For the eleventh 9 123456789012345678901234567890121234567890123456789012345678 23456789012345678901234567890121234567890123456789012345678 9 1 term, k will be equal to 11, so 9 123456789012345678901234567890121234567890123456789012345678 9 123456789012345678901234567890121234567890123456789012345678 9 123456789012345678901234567890121234567890123456789012345678 10~k 1 1! 10~11 1 1! 10~12! 120 9 123456789012345678901234567890121234567890123456789012345678 5 5 5 5 12 23456789012345678901234567890121234567890123456789012345678 9 123456789012345678901234567890121234567890123456789012345678 k21 11 2 1 10 10 9 123456789012345678901234567890121234567890123456789012345678 9 123456789012345678901234567890121234567890123456789012345678 7. If you do not see the answer by visualizing the points, you can always use the 123456789012345678901234567890121234567890123456789012345678 9 9 123456789012345678901234567890121234567890123456789012345678 formula for the midpoint. It is just the average of the coordinates (in this 9 123456789012345678901234567890121234567890123456789012345678 9 1 case, the y coordinates) 9 123456789012345678901234567890121234567890123456789012345678 9 123456789012345678901234567890121234567890123456789012345678 9 123456789012345678901234567890121234567890123456789012345678 y1 1 y2 2 1 6 8 23456789012345678901234567890121234567890123456789012345678 9 123456789012345678901234567890121234567890123456789012345678 5 5 5 4. 9 123456789012345678901234567890121234567890123456789012345678 2 2 2 123456789012345678901234567890121234567890123456789012345678 9 9 123456789012345678901234567890121234567890123456789012345678 8. You can use trigonometric functions to solve this problem, or you could use 9 123456789012345678901234567890121234567890123456789012345678 9 123456789012345678901234567890121234567890123456789012345678 the Pythagorean theorem since this is a right triangle (the measure of the 9 123456789012345678901234567890121234567890123456789012345678 9 123456789012345678901234567890121234567890123456789012345678 angles have to sum to 180 and you know the measures of two of the three 123456789012345678901234567890121234567890123456789012345678 9 9 1 angles, 40 and 50, sum to 90. This means the third angle must be 90.) 9 123456789012345678901234567890121234567890123456789012345678 9 123456789012345678901234567890121234567890123456789012345678 9 2 2 2 123456789012345678901234567890121234567890123456789012345678 1 b 5 c a 123456789012345678901234567890121234567890123456789012345678 9 9 123456789012345678901234567890121234567890123456789012345678 2 9 123456789012345678901234567890121234567890123456789012345678 42 1 ~2=5! 5 c2 9 123456789012345678901234567890121234567890123456789012345678 9 123456789012345678901234567890121234567890123456789012345678 2 16 1 20 5 c 9 123456789012345678901234567890121234567890123456789012345678 9 123456789012345678901234567890121234567890123456789012345678 2 23456789012345678901234567890121234567890123456789012345678 9 123456789012345678901234567890121234567890123456789012345678 36 5 c 123456789012345678901234567890121234567890123456789012345678 9 9 1 65c 9 123456789012345678901234567890121234567890123456789012345678 9 123456789012345678901234567890121234567890123456789012345678 The answer is 6. 23456789012345678901234567890121234567890123456789012345678 123456789012345678901234567890121234567890123456789012345678 9 123456789012345678901234567890121234567890123456789012345678 9 9 123456789012345678901234567890121234567890123456789012345678 9. The mean is the average, so we have to add up all the numbers and divide by 9 123456789012345678901234567890121234567890123456789012345678 9 1 the total: 23456789012345678901234567890121234567890123456789012345678 123456789012345678901234567890121234567890123456789012345678 9 9 123456789012345678901234567890121234567890123456789012345678 7 1 4 1 3 1 6 1 2 1 2 24 9 123456789012345678901234567890121234567890123456789012345678 5 5 4. 9 123456789012345678901234567890121234567890123456789012345678 6 6 9 1 23456789012345678901234567890121234567890123456789012345678 123456789012345678901234567890121234567890123456789012345678 9 9 123456789012345678901234567890121234567890123456789012345678 The median is the number in the middle if you line up all the numbers from 9 123456789012345678901234567890121234567890123456789012345678 9 123456789012345678901234567890121234567890123456789012345678 greatest to least. Since there is an even number of numbers, the median will 9 1 23456789012345678901234567890121234567890123456789012345678 9 123456789012345678901234567890121234567890123456789012345678 be the average of the two middle numbers, {2, 2, 3, 4, 6, 7}. The median then 9 1 9 123456789012345678901234567890121234567890123456789012345678 1 9 123456789012345678901234567890121234567890123456789012345678 . is the average of 3 and 4, or 3.5. The difference between the two is 0.5 or 9 123456789012345678901234567890121234567890123456789012345678 2 23456789012345678901234567890121234567890123456789012345678 9 1 123456789012345678901234567890121234567890123456789012345678 9 123456789012345678901234567890121234567890123456789012345678 9 123456789012345678901234567890121234567890123456789012345678 9 1234567890123456789012345678901212345678901234567890123456789 78

Copyright © 2005 Thomson Peterson’s, a part of The Thomson Corporaton SAT is a registered trademark of the College Entrance Examination Board, which was not involved in the production of and does not endorse this product.

1234567890123456789012345678901212345678901234567890123456789 123456789012345678901234567890121234567890123456789012345678 9 9 123456789012345678901234567890121234567890123456789012345678 123456789012345678901234567890121234567890123456789012345678 9 9 123456789012345678901234567890121234567890123456789012345678 10. Draw this picture! Two lines that are perpendicular to the same line are 9 123456789012345678901234567890121234567890123456789012345678 9 123456789012345678901234567890121234567890123456789012345678 parallel. To see this, draw one line perpendicular to another, and then draw 9 123456789012345678901234567890121234567890123456789012345678 23456789012345678901234567890121234567890123456789012345678 9 1 a third line perpendicular to the second; it will be parallel to the first line). 9 123456789012345678901234567890121234567890123456789012345678 9 123456789012345678901234567890121234567890123456789012345678 2 9 123456789012345678901234567890121234567890123456789012345678 . That means a and c are parallel, which implies they have the same slope, 9 123456789012345678901234567890121234567890123456789012345678 5 23456789012345678901234567890121234567890123456789012345678 9 1 9 123456789012345678901234567890121234567890123456789012345678 11. This might appear complicated, but a few careful substitutions and you will 9 123456789012345678901234567890121234567890123456789012345678 9 123456789012345678901234567890121234567890123456789012345678 have the answer. When, y 5 3, x 5 5 because x 5 y 1 2. 9 123456789012345678901234567890121234567890123456789012345678 23456789012345678901234567890121234567890123456789012345678 9 1 9 123456789012345678901234567890121234567890123456789012345678 Now you need to plug this value of x into the function, 9 123456789012345678901234567890121234567890123456789012345678 9 123456789012345678901234567890121234567890123456789012345678 9 123456789012345678901234567890121234567890123456789012345678 2 1 3 f(x) 5 x 23456789012345678901234567890121234567890123456789012345678 9 1 9 123456789012345678901234567890121234567890123456789012345678 2 9 123456789012345678901234567890121234567890123456789012345678 f~5! 5 5 1 3 5 25 1 3 5 28 9 123456789012345678901234567890121234567890123456789012345678 9 123456789012345678901234567890121234567890123456789012345678 12. The first step is to sketch the inscribed square. 23456789012345678901234567890121234567890123456789012345678 123456789012345678901234567890121234567890123456789012345678 99 123456789012345678901234567890121234567890123456789012345678 9 123456789012345678901234567890121234567890123456789012345678 9 123456789012345678901234567890121234567890123456789012345678 9 123456789012345678901234567890121234567890123456789012345678 9 123456789012345678901234567890121234567890123456789012345678 9 1 9 123456789012345678901234567890121234567890123456789012345678 9 123456789012345678901234567890121234567890123456789012345678 9 123456789012345678901234567890121234567890123456789012345678 23456789012345678901234567890121234567890123456789012345678 123456789012345678901234567890121234567890123456789012345678 99 123456789012345678901234567890121234567890123456789012345678 9 123456789012345678901234567890121234567890123456789012345678 9 123456789012345678901234567890121234567890123456789012345678 9 123456789012345678901234567890121234567890123456789012345678 9 123456789012345678901234567890121234567890123456789012345678 9 123456789012345678901234567890121234567890123456789012345678 9 123456789012345678901234567890121234567890123456789012345678 9 123456789012345678901234567890121234567890123456789012345678 9 1 9 123456789012345678901234567890121234567890123456789012345678 9 123456789012345678901234567890121234567890123456789012345678 The sides of the square are =8 since the area is 8. You get this by working 9 123456789012345678901234567890121234567890123456789012345678 9 123456789012345678901234567890121234567890123456789012345678 backward from the area of a square formula: 9 123456789012345678901234567890121234567890123456789012345678 23456789012345678901234567890121234567890123456789012345678 123456789012345678901234567890121234567890123456789012345678 99 123456789012345678901234567890121234567890123456789012345678 A 5 s2 9 1 9 123456789012345678901234567890121234567890123456789012345678 2 9 123456789012345678901234567890121234567890123456789012345678 8 5 s 23456789012345678901234567890121234567890123456789012345678 123456789012345678901234567890121234567890123456789012345678 99 123456789012345678901234567890121234567890123456789012345678 9 1 =8 5 s 9 123456789012345678901234567890121234567890123456789012345678 9 123456789012345678901234567890121234567890123456789012345678 4 3 2 5 s = 23456789012345678901234567890121234567890123456789012345678 123456789012345678901234567890121234567890123456789012345678 99 123456789012345678901234567890121234567890123456789012345678 9 123456789012345678901234567890121234567890123456789012345678 2=2 5 s 9 123456789012345678901234567890121234567890123456789012345678 9 1 23456789012345678901234567890121234567890123456789012345678 9 123456789012345678901234567890121234567890123456789012345678 The longest straight line that originates and terminates on the circumference 9 123456789012345678901234567890121234567890123456789012345678 9 123456789012345678901234567890121234567890123456789012345678 is the diagonal of the square. Now that you have the side of the square, you 123456789012345678901234567890121234567890123456789012345678 99 1 can find the length of its diagonal because two sides of the square and its 9 123456789012345678901234567890121234567890123456789012345678 9 123456789012345678901234567890121234567890123456789012345678 hypotenuse make a 45-45-90 right triangle. The diagonal is the hypotenuse, 9 123456789012345678901234567890121234567890123456789012345678 9 123456789012345678901234567890121234567890123456789012345678 2 times greater than a side. so it will be = 123456789012345678901234567890121234567890123456789012345678 9 23456789012345678901234567890121234567890123456789012345678 123456789012345678901234567890121234567890123456789012345678 99 1 This means the diagonal of the square is: 9 123456789012345678901234567890121234567890123456789012345678 123456789012345678901234567890121234567890123456789012345678 9 9 123456789012345678901234567890121234567890123456789012345678 ~2=2!~=2! 5 2=4 5 2 3 2 5 4. 9 123456789012345678901234567890121234567890123456789012345678 123456789012345678901234567890121234567890123456789012345678 9 123456789012345678901234567890121234567890123456789012345678 9 123456789012345678901234567890121234567890123456789012345678 9 1234567890123456789012345678901212345678901234567890123456789 Copyright © 2005 Thomson Peterson’s, a part of The Thomson Corporaton SAT is a registered trademark of the College Entrance Examination Board, which was not involved in the production of and does not endorse this product.

79

1234567890123456789012345678901212345678901234567890123456789 123456789012345678901234567890121234567890123456789012345678 9 9 123456789012345678901234567890121234567890123456789012345678 123456789012345678901234567890121234567890123456789012345678 9 9 123456789012345678901234567890121234567890123456789012345678 13. Our range of numbers to meet the conditions is 1000 to 1198. The only way 9 123456789012345678901234567890121234567890123456789012345678 9 123456789012345678901234567890121234567890123456789012345678 to meet the two conditions in the 1100’s is to have the ones and units sum to 9 123456789012345678901234567890121234567890123456789012345678 23456789012345678901234567890121234567890123456789012345678 9 1 15 since with the two 1’s (in the hundreds and thousands place) will make 9 123456789012345678901234567890121234567890123456789012345678 9 123456789012345678901234567890121234567890123456789012345678 the sum 17. But this is impossible, since two numbers that are the same 9 123456789012345678901234567890121234567890123456789012345678 9 123456789012345678901234567890121234567890123456789012345678 cannot sum to an odd number. Therefore, the answer must be in the 1000’s. 23456789012345678901234567890121234567890123456789012345678 9 1 9 123456789012345678901234567890121234567890123456789012345678 1011 is far too less a sum of digits, and 1099 is too great, but not by much. 9 123456789012345678901234567890121234567890123456789012345678 9 123456789012345678901234567890121234567890123456789012345678 Try 1088. It sums to 17. 9 123456789012345678901234567890121234567890123456789012345678 23456789012345678901234567890121234567890123456789012345678 9 1 9 123456789012345678901234567890121234567890123456789012345678 9 123456789012345678901234567890121234567890123456789012345678 9 123456789012345678901234567890121234567890123456789012345678 Section 8 9 123456789012345678901234567890121234567890123456789012345678 23456789012345678901234567890121234567890123456789012345678 9 1 As you might expect, answers will vary. If possible, politely ask a teacher, 9 123456789012345678901234567890121234567890123456789012345678 9 123456789012345678901234567890121234567890123456789012345678 fellow student, or some other person knowledgeable about formal essay 9 123456789012345678901234567890121234567890123456789012345678 9 123456789012345678901234567890121234567890123456789012345678 writing to review your essay and provide feedback on ways in which the 23456789012345678901234567890121234567890123456789012345678 123456789012345678901234567890121234567890123456789012345678 9 9 123456789012345678901234567890121234567890123456789012345678 essay is commendable and on areas where it could be improved. 9 123456789012345678901234567890121234567890123456789012345678 123456789012345678901234567890121234567890123456789012345678 9 123456789012345678901234567890121234567890123456789012345678 9 123456789012345678901234567890121234567890123456789012345678 9 9 1 9 123456789012345678901234567890121234567890123456789012345678 9 123456789012345678901234567890121234567890123456789012345678 9 123456789012345678901234567890121234567890123456789012345678 23456789012345678901234567890121234567890123456789012345678 123456789012345678901234567890121234567890123456789012345678 9 123456789012345678901234567890121234567890123456789012345678 9 123456789012345678901234567890121234567890123456789012345678 9 123456789012345678901234567890121234567890123456789012345678 9 123456789012345678901234567890121234567890123456789012345678 9 123456789012345678901234567890121234567890123456789012345678 9 123456789012345678901234567890121234567890123456789012345678 9 123456789012345678901234567890121234567890123456789012345678 9 123456789012345678901234567890121234567890123456789012345678 9 9 1 9 123456789012345678901234567890121234567890123456789012345678 9 123456789012345678901234567890121234567890123456789012345678 123456789012345678901234567890121234567890123456789012345678 9 123456789012345678901234567890121234567890123456789012345678 9 9 123456789012345678901234567890121234567890123456789012345678 9 123456789012345678901234567890121234567890123456789012345678 9 123456789012345678901234567890121234567890123456789012345678 123456789012345678901234567890121234567890123456789012345678 9 9 123456789012345678901234567890121234567890123456789012345678 9 123456789012345678901234567890121234567890123456789012345678 23456789012345678901234567890121234567890123456789012345678 123456789012345678901234567890121234567890123456789012345678 9 123456789012345678901234567890121234567890123456789012345678 9 9 1 123456789012345678901234567890121234567890123456789012345678 9 123456789012345678901234567890121234567890123456789012345678 9 9 123456789012345678901234567890121234567890123456789012345678 9 123456789012345678901234567890121234567890123456789012345678 9 123456789012345678901234567890121234567890123456789012345678 9 123456789012345678901234567890121234567890123456789012345678 123456789012345678901234567890121234567890123456789012345678 9 23456789012345678901234567890121234567890123456789012345678 123456789012345678901234567890121234567890123456789012345678 9 123456789012345678901234567890121234567890123456789012345678 9 123456789012345678901234567890121234567890123456789012345678 9 123456789012345678901234567890121234567890123456789012345678 9 9 1 9 123456789012345678901234567890121234567890123456789012345678 9 123456789012345678901234567890121234567890123456789012345678 9 123456789012345678901234567890121234567890123456789012345678 9 123456789012345678901234567890121234567890123456789012345678 123456789012345678901234567890121234567890123456789012345678 9 23456789012345678901234567890121234567890123456789012345678 123456789012345678901234567890121234567890123456789012345678 9 9 1 9 123456789012345678901234567890121234567890123456789012345678 123456789012345678901234567890121234567890123456789012345678 9 9 123456789012345678901234567890121234567890123456789012345678 123456789012345678901234567890121234567890123456789012345678 9 123456789012345678901234567890121234567890123456789012345678 9 123456789012345678901234567890121234567890123456789012345678 9 123456789012345678901234567890121234567890123456789012345678 9 1234567890123456789012345678901212345678901234567890123456789 80

Copyright © 2005 Thomson Peterson’s, a part of The Thomson Corporaton SAT is a registered trademark of the College Entrance Examination Board, which was not involved in the production of and does not endorse this product.

12345678901234567890123456789012123456789012345678901234567890121234567 1234567890123456789012345678901212345678901234567890123456789012123456 7 7 12234567890123456789012345678901212345678901234567890123456789012123456 1 34567890123456789012345678901212345678901234567890123456789012123456 7 7 12234567890123456789012345678901212345678901234567890123456789012123456 7 1234567890123456789012345678901212345678901234567890123456789012123456 1 34567890123456789012345678901212345678901234567890123456789012123456 7 7 12234567890123456789012345678901212345678901234567890123456789012123456 1 34567890123456789012345678901212345678901234567890123456789012123456 7 7 12234567890123456789012345678901212345678901234567890123456789012123456 7 1234567890123456789012345678901212345678901234567890123456789012123456 7 1234567890123456789012345678901212345678901234567890123456789012123456 MATH 7 1234567890123456789012345678901212345678901234567890123456789012123456 34567890123456789012345678901212345678901234567890123456789012123456 7 1 7 12234567890123456789012345678901212345678901234567890123456789012123456 Number Number Raw 7 1234567890123456789012345678901212345678901234567890123456789012123456 7 1234567890123456789012345678901212345678901234567890123456789012123456 Correct Incorrect 5 Score 7 1234567890123456789012345678901212345678901234567890123456789012123456 34567890123456789012345678901212345678901234567890123456789012123456 7 1 7 12234567890123456789012345678901212345678901234567890123456789012123456 7 1234567890123456789012345678901212345678901234567890123456789012123456 2 (.25 3 ) 5 Section 2 7 1234567890123456789012345678901212345678901234567890123456789012123456 7 1234567890123456789012345678901212345678901234567890123456789012123456 34567890123456789012345678901212345678901234567890123456789012123456 7 1 7 12234567890123456789012345678901212345678901234567890123456789012123456 7 1234567890123456789012345678901212345678901234567890123456789012123456 Section 5 2 (.25 3 ) 5 7 1234567890123456789012345678901212345678901234567890123456789012123456 7 1234567890123456789012345678901212345678901234567890123456789012123456 34567890123456789012345678901212345678901234567890123456789012123456 1234567890123456789012345678901212345678901234567890123456789012123456 77 1234567890123456789012345678901212345678901234567890123456789012123456 7 1234567890123456789012345678901212345678901234567890123456789012123456 Section 7 2 (.25 3 ) 5 7 1234567890123456789012345678901212345678901234567890123456789012123456 1234567890123456789012345678901212345678901234567890123456789012123456 77 1234567890123456789012345678901212345678901234567890123456789012123456 7 1 CRITICAL READING 7 12234567890123456789012345678901212345678901234567890123456789012123456 7 1234567890123456789012345678901212345678901234567890123456789012123456 7 1234567890123456789012345678901212345678901234567890123456789012123456 34567890123456789012345678901212345678901234567890123456789012123456 7 1234567890123456789012345678901212345678901234567890123456789012123456 Sections 7 1234567890123456789012345678901212345678901234567890123456789012123456 7 1234567890123456789012345678901212345678901234567890123456789012123456 1, 4, and 6 2 (.25 3 ) 5 1234567890123456789012345678901212345678901234567890123456789012123456 77 1234567890123456789012345678901212345678901234567890123456789012123456 7 1234567890123456789012345678901212345678901234567890123456789012123456 7 1234567890123456789012345678901212345678901234567890123456789012123456 7 1234567890123456789012345678901212345678901234567890123456789012123456 WRITING 7 1234567890123456789012345678901212345678901234567890123456789012123456 7 1 2 34567890123456789012345678901212345678901234567890123456789012123456 7 1234567890123456789012345678901212345678901234567890123456789012123456 Section 3 2 (.25 3 ) 5 7 1234567890123456789012345678901212345678901234567890123456789012123456 7 1 1234567890123456789012345678901212345678901234567890123456789012123456 7 7 12234567890123456789012345678901212345678901234567890123456789012123456 7 1234567890123456789012345678901212345678901234567890123456789012123456 Section 8 Go to www.petersons.com/satessayedge/ 7 1234567890123456789012345678901212345678901234567890123456789012123456 7 1 34567890123456789012345678901212345678901234567890123456789012123456 for instant online scoring and feedback. 7 12234567890123456789012345678901212345678901234567890123456789012123456 7 1234567890123456789012345678901212345678901234567890123456789012123456 34567890123456789012345678901212345678901234567890123456789012123456 1234567890123456789012345678901212345678901234567890123456789012123456 77 1234567890123456789012345678901212345678901234567890123456789012123456 7 1 1234567890123456789012345678901212345678901234567890123456789012123456 7 1234567890123456789012345678901212345678901234567890123456789012123456 7 7 12234567890123456789012345678901212345678901234567890123456789012123456 7 1234567890123456789012345678901212345678901234567890123456789012123456 7 1234567890123456789012345678901212345678901234567890123456789012123456 7 1234567890123456789012345678901212345678901234567890123456789012123456 1 34567890123456789012345678901212345678901234567890123456789012123456 7 2 34567890123456789012345678901212345678901234567890123456789012123456 1234567890123456789012345678901212345678901234567890123456789012123456 77 1234567890123456789012345678901212345678901234567890123456789012123456 7 1234567890123456789012345678901212345678901234567890123456789012123456 7 1234567890123456789012345678901212345678901234567890123456789012123456 7 1 7 12234567890123456789012345678901212345678901234567890123456789012123456 7 1234567890123456789012345678901212345678901234567890123456789012123456 7 1234567890123456789012345678901212345678901234567890123456789012123456 7 1234567890123456789012345678901212345678901234567890123456789012123456 1 34567890123456789012345678901212345678901234567890123456789012123456 7 2 34567890123456789012345678901212345678901234567890123456789012123456 1234567890123456789012345678901212345678901234567890123456789012123456 77 1 7 12234567890123456789012345678901212345678901234567890123456789012123456 1 34567890123456789012345678901212345678901234567890123456789012123456 7 7 12234567890123456789012345678901212345678901234567890123456789012123456 1 34567890123456789012345678901212345678901234567890123456789012123456 7 1234567890123456789012345678901212345678901234567890123456789012123456 7 1234567890123456789012345678901212345678901234567890123456789012123456 7 1234567890123456789012345678901212345678901234567890123456789012123456 7 12345678901234567890123456789012123456789012345678901234567890121234567

Scoring Worksheet

Copyright © 2005 Thomson Peterson’s, a part of The Thomson Corporaton SAT is a registered trademark of the College Entrance Examination Board, which was not involved in the production of and does not endorse this product.

81

12345678901234567890123456789012123456789012345678901234567890121234567 1234567890123456789012345678901212345678901234567890123456789012123456 7 34567890123456789012345678901212345678901234567890123456789012123456 7 12 1234567890123456789012345678901212345678901234567890123456789012123456 7 34567890123456789012345678901212345678901234567890123456789012123456 7 12 34567890123456789012345678901212345678901234567890123456789012123456 7 12 1234567890123456789012345678901212345678901234567890123456789012123456 7 34567890123456789012345678901212345678901234567890123456789012123456 7 12 7 1234567890123456789012345678901212345678901234567890123456789012123456 MATH CRITICAL READING 34567890123456789012345678901212345678901234567890123456789012123456 7 12 34567890123456789012345678901212345678901234567890123456789012123456 7 12 34567890123456789012345678901212345678901234567890123456789012123456 7 12 Raw Math Scaled Raw Math Scaled Raw Verbal Scaled Raw Verbal Scaled 34567890123456789012345678901212345678901234567890123456789012123456 7 12 Score Score Score Score Score Score Score Score 2 34567890123456789012345678901212345678901234567890123456789012123456 7 1 34567890123456789012345678901212345678901234567890123456789012123456 7 12 78 800 37 510 60 800 28 500 34567890123456789012345678901212345678901234567890123456789012123456 7 12 34567890123456789012345678901212345678901234567890123456789012123456 7 12 77 800 36 510 59 800 27 490 34567890123456789012345678901212345678901234567890123456789012123456 7 12 76 800 35 500 2 34567890123456789012345678901212345678901234567890123456789012123456 7 1 58 790 26 490 34567890123456789012345678901212345678901234567890123456789012123456 7 12 75 790 34 500 57 770 25 480 34567890123456789012345678901212345678901234567890123456789012123456 7 12 34567890123456789012345678901212345678901234567890123456789012123456 7 12 74 780 33 490 56 760 24 470 34567890123456789012345678901212345678901234567890123456789012123456 7 12 73 770 32 490 2 34567890123456789012345678901212345678901234567890123456789012123456 7 1 55 740 23 460 34567890123456789012345678901212345678901234567890123456789012123456 7 12 72 760 31 480 34567890123456789012345678901212345678901234567890123456789012123456 7 12 54 720 22 460 34567890123456789012345678901212345678901234567890123456789012123456 7 12 71 750 30 480 34567890123456789012345678901212345678901234567890123456789012123456 7 12 53 710 21 450 2 34567890123456789012345678901212345678901234567890123456789012123456 7 1234567890123456789012345678901212345678901234567890123456789012123456 70 740 29 470 52 700 20 440 7 1234567890123456789012345678901212345678901234567890123456789012123456 69 730 28 460 7 1234567890123456789012345678901212345678901234567890123456789012123456 51 690 19 430 7 1234567890123456789012345678901212345678901234567890123456789012123456 68 720 27 460 7 1234567890123456789012345678901212345678901234567890123456789012123456 50 680 18 420 7 1234567890123456789012345678901212345678901234567890123456789012123456 67 710 26 450 7 1 49 670 17 420 66 700 25 450 34567890123456789012345678901212345678901234567890123456789012123456 7 12 48 660 16 410 34567890123456789012345678901212345678901234567890123456789012123456 7 12 65 700 24 440 34567890123456789012345678901212345678901234567890123456789012123456 7 12 47 650 15 410 2 34567890123456789012345678901212345678901234567890123456789012123456 7 1234567890123456789012345678901212345678901234567890123456789012123456 64 690 23 440 7 1234567890123456789012345678901212345678901234567890123456789012123456 46 640 14 400 7 1234567890123456789012345678901212345678901234567890123456789012123456 63 680 22 430 7 1234567890123456789012345678901212345678901234567890123456789012123456 45 630 13 390 62 670 21 420 1234567890123456789012345678901212345678901234567890123456789012123456 77 1234567890123456789012345678901212345678901234567890123456789012123456 44 620 12 380 61 670 20 410 7 1234567890123456789012345678901212345678901234567890123456789012123456 43 610 11 370 7 1234567890123456789012345678901212345678901234567890123456789012123456 60 660 19 410 7 1234567890123456789012345678901212345678901234567890123456789012123456 42 600 10 360 59 650 18 400 7 1 2 34567890123456789012345678901212345678901234567890123456789012123456 7 1234567890123456789012345678901212345678901234567890123456789012123456 41 600 9 350 58 640 17 390 7 1234567890123456789012345678901212345678901234567890123456789012123456 7 1 40 57 640 16 380 590 8 340 7 1234567890123456789012345678901212345678901234567890123456789012123456 56 630 15 380 34567890123456789012345678901212345678901234567890123456789012123456 7 12 39 580 7 330 2 34567890123456789012345678901212345678901234567890123456789012123456 7 1234567890123456789012345678901212345678901234567890123456789012123456 55 620 14 370 38 570 6 320 7 1234567890123456789012345678901212345678901234567890123456789012123456 7 1 37 54 610 13 360 560 5 310 34567890123456789012345678901212345678901234567890123456789012123456 7 12 34567890123456789012345678901212345678901234567890123456789012123456 7 12 53 610 12 360 36 560 4 300 34567890123456789012345678901212345678901234567890123456789012123456 7 12 52 600 11 350 34567890123456789012345678901212345678901234567890123456789012123456 7 12 35 550 3 280 7 1234567890123456789012345678901212345678901234567890123456789012123456 51 600 10 340 34 540 2 270 7 1234567890123456789012345678901212345678901234567890123456789012123456 7 1234567890123456789012345678901212345678901234567890123456789012123456 50 590 9 330 33 540 1 250 2 34567890123456789012345678901212345678901234567890123456789012123456 7 1234567890123456789012345678901212345678901234567890123456789012123456 49 590 8 320 7 1234567890123456789012345678901212345678901234567890123456789012123456 32 530 0 240 7 1234567890123456789012345678901212345678901234567890123456789012123456 48 580 7 310 7 1234567890123456789012345678901212345678901234567890123456789012123456 31 520 21 220 7 1 47 570 6 300 2 34567890123456789012345678901212345678901234567890123456789012123456 7 1234567890123456789012345678901212345678901234567890123456789012123456 30 510 22 210 46 570 5 290 7 1234567890123456789012345678901212345678901234567890123456789012123456 29 510 23 and 200 7 1234567890123456789012345678901212345678901234567890123456789012123456 45 560 4 270 7 1234567890123456789012345678901212345678901234567890123456789012123456 below 7 1 44 550 3 260 2 34567890123456789012345678901212345678901234567890123456789012123456 1234567890123456789012345678901212345678901234567890123456789012123456 77 1234567890123456789012345678901212345678901234567890123456789012123456 43 550 2 250 7 1234567890123456789012345678901212345678901234567890123456789012123456 42 540 1 240 7 1234567890123456789012345678901212345678901234567890123456789012123456 7 1 41 540 0 230 2 34567890123456789012345678901212345678901234567890123456789012123456 1234567890123456789012345678901212345678901234567890123456789012123456 77 1 40 530 21 220 34567890123456789012345678901212345678901234567890123456789012123456 7 12 39 520 22 210 7 1234567890123456789012345678901212345678901234567890123456789012123456 34567890123456789012345678901212345678901234567890123456789012123456 7 12 38 520 23 and 200 2 34567890123456789012345678901212345678901234567890123456789012123456 7 1 7 1234567890123456789012345678901212345678901234567890123456789012123456 below 1234567890123456789012345678901212345678901234567890123456789012123456 77 1234567890123456789012345678901212345678901234567890123456789012123456 12345678901234567890123456789012123456789012345678901234567890121234567

Score Charts

82

Copyright © 2005 Thomson Peterson’s, a part of The Thomson Corporaton SAT is a registered trademark of the College Entrance Examination Board, which was not involved in the production of and does not endorse this product.

Related Documents

Sat Practise Test 1
May 2020 12
Sat Practise Test 2
May 2020 12
Sat Test
November 2019 12
Sat Reasoning Test Lecture 1
November 2019 10
Sat Test 11
October 2019 17
Sat 1
October 2019 19